You are on page 1of 954
Se crete Ts Main etd aes INA [== Tata McGraw-Hill Published by Tata McGraw Hill Education Private Limited, 7 West Patel Nagar, New Delhi 110.008 ‘Course in Mathematics for IIT-JEE 2012 Copyright © 2011, by Tata McGraw Hill Education Private Limited. No part of this publication may be reproduced or distributed in any fotm or by any means, electronic, mechanical, photocopying, recording, or otherwise or stored in a database or retrieval system without the prior written permission of the publishers. The program listings (if any) may be entered, stored and executed in a computer system, but they may not be reproduced for publi ‘This edition can be exported from India only by the publishers, Tata McGraw Hill Education Private Limited. Price: & 625.00 ISBN (13):978-0-07-132928.-6 ISBN (10): 0-07-132928-5 Vice President and Managing Director—MGraw-Hill Education: Asia Pacific Region: Ajay Shukla Head—Test Prep and School: ¥. Biju Kumar Publishing Manager—Test Prep: KN Prakash Manager (Sponsoring): Abhishek Sharma Editorial Executive: Pratibha Singh ‘Asst Manager (Developmental Editing): Anubha Srivastava Junior Manager—Production: Medha Arora Dy Marketing Manager: Niju Sreedharan General Manager—Production: Rajender P. Ghansela ‘Asst General Manager—Production: 8. L. Dogra Information contained in this work has been obtained by Tata McGraw-Hill, from sources believed to be reliable. However, neither Tata McGraw-Hill nor its authors guarantee the accuracy or completeness of any information published herein, and neither Tata McGraw-Hill nor its authors shall be responsible for any errors, omissions, or damages arising out of use of this information. This work is published with the understanding that Tata McGraw-Hill and its authors are supplying information bat are not attempting to render engineering or other professional services. If such services are required, the assistance of an ‘Typeset at Script Makers, 19, A1-B, DDA Market, Paschim Vihar, New Delhi 110 063, and text and cover printed at Gopsons, A-2 & 3 Sector-64, Noida, U.P. 201301 Cover Designer: K Anoop DAALCRZCDAYDB een Contents Syllabus Format tions in this Book 1. Complex Numbers 2. Theory of Equations 3. Progressions 4. Logarithm 5. Permutations and Combinations 6. Binomial Theorem 7. Matrices 8. Determinants 9. Probability 10. Trigonometry 11, Trigonometric Equations 12, Solution of Triangles and Applications of Trigonometry 13. Inverse Trigonometric Functions 14, Cartesian System of Coordinates and Locus 15,_Straight Lines 16__Circles 17. Conic Section (Parabola, Ellipse, Hyperbola) 18, Three Dimensional Geometry 19_Eunctions 20. Limit and Continuity 21. Differentiation 22. Applications of Derivatives 23. Indefinite Integration 24. Definite Integrals 25,_Differential Equations 26. Vector Algebra LT-JEE 2008 Mathematics (Solved Paper-l) HIT-JEE 2008 Mathematics (Solved Paper-I1) HIT-JEE 2009 Mathematics (Solved Paper-1) HIT-JEE 2009 Mathematics (Solved Paper-I1) HT-JEE 2010 Mathematics (Solved Paper-1) HIT-JEE 2010 Mathematics (Solved Paper-I1) 10.1-10.48 1A-1LI8 12.1-12.46 13.24 14,1-14.20 19.1-19.33 20.1-20.44 21.1-21.48 24.1-24.55 25.1-25.38 26.1-26.40 rk, ae. Copyrighted material Format of Questions in this Book SECTION A Straicut Opsective Type ‘This section contains multiple choice questions. Each question has four choices (a), (b), (c) and (d), out of which only one answer is correct. SECTION B Murtiete Correct Answers Tyre This section contains multiple correct answers type questions. Each question has four choices (a), (b), (€) and (d), out of which one or more answers are correct. SECTION C Linkep ComPREHENSION TYPE Each set in this section contains a paragraph followed by questions. Each question has four choices (a), (b), (¢) and (d), out of which one or more answers are correct. SECTION D Waite tHe ANSWER TYPE Each question in this section has an answer which is a non-negative integer. (Contd) xii Format of Questions in this Book SECTION E Matrix Marcu Type ‘Each question in this section contains statements given in two columns, which have to be matched. Statements ‘in Columa I are labelled as a, b, ¢ and-d whereas statements in Column II are labelled as p, q, r and s, The answers to these questions have to be appropriately bubbled as illustrated in the following example. Ifthe correct matches are a—g, a—1, b—p, b's, c—s and d—q, then the correctly bubbied matrix will look like the following. pat REASONING TYPE } Bach question in section has four choices (a), (b), (c) and (d) out of which only one is correct. Mark your choices as follows: (@) STATEMENT-1 is True, STATEMENT-2 is True; STATEMENT-2 is a correct explanation for STATEMENT-1 (6) STATEMENT-1 is True, STATEMENT-2 is True; STATEMENT-2 is NOT a correct explanation for STATEMENT- STATEMENT-1 is True, STATEMENT-2 is False (d) STATEMENT-1 is False, STATEMENT-2 is True Cuapter + ComP_Lex NUMBERS 4.1. DEFINITIONS A number of the form a + ib where a, b € R, the set of real numbers, and i = J—1, is called a complex number. A complex number can also be defined as an ordered pair of real numbers a and b, and may be written as-(a, 6), where the first number denotes the real part and the second number denotes the imaginary part. If z =a + ib, then the real part of z is denoted by Re(z) and the imaginary part of z is denoted by Im(2). ‘A complex number z is said to be purely real if Im(z)= 0 and is said to be purely imaginary if Re(z) = 0. Note that the complex number 0 = 0 + i0 is both purely real and purely imaginary. Itis the only complex number with this property. We denote the set of all complex numbers by C. That C= {a + ibla,b © R}. Two complex numbers z; = ay + ib, and 2, = a + iby are said to be equal if a, = a, and by =by. 1.2 ALGEBRAIC OPERATIONS WITH COMPLEX NUMBERS 1. Addition: (a + ib) + (c + id 2. Subtraction: (a+ ib) ~(c + id 3. Multiplication: (a + ib) (c + id) = (ac ~ bd) + iad + be) 4. Reciprocal: If at least one of a, b is non-zero then the reciprocal of a + bi is given by 1 a-ib a+ib (a+ib)(a-ib) S. Quotient: If at least one of c, d is non-zero, then quotient of a + bi and ¢ + di is given by (a +c)+i(b+d) (ac) +i(b-d) abi _(a+ib)(c-id) _(ac+bd)+i(be~ad) e+di (c+id)(c~id) +d? actbd , ,be-ad C+ ed 1.3 CONJUGATE OF COMPLEX NUMBER Let :=a + ib be a complex number. We define conjugate of z, denoted by Z to be the complex number a — ib. That is, if z= a + ib, then Z ~ ib. Properties of Conjugate of a Complex Number @ 427 RH i) @ Gili) z+ tiv) (v) z= 2 ee zis purely real (vi) 2+ Z =0€ zis purely imaginary. (vil) <2 = (Re(@P + fIm@)P* (iii) (x) &) (xi) (=) slit +0 a (xii) If PQ) = ay + ay 2 tay? +... +a, 2" where dy, aj, ... dy and z are complex number, then P(2) = dy +4,(Z)+4,(z +a, (Z)" = Pe where P(z) = a, +a,z+a,27 ++++4,2" = £© where P (2) and Q (2) are Oe ‘ polynomials in z, and Q(z) # 0, then P@) (xiii) If R(@) R@ 1.2 Course in Mathematics for ITT-JEE j1 42 4) a 4, (xiv) fz =| 45), then bb lt 4s a a where a,, bj ¢; (i= 1, 2, 3) are complex numbers. 4.4 MODULUS OF A COMPLEX NUMBER Let z = @ + ib be a complex number. We define the modulus or the absolute value of z.to be the real number |Va?+5* and denote it by Iz1. ‘Note that 12120 ze C Properties of Modulus ety If zis a complex number, then @ ld=0ez= (i) = igl=ba=bel iii) - 121 S Re(z) slat (iv) lel s Ime) $ tel @) ez ker If z,, % are two complex numbers, then (vi) ey zal = lel gh aay (2 (wii) le lg? + lgP + z+ 1% = ley? + leg)? + 2Re(ziz2) (viii) lay + 2. (ix) Izy - a = lay? + le? - zy -a = Ig? + lef? - 2Re(ziZ2) (x) lay + 2 + ley — 23 = 20a + LP) (xi) Ifa and b are real numbers and z,, z, are complex numbers, then lazy + begl® + Woz, — azyl® = (a? + B) (Izy! + leg?) (xii) If zy, 2 # 0, then Iz, + 2 = lzyP + ley ° a is purely imaginary. 2 (xiii) Triangle Inequality. If 2, and z, are two complex aumbers, then Z, + z3! $ Iz! + lzl. ‘The equality holds if and only if :,z, 20. Hn general, lzj'#Z, +. + APS lz + led # + el and the sign equality sign holds if and only if the ratio of any two non-zero terms is positive, (xiv) Izy ~ 231 S kyl + leat (ev) sy! bel + lel (xvi) Izy ~ 2a 2 Hl! ~ tel (xvii) If @,, ay, a3, are four complex numbers, then k-a@l+k-al+k-ajl+k - a > max {la - a! + la, — at: 1, m,n are distinct integers lying in {2, 3, 4) and m 0 tan“(yixj+e if x<0,y20 -2*)} arg (2)=Jran“(yla)—x if x0 In fact, if z= r (cos 6 +i sin ), then z is also given by -n2 if x=0,y<0 z= 7 [cos (ke +0) + i sin (2k +8)] Caution A usual mistake committed by the students is to take the argument of z =.x+ iy as tan” (y/2x) irrespective of the values of x and y. Kindly remember that tan (y/x) lies in the interval (~ 2/2, 212) whereas the principal value of argument of z lies in the interval (~ , 2). 4.7 POLAR FORM OF A COMPLEX NUMBER Let z be a non-zero complex number, then we can write 7 (os @+ isin 8) lel and 0 = arg (2). where where kis any integer. Euler's Formula ‘The complex number cos 8+ i sin Bis denoted by e or cis @ That is e's tis O= cos 0+ isin O ‘Some Important Results involving Argument If z, znd zy are complex numbers, then @ arg (Z) = ~ arg @) 14 Course in Mathematics for LIT-JEE (ii) arg (zy 23) = ang (e:) + arg (2) In fact, arg (21 2) = where rg (z)) + ang (z_) + 2k 0 if - cary (z,) +arg(z,) Se ke {1 if ~2e-, ys) respectively. Sum By definition, 2, + z» should be represented by the point (4, + x3, ¥ + y)- This point is nothing but the vertex P which completes the parallelogram with the line segments joining the origin with z, and z, as the adjacent sides. See Fig. 13 Poy tai ty Fig. 1.3 Note that the addition of two complex numbers z, and z, follows the same law of addition as that of vectors, represented both in magnitude and direction by the line ‘segments joining the origin and the points representing 2, and 2, for OP, + OP; = OP, + P,P = OP Difference We first represent ~ z, by P’; so that PP’ is bisected at O. Complete the parallelogram OP, PP’. ‘Then, it can be easily seen that P represents the difference As OP, PP; is a parallelogram so P,P = OP’. Using vector notation, we have OP, ~ OP, = OP, + P,O = OP, + OP’; = OP, + P,P = OP = P,P, Pur Fig. 14 Note the complex number z, ~ z; is represented by the vector P3P;, where the points P, and P, represent the complex number 2, and z, respectively. Note that arg(z; ~z,) is the angle through which OX must be rotated in the anticlockwise direction so that it becomes parallel to P,P, Product (Multiplication) Let 2, =r(cos @, +isin 6) and z, =r; (cos 6 + isin 8). Then 225 rrp (C0s 8 +isin &) (cos & + isin A) = 1) ty (cost, + B) +i sin (B, + &)} Thus tz g1= rr, and argtz, 23) = @ + & This shows that modulus of the product of two complex numbers is the product of their moduli, and the argument of the product of any two complex numbers is the sum of their arguments. Using this, we shall derive a geometrical interpretation of the product of two complex numbers. Let £ be a point on the x-axis such that OE = 1 unit. (See Fig 1.5.) Complete the triangle OPE. Now, taking OP; as the base, construct a triangle OPP, similar to OPE so that OP : OP, = OP; OE ie, OP = OP, - OP; : OE = 11 Also 2P,0P = ZEOP, = ZXOP, = 6, Thus ZXOP= 6 + & Hence P represents the complex number for which the modulus is ryr and the argument is 6, + @. That is, it represents the complex number z; z. Complex Numbers 1S Fig. 1.5 Quotient Let 2, = 1, (cos 6, + isin 6) and z, =r, (cos 6, + isin @), We take z) 0, so that r) #0. Now _ Fi cos 8, +i sin A) ry (608 8, + isin 8) feos (6, = 0) + i sin (8, -8)) bd We shall use this to get a geometrical interpretation of the quotient of a complex number by a non-zero complex number, Let P; and P, represent 2, and zy respectively, On OP; construct the triangle OPP; similar to OEP, where E lies on the x-axis and OE = I unit. (See Fig. 1.6.) Now, OP : OE= ir Also ZXOP = 8, - 0 ‘The point P thus represents the quotient z,/23, since its ‘modulus is r\/ry and its arguments is 8, ~ 8,. 1.6 Course in Mathematics for IIT-JEE Remark ) Note that if 6, and @, are the principal values of arg z, | and arg z; then @, + 8, is not necessarily the principal value of arg(z, z2), nor is 6 — @ necessarily the principal value of arg(z;/:3). Interpretation of arg (3) nna If 4, Zp. are the vertices of a triangle ABC described in the counter-clockwise sense, then Fig. 17 Corollary The points zz, 25 will be collinear if and only if angle a= 0 or x, ie., if and only if 2 = is 27 purely real. Fig. 1.8 B Interpretation of ara { Let 2), 2,23 and z, be four complex numbers. Then the line joining z, and zy is inclined to the line joining z, and z, at the following angle: «n(222) Corollary ‘The line joining z, and z; is inclined at 90° to the line joining z, and z, if are x ($2) “84 + ik(c, — 2), where k is a non-zero real 19). ive, if number. (Fij Fig. 1.9 1.40 SOME IMPORTANT GEOMETRICAL RESULTS AND EQUATIONS 1. Distance Formula Distance between A(z;) and B(z,) is given by AB = Izy a Ble.) AG) Fig. 1.10 2. Section Formula The point P(z) which divides the join of the segment AB in the ratio m : n is given by ee) sr) AG) Fig. 1.11 3. Mid-point Formula Mid-point M(z) of the segment AB is given by 1 ze t+) Be) Me) 4G) Fig. 1.12 4. Condition(s) for four non-collinear A(z,), Blz2), Ces) and D(zq) to represent vertices of a (i) Parallelogram The diagonals AC and BD must bisect cach other t 1 1 quits Faery ° aytyenty Dee cen Ai) BG) Fig. 1.13 i) Rhombus (a) the diagonals AC and BD bisect each other ° +e + % and (b) a pair of two adjacent sides are equal, for instance, AD = AB yak, 2)! Dew ce) AG) Bey Fig. 1.14 Complex Numbers 1.7 (iii) Square (a) the diagonals AC and BD bisect each other 2 ytnenty (b) a pair of adjacent sides are equal; for instance, AD = AB y= al=ly- a! (©) the two diagonals are equal, that is, AC = BD € Izy ~ a1 = ley - 2h Di) cus) No asd Aw) Bu) Fig. 1.15 (iv) Rectangle (a) the diagonals AC and BD bisect each other oe atnenty (b) the diagonals AC and BD are equal ky -aely- 2h Des) Ce) Pe ae | - aie AG) Be) Fig. 1.16 ‘Showing that four non-collinear points form a 1.8 Course in Mathematics for IIT-JEE 5. Centroid, Incentre, Orthocentre and Circum- centre of a Triangle Let ABC be a triangle with vertices A(z,), B(z3) and Cz), (i) Centroid G(z) of the AABC is the point of concurrence of medians of AABC and is given by 1 t= 5 Gitar 4 5 e 3 D c Fig. 1.7 (ii) Incentre (2) of the AABC is the point of concurrence of internal bisectors of angles of AABC and is given by az, +be, +62, atbte Fig. 1.18 ii) Circumcentre S(z) of the AABC is the point of concurrence of perpendicular bisectors of sides of AABC and is given by (- lal H@-vlal + -adlal %— 4) + HG — 4) + HZ — z= Fig. 1.19 (iv) Orthocentre H(z) of the AABC is the point of concurrence of altitudes of ABC and is aiven by 2a af iiah ad 2% Wellaf ay a 5 i [igh gt = (an A) s+ (an BY, + (tan O)2, tan A+ tan B+ tan C px (2S00A) 5 + (b see Bz, +(e 80 Oe, asec A+bsec B+csecC Ati) or or aD cs) Fig. 1.20 Remark In case circumcentre of AABC is at the oti orthocentre of triangle is given by z; + z) + 23. See Example 22. Euler's Line ‘The centroid G of a triangle lies on the segment joining the orthocentre H and the circumcentre $ of the triangle. G divides the join of H and S in the ratio 2 : 1 G s (Onthocentre) (Centroid) —_(Cireumcentre) Fig. 1.21 Thus, 1 to = 3 (en +2zs) 6. Area of a Triangle Area of AABC with vertices A(z,), B(2,) and C(2,) is given by BG) ces) Fig. 1.22 7. Condition for Triangle to be Equilateral Triangle ABC with vertices A(z)), B(z.) and C(z;) is equilateral if and only if ° ° ° tn gy ° 14 4] =0 an 2 Complex Numbers 1.9 1 where reseitatad Ath) a iN Bi) Clts) Fig. 1.23 8. Equation of a Straight Line (@) Non-parametric form ‘An equation of a straight Line joining the two points A(z;) and B(z,) is or ark o 7 a)+4%— 4% =0 PO Bie) in) Fig, 1.24 (i) Parametric form ‘An equation of the straight points A(z,) and B(z,) is remy +(1- Dy where 1 is a real parameter. (iii) General Equation of a Straight Line The general equation of a straight line is azt+ai+b =0 where a is a non-zero complex number and b is a real number. 9. Complex Slope of a Line If A(;) and B(Z,) are two points in the complex plane, then complex slope of AB is defined to be joining the Be 4 ‘Two lines with complex slopes Hi, and Hare parallel, if) = Gi) perpendicular, if 1 + py = 0 ‘The complex slope of the line Gz+az +b=0 is given by ~(a/a) 1.10 Course in Mathematics for IIT-JEE 10. Length of Perpendicular from a Point to 2 Line Length of perpendicular of point A(q) from the line az-+az+b=0 (a€ C- (0), be R)is given by faa+ aw +b) al Fig. 4.25 11, Equation of Circle ) An equation of the circle with centre at zo and radius ris k-al=r or = x + re®, 0.5 8 < 2x (parametric form) or -4%-%2+4%-7 =0 Fig. 1.26 (ii) General equation of a circle is etarta+b=0 a where a i$ a complex number and b is a real number such that aa ~b2>0 Centre of (1) is ~ a and its radius is Yaa —6 (iii) Diameter Form of a Circle ‘An equation of the circle one of whose diam- ‘eter is the segment joining A(z,) and B(z,) is 2) @-%)+(@- 4) @-a)=0 Pa) AG) Bie) Fig. 1.27 (iy) An equation of the circle passing through two points A (z,) and B (z,) is -2) (E-H)+@-H)e-2) zi +H 4 =o le % where k is a parameter, (¥) Equation of a circle passing through three non-collinear points. Let three non-collinear points be A(z,), B(z2) and C(z,), Let P(z) be any point on the circle, Then either (ZACB = ZAPB (when angles are in the same segment] ZACB + ZAPB = [when angles are in the opposite segment} Pe) a Vo ete ae} Pe Bt) a4 a fusing arg ( arg (2, %)= arg (2) + arg (z:)) (vi) Condition for four points to be concyclic. Four points 2,, 25, z3 and zy will lie on the (4-4) same circle if and only if is purely real, 1.11 RECOGNIZING SOME LOCI BY INSPECTION @ arg @) = @ Wf @ is a real number, then arg (2) = @ represents a ray starting at the origin (excluding the origin) and making an angle a with the real axis. See Fig. 1.29. Fig. 1.29 (ii) arg @- 2g) =@ If cis a real number and z) is a fixed point, then arg (z ~ za) = @ represents a ray starting at za (excluding the point z)) and making an angle o with the real axis. y Fig, 1.30 (iii) If 2, and 2, are two fixed points, then ke-gl=k- a represents perpendicular bisector of the segment joining A(z,) and B(z,). Fig. 1.31 Complex Numbers 1.11 (iv) If 2; and z) are two fixed points, and k > 0, & # 1 is a real number, then represents a circle, For k = 1, it represents perpendicular bisector of the segment joining A(z,) and B(z,). W) Iz-altlz-glek Let z, and z, be two fixed points and k be a positive real number. (a) If k > lz, — , then lc -2yl + le — z= k represents an ellipse with foci at A(z,) and Be) and length of major axis f__» D Yee Fig. 1.32 (b) If k= zy — zh, then ke-al+k-al=k represents the segment joining 2, and x. (©) If k < lz, ~ oy, then len altize-alek does not represent any curve in the Argand plane. Wakz- lek Let z, and z) be two fixed points, k be a positive real number. (a) IK # ley ~ zl, then wi) k-al-k-gek represents a hyperbola with foci at A(z;) and BC) Ae) BU) Fig, 1.33 (b) Ik = ly -zphthen tk-z-k- all represents the straight line joining A(z,) and B(z;) but excluding the segment AB. See Fig. | AG) Bs) Fig. 1.34 1,12 Course in Mathematics for IT-JEE (vii) Iz — 2,P + Iz - 23P = tz, ~ 21 If z, and z are two fixed points le = 2P + le - 2p? = ey 2 represent a circle with z; and z; as the extremities of a diameter. See Fig. 1.35. then Bee) Agi) Fig. 1.38 cw n(8)n0 Let z and z, be two fixed points, and abe a real number such 0 < @ S a. (a) If 0 nee ae 2° lal BG) A(z) ° @ 0 Fig. 1.41 1.13 DE MOIVRE’S THEOREM AND ITS APPLICATIONS: (a) De Moivre’s Theorem for integral index. If n is an integer, then (cos 8 + i sin 6)" = cos (n®) +i sin (n®) (b) De Moivre’s Theorem for rational index. If nis a rational number, then value of or one of the values of (cos + isin)" is cos (n®) + i sin (m6). In fact, if n = plq where p,q €1,q>0 and p.q have no factors in common, then (cos @+ i sin @" has q distinct values, one of which is cos (n®) + i sin (n8). Note } ‘The values of (cos @+ i sin @”* where p,q € 1. q #0, hef (p,q) =I are given by os| Leake] vial Bakes | | 7 q | where k=0,1, ‘The nth Roots of Unity By an nth root of unity we mean any complex number z which satisfies the equation gel ay Since, an equation of degree m has n roots, there are values of z which satisfy the equation (1). To obtain these n values of z, we write Complex Numbers 1.13 1 = cos (2km) + i sin (2kx) where ke Tand = ze cos (242) + sin (2) n using the De Moivre's Theorem] where k= 0,12 0.01. Note | We may give any n consecutive integral values to k. | For instance, in case of 3, we may take ~ 1, O.and | and in case of 4, we may take ~ 1, 0, 1 and 2 | or—2,-1,0and 1. 22), 5 .( 2" Notatio 7 22) 3) mn =cox{ + sin( 24 By using the De Moivre’s theorem, we can write the nth roots of unity as aay... a. Sum of the Roots of Unity is Zero We have Le ot. tat ts be re But af = 1 as @is a nth root of ur l+tor..+a" Also, note that 1 xo Writing nth Roots of Unity When n is Odd If n= 2m + 1, then nth roots of unity are also given by exo) 22) where m,—(m—1), 5-1, 0,1, 2, ...m. Since cos (- 242) = cos{ 24) wt 2a we may take the roots as 2k) 5; gin (2H 1, cog 7°) + sin (2) where k= 1,2, 00m In terms, @ we may take nth roots of unity to be 1, a*", oa ‘Writing nth Roots of Unity When m is Even If n= 2m, then nth roots of unity are given etl eQs@..t 1.14 Course in Mathematics for ITT-JEE. wien ene 2E 2m: Cube Roots of Unity Cube roots of unity are given by 1, «, a, where ovo) Bae Some Results Involving Complex Cube Root of Unity (@) @ w=) G) L+a+o=0 Gi) P-1=@-)O-9-a) (iv) @and af are roots of 7 +x + 1=0 () a - B= (ab) (a - bo (a- ba) Wi) +B + 2 be ~ ca - ab = (a+ bar cal) (a + be + co) @ +b +0) -3abe = (a+b +0) (a+ bo + ca) (a + ba + co PH = + 1) (e+ oe + oF) (ix) a’ + bP = (a + b) (a + bo (a + bar) (x) Cube roots of real number a area’, a' ax.a\* af, To obtain cube roots of a, we write x° = a as y' = 1 where: yeaa’, Solution of (vii) (viii) Lare 1, @ a. xealal? aa nth Roots of a Complex Number Let ¢ #0 be a complex number. We can write z in the polar form as follows: z= r(cos 8+ i sin where r= lzland @= arg (2). Recall - r< OS x. The nth root of z has n values one of which is equal to ae sa [cos( " )ésin ( 5) and is called as the principal value of To obtain other value of Vid, we write 2 as z=r {cos (8+ 2k) + i sin (6 + 2k] coe( 2224 sisal orth 13 gf = af where k= 0, 1, 2,0... 2-1 and @= cos 2 + isin 2% is a complex nth root of unity. ‘Thus, all the m nth roots of z can be obtained by multi- plying the principal value of V/izi by different roots of unity. Rational Power of a Complex Number If z is a complex number and m/n is a rational number such that m and 1 are relatively prime integers and n> 0. We define a= (2)" ‘Thus, 2%" has n distinct values which are given by ~ ~(f3)" [so ( 42k) ” where k= 0, 1, 2, ra))] SINGLE CORRECT CHOICE TYPE Example 1 The number of solutions of z+ 2 = Ois (a) 2 ) 3 4 @s Ans. (d) Solution 2+2=0 = z = lPe=t ‘This equation has four non-zero and distinct roots. ‘Therefore, the given equation has five roots. TIP It is unnecessary to find roots of 2 + 1 = 0 Example 2 Let p and q be two complex numbers such that g # 0. If the roots of the equation x° + px + q? = have the same absolute value, then pig is (a) an integer (b) a rational number (©) a real number (@) acomplex number. | Ans. (¢) Solution Let x,, x, be the roots of 7 + px + q? = 0 and Ix = by! =r, Note that x), = q? #0. Now, PL Gtx) 2 Retmi)+ 2 r = © is areal number. q Next, as — Re (xj, ¥)) Sly Hl = by xl = byl byl = 7, we get 2 op = Fes ¢ 7 ‘Therefore, pig is a real number. Example 3 If k>0,|zl=and =-24+2=0 k Sage then Retw) equals @o (b) k 1 1 ©) k @) - t Ans. (a) Solution |zl=k= 12 =# or ? Next 2Relw) = wiv + rk Riz, we get 2 Re (w) +k Using < ae +4 eammeced 2 (b) 1 @ v2-1 Solution Let |z + Now, the identity, gives us +3 let zl > a’ $2+3aa'-3a-250 = (@-2 (a+ 1 s0-a-2<20 => as2 Example 5 If ic’ + 2° 2 +/=0, then | zl equals @4 (b) 3 @2 @t Ans, (6) Solution We can write the given equation as Complex Numbers 1.15 2@-)+i@-)=0 @+)G-)20 = Za-izei [2h =l-i and lel = li IP= 1 and kel = 1 l= Qugug ©, then value of n Example 6 If @= cos = +i si n Leora... 0" is (@) l+i (b) 1 + i tan (win) (©) 1 + i cot (w#2n) (a) none of these. Ans, (0) Solution We have Salt a+ @ts ar! Example 7 For any complex number z, the minimum value of (zl +1z—2il is @o 2 Ans. (0) (b) 1 (d) none of these 1.16 Course in Mathematics for IT-JEE Solution We have, for 2 eC Iil = lz + Qi DNS lel +124 ~ zl > 2s lz + lz -2i ‘Thus, minimum value of Iz! +z —2#/ is 2 and it for any z lying on the segment joining z = 0 and Example 8 For all complex numbers z), 2; satisfying el = 12 and [z, ~ 3 ~ 4ii = 5, the minimum value of attained lz zhi (a) 0 (b) 2 ©) 7 (d) 17 Ans. (0) Solution | = 12 implies that z, lies on the circle with centre C, at the origin and radius 12 whereas ley — 3 —4i= 5 implies z, lies on the circle with centre at C, (3 + 4i) and radius 5, See Fig. 1.42. The quantity Jz, — 2) will attain the least value 2 when z, and z, lie ‘on the line joining C, and C, i.e. on the line z = (3 + 4i)r Fig. 1.42 In fact, when we take z; = 6 + 8i and z, 2 aed we 12 obtain lz, pg BFAD (6+ 81 2 |= 2 +402. Example9 If ies on the circle le 1!= 1, then == equals (@) 0 (b) 2 = (©) -1 (d) none of these Ans. (a) Solution Note that lz — 1 = 1 represents a circle with the segment joining z = 0 and z = 2 + Oi as a diameter. See Fig. 1.43. If z lies on the circle, then car] is purely i imaginary. y Fig. 1.43 Example 10 If 2), 2» t3 are complex numbers such that Ae = leg = ley = Ly then Izy +z) + 23h is (@) equal 10 1 (b) less than 1 (©) greater than 3 (4) equal 10 3 ‘Ans. (a) Solution Since ley! = | 1, we get 4 1tQ+a Example 11 If @ is an imaginary cube root of unity, then value of the expression 2A +e (1+ @) 432+ 24 o) tant nt int @ (n+ a) @ trast en ny Fane? =n 1 a (© gnn+d?—n (@) none of these Ans, (a) Solution rh term of the given expression is CHD G+ Oe ayers) © value of the given expression is Deve ol 4 (ntl +n Example 12 The greatest and the least value of ley + cyl if 2, = 24 + 7i and [zp = 6 are respectively (@) 31, 19 (b) 25, 19 (©) 31, 25 (@) none of these Ans. (a) Solution Note that zl = 6 represents a circle. As 6, ley #zql = 12-6 24-7) represent distance between a point on the circle lel = 6 and. the point C (- 24-7). le, + 2,1 will be greatest and least at points B and A which are the end points of the diameter of the circle through C. As OC = 25, CA = OC ~ OA= 25 ~ 6 = 19 and CB = OC + 0B=25+6=31 cp-24-70, Fig. 1.44 Alternative Solution lel = 6 => z, = 6" where OER. lz, +2)!" = 124 + 71 +6 (cos O+ isin Ol” = (24+ 6 cos OF + (7 +6 sin OF = 576 + 36 cos? 0+ 288 cos 6+ 49 +36 sin? 0+ 84 sin @ = 625 + 36 + 12 (24 cos O+7 sin = 661 + 12 (25) sin (8+ a {put 7 = cos cand 24 = rsin a} = 661 +300 sin (4+ a Thus, greatest possible value of Izy + 221° is 661 + 300 = 961 and the least possible value of Iz, + za!” is 361. 2+ greatest and least possible values of Iz, + zy! are 31 and 19. Example 13 If z, and z, are two complex numbers such ace thea att that (a) ky ke R tb) = ihe, kER @xy=n (d) none of these Ans. (b) Solution Clearly both z, and z, cannot be 0, Suppose 2 #0. We can write or as Hy] | 41 a | |e This shows that z;/z, lies on the perpendicular bisector of the segment joining A (1 + i) and B(L + i0) [See Theory]. ‘Thus, 2:/z2 lies on the imaginary axis. 2/2) = ia for some a € R. = aft, = Via = ~ ila = = ike, for some k €R Complex Numbers 1.17 Alternative Solution 00), A(z), BCG, + %) and e(@,) are the vertices of a parallelogram. Its diagonals OB(z, + z;) and CA(z; ~ z:) will be equal if and only if OABC imaginary. Example 14 For complex numbers: 2, and, satisfying 1-13 2 is a rectangle, that is, z/z, is purely yn% are the vertices of a triangle which is (a) of area V3 (b) right-angled and isosceles (©) equilateral (d) obtuse-angled and isosceles Ans. (¢) Solution We have uz Thus, tz, ~ zl = lay zg! = ley — 1 Hence, z;, z, and z; are the vertices of an equilateral triangle, Example 15 If @ > 0, and the equation lz ~ al + lc ~ 2al = 3 represents an ellipse, then a lies in (@) 0,3) &) (2, VB) © 03) @ a, V3) Ans. (c) Solution The equation |z -a°{ + (z~2al = 3 will represent an ellipse if la - 2al < 3 [See Theory) @ 30] Example 16 If @ is real and z,, z> are connected by 23423 + 22, x, cos O= 0, then triangle with vertices 0, z, and zis (a) equilateral (b) right angled (©) isosceles (d) none of these. 1.18 Course in Mathematics for IMT-JEB, Ans. (c) Solution 2) +23 +2; = (slg) ee z:c08 0=0 = (i sos0] = = (1 ~cos? ) =—sin? @ 2 > AL =-cos 6 isin a > | = (cos) +sin? 8 = 1 a > Igl=iz) => tz,-Ol=z)-01 ‘Thus, triangle with vertices 0, z, and z, are vertices of an isosceles triangle. Example 17. 1f| <—4| = 2, then the greatest value of lztis (a) 1+ v2 (b) 2+ V2 (©) V3 +1 @) V5 +1 Ans. (@) Solution We have |z! =| 2 =~ ls 24 = InP <2izt+4 => (zl- pss > ln-1< V5 = iis V5 41 Be 2 ‘Therefore, the greatest value of [zi is V5 + 1 which is attained when z= V5 +1 Example 18 Reflection of the line @z+a real axis is 0 in the ) 2 == a2 (@) none of these (a) @2 + az=0 ©) @+aye+z=0 Ans. (a) Solution Let.a= a+ iPand z= x + iy, then @z+az=0 becomes ax + By=0 oF {ts reflection in the x-axis is a ys gt or ax-fy=0 * (PE or az+az=0 Example 19. Ifa, f, x Sare four complex numbers such vr, 1 _ ahi that 5 is eal and aS By # 0, then z = 57, te R represents a (a) circle (b) parabola (c) ellipse (d) straight line Ans. (d) Solution Note that 540. Let & =k, where ke R. Two cases arise: Case 1. k=0. This shows that z lies on the line passing through Sand parallel (0 the complex number a6. Case 2.k #0. In this case y# 0 and ath = £(4)-44) ko+ ~ kok ar)” ilk ee ~ Hes) sr) ‘This is a straight line passing through a@/yand A/6. Note that the condition ad 72 0 is necessary otherwise. asp _Balprt yr SO yld+t 8° and we merely get a single point in the Argand plane. Example 20 If n> 1, the roots of (2 + 1)"= z" lie ona (a) circle (b) ellipse (c) hyperbola (d) straight line. Ans. (d) Solution Let be complex number satisfying the equation (z+ I= 2 then Met DMatZt > bce tzi" => bee =let This shows that z lies on the perpendicular bisector of the segment joining A(~ | +0/) and B(0 +40). That is, zlies on the line z= — 1/2. TIP us unnecessary to find roots of (z+ 1)" = 2" Example 21 If z, and z, are two non-zero complex numbers such that Izy + zl = ley + Izy, then arg (<;) ~ arg (a) ~ 2b) Ans. (d) -m2@) ma 0 Solution Let 2, = r\(cos 6 + i sin 6) and 2) = ry(cos 8 + i sin ) where r= Keil, ry = 6, = arg (z;) and 6 = arg (z,). We have a, +f =r} +7 + nr, cos (8, - 8) (7, + 1)? + Uryry {cos (8, — 8) — 1) Now, le, +2; = cos (8-8) Alternatively, © 2/2) is a non-negative real number e ‘arg(z;) = arg(e;) Example 22 If A(z,), Bt, triangle such that iz! = leh ABC is =, =0 =O lg 4zgle ly! + lel ) and C(z;) are the vertices of a ley! > 0, then orthocentre of @ atta Wb) 212 (©) 0 (d) none of these Ans, (a) Solution As \z\| = lz)! = lzyl, circumcentre of AABC is 250. Also, centroid of AABC is zg = ; 142,42) If zy denotes the onthocentre of AABC, then 1 3 (Gy + 223) = i @ taroet (ey + 20) = mMEN+A+E Example 23 If lz 25il < 15, then Imaximum arg (z) - minimum arg (2)! equals (@) 2 cos" (3/5) (b) 2 cos"! (4/5) (©) 2 + cos! (3/5) Gd) sin (3/5)—cos™ (3/5) ‘Ans. (b) Solution Af \z - 2Sil < 15, then z lies either in the interior and or on the boundary of the circle with centre at C (25i) and radius equal to 15, From the Fig. 1.45, it is clear that argument is least for point A and argument is seme for point B. Os _ 20 i a From right AOAC, cos & a= "oe = W2-8 — = cos" (4/5) Now, for lz - 25i $15 Complex Numbers 1.19 Fig. 1.48 maximum (arg z) ~ minimum (arg 2) = larg (B) ~ arg (A) = ZBOA = cf = 200s" (4/5) Example 24 If z, + 2, +25 = 0 and lal = lel = be then zp +23 +38 equals (a) - b)- © ° @3 Ans. (©) Solution Om (6, ata? | + ay? + ay + Alene + 23% + 2122) toad agg tl a 2 & ity (+H +B) = 0 But 2925 #230) + 2122 = 212285 ( Thus, 27427 +27=0 Example 25 If lq! = ley = layt= 1 and z, +2) +2 =0, then area of the triangle whose vertices are 2}, 2» 25 is (a) 3434 ) 34 @1 2 Ans. (@) Solution tz, + 2xF + lg, ~24? = AUlz,!? + le) = bath -o s+) = y= nP=3 > ky-gle Similarly, te, - 4) sky - a = V3 Thus, & whose veniegs ae yz; and zis equilateral and its side is of length V3. ‘Therefore, area of triangle - Seay 8 1.20 Course in Mathematics for ITT-JEE. Example 26 Shaded region is given by (@) e+ 2126.0 arg @ sE (by I24 21 26,0 Sarg (2) $F © e+ 2186,05 arg (@ s> (a) none of these. cas 35) aa) Fig. 1.48 Ans. (b) Solution Note that AB = 6 and 1+ W3i=- 243 + WI =-2+6 18; 2 (om E sisin 5) == 246 [008s 3 ZBAC = x13 ‘Thus, shaded region is given by le+2126 and OS anges n/3 Example 27 W-mearg(@)<— 5, then arg 2 arg (- 2) is @x Ans. (a) Solution Let arg (2)= @, then arg Z =~ @and arg (~: ~ m~ @ See Fig. 1.47 Thus, arg (Z)~ arg (-2) =-0-(n-O=% x g ®r OF W-> Example 28 If k >I, z)l 0,lzl=Iwl=k,anda@= —=—"_, Reo then Re (@ equals (a) 0 (b) k/2 © k () none of these Ans. (a) Solution But a@ = = Re(ay = 0 Example 30 Roots of the equations are + I= @- 8 a a oo 1an(2).200n (28) seal? 0 soe (2) 200 (28) (6 tom(2} 210m (24) (a) none of these Ans. (b) Solution For z# 1, (1) can be written as [For k = 0, we get 4) If we denote this value of z by z,, then oe ena tg tat > Gas ~ a _ =- tou (E) k= 2-112 2 Example 31 If 2 ¢ © ~ R then *2*2 eR, is tee equivalent to say f izi< 13 (by tzl=t @)tzi=2 @lzl>3 Solution As ° ° [vz# Zz) Example 32 Let z,, 2». 25 be three distinct complex numbers lying on a circle with centre at the origin such that 2, + 2323. 2p + 2,2) and zy + 22) are real numbers, then 123% equals @=1 (b) 0 © (d) none of these Ans. (¢) Solution As 2), 23, 2 lie on a circle with centre at the origin, Complex Numbers 1.21 lz! = legl = Izy! = r{say). As atone Ry toys Rt _ Pen +e) aay 2 : = . Stee a) 4h AaAtry Sinitaty, 2 = BBR ® Aas BRtE Pr +h % and = Btak @) From (1), (2) and (3), we get Plates | otny | ytny ae 4ytey gytry yn trey = 1453 7G +85) 2% ty -(% +4r'%) 2 aXy-D Lgl (j-“ky-P) aor “ Hence, 4Byel Example 33 If the points z, ~ iz and | are collinear then lies on (a) a straight line (bya circle (c) an ellipse (6) a pair of straight lines. Ans, (b) Solution As z, ~ iz and | are collinear 22) z 2] o=|- & t=cale -2 i rid roid (taking -/ common from Rs} 1.22 Course in Mathematics for ITT-JEE 1 = ij=0 1 using C, + C, ~ Cy and C) > C, ~ C) = @-DE+)+G-NE-)=0 => a-i+k-t+-H-7+ = 2-1 -iz- (1 + DF =O = = ‘Thus, z lies on a circle. Example 34 If |z il = 1 and arg (z) = @ where Be (0, 4/2), then cot O~ 2 z equals (a) i b)-i (oe iti Ans. (©) Solution |2—1= | represents acircle with centre at and radius 1. As 0 < @< 2/2, ties on the semi-circle in the first quadrant. See Fig. 1.48. As ZOPA=n/2 and ZPOA = mI2 ~ 8, we get P cos (-0) = 5 2°)" OA la = sin 0= > Also, since ZPOA = x/2 ~ 2S) om(] @ [sin @+ i cos A] = 1 + i cot @ cot @=9 cot O- ALi Pe Ki oO Fig. 1.48 Example 35. If! 122, +32, +4241 + 642,29 equals (a) 24 Ans. (a) Solution Bilesl=4 and 4, then absolute value of Bza25 +2723¢) (b) 48) 72) 96 182905 + 27,2) + 62,20 8 27, 6 & lei lzal Lzyl 8 (24) (4) = 96 4 Example 36 and +2445 J2+7, then the complex number zy 2 + 25 2+ %) Za is (a) purely imaginary (b) purely real (¢) positive real number (d) none of these Ans. (a) Solution — \V2 tf tytn eae = B= leP + leg? + leg? +2 Relais + 252) +2122) = Rete + 28) +22) = 0 Example 37 fa, b,c are distinct integers and vs 1 is cube root of unity then minimum value of x= lat bos col + la + ba + cal (a) 6V2 (b) 3 ©) 42 (d) 2 Ans. (a) Solution — Letz=u+bo+cat Then a + bors coh) (a + bee + cao) P+ BP + ~ he ~ ca ~ ab 4 2 3 5 Wb-eP +a) + @- bY} il + (Zi = 2b = 2 Kb- 0) + (e-a) + (ab) As a,b, © are distinet integers, minimum value of (b-cF +(e -ay +a by is Pe P+? 26. ‘Therefore minimum value of xis 6/2. Example 38° Letr,>Oand 2, = 7; (cos a% + isin aj) for k= 1, 2,3 be such that ot Stat 42 4 Let A, be the point in the complex plane given by w, = cos 2a, + isin ery a 2, 3. The origin, O is the (a) incentre of AA\A,Ay (b) orthocentre of AA\A,A; (©) citcumeentre of AA\AyAy (d) centroid of A.A\A,Ay Ans. (d) =0 fork Solution zis purely real. Example 40 2, and z, lie on a circle with centre at the origin. The point of intersection z, of the tangents at z; and zis given by 24% les as @ 7&+%) (o) n+y ‘Complex Numbers 1,23 fd eis) « Ans, (b) Solution Refer Fig. 1.49. As AOAC is a right triangle with right angle at A, 12,1? + Iz3 2)? =lz,!" +n => layP ~ 252, - Z423=0 = wo ‘Similarly, 2%, -%- % 20 Q) ‘Subtracting (2) from (1) we get = = Fig. 1.49 Example 41 Let z= cos 0+ isin @ Then value of us Lin") | at O= 3° is fa) - 1 2sin3° Ans. (©) Solution Emcor mel [SS 1.24 Course in Mathematics for IIT-JEE = =I ie gl! 08300 — isin30 a] a sin 2sin 3° Example 42. Let z= + iy be a complex numbers where x and y are integers. Then area of the rectangle whose vertices are roots of is (w) 80 (b) 48 ©) 56 (@) 9% Ans. (a) Solution = = 2 + Flor DF + & + HY) = 738 > (2 + FQ? ~ 2y") = 738 f= yt = 369 <2 x 2 369 = Wed yess yYoyss4 Thus, roots of 22°42 738 are + 5 44. ‘Area of the rectangle ABCD whose vertices are # 5 + 4/is (105 (8) = 80 » seu | sew Br A } —| «sa i Fig. 130 Example 43. A man walks a distance of 3 units from the origin towards the north-east (N 45° E) direction, From there he walks a distance of 4 units towards the north- ‘west (N 45° W) direction to teach a point P. Then position of P in the argand plane is tw) 30 + 4 () 4 + 30% Ans, (3) Solution Suppose the man reaches A afier walking 3 units in N 45° E and Bafier walking a distance of 4 units in N 45°W. Position of A in the Argand diagram is 3e™*. Let position of B be =. Since ZOAB = 702, we have (b) (3 ~ 4e'** (d) B+ 4ie™ se . 73" n3git!4 => aa nat = er = = Fig. 1.51 Example 44 A particle P starts from the point z= 1+2i, where i= J=1. It moves first horizontally away from the origin by 5 units and then vertically away from the origin by 3 units to reach a point z,. From z, the particle moves V2 units in the direction of j +j and then it moves through an angle #2 in the anticlockwise direction on a circle with centre at origin, to reach point Zp. The point z, is given by (@ 6+ Ti (b) — 7+ 61 ©) 746i (@) -6 +7 Ans, (d) Solution Affix of Bis (1 +2i) +5 = 6+ 2i Affix of Cis (6 + 2i) +31 = 6+ Si I+i Affix of Dis (6 + 5i) + V2 ( Zz) Affix of Eis (7 + 6fje™ = (7 + 6i)i Example 45 Let a, b, ¢ be three points lying on the circle lzl = 1 and suppose a (0, 7/2) be such that a+ bcos & +e sin @=0, then @) b=tic (b) 2a? + B+ €’ (a+b +c=0 (A) none of these Ans. (a) Solution | =\-al*=|b cosa +e sin al? = Wl? cos? @ + lel? sin’ @ + (62 +c) cos asin @ (+5) cb dag ra14 FF (sin 2@ ae a2 Gin 2@ =0 9 +2 =0 => beric SECTION It MULTIPLE CORRECT CHOICE TYPE w rrampless 16s § (sin? ion?) te @) S+5=0 (b) SF I = © WS=zatd — @ s-5 =0 2 Ans. (2), (b), (©) Put = cos 2% + isin Solution 7 so that for 1S k < 10 sin 2 — jog 24 1 i ( 2 = -ifcos—* + isin = -iat [De Moivre’s theorem} thus, x 0 i 1 $= -iFot = mina) 2 € 1-o 0 But a! = cos 2+ isin2n=1+i0=1 S=i ‘Complex Numbers 1.25 > 0,85 =1 1 d IS = Vt an¢ vs Bae Example 47 Iflz!=le=lzsl=1 and Seley — ay + ley ZF + ley 2 Let = minimum value of S and maximum value of $ 8245 2) 25 Vary on Lzl= 1, then (@) a=0 () B=9 © B=3 @ a=2 Ans. (a), (b) Solution Note that $2 and $ = 0 for 2; = 2)= 2 Thus, a=0 Since la - bP = lal? + lb? - Gb-ab, we get Nag = 2g + Ney — ay? + zy - 23? = 6 ~ (2985 + Zoey + 92) + Faz + 222 + Ziey) =6-2@+M-HGtW-Aagt+a MW Also OSiy + y+ ar SlyP + kzP + ky +2, (+) tity nate Si G+tHtheryt+AGq+y23 ‘Thus, from (1) and (2), we get fa - af + key - P+ ky - als 9 The value 9 is attained when 1 y=@ where «#1 is a cube root of unity. Example 48 Suppose z, +2) +2, +2,=0 and y= leg = leg = keg = 1. Af 2, 23, 25 & are the vertices of a ‘quadrilateral, then the quadrilateral must be a naa (@) parallelogram (b) rhombus (c) rectangle (@) square Ans. (a), (e) Solurion +++ y= a) = 4+R+h+y=0 tot ot = Stott 424 4 3 ake tana t tte + n= 0 2 1.26 Course in Mathematics for 1T-JEE Also, leyzgzyzl= 1 ‘The equation whose roots are 2), 23. 2 and zy is @- a @-WE-wWE-wW=0 @) 4 or ae 0 fuse (1) and (2)| where b = (2) + 2) (ay + 2) + 242) + Et and doy Clearly, if ais a root of (4), then ~ @is a root of (4). Thus, root of (3) are of the form + a + B. If 2) + 2)= Othen zy +z, =0 ‘Zp Zyp Zy are vertices of a parallelogram. (Similarly for other pairs) Also, tay = zF = tejP + 1 2-26 =» ly, - Similarly Iz, ~ gl = 2 ‘Thus, 2). 2». 25 Zar vertices of a rectangle. Example 49 Let P(x) and Q(x) be two polynomials. Suppose that f(x) = Pir) + x O(c) is divisible by x? + +1, then (a) P(x) is divisible by (x — 1) but Qlx) is not divisible by x — 1 (&) Q(2) is divisible by (x — 1) but PLx) is not divisible by x ~ 1 (©) Both P(x) and Q(a) are divisible by x ~ 1 (d)_ f(x) is divisible by x - 1 Ans. (0), (@- Solution We have x? +x +1 = (x~ @) (x~ 0"). Since f(x) is divisible by x°+ x + |, f(@) = 0,/(") =0. Plo) + wo") = 0 => PU)+@Q)=0 1) and PCF) + af Q(ah) = 0 = P(1) + @ QC) = 02) Solving (1) and (2) we obtain PUly= 0 and QC) = 0. Both P(x) and Q(x) are divisible by x ~ 1 > PO?) and Q(x") are divisible by x° - 1 and hence by x - 1 Since f(x) = Par) + x QO), we get fix) is di byr-1. Example 50 If z that order, then @ atyzaty (b) Iz, ~ 21 (c) tz, - 2, (@)_ (2 ~ zz ~ 2) is purely imaginary. Ans. (a), (0), (6), (A) ible 3+ Z4 are the vertices of a square in | Example 52 | unity, then Solution Let the four points represented by ¢y, £3.25 and 24be A, B, Cand D respectively. Since ABCD is a square, the mid point of AC = the mid point of BD = $e $y) OF tae ty Also, = lyenlely z= 241 Since diagonals ofthe square ABCD are equal AC = BD ot “I al Also, since AC L BD, (2, ~ imaginary Example 51 ‘numbers such that 12) + If, and z, are two non-zero complex 2\F? + 12p0, then (a) 2,2y is purely imaginary (b) gy/z2 is purely imaginary 2+ F220 are the vertices of a right triangle. Ans, (a), (b), (c), (d) Solution + laP = + 2) sla P + 1gP = at beg? = layP + leg? = - > F is purely imaginary. Since {z,1? is real, we get purely imaginary purely imaginary, arg(<;/z,) = 1/2 | equats (a) 2-1 (b) "C+", + +a (©) Peg tte) +o + GN = {d) none of these. Ans. (a), (b), (©) Solution We have -DE-@- a)... = (-OG-@) 2. we get 2-1 (2- a (2- a")... 2- a!) = > Example 53 Ifa,b, care real numbers and zisa complex number such that a° + 6° +c? = Land b+ ic=(1+a)z, then Liz equals @ oie (b) arid ia Ite -¢ lea ®) ae © ore Ans (b). (6) btic b-ic Solution We have z= =z Ita Ita =a “(tay (+a) Lea We have Utiz _ Uti Wiz _ Leilet 2-22 Tniz “Isiz -ig—3)4e8 Itat+2e+l-a 2(atib) _ arib *20Fe) ~ ° - atib a-ib Tee “anib 1 @ © UF ea=ib) ~ From (1) and (2) we get (b) and (c) are true. Example 54 Let a be a positive real number and let then (@ mintel = H( Ja? +4 ~ |. we get : lies on a circle. [See section 1.11 in Theory] Example 56 If 6,, 8, and 8, are four real numbers, then any root of the equation sin 6, <' + sin @, 2° +sin 0,2 +5in @,=3 a lying inside the unit circle Iz! = 1, satisfies the inequality (ay 1128 (b) Iz} > 28 (©) I< 12 (@) ial > 12 Ans. (b).(d) Solution Uf z satisfies (1), then z # 0 for otherwise, sin 8 = 3. Also 3=(3islsin @ 2’ + sin @ c+ sin & z+ sin Bl S isin Oi iz!) + tin Ol izi? + Isin Bl bal + isin Al Sid + 1P ell +1 tsin 1S 1] Statist te Hal + alts .. 29 ble < UB 29 lel > 28 Thus, z! > 2/3 and hence tl > U2 Example 57 If @ is a root of ° +2'+2+3=0, then tw) l@i2t (by lac (©) @ lies on of outside the unit circle Izl = 1 (A) ar fies inside the unit circle Iz1 = 1 Ans, (a). Solution Suppose |at< 1. We have [sla +a’ sai slaitiat Haicl+1+t 2363 A contradiction Thus, lait => lies cither on the of outside the unit circle | Example 58 If z #0 is a complex number, then z, iz, ~ 2and ~ ig are the vertices of a (a) square (©) rhombus Ans. (a), (b), (¢) Solution Suppose z, and ~ iz are represented by A, B, Cand D in the complex plane. We have in-poin of ACs + (2+ (2) =O, and (b) rectangle (d) none of these mid-point of BD fle+eiy=0 ‘Thus, the diagonals bisect each other. Also, ACzl- 2-21 = 2izl and BD =~ iz ~ iz = 21~ ii tzl s AC= BD Finally, AB = lic - zl = 1/- Wizt = V2 Izt and BC = V2 tal. Hence, ABCD is a square. Example $9 The equation | tel == ielat-1-illel Iz~ 2) =k (where isa real number) will represent a circle if. (@) kz, 2! 1 (b) k Ste) - 2h () none of these (©) ke Ans. (c) Solution \z— 2," +1z- 2! =k implies IzP + Iz, ~ 2Re(zz,) + IeP + leg - 2Re(zz) = k => Yel - Relelz, + F)) =k - Mz P + lz?) This will represent a circle with centre at +(z, + 23) and radius 1 Jak =|z,—2:) provided k2 4 12; -z ‘Example 60 The locus of z satisfying the inequality logiasy |Z + 11> logs Iz - 11 (a) Re(z) <0 (b) Re(z) > 0 (©) Im(z) <0 (a) Imz) > 0 Ans. (a) Solution We have logs z+ 11> logs = Ize liciz-th [v 0<08<1) => (x4 1+ < e- 1)? + y [where z = x + iy] = 4r<0 or xO or RKO Example 61 For any two complex numbers z,,z, and a, beR, Laz, + beyl? + tz, -az,P = fa) (a? + 6) (P+ 12, P) (b) (a? - B) (lzyP = 12, P) ash (a) none of these. Ans. (a) Solution We have Lazy + bey? + bbz, - az } + |be,)? +|az,?? =2R (be, az;) PlayP + Biz. + 2abRI + Dz)! + alzy? - 2abR(z,22) [+ a and b are real numbers} P+ by (lz + 1eyP) Example 62 If lz? - I= 120 +1, then z lies on. (a) the real axis (b) the imaginary axis ©) y=x (@) circle Ans. (b) Solution 124 (-Dl=iZl+l-t) <= 2i(-1) is non negative real number <> zis purely imaginary Example 63 if @# | is acube root of unity and wand @* ‘satisfy the equation = lazy? + tbe, coe theth e? o atx b+x ctx dtx x then value of +! 44; avi +l ctl del @ (by 2 © 3 (0 Ans, (b) Solution We can write (1) as Exh + o(c+x) d+x) sata (b+ a (c+ d+) = Ext + (bc die’ + (be + cd + bdyx + bed) | Example 64 Complex Numbers 1.29 att arbres de + (ab + ae + ud + be + hd + cde + (abe + abd + acd + bed + abcd) at sarbece de +0r = (abe + abil + acd + bed) This is a fourth degree equation whose two roots are we and w*. Let a, Bbe the two other roots, then (a+ B (w+ 02) + aB+ oa =0 [sum of the products taken two at a time] (a+ B)(-1) + af+t=0 (l-amO-f=0 a=1 o B=1 = dwied = 0 ~bUu | satisfies the equation (|). If sin’ x + cos" v= A + # cos dx, then 5 1 a) A= 2 (e) B= + WAG Ba 5 3 =i id) Be {c) A 3 dB a Ans. (b). Solution We have A+B cosdr= + [te +e!) — (eo) 64 veewy $C NT 1 5 tet 5 = hc CE Ce") + "Cle 1 eet aot = = [6teM +) + 20) 22 3 106) 2 cos 44) + 20) = cosare 3S Thus A=S& and B= V8 Example 65. Letz), 2», 2, be complex numbers such that 1 and value of lz, + zy + 23! ean be @ 1 (by 2 (©) 3 @) 4 Ans. (a), (b) Solution We have z;) +2) + 5) = = = daa aap + 1.30 Course in Mathematics for IIT-JEE Lenten? apt ty) CG) + oy + ay Baty - OH) TD) za + 23) (ey + 2p + 25)? — Hees + 2521 + Mh = P= Balaaey + 221 + 212) + Eyes = where = z= + +% rota = bessafa(tetet bd = ye (321 + +H)-4) (leh = leh = tel = 1) = 229m Bez - 4] = ley zg lag! Biel? - 4b > lef - Ble? ~ 4 = 0 If kl > 2/3, we get keP - 3k? +420 = (el ~ 2) eP - kl -2)=0 = (lal = 27 (a + 1) =0 => td -2=00rlt=2 Ik < 23, we get LP + 3kP-4= 0 = (l= 1) UP + dtd +4) = 0 3 id-l=sO> k= Example 66 If sin a+ sin 8+ sin 7= cos a+ cos B+ cos ¥= 0, then (a) cos 2a + cos 28 + cos 2¥= 0 (b) sin 2a + sin 28+ sin 27= (©) cos (8 +7) + cos (7+ a) + cos (a+ A) =0 (d) sin (B+ y) + sin (y+ @ + sin (a+ f= 0 Ans. (a), (0), (€), (A) Solution Leta =cos a+ isin a, B= cos B+ isin Band c= cos 7+ isin then atb+c=0 a rat and ater = beteatab=0 @ From (1) and Q) @+Pec=0 @) From (2) cos (B+ 7) + cos (7+ @) + cos (a+ B) + i [sin (B+ + sin (y+ @) + sin (a+ B= 0 From this (c) and (d) follow. Similarly, (a), (b) follow from (3). Example 67 cos 7= 0 then (a) cos 4a cos 48 + cos 4y= 0 (b) sin 4+ sin 48+ sin 4y= 0 (©) cos 3a¢+ cos 38+ cos 37= 3 cos (a+ B+ (@) sin 3er+ sin 38 + sin 3y= 3 sin (a+ B+ Ans. (a), (0), (©), (@) Solution (a) (b) follows from (a) and (b) of Example 61. Using notation of Example 61, @ + bi +c) ~3abe =(a+b+0) (a+b? +2 be -ca~ab) =0 From this identity both (c) and (d) follow. Example 68 If «# | isa cube root of unity, then sum of the series S=14+20+30'+...+3n a"! (ne Nis 3n If sin a+ sin B+ sin y= cos @+ cos B+ — F.. @ 5 (b) 3n(w? - 1) ©o (@) none of these. ‘Ans. (a), (b) Solution 1+ 2x 4304... + 3ne™! = tere ete] 45 (=) 0-Gnt x) +) (=a? Put x= ay we get 1+ 2@+3e +... +3n a! -@) (1-3n-1) +(@-@) (-ay (o? mie ye -) Example 69 Suppose A(z,), B(z,) and C(z;) are vertices ‘of a triangle lying on the unit circle tz! = 1. AD is altitude of the AABC meeting the unit circle in E. (a) orthocentre of AABC is z, + z +25 (b) affix of E is 2322, (©) ifz;?= 2925 and z,"= 252), then AABC is equilateral. (@) if zy +2 =0, then AABC is a right angled. Ans. (a), (6), (©), Solution Centroid of ABC is eg = ; +2 + oh ccircumeentre of ABC is zs = 0. AS zg divides the join of sand zy (orthocentre) in the ratio 1: 2, we get = As E lies on the circle l affix of £. Thus, amy t = Next, = 242) Thus, 2)? + zy > AARC is equilateral Lastly, if 2) + 23=0, z= 2), that is, orthocentre of ABC is at A => AABC is right angled at A. | having one end point as z. Thus, ~ iw = #0 = | Therefore, e+ 21 =2 => ld = 1. Complex Numbers 1,31 Example 70 Let z and w be two complex numbers such that [ch 1, tw € 1, ke + fwl = lei then z can be @ 1 (b) = 1 @i (a) =i Ans. (a), (b) Solution Note that z, w,~ iw and iW lie inside or on the boundary of the disc Iz1 $ 1. Since lz + / wl= lz ~ 2 1= 2, iw, £1 are the other end points of the diameter we get Now, COMPREHENSION TYPE. Paragraph for Example Nos. 71 to 75. If 0%, yy oy Gy re FOOK nth roots of unity. then ke ke @%= cos +isin —= a n where OSkSn-1. Also MTS @)~ a). a) a) Example 71 Value of (1 + @%)(1 + a)... (+ a,..)is @ 3 ¢b) Gy" om @) teen Example 72 Value of (1 ~ aj)... (1~ @%.,) is @n (b) n=l © Cy" @o Example 73. Value of 2-tsin() sin ( (an (b) 1 fe) n-t (d) nl Example 74 Ife =5 in (1), then value of G+ vf ~2eeos +1}(:? +22 cos +1} is 1 wo) 2 1 do Example 75 Value of sin : cos . is 1.32 Course in Mathematics for IIT-JEE fa) 4b) 12 (©) 3 Ans. 71, (d), 72. (a), 73. (a), 74. (c), 75. (a) Solution 71. Put x =~1 in (1) so that CDT~ Ls Gl~ @) Cl -~ a). a) SCI + Cy = + a) + @,_)) (14a) (14a). (14a = 1+ C1 72. Note that @& = | and (1) can be written as x @ ua (- @) &- &).. &- @)) = sleet. tat Put x= 1, so that (1 = @%) (1 = @) ~ (1 = ay.) 73. We have for 1 emo) (2 74, For n=5, note that o Thus, (1) can be written as ee (x — 1) (x - aa - &) e- &) x - &) ==) [P- (a+ &) x41) tP - (a, + @ r+ 1) Putting x=~z, we get o+leeD +2002 41) 5 (2 +2054 :+1) @) Now use cos (42/5) = - cos (1/5) 75. In (3) put z= I, so that Paragraph for Example Nos. 76 to 80. If mis a natural number define polynomial P,(x) of degree nas follows: cos nB= P, (cos 8) For example, P,(x) = 2x7 ~ 1 and P,(x) = 4x° ~ 3x. Example 76 EP (0)+ Ps (2) equals (@) Py.z @) © P,@ (b) Py, () + Py) (€) Ppa (2) - Py Example 77) (x+ yx -1)"+(r— yx" =1)" equals (a) P, @) (0) Past + Par (c) 2P, (x) (d) none of these Example 78 P,(x) equals (a) 36x° ~ 45x4 + 18 - 8 (b) 32x° ~ 48x* + 187 - 1 (©) 36x° — 48x4 + 18? — 5 (d) none of these Ans. 16, (6), 77. (6) 78. (b) Solution 76, We have P\(x) = x and cos (n+ 1) 0+ cos (nm ~ 1)O= 2 cos (n® cos O => Pag (2) + Pps (8) = 2x P(x) P= H [Py @) + Py I 2x 77, Putx=cos Oso that yx? -1 =isin @ Now, (r+ yx? -1)"+ (r= x? = 1)" = (cos @+ i sin BY" + (cos @ 2 cos (nd) = 2P, (cos 0 Thus, r+ yi? =1)" + yi? -1)" = 27, 78. 2elx)= (x4 Ye? = 1) + @- YP -1)% = 2a? + §C, AUP — 1) + 8C, 22 - 17 + Cy? = 19) in Oy" [using DeMoivre’s theorem] Pox) = 32x° — 48x + 18x? — 1 Paragraph for Example Nos. 79 to 83. For non-zero complex numbers z, and 2; define the dot product and the cross product of z, and 23 as follows: =x hy tM Ja = Re(HE) Cross product: 4! ey! sin 8 Fig. 1.54 Example 79 z, +z, + i(c, x 2)) equals @) 2% ) ¢ © Fz «@ Example 80 z; -z; = 0 is equivalent to say (a) zfs; is purely imaginary {b) 2)/2y is purely real (©) 4 = 00rz, =0 (d) none of these Example 81 OP coincides with OQ if and only if (@) a ) x20 © y+aty Xx (d) none of these Example 82 The area of parallelogram with sides OP | and OQ is | fa) Izy +23) (b) tz x zh © | + ley x 2h (©) tay + 23h ey X zh Example 83 The length of projection of OP on OQ is (a) bey + 2Vleyh (b) bey + zag (©) ky + el (@) kz, xa Ans, 79. (¢), 80, (a), 81. (b), 82. (b), 83. (b) Solution 79. 2) +2 + iz) Xz) = (x) a) +9 99) + HO HD) = (n — ) ( + = 0 Rez 2 ) 80. =0 Complex Numbers 1.33 is purely imaginary 81. OP coincides with OO = 6=0e4xn=0 82. Area of parallelogram with sides OP and OQ = 2 [area (AOPQ)] = 2 Uy! Lzal sin A = 2s) x ey! 83. The length of projection of OP and OQ fia Mzleos = lig cos 61 Paragraph for Example Nos. 84 to 86 Let a B. ybe three real numbers such that a + 6 + 7° — a+ip Iny Jz? equals y= Oandz Example 84 @ 7 y oT ¥r Example 85a equals @ Ans. 84 (¢), 85 (a), 86 (b) ror a-y ese Solution 84. {2° 85. From Example 84, Fe Leis 1.§M Course in Mathematics for IT-JEE z+E > a= + 2)0-9= Tye -* z+t 86, = > p= 20+IzF) Paragraph for Example Nos. 87 to 91 Let Ae Cand 2, zz) € Care such that Example 87 If 2 is a given purely imaginary number then all the triangles with vertices z,, 2, zy are (a) similar (b) acute angled (©) equilateral (d) obtuse angled Example 88 Value of 2 for which triangle with vertices 2. 23,2) i8 equilateral, is 5 (-va) ) $(+ 3) (©) V3 (a) ~V3i Example 89 If Ze R, then 2), 2 23 (a) lie on a circle (b) are vertices of a right triangle (©) lie on a straight tine (@) lie on a parabola Example 90 If 2 = it (6 R) and z, z are fixed, then locus of z, is (a) a straight line (b) an ellipse (©) a circle (@) & pair of straight tines Example 91 If A= e" (¢ R) and zp, z, are fixed, then locus of z, is (a) a straight line (b) an ellipse (©) a circle (4) a pair of straight lines Ans. 87. (a), 88. (b), 89. (6), 90. (©), 91. (@) Solution 87. Let A= ia where a ¢ R. Let zy. Z9. 23 and Wy, Wa, Wy be vertices of two triangles ABC and PQR respectively, such that 3=™ sia ow Q) = BAC = ZOPR = aid (3) From (2) and (3) we get ABC and APOR are similar. 8. 4-25 A - 2) 4 y=(l- ay + Ay => gylies on the line joining z, and 2. 90. As a is purely imaginary, angle between the 5m segment joining zz and z,z; is a right angle. = 2; lies on a circle with the segment joining zz as diameter. SAA) tet =1 2a = 2 is equidistant from z, and z5 => 2 lies the perpendicular bisector of the segment joining z, and 23. Paragraph for Example Nos. 92 to 95 Let A, B, C be three sets of complex numbers defined as an. jz: Im(z) 2 1) Bs{z:k-2-#=3) C= {z: Re(l — dz) = ¥2). Example 92 The number of elements in the set AOBNCis @ 0 (b) 1 © 2 @ = Example 93 Let z be any point in A.A B 4 C. Then, Ie 1a + lz— 5 ~ il? lies between (a) 25 and 29 (b) 30 and 34 (©) 35 and 39 (d) 40 and 44 Example 94 Let z be any point in A 0 BC and let w be any point satisfying iw —2—i1< 3. Then, t2l —lwi +3 lies between (a) ~ 6 and 3 (by ~ 3 and 9 (c) ~ 6 and 6 (@) ~ 3 and 6 Example 95 If zis any point ABC, then equals (a) a2 (b) xf (©) 2/3 (a) A/4 Ans. 92. (b), 93.(c), 94.¢b), 95. (a) Solution 92. A consists all point in the half plane Imz 2 1; B all points on the circte lz - (2 + i)! = 3 and C all the points on the line x + y = V2. These three curve intersect in just one point as shown in Fig. 1.51 y fh he aye 3 v2 * Fig. 155 93. Points A and B are end points of a diameter of le ~ (2 + il = 3. Affix of A is - 1 +i and that of BisS+i, k+l-P4ke-5 PA’ + PB = ABP where lies between 35 and 39. 94. lz} -twil siz wi Here z is fixed. Thus distance between z and w would be maximum for diametrically opposite points. 2 Wizl-twilsiz-wi<6 => -6~I ltar+a, > 44, (cosa +i sin a) +2, (Cos 2+ i sin 2a) +o +a, (cos nar + i sin na) = 0 {using DeMoivre’s Theorem] Equating real parts, we get 1 +a, cos. a+ a, cos 2a+ > lay 60s @ + a; cos 2a + «. -. + a, cos na = 0 +a, cos nal = 1. Example 97 If c#0 and 2 + cos 6+ isin @= 3z, then find the value of 2(< + 2) ~ 120? Ans. 3 Solution We have 9 Jer = (2 + c086)'+ sin? @= 5440088 (I) and (2) = 3= AE +2)-1ee Example 98 If <= (03, find the smallest value of positive integer n for which (2 + 177) = 2 Ans. 10. Solution Weave = 2 (A3~i) = —Lit+iV3) =ab where @ # 1 is a cube of unity. Also, msi ‘Thus, +P) = iw +0" =[t- oI = - Also, vet o™ 1.36 Course in Mathematics for IT-JEE ‘The given equation becomes -@=h a" = fo" *=-1 This is possible if m is of the type 4k + 2 and 2n ~ 2 is a multiple of 3. ‘That is, 2(4k + 2) —2 = 8k + 2 is multiple of 3. The least value of k for which this is possible is 2. Therefore, n = 10. Example 99 If 1, x,, x, x; are the roots of x*— 1 = 0 and @is a complex cube root of unity, find the value of Ans. 1 Solution We have a1 (e- 1) =x) (X-y) (F-4). (@' -x)(@ -x,)(@" -x,) (@=x,@—x,(@=%) 1 (@* =a" -x)(@* orl (@-I(o-x)(@~x, “ot 1 @y- Thus, - wat (@+)(@=1) © Example 100 If z, = (1+iv3) find the value of VB Imi Z 9. Ans. 1536 Solution ts = (1+iv3) (1-13) = (1+ v3) (0439 => 3 lm (25%) = 3(2%)=1536 Example 101 Leta,b,c,de Randa’ +b?=4,7 += 2 and a+ib=(c + id) J/x+iy, find the value of x7 + y*. Ans. 4 atib Solution Jatiy = ia _ (atiby => x+i= |g (a+ib?|_la+ = tee l= Cy iay| fetid o+P = vty = aya = eyed Example 102 Ifa, b€ find the number of complex numbers a + ib for which (a + ib)" = (a — ib)’. Solution Let z= a+-ib, the given equation can be written as => bP tz =» le? = tet » ld =0 or lal’ = = ld =O or kel =1 If id =0, then z =0 If lel = 1, we get Z = I/z and the equation becomes 1 > esl ‘This equation has 5 roots. ‘Thus, there are 6 complex numbers satisfying the given equation, Example 103 If the complex number z is such that iz ~ I'S 1 and lz - 21 = 1 find the maximum possible value ll*, Ans.3 Solution \z~ 1S 1 represent the interior and boundary of the circle with centre at I + Oi and radius 1 and lz ~21= 1 represent circle with centre at 2 + Oi and radius 1. Clearly the points z satisfying Iz - 11 $ 1 and tz ~ l= 1 ie on the are DAC. 2 OA Sil $ OC (= OD) As ZOCB = n/2, OC? = OB - BC’ = 4 Thus, Solution 251. 5 jest => ip - 25P = 25 ke - 1P = td? - 252- 25 Z + 625 = 25 fle? - => Dalz = 600 = Nei? = 25 = ld=5 Example 10S If 0 < arg z < n/4, find the least value of v2 le=al. Ans. Solution 0 S arg (2) $ x/4 represent the region of the complex plane lying in the first quadrant and bounded by the x-axis and the line The least value of lz — is the length of perpendicular L from (0, 1) to y = x, which i : ©, )toy=x, whichis 7 ‘Thus, least value of J2 lz ~=1 Fig. 1.57 SECTION V MATRIX MATCH TYPE Example 106 Number solutions of @) 2+ =0 (1 ) 2+ =0 @ 3 ©) 2+ 8% =0 o4 @ iz-2 =1 (8) infinite and Iz ~ l= 2 Ans. pars * @@OQ b @@O® @C@O® ®®2O® ao Complex Numbers 1.37 Solution @ 2=- =» Wd=id=0 or id=] i id= 0, then z=0 1 tdel, then 2=-leceti . +121 =0 has 3 solutions. (b) For 2 #0, 2 = > tk Each complex number of the form a(1 + i) where ae R satisfies the given equation. (© (21 = 1-821 = kP= Bela kl=0 or li=8 i kia B cd = 64 = = z= = 8, ~ Bw, ~ Sw? Thus, 2 + 8 0 has 4 solutions. (d) Circles tz ~ 21 = 1 and Iz ~ 1) = 2 touch each other atz=3, Thus, z— 211 and tc ~ Il = 2 has just one solution. Example 107 If a,b, care distinct integers, and a+ 1 is ‘a cube root of unity, then value of (a) lat boreal? (py i (b) a+bo+e@P = (q) 21 +la+ bal + cal? latbat+ co a+ bar +ca| (d) la + ba? + coal © (22 (s) 2 Ans. s part *M@OQ >1@®@@OO © @@OQ *@@OQ) Solution la + bw + ew" = (a + bw + en’) (a + biv + cH) = (a + bw + ew) (a + bw + cw) aa +B +c be-ca-ab = tbo +(e-a + @- 1.38 Course in Mathematies for IT-JEE are distinet integers the least value of +(e-aP + (a~ bP 22 la + bo? + cal? = la + ba? + cal? sla + bas cor? Tip By just noting that la + bos caFl = la + ba? +. cad we ‘can guess all the answers. Example 108 Centre of circle +i -4i = 20 (pli i|=4 (@) $3 + 07 - + de (-4-i -(1-F+7=0 nia () 142i Ans. Solution (a) The given equation represents a circle with the segment joining 2 and 4/ as diameter. Therefore, its centre is 1+ 2i (b) 4iz- IP tee P 2 -2-T+#) Z+r+i+l Sz-SE+3=0 his circle with centre at $/3 + OF (©) It represents a circle with centre att —i (d) Diameter of the circle is the segment joining = 3-44 and ~$ + 2i. Thus. its centre to - 4 — i, Example 109 Modulus of complex number whose reciprocal is (py 10 +8" a? +e =a @ fe roee? i lal ore Varoree . oy Se eae atibtic lallb +t Ans. pars *@@@Q i OXOTOTO) @@O®@| @@OO) atbtic ab +iac 1 y@+byP +e = it ager lalybF +c = 8° oe Solution @ 1 lb-el = tel * Jarve® fare ing tye NO tP a te? baa ab +c) = (a+ibya-ie) Va? +b fa? +e l= - Mt lallb +el 1 Ve +Geo) - l= fa +o+o? Example 110 Modulus of z= yee. +0088 —isind when (a) -#<0<0 po (b) @=0 @ tan( $8) (© 0<é arg (1 +2°)=3a/2 (b) 1-24 =1-cos da@—isin dar = 2 sin’ 2@- 2i cos 2esin 2a =2 sin 2@ [cos (2a— x/2) +i sin 2a 712)] = arg(1-2')=2a-an , (oy MEE eee g( EZ) isin 22 1-2* sin(2@) if-2<0<0 Complex Numbers 1.39 4 a+ a 2-1 ad o4l _ sina) cos (32/2) __sin(2a) © cos (322) ze Thi we ae (5 +1 Example 112. Match the equation on the left with the ccurve they represent on the right (@) e-31+le- A= 10 (p) circle 22-3 ) FS |-2 (q) hyperbola @e2+P=5 (®) straight line 2-6 . @ z4l. 3 (6) ellipse Ans. ros pa @@O® ®@OO ®@O® 4@@O® Solution For (a) and (d) see theory. (b) can be written as =| =i = z lies on the perpendicular bisector of the segment joining 3/2 and i (©) Letz=xtiy = Per-y 42a ‘Thus, > oe = ‘Thus, z lies on a hyperbola. Example 113 Value of (a) 4 a (ft — at) (po if a+ ip p20 is a root of z= 1 a () pet +2-2 @t z fez et 1.40 Course in Mathematics for IT JEE | ws aif (5) 10 Zs Zap Spe Zy Fopresent vertices of a parallelogram, pars *@@OG@) ‘WOOO * @@@OO) 1@@OO0) (a) Let z= a+ iff. then 2 Also, i2l* Now, 4a (fp ~ at) = daft ~ of) eh 1 > a & Paces 2] Ans. la id=l= Fade all iie-va)] +kled (@) 2. 2» Z5, & are venices of a parallelogram if and only if diagonals bisect each other. io Yc, x. then value of a Example 114 If (1 +.0) a) Cy Op + Cy = + Cy (pO W)C,- Ct Oy - = Gy = @ 2% 2-1) ©) Cy + Cy + + Coon (OF ed WC, Cyt + Cy (sy - 2 Ans. Pars *@@OQ > @@OQ © @@OO ‘@@O0 Solution Put asi we = Yew =a+9” ms 50 x xy” =2 {cos4 isin) 4 4 = 2-1 +08 Oy Cp + Cy = a + Cyqge - 2” ay and Cy = Cy Cg= = Gy 0 @) Also Cy + Cz + .. + Cig = 2” QB and Cy + Cyt + Cy = 2™ a) From (1) and (3) Cot Cyto + Ciqy = 2” (2-1) From (2) and (4) Cy + Cy + 2s + Coy = 2 Example 115 2, z»,<; are vertices of a triangle. Match the condition on the left with type of triangle on the right (a State (p) right angled = yl + x2) + Zyly - 0 ()obtuse angled \< 0 (isosceles and right - angled (s) equilateral *@@OO) + ®@OO Solution (a) See Theory (b) As s purely imaginary the triangle is right Bnh angled with right angle at z. (©) Itis an obtuse angled triangle. (@) AS ky ~ a= ley) and 3% imaginary, triangle is right angled and isosceles. ‘SECTION VI REASONING TYPE is purely Example 116 Statement-1: sents a straight line, Statement-2: Perpendicular bisector of a segment is a straight line. Ans. (a) Solution | + 424 = cis equidistant from A (2) ana (-3-21). ‘Thus, z lies on the perpendicular bisector of AB and hence is a straight line. Example 117 Statement-I: |z + i” + le — 31 represents a circle. Statement-2: The set of points in a plane such that sum Of the squares of their distances from two fixed points in a plane is a constant is an ellipse. 16 Ans, (c) Solution z+ iP +1¢-31 = 16 = 2 -(34+2-G-)E -6=0 which represents a circle. Example 118 Statement -1: a, 6, ¢ are three distinct real numbers and @% | is a cube root of unity, then a+bwrew® aw +b+ow Statement-2: If ¢ # 0, then Complex Numbers 1.41 Ans. (b) Solution We have lat barca =\aw + ber carl = lo(ac? +b + ca) sled lack + b+ coh = lack +b + cab [vlal=1) la+ha+co" = jaar? +b + co Also, since IE | = lel, we get LW 240. Example 119 Statement-1: a, 6, c are three non-zero real numbers such that a +b+e=Oand z,, zp. 25 are three ‘complex numbers such that az, + bz, + €z3 = 0, then 2), 25 and zy lie on a circle ‘Statement 2: If z), z; and z, are collinear then a1 2%) yh Ans. (d) Solution atb+e=0 = c=-(a+b) az, + by +0252 = az, + bz: ~ (a+ b)z=0 5 op = Mt bey a+b => 2 divides the segment joining z, and z, in the ratio bea => 2, Z) and 2, are collinear, For truth of statement-2, see theory, Example 120 Statement-I: If z € C, least value of lel+le~ I+ le~2tis ‘Statement-2: If >0, then lz ~z\1= Iz~ zy represents the perpendicular bisector of the segment joining z, and z). ‘Ans, (b) Solution We have yell+R-atle-U2k+Q-g+k-i =2¢iz-1 2+ least value of y is 2 which is attained at 2 For statement-2, see Theory. Example 121 Statement-1: Two nonzero complex numbers z, and z; lie on a straight line through origin if aand only if z,Z is res 142. Course in Mathematics for ITT-JEE Statement-2: Two non-zero complex numbers z, and z, always lie on a straight line passing through origin if and only if 2 is real, Ans. (a) Solution 2, will tie on a straight line through the origin if the origin © divides the join of z,, zy is some ‘ato, +h, A. for some ke R Vk keR = - kiP e R. eRe gneR Example 122. Statement-1: in the complex plane, then AC? + BD? + AD* + BC? > AB’ + CD? Statement-2: If then ‘A,B,C, D are four points are four complex numbers ly +m -y-aP20 Ans. (a) Solution Let and 2, represent the points A, B, C and D in the complex plane, then AC? + BD? + AD? + BC - AB? ~ CD* ~ 2ReleS + %) + Rely ZH B-m4-2P20. EXERCISE SINGLE CORRECT CHOICE TYPE 1 (b) k= 0 @ O, ay is & permutation of (d) none of these |. U3. Z5. 25 are three complex numbers, then key + oP + ley + af? + le, + 2:7 equal (a) 2z, +m + oF (b) kz + ay + BP (©) 2igy + 2y + 2yP - tay? - ey - tes (d) none of these Weyexy+iyy, 22%) + iy then % 2 q y,|eauals @ 21% - 2% () Bn 4 (© key ~ ky? (A) te,P? = ley = 23? ». If Re(a), Re(b) > 0, and x = la ~ bl—1a + bl, then @x<0 (b) x >0 (x21 @)O0 (x21 @O1 (b) Il = 1 @ O, then zz + az + bz + ¢ = O represents (@) a circle (b) @ parabola r>0 Complex Numbers 1.43 (©) a straight line (@) finite number of points in 19. 1f@ =b,ce R, and Ibi" 0} (b) (zs Inez) < 0} (©) (2: Imiz) > 0) (0) (z= Retz) = Im(=)) Mn 23and 1, ay ~Sish+1 4, , are nth roots of unity, then value ofS. aay, is fa) 0 ib) {c) -1 td) (-1" |. The quadrilateral formed by the points with affix 0,2, 2+ icandicis (a) parallelogram {b) rhombus (e) rectangle (d) square Iflg-2= 2-1, then Mz! -4Re(z) equals (0 ty 1 jl (d) none of these ‘Sum of the common roots of 2% +2! +1 =O and Dele edeth=Ois (a) 0 tb) = fe) I (a) 2 Af.z, and z; lie on the same side the line @z+ @z +b = 0, where a € C, a #0, b € R, then the ratio +h Gz, +42, +b (a) purely imaginary (b) a positive real number (c) a negative real number (d) none of these If z, and z, are the roots of the equation 2 + az +b , then prove that the origin, z; and z, form an ‘equilateral triangle if and only if @ a =m tb) b? = 3a (©) a+ 3b=0 () B+ 3a=0 Ifa > 0 and clcl + az +2a=0 then z must be (a) purely imaginary (b) a positive real number aa+ is 7. 38, 39. 41. a2, (c) a negative real number «0 3 . Mf ——— =a + ib, then (a—2y' + equals, 2+cosd+isind (a) 0 ib) 1 w= @ 2 Ifz=4+ V7, then value of 2’ - 427-92 +91 equals (a) 0 tb) 1 w-1 @2 The number of complex numbers satisfying the equation {z| = 2 and Ici = Iz His (@ 0 (1 ©2 (@) infinite ite= 7 > + then distance of point = from the me fine joining z, + 1 and z+ Lis (0 1 eo @ mal mei fy. 23¢25 are three complex numbers such that 42, = Taq + 3cy = 0, then z), 23. 23 are (a) vertices of a scalane triangle (b) vertices of a right triangle (©) points on a circle (d) collinear points ‘SECTION II MULTIPLE CORRECT CHOICE TYPE If a complex number z has modulus | and argu- ‘ment #73, then 2? + z (a) is purely imaginary (b) has modulus V3 (ce) lies on the imaginary axis (d) none of these If 2, =a + tb and z, = ¢ + id are two complex numbers such that 12)! = |zp! = 1 and Re (2, 0, then the pair of complex numbers, w, = and w, = b + id satisfy (a) Wwl=t (©) tw, Wy (b) beg = 1 (@) ReHw,) = 0 45. 41. 48. 49. (©) xy + L 22.005 (04+ 9) y (d) none of these If zy, z3 are two complex numbers satisfying the relation Poimts A, B and with affixes z,, z: and (I~ 2, + izy are the vertices of (a) an isosceles triangle (b) an equilateral triangle (©) a right triangle (4) an obtuse angled triangle If Iz, ~ zl = bay + Lzph, then (a) arg ( = : by a(S )- © (©) 4%, 2 $0 (d) 2), O, 2 lie on a straight line, ‘Af z, lies on [zI = I and z, lies on Izi = 2, then (a) 3S 12, - 22155 (b) 1S iz +a153 (©) 12, - 32125 @ ty -a121 If z satisfies the inequality Iz ~ (a) min (arg ()) = tan (3) ~2i) SL, then 4 (©) max (arg (2) = 5 (©) min (zt) = v5 -1 (d) max (iz) = V5 +1 If 0, z, and z, are vertices of an equilateral triangle, then (a) tel = leg! = ly (b) (+ 23 = 32 (e) larg (z,) ~ arg (ql = 23 (A) ey + 2yl = Mey + legl | Complex Numbers 145 50. Which of the following value(s) of &, the equation ‘ represents a circle? (a) 3 (by 2 os Wr 51, Letze Cand U4 elle Uh, then (a) minimum value of x is 2 minimum value is attained on at least three points minimum value of x is attained at infinite number of points maximum value of x is attained on the circle la = V2 52. If2,,z,and z, are vertices of an equilateral triangle whose orthocentre is, the origin, then @) 444520 (b) ley! (©) az + (4) Oz, is perpendicular to 2325. 53, If the point , is the reflection of a point z, through the line bz + bz =, where b #0, then bz, + bz, is (a) proportional to Ie, (b) equal to c (©) inversely proportional 10 & (@) none of these 54. Equation of line making equal intercepts on the axes and touching V2 i= (@rtyel @x-ysl $5. If'z). 2) € C ate such that (by © () then (@) Igh= 1 (b) teh = 1 (use GER Wyse*GeR 56, If z and zx are two complex numbers such that is purely imaginary is purely imaginary 1) is always real, then z ean fie on {a) the real axis (b) the imaginary axis (©) a circle (d) a parabola 1.46 (Course in Mathematics for ITT-JEE 58. If z; and z, are two complex numbers and ¢ > 0, then Izy ~ zx! 2 (Ie) key! + (1 - Ve) kz is. (a) true for all z, 23 € © (b) true if 2; = % () false for all z,, z) € C (d) teue if z/t5 $9. If z € C, the minimum value of lel + lz ~ St is attained at (@ 2=0 () 225 © z2=52 (d) for all z € (0, 5) 60, The curve represented by ze? __,, de® 2+0080-+isind (a) never meets the imaginary axis (b) meets the real axis in exactly two points (©) has maximum value of I:l as 3 (d) has minimum value of [zl as | 61. Which of the following satisfies lz + I= 2+ 2421 fa) lei (b) ~Z 6 © ; =i @ 0 62. If complex numbers z;, z. z) are the vertices of a triangle, then the complex number z which makes the triangle a parallelogram is given by @) 4 +n-% ) a ta-% © a+%-% @ytata 63. If zj.Z5y 005 lie on lel = rand Re (& ye } jattat then @ ¥ y=0 ) |Sz,| <0 j @ y+=o (@) mone of these py 64. Let S denote the set of complex numbers z such that logy, (log 42 (Iz? + 4lzl + 3)) <0, then S is contained in (a) @, 1) (b) {z | Re(z) > OF (©) [z1 Re(z)> 3) ) none of these 65. Ifze Cand log,,, 913 + 4il 0,b>0 102. 103. 104, 105. (©) log td + cos" tl () eS (d) log tz- 1 + sec" el (s) led 21 Centre of circle ® = (p) 543% (b) exseate2(ip} @ 144 teR " (©) passing through wo 3, -V3i, 3 (d) having join of 3 + Si and - 1 + 31 as diameter If @, Bare roots of 2 sin’ 6-2 sin 26+ 1 = 0 where 0 < @< a2 and nis an integer, then value of () 1 (a) (ay (p) 2" cot" @ (b) (a+ By" (q) cosec™ 8 Oa+fs £8) 2 cosee” B cos (n (@) lat + a" (3) 2 cosec" @ For z= x + iy, define e* = e“(cos y + i sin y). The set of complex numbers z such that (p) (2+ QnziIne 1) - i (@) (log 2+ iQn-V3)rlne I) () (Qn+ Dailne (8) (log 2+ 2n!In € 1) SECTION VI REASONING TYPE Circum-centre of AABC with vertices A(z,), B(z,) and €(25) is S(z9)- The altitude from vertex A to side BC meets the circumeircle at E(0,k# 1, then represents a circle Statement-1: If 27 — 2+ 1 =O and number, then n Jens A] Ver where [x] denotes the greatest integer < x Statement-2: If w # 1 is a cube root of unity, thea toa, JO-1 iff is nota multiple of 3 whe its : " 2 ik is multiple of 3 Statement-1: Two lines + az + by = 0 (where a, a, € C,a, a, #0 and is @ natural +i +b=0 = 0, 4,2 b, b, © R) are parallel if and only if is real ma Statement-2: Two lines az +a2+b=0, a2 + + by = 0 (where a, a, € C, a, a, # 0 and b, b, € R) are perpendicular if and only if * is 4 purely imaginary. ‘Statement-1: The minimum value of ali 1 1B = a4] 2 Statement-2: The maximum value of 2i LO 112, . Izy, . IF roots of the equation 2 Complex Numbers 1.49 Statement-1: Let m,n be (wo natural numbers such that m = a? + D? and n = c® + d° for some a, b,c, d € I, then there exist ¢, fe T such that mn eer Statement-2: If z,, 29 C, then lz, + z3!= lei + ley iff and only if z,29> 0. SECTION | SINGLE CORRECT CHOICE TYPE Wie =legh=teyl= 1 and 2, +220, then zy, 25, zy are the vertices of a triangle which is (a) right angled (b) isosceles (€) obtuse angled (d) equilateral z; are the vertices of an isosceles triangle with right angle at z, then value of (b) 1 td) If zy, 2p, 2) are the vertices of an isosceles triangle, then value of fe) -1 (- a) + G4) + | -aP is @ 0 (b) © -1 (a) 3 If roots of the equation =" + az + B=0(a, Be C) are real, then value of (Im By" + (Im @ (Im (a B)) is @o (b) 1 (c) -1 Wi + art B= are purely imaginary, then value of (im BY - (Re @ (Re (@B)) is (a, Be C) (@ 0 tb) I © -1 i . If roots of the equation z’ + az + B= 0 lie on tel = 1, then (a) 2 lm a@=1- IF (b) 2 iim a = pr - (c) lm @ (d) none of these If z, = 1 + fand z, = | ~éare the two vertices of an equilateral triangle then third vertex is given by @) 4G (b) 123 (©) 341 (d) none of these } Khare 0, lat 1. then al the circles passing through and (a) are concentric with the circle [el = 1 (b) co-axal 1.50 10. uM. 12, 13. 14, 15. 16. 17, 18, Course in Mathematics for ITT-JEE (©) are orthogonal to the circle leh (d) none of these }. If Re(z) > 0, then value of the | +z +... + z"lcan- not be less than yw t i (a) le ia (b) tel" + 7 (©) mist (4) niet +1 If zl = 1, then z can be written in the form @ aero 8 wem lax laix (© 2% eR) —@) none of these ms If? 4x41 =0, then value of (=+4)-(°+ x is (a) 100 (©) 50 If z, lies in Iz — 31 < 4, zy on lz - N+ iz + Wed and A = lz, ~ 27, then @) 0S AS 152 ) 0< As 152 @) OSASI72 @ OsA0 (d) Re(2) <0, Iem(z) >0 Wiel = 3, then a equals @c (b) () 32 d) 19, 21 22, 23, 24, Ifp, g, rare positive integers, then remainder when. 4 oly is divided by x7 +x+1 (@) 0 (1 x1 (x= Ifz lies on the curve le ~3~4i1= 3, then least value of Izlis (a) 2 (b) 3 © 8 (@) none of these ‘SECTION It MULTIPLE CORRECT CHOICE TYPE If the imaginary part of the complex number (2-1) (cos ai sin @) +(2= 1)" (cos a+ isin a) is zero, then (@) ial= 1 © arg @) (b) k= 1 @ (@) ag (z- =a bE + d=0 where Two lines d+ az + c=0, be a,be C-{0) ande,de R, are (a) parallel iff ab=ab (b) parallel iff is real (©) perpendicular iff ab +ba = 0 (@) perpendicular iff = is imaginary Let, 23 € C2) #2, (a) Equation of straight line through z, and z, is 0 z (b) Equation of circle with 2; x as diameter is De - 4-2 = ky - (©) Equation of any circle passing through z, and z, te -2)- ei -asame( i )eo any where 2 R. (@) none of these Let two given circles be Cy: Etta +e=0 and C,: F+he+bz+d=0 wherea,be C and cde R. (a) Radius of circle C, is Vial’ —¢ (b) C, and C; will touch externally iff la ~ ble Jlaf—c + yibP-d ©) C, and C, will intersect orthogonally iff ab + ab seed 26. 21. 28. (d) radical axis of C, and C, is Gd + a) E+e-d=0 Let C be the circle te = al = kle - 2 where z,2)€ C,zy#2y and k>O,k# 1. 74 (a) centre of C lies on the line re (b) radius of C is ae (©) mid point of the segment joining z, z; lies on the circle (d) none of these Let c > 0. For the ellipse ~ cil + lz + cll = de (a) eccentricity is 172 (b) one of the directrix is z = dei (©) minor axis is the real axis (d)_major axis is the imaginary axis ABC is an equilateral triangle whose circumeentre is at the origin. Let the point A be represented by -1+ fii. (a) length of side of AABC is V3 (b) centroid of ABC is -1 +i (©) orthocentre of AABC is 0 (d) in-radius of AABC is 1. If z, and z, are fixed complex numbers, the locus of zo that (a) a circle (b) a semicircle (c) a straight line (@) perpendicular bisector of segment joining, 3 and Zp, Which of the following represent a straight line through z, and z3? (a) 2=241(@%-ahteR (b tn aes ) ao Para oj Veo 4%1 "eR 30. Complex Numbers 1.51 Ifa #0, then we( } = | represents (a) a straight line (b) rectangular hyperbola (c) a hyperbola with eccentricity JF (@) part of circle with centre at a. ‘SECTION til COMPREHENSION TYPE Paragraph for Question Nos. 31 to 34 31. 32, 34. For zextinnye R, define se (os y+ isin y), sinh 2= 5 (ee) to. cosh 2= 5 (e+e) For z € C, conjugate of e* equals tb) e* @e (cosh 2)° + (sinh 2)? equals (@) 1 (b) +1 (d) none of these sinh z holds if z equals (a) Qn ~ 1x2 (b) 2ne (©) Qa + Dai (d) Qn- Iain ne L sinh z= 0 if z equals (a) lami (©) Qn - Dai (b) Qn - 1)xt (d) Qn = Imi, net Paragraph for Question Nos. 35 to 38 35, 36. 37. If z), z are go complex numbers representing points A and B, we define complex slope of line AB Complex slope of line a: + af +b=0 where ae C and be Ris a a « -“ If wand 1’ are complex slopes of two perpendicular lines, then (a) al =-1 (b) w= yf eM? ute =0 (a) ga = 1 If wand 4/ are complex stopes of two parallel tines then @ (by ails ©@ = 1.82 38, 8 4. 42. 43. Course in Mathematics for IfT-JEE @) wee (b) ml = -1 fe) a = @) ws Ifa line makes an angle @(¢ 2/2) with the positive direction of the x-axis, then its complex slope equals (a) eX? tb) (©) ie? {d) none of these SECTION IV INTEGER ANSWER TYPE Find the number of complex numbers which are conjugate of their own cube. If w #1 is a cube root of unity, and a+b = 11, @ + b* = 1001, find the value of (aw? + bw) (aw + bw”) IF teh = tel = leg! and arg (2)/2,) = 60°, find the de- 4 gree measure of arg | = 2% Let eccentricity of ellipse with focii z) and zp be 173. If Izy = z3f = 5 and z lies on the ellipse, find maximum possible value of lz ~ z! SECTION V MATRIX MATCH TYPE If @# 1 is acube root of unity, a, 6 € R then value of (a) (a +b) (am + bat) (p) 6ab (aah + bon (b) (a+ bP +(aw+ bay + (q) a+ b+ (aa? + bax? 1 - 3ab (©) @+b+D@t bus a) (a+ ba? + @) are @) (a+ bas cory + (s) Qa-b-c) (a + ba? + cay® (Qb-c-a) (Qce-a- 6) ttre 14, then value of (@) l-wew-w4e. @) 7(5+ Vii) (b) 1- www +. @ Z(0-V3i) 45. 47. 48. 1 wy lz RO+v) ©) ltwew ewe. @ Lew ewe. (a) Gis the greatest and L be the least value of tz — 21 if z satisfies: ie+243ds1 (b) G is the greatest and Lis least value of Iz] when z satisfies ~ Sst (©) Gand L are the roots of f? - 10r + 36 = 0. (d) G is the sum of infinite geometric series 5.5 54345 2°32 (p)-L+G=10 (q) LG = 624 () G-L=2 REASONING TYPE Statement-1: Let ABC be a triangle and P be a point on the circum-circle of AABC, then PA® + PB + PC’ is independent of the position of point P. ‘Statement-2: Let A, B, C be three points in a plane such that CA? + CB? = AB7/2, then ACAB is an isosceles triangle. Statement-1: Centre of circle Statement-2: Radius of circle Let ABC be a triangle and P be any point in the complex plane. Statement-1: BC° + CA? + AB? S3(PA? + PB? + PC?) Statement-2: BC’ + CA? + AB? = (PA? + PBP + PC?) € P is centroid of AABC. Complex Numbers 1.53 Past Years’ IIT Questions : yeaytbB and z=aB+by where and Bare the complex cube roots of unity, show 11978] (1979) . If the cube roots of unity are 1, @, @, then roots of the equation (x -1)° +8 = 0 are (@) -1, 1+ 2@, 1+ 20° (b) -1, 1 2a 1 = 207 © -1,-1,-1 (@) none of these (1979) . The smallest positive integer n for which (5) is (a) 8 (b) 12 (©) 16 (@) none of these [1980] . Find the real values of x and y for which the following equation is satisfied G+ix-2i | Q-3dy+i 3th 3-i . It is given that n is an odd integer greater than 3, but not a multiple of 3. Prove that.’ +7 +x is a {1980} factor of (x + 1)" —x"— 1 {980} . True/False For complex numbers z; =, + iy; and z,= x + iy. we write z, 2p if x, $x) and y, $y, Then for all complex numbers z with 1. z, we have (1981) . The complex numbers z =x + iy which satisfy the equation lie on (@) the x-axis (b) the straight line y = 5 (©) a circle passing through the origin (d) none of these 11981) 10. ML 13. 14. 15. 16. 17, . Let the complex numbers z,, zy and z, be the vertices of an equilateral triangle. Let zy be the ccircumcentre of the triangle. Then prove that g+deg=3g (1981) Ifz= ( fa) Re(z) (b) Im(z) (c) Refz) > 0, Im(z) > 0 (d) Re(z) > 0, Im(z) < 0 [1982] ‘The inequality lz ~ 41 < Iz ~ 21 represents the region given by (a) Re(z) > 0 (b) Re(z) <2 (©) Retz) > 0 (@) none of these [1982] If z =x + iy and w = (1 - iz)Mz — 8), then hl = 1 implies that, in the complex plane, (a) < lies on the imaginary axis (b) ¢ lies on the real axis (i ‘on the unit circle (d) None of these [1983] The points 2,23. 23.2% in the complex plane are the vertices of a parallelogram taken in order if and only if @yty=nty O) at yraty a+ +4 (d) None of these (1983) Prove that the complex numbers z), z and the origin form an equilateral triangle only if zi + 23 11983] = 4220. ‘True/False If the complex numbers 2, 2 and z3 are the vertices of an equilateral triangle such that ly! = lel = ley) then zy + zp +2540. 11984) If a, b, ¢ and w, v, w are complex numbers representing the vertices of two triangles such that ~ Na+ rb and w=(1 ~ pu + rv, where ris a complex number, then the two triangles: (a) have the same area (b) are similar (©) are congruent (d) none of these Itz, =a + ib and z) = + id are complex numbers such that [z= Izy! = 1 and Re(z, Z,) = 0, then the pair of complex numbers w) = a+ ic and wy = b+ id satisfies: [1985] 19, a. 24. 25. 26, 27, Course in Mathematics for IIT-JEE (a) bot = (b) bey = 1 (©) Re(w,,) = 0 (4) none of these [1985] True/False If three complex numbers are in A.P., then they lie ‘on acircle in the complex plane. 11985) Let z, and z, be complex numbers such that z; #23 and (g,! = [z31. If z, has positive real part and z) has Qty, negative imaginary part, then > , may be {a) zero (b) real and positive (c) real and negative (4) purely imaginary (e) none of these [1986] . Show that the area of the triangle on the Argand diagram formed by the complex numbers z, iz and = Lin tiis 5 11986} ‘Complex numbers 2), 23, 23 are the vertices A, B,C respectively of an isosceles right angled triangle with right angle at C. Show that = 2) = Ay - 2) Ga) {1986} If z, and z, are two nonzero complex numbers such that, + z= ey + 3), then arg 2, ~arg z, is equal © z @-# () -5 fk 0 ‘d) = © w $ @n (1987) . The value of 3 {sn 2tt — soon 24) is @-1 (b) 0 =i @i (©) none of these 11987) The complex numbers sin x + i cos 2x and cos x ~ isin 2x are conjugate to each other, for 1 x= (m5) (d) no value of x [1988] For any two complex numbers z), 2) and any two real numbers a and , laz, - bz, + Ibz, + az; True/False The cube roots of unity when represented on the Argand diagram form the vertices, of an equilat- eral triangle. 11988) If @ & yare the cube roots of p, p <0, then for any +, yvand z, @) xanw () «20 {1988} 28. 2. 3. 32, 33, 34. 35. 36. a+ yp xB+ yy + ca” Ifa and b are real numbers between 0 and 1 such that the points z, =a +i,z)=1 + biand z= 0 form an equilateral triangle, then @ = and b= e {1989} Let 2, = 10+ 6/ and z, = 4 +64. Ifz is any complex (1989) number such that the argument of 2 then prove that Iz ~ 7 ~ 91 11990} Column Matching Type 2#0is acomplex number. Column 1 Column 2 (192) ABCD is a thombus. Its diagonals AC and BD intersect at the point M and satisfy BD = 2AC. If the points D and M represent the complex numbers 1 +i and 2 ~ i respectively, then A represents the complex number __or__. [1993] Prove that 2 ay" Cy, 20 where k = (3n/2) and n is an even positive integer. [1993] Suppose z), £3. 2; are the vertices of an equilateral triangle inscribed in the circle tol = 2. If 2, = 1+ iV3- then (1994) Let cand #1 jon +xed= ea 0. The equation whose roots are a'”, 8” is @ P-4-1 (b) Pare 150 © P+x-1=0 @ P+e4150 [1994] If «is an imaginary cube root of unity, then the value of : {1994} If @(¢ 1) is a cube root of unity and (1 + 097 = A + B @ Then A and B are respectively the numbers: fa) 1 (©) 1,0 (b) 1.1 (a) - 1,1 [1995] 37. 38. 39, 40, 41. 42, 43. 1. Let be + bE If @(# 1) is a cube root of unity, then 1 Itito i | [1995] -i -it@-l -1 equals (a) 0 ib) 1 @i @) @ (1995) Let z and w be two non-zero complex numbers such ‘that izi= Iwi and Arg z + Arg w = . Then z equals (a) (b) -w © w (d) -# 11995] Let z and w be two complex numbers such that lS 1, bwt ST and ke + fl = le - diel = 2, Then z equals (a) Lori (b) for ~i (© Lor-1 @ ior-t [1995] Miz +2 ~z += 0, then show that Id = 1. [1995] Ife S 1, wl 1, show that Iz —wl? € (lel—twl)? + (Arg c—Argw)? [1995] For positive integers my, m, the value of the expression CFD EFA + 4 PY +47), where i = Ji, is areal number if and only if (a) m=m+) (b) m= m1 © m =m (d) n> 0,m, >0 [1996] ‘The value of expression 12 - a (2~ a?) +23 a) B-aP) +... +(n=1) (1 a (n-@), where @is an imaginary cube root of unity is__ [1996] |. Find all non-zero complex number z satisfying 11996} b #0 be a line in the complex plane, where 5 is the complex conjugate of &. If a point z; is the reflection of a point z, through the line, then show that c Ebtzb [1997] ). Let z, and z, be the roots of the equation 2” + pz +4 =0, where the coefficients of p and q may be complex numbers. Let A and B represent z, and z, in the complex plane. If ZAOB = ar# 0 and OA = OB, where O is the origin, show that {1997} 41, 49. 50, St. 52. 53. 55. Complex Numbers 1.85 Prove that : 2kn\__n ink) a |.-% be deo 2 } 2 where n 2 3 is an integer. [1997] |. If wis a cube root of unity, then (1 + @~ oy" equals (a) 128 @ (b) - 128 @ (©) 128 @ (@) ~ 128 [1998] ‘The value of the sum ser +i") where = ¥=1 equals @ i (b) i= (©) -i a0 (1998) Let AeA AyA,A Ay be a regular hexagon inscribed in a circle of unit radius. Then the product of the lengths of the line segments AA), AgAy and AgAy is @ 3 8 B © 3 @ ah [1998] Ifi= J-1, then Ry a 445|- +3 SS} is equat to @ 1-3 (b) 1+ iV ©) is ) ~ i3 (1999) For complex numbers z and w, prove that tal w = bw if and only if z= w or gw =1 [1999] ‘If arg (z) < 0, then arg (—z) ~ arg (z) = wx (b) -# x g @ -F @ 5 {2000} |. If zy, 29.2) are complex numbers such that Iz,!= kz! = [then le, +2) 4 21is (a) equal to | (b) less than 1 (©) greater than 3 (@) equal to 3 {2000} Let z; and z, be 1" roots of unity which subtend a right angle at the origin. Then n must be of the form (a) 4k +1 (b) 4k +2 (c) 4k +3 (d) 4k (2001) 1.56 Course in Mathematics for IIT-JEE 56. The complex numbers z), z, and zy satisfying are the vertices of a triangle which is (a) of area zero (b) right-angled isosceles (©) equilateral (d) obtuse-angled isosceles [2001] 37. . Then the value ofthe determinant (a) 3@. (b) 3a(@ = 1) () 30° (@) 3011 - @ (2002) 58. For all complex numbers z,, z» satisfying lz,l = 12 zy ~3—4il= 5, the minimum value of tz, ~ zl is @ 0 (b) 2 (©) 7 (d) 17 (2002) 59. Let a complex number @ a # 1 be a root of the equation Pel where p, q are distinct primes. Show that either Leata+...¢a?'=0 or l+atar+.. + a"! =0, but not both together, [2002] 60. Ifizi=1andw (where z #~1), then Re(w) equals 1 iz+1P @o (b) ~ A © TF (2003) 61. If z; and z, are two complex numbers such that lz! 1-2. <1 and > 1 then sow tat f= 0, k# 1, show that pa represents circle. Find its centre and radius. (2004) Let a, b and c be three integers such that at least two of them are unequal and @(# 1) is a cube root Of unity. The least value of the expression la + ba +carlis @o © Mm (b) 1 @ 12 (2005) . The shaded region in Fig. 1.59 is 8 Fig. 1.59 MW < 2, larg (2 + 1) < #14) +1 <2, larg (2 + DI < #2) = I> 2, larg (2 + 1) < #14} rle+ I> 2, lang (z+ DI < A/4) Letw= a+ if, B40 and #1. The set of val- tues of z for which in (SE) =o I-z (b) {z2z d) {z: a) ( © rkl=1) iy ) ze 1) [2006] . A man walks a distance of 3 units from the origin towards the north-east (NV 45°E) direction. From there, he walks a distance of 4 units towards the north-west (N 45°W) direction to reach a point P. Then the position of P in the Argand plane is @ sere ai (b) GB = die’ (©) (4 + 3ne™ ae (2007) z @) GB + 4ie on (a) a line not passing through the origin (b) Il = V2 (©) the x-axis (@) the y-axis [2007] A particle P starts from the point zp = 1 + 2i, where i= J-1. It moves first horizontally away from ori- gin by 5 units and then vertically away from origin by 3 units to reach a point 2). From z; the particle moves 2 units in the direction of the veetori + j and then it moves through an angle ; in anticlock- wise direction on a circle with centre at origin, to reach a point z,. The point zis given by (a) 6+ 71 (b) ~ 7 + 64 (c) 7 + 61 ) ~ 6471 [2008] Paragraph for Question Nos. 71 to 73 n. 72. 23. 14. 15. Let A, B, C be three sets of complex numbers as defined as follows: A= (z: Im@) 2 1) Bx (zle-2-i=3 C= {z Re (1-9 = V2) Then number of elements in the set AN BAC is @o (b) 1 (© 2 @) = Let z be any point in AV BOC. Then lz +1 - iP + \z~5 iF lies between (a) 25 and 29 (b) 30 and 34 (©) 35 and 39 (d) 40 and 44 Let z be any point in A. B 7) C and let w be any point satisfying tw ~ 2-1 <3. Then Izl—Iwl+ 3 lies between (a) - 6 and 3 (b) ~ 3 and 6 (©) ~ 6 and 6 (@) ~ 3 and 9 12008) Let: = cos 0+ i sin @ Then value of fe ry mer = at 0=2°is 1 1 © ar © Ser 1 1 © Fane © Fine 12009} Let z = x+ iy be a complex number where x and y ‘are integers. Then the area of the rectangle whose Vertices are the roots of the equation 22°42 =350 is (a) 48 © 40 (b) 32 (@) 80 {2009} 4. 43. 4s. 47. 59. 61. 63. 65. 10. 4. B. 82. £38 -@ 2 @ Cb) 6. (a) (a) 14) AS jd) 18. (@) 19. (b) 26. (b) 27. .(b) 30.) 31. @) . (b) 34. (a) 35. ©) (ce) 38. (©) 39.) 51. (a), (b), (©), @) 53. 55. 37. SECTION | ® 10@ (b) 22. (a) 23. SECTION tt (a), (b), (€)42.(a), (6), (©), @ (a), (b), (€)44.(a), (©), (4) (@), ©) (a), (0), (©), @ (b), (€) 50.0), (©), (@) 46. (b), (©), @ 48. (a), (0), (©), (a), (b) (a), (b), (©), (@), © 58. (a), (b), (©), (d) 60. (b), (©), © 62. (a), (b), ©) (a), (0), (c) 64. (dy (b), @) ‘SECTION It (a) 67. (d) 68. (d) (bo) 71. (@) 72. (b) (d) 75. (©) 76. fa) (©) 79. (b) 80. fa) (83. (b) BA, Gb) SECTION 5 87.5 88, 22 91. 10092. 2 95.128 96. wou 89, 93. 97. @ ) . (e) . (©) . (d) ©) © - (b) (b) @) ). (b) 3. 7. a1. 85. © @ (b) ) 14 2 1.88 Course in Mathematics for IIT-JEE 101, aces eeg es aeg es aoe es aoe es aogce 108. 108. al 4s. (by (a) @) (b), (d) (a), (b) SECTION VI 106. (b) 109. (a) 112. (by SECTION I 2. @) 3.@ 6. (d) 7. (b) 10. (b) 11. (b) 14. (d) 15. (b) 18. (a) 19. (ay SECTION Ii (a), (©), (0) 28. (©), (4) (a), (b), (€)30. (a) . (b) ©) aogs ao oe aeoge SECTION It 32.@) = 33. © 36.) 37. @) SECTION IV 40.91 41. 30 SECTION V 107. (a) 110. (b) 4. @) 8. ©) 12. @) 16. (@) 20. (a) 22. (a), (), (), (d) (a), (b), (€)24.(a), (b), (€). (4) 26. (a), (b», (©) 34. (a) 38. (a) 42. 10 SECTION VI 46. {c) 47. (b) 48. (b) PAST YEARS' IT QUESTIONS 3. @) 4.@ S.x=3y= 7. Tue 8. (a) 10, (b) u. @), 12. () 13. (b) 15. False 16. (b) 17. (@), (6). (©) 18. False 19. (@),(@) 20. (©) 2. (@) 24. (a) 25. (+P) (la? +lzP) 26. True 27. worw? 28. 2-¥3, 2-V3 30. (i) © (b), Gi) @ @) 31. 3-i2 or 1-312 33. ~2,1- VBE 34. @) 35. (©) 36. (b) 37. (a) 38. (d) 39. (©) r 42. (@) 43. 4 re IP On-1) 4 48. (d) 49, 50. (©) 51. ©) 53. 54. (a) ‘5S. (d) 56. ‘57. (b) ‘58, (b) 60. 63. (©) kla- Bi 64. us = TEE 65. (b) 66. (d) 67. (d) 68. (d) 69. (3) 10. (d) 71. (b) 2. ©) 73. (b) 74. (d) 75. @) HINTS AND SOLUTIONS 1. ky -zP=- 3% aP = kyP +P 2%, - B22 =14RP kf - 3-2% keyP leg? Ie, - iz, +1 =0 (k,P = 1) (oP -1 As izy1 # I, we get let” I 1 1 Pud 2. |ax, az ayletarl, 2 2 a ug or ile Complex Numbers 1.59 As lal’ > 0 we get Sle. ay 24) = lazy, a2, 24) 1 1 ry fraud 3. A=|,ta gta ytal=|s, n+a z+ Z4a) |z [using Ry > R)—aR, and Ry —> Ry aR] A. ley t agi? + lag + P+ ley +P? = Aly P + lef + lesP) + QZ, + Hz teyH+ Hey +234 a) None of the expressions in (a), (b), (c) equals this expression. i 2 4+% 4-3 lath 2) 7% 4-3! fe a 2222, a- | 2, using C, > C, - Gand C4 G+) 6. a-bP 1a + bP = -ab-ab -Gb —ab = -(a+ 2) (b+ b)=—4Rea)) (Re()) <0 => fla bl +10 + bi) <0 a alz)(l~ 2a) . 1 © (=a) (t~ za) 1 = Ey- x 8 rele of 4 (n+ 5%) L — —A fig +12,P-32,-22, gllaP Hah 28-3 wlaP ley +21z;llep!] =0 a) = a@i+bz+7=0 Q Multiplying (1) by and (2) by b and subtracting, weet t= TF 16F 11. From problem 10, we get (al? = IbP)e = be - ae LHS.=0 and R.HS. #0 1.60 Course in Mathematies for HTT-JEE 12, az+ e+e=O a = Gaz + aE + Gc=0 = bbz + aE + bE =O = b:+Gz+0=0 Q From (1) and (2) we get (at djz+ (G+ bEte+e=' ‘This is of the form Az+ Az +a=0 where A =@+ b and @=c +Ze R and thus represents a straight line. Also, Az ~ = Ay = JE+|R-Aal =0 (AZ -2)F+|R-Azf=0 (1) As itis of the form = Ale = (Ay Ge+az+e=0 where @= Az, ~<, and c € R, the given equation represents a straight line. 14, If mis slope of the line (1) in problem 13, then m= REO, ig = SEE Ima) ~ in ltim__a@ ? i=in & Se Aw tottand cose ising t+ttan@ cos@+isin® = (vos O- isin OP =e 16. Asik~aP =r or te-al=r => zliesona circle with centre at a and radius r. a Q) Subtracting (2) from (1) we get (a-b)z+ (b- Substituting this in (1) we get a quadratic equation in 18, Potting 7=b and = in(2) above we get B+birherc=0 = i+ bP = bP -c @) ‘This is a circle with centre at — b and radius ./\b! 19. If1b? W-@ ols 1 Wal ol 1 T=lallol Also, tal — Iwi < lu — of Mal-toll | u-v Thus, T=luilvl ito 25, Let a= a+ if, 2=x-+ iy, so that de+az+e=0 becomes 2(ax + By)+e=0 a a Let my =slope of (1) = — 5 26. 27. 28. 30. 31. 32, 33, Also, m, = slope 0A = é o=|i -i || lic i = (2- DE -YFG-DE +)=0 = 2eE - (1-2-0 +E = => zlies on a circle tei? - Siz1+ 1 =0 = t= Stv2t 2 = z= OeViie where 06 R. O 0 AST, Gi yy... GF, are roots of x"— 1 = 0, sum of the products taken two at a time is 0. = Sua)y+Daa,=0 > Eaa=-(Za}-t The point (0), A(z), Btz + iz), Cliz) are vertices of a square as (i) the mid point of OC = the mid point of AB (ii) OC = AB = V2iz1 (ili) OA = OC = Iz ie-2P s4le-1? = WP - 2-22 +454 -2- 24+ = Be - 2 +E) = Bid? - 4Re(z) = 0 t+ De -z4+ 1) +24 =0 > G+D@+z+)=0 = re-load ‘out of these @and of satisty 24 IM 4 10 Produce the join of z, and z; to meet az +az +b=0 in z. Suppose z divides the join of z, and z) exter- nally in the ratio & : 34, 35. 38. 4l Complex Numbers 1.61 As zlies on dz+ az +b=0, (2, — ke,) + alZ, ~ B+ bk Bret h ps y tae, +b > te Now, 0, z),z2 form an equilateral triangle 42225 ee O+ ded = 0g, +2) + 2 2 @+z) e @=3b ald +al) =~ 2a = z As a> 0, ld 20, zis real and negative. (1+ 2c0s0) + 2isind (a -2)+ ib ‘ O°" B+ 6088) isin (1+ 2¢086)" + Asin’ 2+ cosy + sin’ @ = (a-27 +h = = St4e080 5+4c056 -8+2=0 J: + 91 by bwyel= 1 Also, Re( i, w.) = 2eos O=x+ Mx = x= cos O+ isin 8 Similarly y= cos + ising + 4 = cos (08) + i sin (0 + cos (n® ~ i sin (n = 2 cos (n@) Next = 2 cos (0+ @) + isin (0+ 9) y =42% 2200s (0-9 yx Lastly, ay = cos (8 + 9) +1 sin (8+ @) = wrt arcs (04% y leyley + M= lesley ~ I = _z/z, lies on the perpendicular bisector of -1 and | = 2,/z; lies on the imaginary axis = 2; = ike; for some ke R Also, the join of O and z, makes an angle of #/2 with the join of O and z; 46. ‘Thus, A, B, C are vertices of an isosceles right triangle. ley — ay)? = ley? + lagl? + ley ead => lah +k Ey — By = ley? + leg + lez => 4% + F250 Also, since ley ~ zl = key - Ol + Izy 01 a We get 2}, 23, 0 are collinear. In view of (1), z, and z, lie on a straight line on a straight line opposite side of O and 48. 49. 50. Si. => lz, —22,!= (cos 8-4 cos 9)? + (sin 8-4 sin 9)? = 1416-8 cos (@- 9) 2 9S lz, - 22," $25 Similarly, 1S lz, + 21 $3, le, ~ 23121 and tz, - 32125. Refer Fig. 1.60 yt | a @ “ par @ FL ee oi Fig. 1.60 Min (arg 2) = @ where y = (tan @) x is tangent to the circle (x= 1P + -2F=1 oo) fran er 2] = (tan a 2)? = sec’ = tan?a— 4 tana’ + 4 = sec?a = tana = 3/4 Min (arg z) = tan’! (3/4) Clearly, max (arg z) = /2 Also, Min (le) = OD and max (lel) = OB where ODB pass through the centre of (1). Now, OD=0P-1= V5-1 and 0B =0P+1=V5+1 (a) is true as sides an equilateral of triangle are equal. For (b) note that O + ef +3 = 0G, +2) + 212 = + 2) = Baz As Zz, Or, = PB, (©) is true. For all values of k except k = 3. In case k= 3, the equation represents a straight line. xell-zltlz4 M+lel 2M 2) ++ Die lel22 412122 ‘This value is attained at z =~ 1.0, 1. 52. Inn equilateral triangle orthocentre, cireumcentre and centroid coincide. For (¢), recall for an equilateral triangle (ay + 2p + ty = Bayes + aye) + ‘53. If zis any point bz +5 e-alek- = Bet (g- This line is identical to the line be+bE-c=0 $4. Each of the given line is tangent to x7 + y* = 1/2 aPsll~z,2 al 22 @ IyP tlh -a%, (kPa -1)=0 56. ley + ay = tay + ley? ° purely imaginary e i a) is purely imaginary 2 ‘As £1 is purely imaginary, Oz,,23 sa right iangle. z 57. or lies on a) ft 2) S yz t= (vers N{Jpts) $ dle\P + (le) keg? ‘Thus, from (1) key — zg) 21 -c) bey? + 1 = We) ke, 59. Since lz] +15 ~ 212 lz + (5-2)! =5, minimum value ‘Complex Numbers 1.63 61. As LHS. is real, Im(z) =~2 62, Complete parallelogram taking BC, CA and AB as iagonals. 63. Let Thus, Sis a real number, so ReS=S=0e Sy =0 r=) = ds a or a1 = = 20a Ft so Ba =o? 2 me 64. logya (cl? + 4lel +3) > 1 => Il? + 4lcl +3 < 2. Not possible 65. logics 4 (5) < log), (13) = ketil>t 66. = 0 =e" cos y=0,e"siny=0 Ase" #0, cos y'= sin y = 0, Not possible 67. sinz=O => => & cos 2x = le sin 2x=0 = sin2x=0 = x=nA/2,n€1 7 cos (ny) = 1 = GleP al = nem and erent 68, Similar to 67. 69. sin(iy)=0 => €%=1 > y=0 70. €° cosy, = €" cos yy et sin y = > yenety, Asin problem 67, y, = 2n+ 9. = =m Thus, one ‘ 2 524208 1.64 TL. . Can be checked easily by taking 74, e 15. 16. 78. 80. ‘Course in Mathematics for IIT-JEE logr=1 e log lel = 1, arg ge-2 Jes = glo glare) = bale log (-i) = log I-il + iarg (-f) =~ ni Note that "= 1 and 1, @ a7,..., 0"' are roots of 120. Thus, 0+ 1) @— a) (e-@)... a") @- a) 2 5 Use (x~ @) (x @) 21° —2reos > +1 ete. eta ge In 78, divide both the sides by x"! to obtain (+4-200s#](x+ 120052) txt x ” x n (1rt-2o0s z Now, put x = cos@+ isin® From 79, Take limit as 6 0 to obtain =2'(28in?@ = 222)..( in) n=? (2sin 2) 2sin Fe | asin? Se znd 2a n-2 2sin~ sin" || 2sin = sin" 2n dn Qn On 82. aB= (w+ of + a) (a+ w+ of) =34(a+f)=2 83. Desired equation is (a+ P+ aB=0 1 Vii) But mas sin2 + sin 2% ~sin®>0 ut Im ars sin + sin SE —sin ot $h1+s) 85. () Sw! =142a= V7 > —Seyn-3 -da pansy y => ¥+3y-450 = ysl = not possible. ye-4 = Pel = x=tl Thus, Lal + Iyl= 5 88. Letz=xtiy xt iy+\2r+? +2y +1=0 =» e+ (x4? +2y? = 0, y =-1 x4 Axo? +2=0 > xt + 2=07 > Pedrede0 = ‘Thus, IPede 1s 89. 4 + Dz) + il S411 +P + 4a ke be sin (7- A) + casin(a— 9 +absin(f- a =0 W=3 pota apzoresd{ tried) 234+." =34C1) @ kfeil-g" = = z lies onx = V2 Similarly for (b) (a) Both the roots of x7 + ax +b = 0 non-negative ifas0,b 20 (b) x? + ax + b = 0 has one non-negative and one negative root if b < 0 (c) ° + ar +b = 0 has both negative roots if a>0,b>0 (a) sin” Ia is defined for all z with Ic} $1 (b) tan“! Ia is defined for each z € C. (©) log tal is defined for Iz > 0 and cos” kel for bist {d) log Iz - 1) is defined for |x - 1| > 0 and sec! id for i 2 1. -9 z-l (b) kz - (5 + 38) are (a) (©) 3, ¥3i and V3i are equidistant from 1. (d) Mid point of the diameter is the centre a, B= cot Oi Also, osec’@, a+ B= 2cotO @ + ff =2 cosec"Bcos 8 and lal" +/A" = 2 cosec"@ =0 = siny=Oundcoxy>0 = y=2nmne (a) e'cos y= 2, e'sin Also, e=2 = x= log? Thus, z =log2+2nai,ne 1 (b) e’cos y= 1, e*siny = V3 ‘Square and add to obtain ead = ea? = x=log? cosy = 1/2, sin y= -V32 1.66 Course in Mathematics for HT -JEE = yp =2ne- ais a = log? +i Qnz- #73), ne 1 (©) ecosy= ee! siny=0 ‘Squaring and adding sedans? cosy stsinysO = y=2ne Thus, 2 =2+2nzine 1 @) e* cos y=-1,e*siny=0 ‘Square and add to obtain x = 0 cosy=-I,siny=0 = y=Qn+DaneL 105. Centroid G of AABC is ze + tp + a). Let Orthocentre H of ABC be z, As G divides the join of H and $ in the ratio 2: 1, we get Wy — Hy 4% fori=1,2,3,4 23, 24 lie om a circle with centre at zp ly - ig =r = wi for i= 1,2,3,4. Now, from (1), we get wom, Plny-P lig 2, 2 - wom P/n Foy = em 183i] =0 We — wy wm 2 wm 106, Take conjugate of the given expression to see that 2, % + (2, & + Fz) + czy % is real. For statement-2 see Theory in chapter 2. 107. Use Pythagoras theorem and statement-2. 5 _|2z+3|_ 2|2+3/2| tos. f-BS4 24/3) => we smn) 3 BB z+¥3| 2N30 2 represents a circle. For statement-2, see Theory. 109, For statement-2, note that = Lif kisamul- tiple of 3, and i —& is not a multiple of 3, then one of @, @ equals w and other equals a” Next, note that roots of z°-z + 1 = 0 are - @and - 0? =~ @~ Ii, Now, use Ghee? =i- a+ corr __ J if is not a multiple of 3 ~ {\ if k is a multiple of 3 110, Let a= a+ (Band a, = a, + if, Now, the two lines are given by 2 (ax + fy +b=0 wM and 2 (ax + By) +b, =0 2 ‘The lines (1) and (2) are parallel if and only if oe a BOB ip oo «Sib G+, @,-iB o 2-[4) eo 4 isrea a le a and only if ee S25“ is purely imaginary a a a 4 4 A= CoaF veace 10 6nd -— © $+3sind + {(@? is maximum when cos@ = —1, and maxi- ‘mum when cos@= 1. ‘Thus, 1/2 < f(® <1 12. m= la + ib and n= le + idl min = (la + ib le + idl)? = Mac ~ bd) + ifad + be)? seep where ¢= ac ~bd, f= ad + be € 1. Statement-2 is false, since Complex Numbers 1,67 ey + zp? = (lel + Leal)? leyl* + kept? + 2Re(z, Z) slay? + leg + lay teal Refz, %) =z 2) 41%20, 2.1 QUADRATIC EQUATIONS ‘An equation of the form art bret c=0 a where a #0, a, b, ¢ © C, the set of complex numbers, is called a quadratic equation. The numbers a, b, ¢ are called the coefficients of the equation. A root of the quadratic equation (1) is a complex number @such that aa + bat+c=0 ‘The quantity D = b* — 4ac is known as the discriminant of the equation (1). Roots of (1) are given by the quadratic formula p= 2b AND. 2a 2.2 NATURE OF ROOTS 1. Ifa, b,c Rand a #0. Then the followings hold good: (a) The equation (1) has real and distinct roots if and only if D > 0. (b) The equation (1) has real and equal roots if and only if D = 0. (c) The equation (1) has complex roots with non-zero imaginary parts if and only if D<0. @) p+ ig (p,q € R,q#0) is a root of (1) if and only if p — ig is a root of (1). 2. If a, b,c € Q and D is a perfect square of a rational number, then (1) has rational roots. 3. Ifa, bce Qandp+ Jq (p,q € Q)is an irrational root of (1) then p — J@ is also a root of (1). 4. Ifa=1,b,¢€ Land the roots of (1) are rational numbers, then these roots must be integers. 5. If (1) is satisfied by more than two distinct ‘complex numbers, then (1) becomes an identity, that isa=b=c=0. Cuarter 2 THEORY OF EQUATIONS 2.3. RELATION BETWEEN ROOTS AND COEFFICIENTS If and Bare roots of the quadratic equation (1), then a+p=-4 and ap @ ‘A quadratic equation whose roots are cand Bis given by (x ~ @ (x f)=0 24 QUADRATIC EXPRESSION AND ITS GRAPH Let fix)= ax’ +bx +c, where a, b, ¢ © Rand a #0. We have fay=a[ebe+£] =al 2 = al( When is a Quadratic Expression Always Positive (Negative)? It follows from (2) that f(x) >0 (< 0) Vxre Rifand only if a>0(<0) and D=b?—4ac <0. See Fig. 2.1 (Fig. 2.2). Also, it follows from (2) that f(x) 2 0(s 0) Vx Rif and only if a > 0 (< 0) and D = b? ~ 4ac In this case f(x) > 0 (<0) for each x € R, x4 —b/2a. and the graph of y= f(2) touches the x-axis at x= ~b/a. @Q) yy Deb—dec <0 and a>0 fix)? 0 VreR (- b?2a, (4ac - bV4a) 2.2 Course in Mathematics for U'T-JEE. x AG 82a, (hac = #24 T | | foreo | YaeR | De bP~4ac<0 anda<0 Fig. 2.2 ‘Sign of a Quadratic Expression If D =? — 4ac > 0, then (2) can be written as rene l(-o 2) (FS ]] 2a where a= a ? — ac > 0 and a > 0 then >0 forxB fay If D= <0 foracx 0 anda <0 then <0 forxa fix) |>0 for P 0, then f(x) attains the least value at x= —b?a. This least value is given by b)_ 4ac—b* si Cz) sa Ifa <0, then f(x) attains the greatest value at x = —b/2a. This greatest value is given by b dac s(-2) 2a 4a 2 2.5 POSITION OF ROOTS OF A QUADRATIC EQUATION Let fix) = ax? + bx + ©, where a, b, ce R be a quadratic ‘expression and let a real number. Conditions for Both the Roots to be More than a Real Number k. If a>, then the parabola y = ax? + bx + ¢ opens upwards and intersects the x-axis in crand f where z a p= b+ yb" - 4ac 2a In this case both the roots @and f will be more than k if k lies to left of both wand f. See Fig. 2.5. a> 0B —dac>0 Fig. 2.5 From the Fig. 2.5, we note that both the roots are more than k if and only if D>0 ik 3 Git) fH > 0 a In case a < 0, both the roots will be more than k (refer Fig. 2.6) if and only if -> @) D>0 Gk0 Fig. 2.6 Combining the above two sets of conditions, we get both the roots of ax? + bx +e = are more than a real number kifonly if @D>0 dec Gin gh >0 2a Conditions for Both the Roots to be Less Than a Real Number k Both the roots of ax’ + bx + ¢ number k if and only if are less than a real @ D>0 k> me Gi) af >0 2a 26 CONDITIONS FOR A NUMBER k TO LIE BETWEEN THE ROOTS OF A QUADRATIC EQUATION The real number & lies between the roots of the quadratic, equation f(x) = ax? + bx + c= 0 if and only if @ and f(k) are of opposite signs, that is, if and only if (f) a>0 (i) D>O Gil) fi) < 0 {See Fig. 27] or fi) a<0 fi) D>0 iii) fk) > 0 [See Fig. 2.8] Theory of Equations 2.3, Combining, we may say k lies between the roots of fle) = ax + bx + c= Oif and only if () D>o Gi) aff) <0 40, B4ac> 0 yrarthete Fig. 28 2.7. CONDITIONS FOR EXACTLY ONE ROOT OF ‘A QUADRATIC EQUATION TO LIE IN THE INTERVAL (,, 2) WHERE ky < ky Ifa>0, then exactly one root of f(x) = ax’ +bx+e=0 lies in the interval (k;, k3) if and only if f(k,) > O and f(k,) <0. Also, exactly one root lies in the interval (kj, &) if and only if f(k,) <0 and ft) > 0. See Fig. 2.9.-Thus, ifa>0, exactly one root of f(x) =ax" + bx-+e=0 lies in the interval (k,, k) i and only if f(y) fk) < 0. a>o Si)=ac+be+e Fig. 29 Similarly, if a <0, exactly one of the roots of f(x) = ax’ + | bx+c=0 lies in the interval (k,, kp) if fe) fk) <0. a 0, both the roots of f(x) = ax? + bx + c= 0 lie in the ) imeerval (&,, &) if and only if 24° Course in Mathematics for IMT-JEE @ D>o0 i) 0 and fib) > 0 See Fig. 2.11 a>0 Flay= ar? + be +e Fig. 2.11 In case a <0, the conditions read as () D>0 (i) < -2ek 2a Gil) f(k,) < 0 and f(k, < 0 See Fig. 2.12 f)=ar+hx +e Fig. 2.12 2.9 CONDITIONS FOR A QUADRATIC EQUATION TO HAVE A REPEATED ROOT ‘The quadratic equation f(x) = ax’ + bx + =0,a# Ohas @ as a repeated if and only if f(a = 0 and f(a) = 0. In this case f(x) = a(x ~ @*, In fact a= ~ bia. See Fig. 2.13 and Fig. 2.14. a>0 Sx) = ax-aP L@H=0, (a= 0 Fig. 2.43, a<0 fix) atx a Hao, f@=0 Fig. 2.14 2.40 CONDITION FOR TWO QUADRATIC EQUATIONS TO HAVE A COMMON ROOT ‘Suppose that the quadratic equations ax” + bx + and a’ + Bx +e = 0 (where a, a’ # 0 and abl — a’b #0) have a common root. Let this common root be a Then ad + bat and da + bate =0 Solving the above equations, we get (ab’=a’b) = (ac —ae"y =(be" — b’e)ab’ — aby This is the required condition for two quadratic equation to have a common root. See Fig. 2.15 ar +hete yra'e+ Bere yede+hrte yrarls inte Fig. 2.15 How to Obtain the Common Root? Make coefficients of x7 in both the equations same and subtract one equation from the other to obtain a linear ‘equation in x. Solve it for x to obtain the common root, 2.11 CONDITION FOR TWO QUADRATIC EQUATIONS TO HAVE THE SAME ROOTS In case a’b ~ abt = 0, then Sat, a ve In this case both the equations have the same roots, 2.12 EQUATIONS OF HIGHER DEGREE The equation fle) = age +a! + ah? + aye ta, =0 where dp. dy. *% dgts dy € C, the set of complex numbers, and do # 0, is said to be an equation of degree n. ‘An equation of degree n has exactly n roots. Let a, &, ss, 4, © C be the n roots of (1). Then F(x) = ax = &) (= @&) - (= @) Also a, +a, +--+ a,=-“L and 4% eM Qa, --- a= (-1)" 40 Gy : 2.13 CUBIC AND BIQUADRATIC EQUATIONS If @ B, y are the roots of ax? + bx? + cx +d =0, then c as psy=-%, brs yas of = ; Also, if @ B, 7 Sare the roots of the equation ax! + bx’ + cx" + dee =0, then at ft y+ b= bla,(a+ p(y+ 5)+ ap 7S=cla OB (y+ 6)+ ¥8( a+ B) =— dla, aByS= ela, 2.414 TRANSFORMATION OF EQUATIONS a and apy=~ 4. We now list some of the methods to form an equation whose roots are given in terms of the roots of some other equatio Let the given equation be aX tay tt ta x ta, =0 w Method 1: To form an equation whose roots are k(# 0) times roots of the equations in (1), replace x by wk in (1). To form an equation whose roots are the negatives of the roots in equation (1), replace xby —xin(1). Alternatively, change the sign of the coefficients of x" 1,09, x8, ete, in (1). ‘To form an equation whose roots are k more than the roots of equation in (1), replace x by x—kin (1). Method 2: Method 3: ‘Theory of Equations 2.8 Method 4: To form an equation whose roots are reciprocals of the roots in equation (1), replace x by I/r in()and then multiply both the sides by x". To form an equation whose roots are square Of the roots of the equation in (1) proceed as follows: Method 5: Step 1 Replace x by Vx in (1) Step 2 Collectail the terms involving Vx on one side. Step 3 Square both the sides and simplify. For instance, to form an equation whose roots are squares of the roots of 2° + 27 x +2 = 0, replace x by Vr to obtain ave +2e-vx +2=0 = Ve@e- Is ‘Squaring we get x(x ~ 1? = 4 + 1 -6x°-Tx-4=0 Method! 6: To form an equation whose roots are cubes of the roots of the equation in (1) proceed as e+) follows: Step 1 Replace x by x, Step 2 Collect all the terms involving x" and x” on one side, Step 3 Cube both the sides and simplify. 245 DESCARTES RULE OF SIGNS FOR THE ROOTS OF A POLYNOMIAL Rule t: The maximum number of positive real roots of a polynomial equation Fla) = ay xX + ay 2 + a +a yxta, =O is the number of changes of the signs of coefficients from positive to negative and negative to positive. For instance, in the equation x’ + 3x° + Tx ~ 11 = 0 the signs of coefficients are tee As there is just one change of sign, the number of positive roots of x° + 3x7 + 7x— 11 =Ois at most | ‘The maximum number of negative roots of the polynomial equation f(x) = 0 is the number of, changes from positive to negative and negative to positive in the signs of coefficients of the equation f(- x) = 0. Rule 2: 2.6 Course in Mathematics for IMT-JEE. 216 SOME HINTS FOR SOLVING POLYNOMIAL EQUATIONS 1, To solve an equation of the form (x-a)*+(x-by =A put £ eS In general to solve an equation of the form (= a)" +(e ~ bP =A where n is a positive integer, we put gees 2, 3 2. To solve an equation of the form ay (FOO) + a,(fONy" + ay = 0 a) we put (fix))"= y and solve ay 7 +a, y +a, =0. to obtain its roots y, and y>. Finally, to obtain solutions of (3) we solve, ny" and (FQ) = yy 3. An equation of the form (ae + br + ¢)) (ax? + be + 6) « (al + be +e) A Where ¢). ¢ “++. Gp A € R, can be solved by putting ax? + bx = y. 4, An equation of the form (=a) (%- b) (8-0) e- dy AP where ab = ed, can be reduced to a product of two quadratic polynomials by putting y=x+ 2 5. An equation of the form (e~ a) (xb) (e~ ) e~d) =A where a 1), then we can write fix) = (x= a g(x) where g(a) #0. Also, (9) = 0 has eas « root with multiplicity rok IF f(29 = 0 has n real roots, then f(x) = 0 has (n= 1) real roots. It follows imme Rolle's Theorem. IF f(x) = 0 has a distinct real roots, we can write FAR) = dy (& — 0%) (~~ where aj, d+ fo) = 0, ‘We can also write £0 1 fo” Axnw Result 2 ‘ely using Result 1 and Result 3 (~ @,) @, are n distinct roots of SOLVED EXAMPLES: SECTION | SINGLE CORRECT CHOICE TYPE Example 1 The number of real roots of (x+3)4+ (x4 5)'=16 a) is @ 0 (b) 2 4 (d) none of these Ans, (b) Solution Pury=x+ S23 =244 [See 2.16 in Theory] ‘The equation (1) becomes O- +04 Dt = 16 Ay +6? +1) =16 yt + 6-720 O' + DG? -)=0 fe-7 of yal ystv7i or y=el xe-44V7i orxs—4215-Sor- the given equation has two real roots, 0 puuure Example 2 The product of real roots of the equation Lat ~ 26 12-2750 i) is (a) -3" fe) -3" dy - 3" Ans, (a) Solution Put ixl** = y [See 2.16 in Theory], so that (1) becomes (by - y ~%6y-27=0 = O+)0-M=0 > y=-10r27 As xl 2 0 Vx, we get y = 27 eae product of roots of (1) is 3°) (— Example 3 Sum of the non-real roots of W4x-20? 44-3) 12 a is @1 Ans. (b) Solution Put. +x = y [See 2.16 in Theory}, so that the equation (1) becomes O-DO-D=12 y~Sy~6=0 (b) = 1 ©) -6 6 = = Y-9G+D=0 = When = -y = 6, we get +4 -6 = (+3) (250 of x= When = y=-I,we gettext = x= w, w? and their sum is —1. Example 4 If tan A and tan B are the roots of the quadratic equation x? - px + ¢ = 0, then value of sin? (A +B) is 2 2 P P @) + b) > p+¢ © (q+ py 2 P i) 1- 5 —_, © (d-qy d-gh +p Ans, (d) Solution We have tan A+ tan B= p, and tan A tan B= tn AytnB p tan (A +B) = = = tan A tan B tan? (A+B) sec’ (A+B) Now, sin’(A+B)= - l-a* ‘Theory of Equations 2.7 ExampleS If x € R, the number of solutions of Vout -J2x-12 Lis ft (by 2 (© 4 @) infinite Ans. (a) Solution For the equation to be defined, we have 2e+ 12 0and 2e-120 => 2x2~1 and 2x21 x2 1/2. We rewrite the equation as Vivi = + Jeni ‘Squaring both the sides, we get Qee l= 14 2-14 2y2eRT > 1=2V2x-1 = 2-114 = x= 5K Since x = 5/8 satisfies the given equation, the given ‘equation has just one solution. Example 6 If l,m, mare real, + m0, then the roots of the equation (4 mje ~~ myx ~ 2 (L+ m) = 0 are (a) real and equal (©) real and unequal Ans. (©) Solution Discriminant of the given equation is D=9 (lm) +8 (+ my As 14 m#0, (14m)? > 0. Also, (1m)? 20. Thus, D>0. Hence, roots of the given equation are real and unequal. Example 7 The real value of a for which the sum of the squares of the roots of the equation 2° ~(a=2)e-a~ 1 =0 assume the least value is @ 0 (b) I @2 @3 Ans. (b) Solution Discriminant of the equation is (a- 2) +4(a+ 1) (b) complex (d)_ none of these das bedasd sa’ +8>O0asae R. =. Roots of the given equation are real. Let these roots be wand £. Then a+ f=a-2, af=~—(a+!). ‘We have @ + B =(a+ BP -2aB=(a-2P +2(a+1) sa -da+4+42t2=a-2a+6 =(a-17 +5 @ + B? is least when a= 1. Thus, 2.8 Course in Mathematies for HT-JEE Example 8. If p and q are distinct primes and x’ -pr+q = 0 has distinct positive integral roots, then p + equals @5 7 19 40 Ans. (a) Solution, Let a and f be wo roots of 7 — px +q = 0 and assume a< fi We have a+ B= p, ap=q Asqis prime, we must have a= 1, B= 4. Thus, p=l+g—p-—q=1. As p and q are prime, this is possible if p = 3, q = 2. Example 9 The real values of a for which the quadratic equation 3x7 + 2(a’ + Lx + (a? ~ 3a + 2) = 0 possesses roots of opposite signs lic in @ =I) ) C=, 0) 3 2 (d) | =, (©) (2) @ Q 2) Ans, (c) Solution The quadratic equation 3x7 + (a? + I) x + a? ~ 3a + 2 = 0 will have two roots of opposite signs if it has real roots and the product of the roots is negative, that is,if 4a? + 1P 3a+2 <0. 3 Both of these conditions are met if @-3a+2<0 ic. if @-1)(a-2)<0 or l 0 for all xe R, Let g(x) = f(x) + f(x) + f(x). Then @) g@)>0¥xER (b) gS OVER (©) g@) =0VeKeER (d) g(x) = 0 has real roots, 2 @ -3a+ 220 and Ans. (a) Solution Since f(x) >0 Vx € R, a>Oand b* ~4ac<0, We have f’(x) = 2ax+ band f” (x) =a. Thus ei) = ar + bx tc + lav +b + 2a = ax + (Qa+ be + Qa+ bee) a>Oand D= (2a +b) ~ 4a(2a +b +0) = 4a? + dab + b - (8a? + dab + dac) We have = 4ac - 4a? < 0, since b? ~ dac < 0. Thus gx) > 0¥ re R. Example 12, If b <0. then the roots, and x ofthe equation 2c + 6x + b = 0, satisfy the condition (xy/x,) + (iin) < k where & is equal to. (a) -3 @) -5 Aus. (d) (©) -6 (@)-2 Solution "The discriminant of the quadratic equation 2x? + 6c +b = 0 is given by D = 36-86 > 0. Therefore, the given equation has real roots. (ay +2,)' - 24, x, i . 2612) 18 4g 2 be b <0) Example 13. If ae + br +c, a,b, ¢ € R has no real zeros, and if ¢ <0, then, @aO ©) a>0 (d) none of these Ans. (a) Solution Let fix) = ax? + be +e. Since f(x) has no real zeros, either f(x) > 0 o f(x) < 0 for all x © R. Since 10) = © <0, we get fix) <0 V x € R. Therefore, <0 as the parabola y = f(x) must open downwards, Example 14 If x is real, then the maximum value of y= 2a - (seve +67) @a+e (bo) a te) fa +o? @ aa re Ans. (a) Solution Let rare [Pap 1 yxith re yeae e y= 2k and ne eth Thus 2a (ree eo") = (2 2-1) o at P +P =a +b - @ - at + PY P+B-(a- 17 Therefore, y = (a ~ x) (reve ve") Sa? +B, Hence, the maximum value of y is a® + b?. This value is attained att=aorx=(a°—b'V2a. Example 15 If both the roots of the equation 2X — 6ax +2 -2a + 9a’ = 0 exceed 3, then (a a Hae © a> 32 @ a<52 ‘Ans. (b) Solution We can write the given equation (x - 3a) = 2a -2 Note that ¢ 2 | and x= 3at (2-2 Both the roots will exceed 3 if smaller of the two roots exceed 3, that is, if 3a-J2a—2 >3 3(a-1)>V2 Ja-1 a> 1 and Ja-1>y2/3 2 a>l+o= 4) = abes oo) 9 Example 16 If the equation /x+1-Vx =a has a solution, then fa) OI @asl 1 Soluion. a= Te ‘Thus, 0 Y3sks3 ‘These extreme values are attained at x = {and x= Example 22. Let a > 0, b> 0 and c > 0, Then both the roots of the equation ax? + bx +.¢=0. (a) are real and negative (b) have negative real parts (c) are rational numbers (@) none of these. Ans. (b) Solution We have D = b? — 4ac. If D> 0, then the roots of the equation are given by -b+VD xu OED 2a As D = b — 4ac 0, ¢ > 0), it follows that the roots of the quadratic equation are negative. In case D<0, then the roots of the equation are given by -btiv-D 2a which have negative real parts. Example 23. Let a, b, c be non-zero real numbers such that \ fet +e) (ae + br + ode ° 2 = fle +e) (ar + br + 0) de ‘Then the quadratic equation ax? + bx + ¢ = 0 has (a) no root in (0, 1) (d) at least one root in (1, 2) {c) a double root in (0, 1) (4) none of these, Ans. (b) Solution Let fixj=(e* +e) (av + bx +0) We have free = Jrevac + Jroa o > 1 = Jrovee =0 1 2 Iff()>0(<0) ¥.xe [1,2], then Jresas >0(<0). Thus, fla) = (&* + €") (ax? + bx + c) must be positive for some value of xin [1,2] and must be negative for some value of x in {1,2}. Ase" +e°2 2, itfollows that if g(x) =ax’ + bx+c, then there exists some a, f € [1, 2} such that ¢(@) > 0 and g(f) <0, Since g is continuous on R, there exists some ybetween @and such that ¢(7) = 0. Thus, ax? + bx + c= Ohas at least one root in (1, 2). Example 24 The equation Vxt3-4¥x-1 +y¥x+8-6yx-1 =1 has (a) no solution (b) only one solution (©) only two solution (@) more than two solutions. Ans. (d) Sohaion Pu Jx—T=t orx=f +1 so that the given equation becomes VP 44-414 VP 49-61 = 1 ot Ver=29? + Vr=3F = 1 or [e-2|+|t-3]=1 This equation is satisfied for all values of f lying between 2and 3, i... 2$1$ 3. Thus, the given equation is satisfied for all values of x lying between 5 and 10. Example 25 The equation 3°-' +5"! = 34 has (a) no solution (b) one solution (©) two solutions (4) more than two solutions Ans. (b) Solution It is quite clear that x = 3 satisfies the given ‘equation we now show that there is no other solution, Note that y = 3°! and y = 5*~' are both increasing functions of x (exponential functions with base greater than 1). ‘Therefore their sum y = 3°! + 5°! is also an increasing y Fig. 2.18 function of x. This means for x <3, 3°"! +5‘! < 34 and for.x>3, 3°! +5"! > 34. Thus, the equation has no other solution. (Fig. 2.16) Example 26 If the harmonic mean between roots of (+ V2) -br+8+ WS =0 Uy is 4 then b equals @2 () 4-5 3 @ 4+ V5 Ans. (d) Solution Let a Be the roots of (1), then 2 b 84295 an 28 = a+B S42 ~ 2543) = bn4e 5. Example 27 In a triangle POR. ZR = w/4. If tan (P/3) and tan (Q/3) are the roots of the equation ax’ + bx + ¢ = 0, then (a) a+ (b) b+c=0 (a+ (d) bee Ans. (a) Solution (5) om (3) --2 ‘Theory of Equations 211 f gee “ (5) mn (5 o Now, 2R = n/4 => P+ Q= 3/4 vet (2) = mee eames 4) ~ 1=tan(P/3)tan{Q/3) = abla, -b I-ca a-c = athe Example 28 Ifa $0, then number of real roots of 2 + dale ~al + 3a =0 is (a) 0 (b) 1 (2 () infinite Ans. (b) Solution Put x ~a=tand rewrite (1) as (a +1) + 2altl + 30° =0 = P+ dati + 4 =O Q Ift<0, we get? + 4a? = 0. Not possible. Therefore ¢2 0. ‘Thus, (2) gives + 4ar + 4a" > te-2 = xe-a Example 29 Let (ay ay, ay ay ds) denote a rearrangement of (3, — 5, 7, 4, ~ 9), then the equation ax +a, ta,2+a,x+a.=0 has {a) at least two real roots ¢b) all four real roots (©) only imaginary roots (@) none of these Ans. (a) Solution x= 1 is always a root of the equation, Example 30 If three distinct real number a, b and © satisfy (a +p) = bb +p) = Cle + p) where p © R, then value of be + ca + ab is (a) -p (b) Pp w) 0 Ans. (c) Solution @) pr If value of each relation is k, then a, b, ¢ are roots of s+ pr? == 0. Example 31 The number of integral roots of the equation ate Vr8 +20 222 fo tb) 2 © 4 (a) 8 2.12 Course in Mathematics for IT-JEE Ans. (b) Solution Put x‘ = y and write y + 20= (22 -y)* = ye 16, 29, But y = 29 is not possible and x‘ = y = 16 = x= £ 2 as .ris an integer. Example 32 The product of roots of Were Box =1 w (a) -21 Ans. (b) Solution Cubing (1) we obtain Bere 8-14 564- VF a1 = (4-2-5 364-2 =- 125 189 => x= 43/21 Example 33 Let § denote the set of all values of the parameter a for which is (b) - 189) -9 d) -5 > r+ VP 4a sa aw has no solution, then S equals @) C11) b) == 1) fe) {- (d) ©, ©) Ans. (b) Solution If a = 0, (1) holds for all x S 0. Write (1) as Yana = date => 2ax = ala - 1) 3 x = (a - YR Substituting this (1) we geta~1+la+ Il =2a ‘This holds if a 2 ~ i. The equation has no solution if a-<— 1 Example 34 The number of roots of the equation ) is @o m1 @©2 @3 Ans. (d) Solution Put x= Wu, Vi Ww @ (1) and (2) give uty = 3912+ = yt = When uv = When inv = ~ 49/12, we get x =~ (5 + V73 yd. Example 35 If all the roots of x + px +q=0p.qe R, q #0 are real, then (a) p<0 () p=9 (©) p>O () p>q ‘Ans. (a) Solution If @, & yare the roots of a + pe +q=0, then a+ B+ y= 0. yt yr af =p. apy=-4 As q# 0, none of « B, ys zero. Now, use pa(a+ Br yy-(@+ B+ P< 0 Example 36 Let S denote the set of all real values of a for which the roots of the equation Dart a? —1= a lie between 5 and 10, then § equals (@) 1,2) (b) 2,9 © 49% (d) 6.9) Ans.) Solution We can write (1) as W-aPeloesatl Now, S Ita and (1 + a) (1 +a) ~ 3a + 4a] > 0 ° 4 (la + 16) <0, Qa-1) (1+ a>0 and Qa +1) (1 +a)>0 -8 sastac-tora> ie anda <= Lora>— 12 2 =! sac-1 7 Example 38 Let S$ denote the set of all values of S for which the equations 2x? = 22a + Ix + ala + 1) = 0 has one root less than a and other root greater than a, then S equals (@) @.1) ) G10) © @.12) (a) none of these Ans. (@) Solution The required a satisfies the inequality 2a? - 22a + Ia +a(a+ 1) <0 © alatl)>0erae (-,-1)U(0,~) Example 39 Let a, 6, p,q € Q and suppose that fis) = P+ ax +b =O and gis) =x) + pr +g =0 have a common irrational root, then (a) fod divides gQ) (b) gy) = xf) (ec) gx) = e~b~q) fix) (4) none of these. Ans, (a) Solution Let a R ~ Q be a common root of fix) = 0 and g(x) = 0. Then @ = ~ aa ~ b, Substituting this in @ + par q=0, we get (@-b+p)atab+q=0 As @is irrational and a, b, p,q € Q. b-ayg This gives, g(x) = (x ~ a) flv. Example 40 The number of irrational solutions of the equation Vesti ity veil 24 w is @ 0 4 Ans. (©) tb) 2 @ Solution Put Vx? +11 =1o that (1) can be written as Ve er-M ave? ar = 4 Q We also have the identity (@+1-1)-@-1- a2 @) Dividing (2) by (3) we get Vearu-ve fils 5 4) ‘Adding (2) and (4), we get af er=mi) = 44 12 Theory of Equations 2.13 = Per-Ma4er4 Pe = t=4-ax2 45 SECTION II MULTIPLE CORRECT CHOICE TYPE Example 41 Let a, b, c, p, q be five different non-zero real numbers and x, system of equations be three numbers satisfying the then (@xtytczatbte-p—q x= % 4 (© y= GaPb= pe~p) PiP~) @ p= SEMO-9e-@ qg9~P) Ans. (a), (b), (€), (a) Solution Note that a, b, ¢ are roots of xy <4 +e 1 i=p tq © xt = p) (t= 9) + yur - g) + Ht p) = rp) t-4) eo P-Wsytrep tage + 1p + gx + gy + pit - pax =0 From here the answers can be given easily. Example 42 Let a, b, c, d be four integers such that ad is odd and be is even, then ax* + bx’ +ex+d=0 (1) has (a) at feast one irrational root (b) all three rational roots (©) all three integral roots (d) none of these, Ans. (a) Solution Putting ax = y (1) can be written as yi + by? + acy + ad =0 Q) 2.14 Course in Mathematics for ITT-JEE If (1) has all rational roots then (2) has all integral roots. If a, & are roots of (2) then a, f y are divisors of a’d and as stich must be odd integers. Now —b=at Bry eo bis odd ac = py + y+ af © cis odd => be is odd. A contradiction. Example 43 Let abe a repeated root of pQx) = + Bax? + 3bx + ¢ = 0, then (@) @ is a root of x7 + Jax +b = 0 and c-ab ©) a= =) ab-c ©) a= Dak (d) @is a root of ax* + 2bx +c = 0 Ans, (a), (b), (@) Solution ais also a root of p'(x) = 0 or. x7 + 2ax+b=0 Now a? + 3ad + 3ba+e =0 @ + 2aa+b =0 a => aa +2%bare =0 2) Eliminating @ from (1) and (2), we get 2a? - bya + ab-c =0 _ er ab © 2a? =b) Example 44 Let a By be roots of 2 - px? + qx r= 0, then (a) equation whose a” - fy f - ya, 7 ~ ais + (3g =p)? + qq - pe +g) -p'r=0 (b) a permutation of a A. yis inGP.if q =p" (©) Square of one of the roots will be additive inverse of the product of the other two if =a gepr (@) none of these Ans. (a), (b) Solution y = a? ~ By= ae = > a-1y4@ Pp AS @is a root of the given equation 1 Sorm- 2 orar+torg-r=0 P Pr P = (ta -Pot@+ry+@-pr=0 = y+ Ga — pi? + 48q- py +a pr=0 Ifq’~p’ r=0, then y = 0 is a root of the above equation. Example 45 Let a, 2, y be roots of x° + px+q=0, then (a) an equation whose roots are a’, £°, 7 is 2 + 3g" + Bq? + py +g =0 (b) at + fi + p = 3apy (©) an equation whose roots are B+ 7, 7 + a, + Bis x + 6gr + (p+ 12g? + 3p'g + 8q° = 0 wer oe pam Ans. (a), (b), (©), @) Solution y yt py +g =0 p= azyl = +a) =- py => vit day? + Gq? + phys gi =0 a a+ B+ 9 = 3q = 3apy Next y=, +p =-3q-a° = a@ =-3q-y As @ is a root of (1) (© 3q~ yh +3q3q-) +9 +p) CurDeg => + 6qy? + tp + 124) + Sp'g + 89° = 0 Lastly, a? + P+ fp (a+ B+ py - Upy+ yrs ap >» (e+ p+ Py - UP P+ fa + ef) apis Bu PP + Pas eR Br+ y+ of? - 2apar B+ =p Example 46 Ifa e tand (ea) 1) 4224+ +0) a for b, ¢ € 1, then fa) a=% be (b) @a=Iibe @a=%be Ans. (a), (b) Solution For (1) 10 hold (ea) (x- 12) +220 ‘must have integral zeros, Ife I, then (ea) (x- 12)= When 13,-14 Example 47 If the equations x° + bx + ¢ = 0 and bx" + cx + 1 = 0 have a common root then (a) b+c+1=0 (b) Pee +1 = be. (©) (b- cP += 1P +@-D=0 Wd) b+e+l = be Ans, (a), (©) Solution If @is a common root of the two equations, @ +barc=0,be +ca+1=0 Multiply the first by b and subtracting we get @-ea+be- 120 a= nh Thus, . (1 = be? + b(t = be) +b) =0 = Beet + 1 —3be = b+e+ DP +e +1 -be-b-0) = pores DIb-0F + @-1P +- DA Example 48 Let cand f be two distinct real numbers and p(x) be a quadratic polynomial such that pla) = cand pif) = B. then (a) p(p()) ~ x = 0 has at least two real roots. (b) ar and f are roots of p(p(x)) ~ x= 0 (©) plpla)) = x for each xe R (a) none of these. Ans. (a), (b) Solution Use pla) = pla) = a and p(B) = p(B) = B Example 49 The quadratic equation ¥-(m- 3+ m=0 a) Theory of Equations 215 has (a) real distinet roots if and only if me (I), ») (b) both positive roots if and only if m € [9, =) (©) both negative roots if and only if m € (0, 1] (d) at least one positive root if and only if me 9%) Ans. (a), (b), (©) Solution (1) will have real and distinct roots o Dein 4m > 0 em Wm +9>0 me NU O,9) Roots of (1) will be both positive if D20,m-3>0, m>0 3 m29,.m>3. me (9, =) Roots of (1) will be negative if D20,m—-3<0,m>0 => mS1,0 p-az=5-y = (B+ a ~ spa = (5+ yf - 467 b -ac Next, a & x dure in GP, a B = z é 2.16 Course in Mathematics for IfT-JEE wow ac” opr Finally. 1/a, /B are roots of ex’ + 2bx + a= 0 and 1x, Wéare roots of rx? + 2gx + p = 0. Next, He, VB, Uy Ware in A. Example 51 f'a(p +)" + 2bpq +c =O and ap +r) + bpr+c=0, then 2atbp (a) germ -20the @ (by gr=p+ a (©) q~ A= 5 Qa+bybp! ae (a) g- n= Np = 4ac Ans. (a), (0), (©) Soluiion q and rate roots of a(p + xP + 2bpxrte=O ar + Yat bypr t+ ap+c=0 2a+hp o8 @ @ nqers = Bin Next (g—0F =(q +8 - Agr = + (a+ bp - ap? — acl a 4 [ab + 6) p? - ac] Example 52 Let a, 6, ¢ be three distinet real numbers: such that each of the expression ax? + br +c, bx’ +ex+a and cx” + ax + b is positive for each x © Rand _ bet eatab tet @= then, athe @)a<4 (b) a@ V4 @) art Ans. (b), (©) Solution According to the given conditions a> 0,b? 0, c <4ab; ¢>0,a° <4be +P +e «< Abe + cat ab) 1 > <@ 4 Also a? +B? + 2 = (be + ca + ab) = Edo + (e-aF + (a-bF1>0 = al. Example 53 Suppose a, b, ¢ are positive integers and (fix) = ax’ — be + ¢=0 has two distinct roots in (0, 1), then (@a25 (b) b> 5 (©) abe 2 25 (d) abe 2 250 Ans. (a), (b), (©) Solution (0) and f(1) are of the same sign, thus, cla-b+ > 0>c@-b+021. Suppose f(x) = a(x ~ a) (x - B) afl ~ a= pet But afl - @ = 14-(2- a s U4 ‘Thus, aa (1 — @) fl — B) 1, we geta>4 Asae haz 5. Also, b? 2 4ac 2 20 = b 2 5. Next asa 2 5,b 2 5,c 2 1 we get abc 2 25. Example 54 Let fix) = ax” + br + c, where a, bce R. Suppose I/(x)! S 1 ¥ xe (0, 1), then (a) al <8 (b) bi <8 ids (d) Lal + 161 + tel $17 Aas, (a), (b), (¢), (d) Solution Putting x = 0, 1, 1/2, we obtain Ids llatbeds 1, 1 +obtds azbeqst = -ISeSbL-lSatbecesh -4Sa+bedes4 > -4S4atdbode s 4 and -4S-a-b-4es 4 Adding we get -8S3a+Bs8 Also -8SatBs8 -l6s ls b>us8 Since -Is-cS1,-8s-as 8 we get -165 HS b> S8 ‘Thus, lal + Ibi + Il < 17, Example 55 Let a, b, ¢ > 0 and a, b, ¢ be in G.P. and @# 1 be a cube root unity. The equation ax? + hx +¢=0 has (a) real roots (b) imaginary roots (©) roots with negative real parts (d) roots are aah, cua” where a = bla Ans. (b), (©). (d) Solution b? = ac and = BEN tae _ Vat + ia 2a 2a = (SSE tate 2 Na aa Example 56 If the equation ax’ + bx +¢ = 0 (a> 0) has two roots cand such that a<~2 and > 2, then (a) # -4ac>0 — b) c< 0 (©) a+lbl+c<0 @) da42bl+e<0, Fig, 2.17 Ans, (a), (0), (€), () Solution Since the equation has two distinet roots @and B. the discriminant b? — 4ac > 0, we must have fa) = ae +bxt0<0foracx

0¥ xifa+b>0 @ pix) <0 V xifatb 0, then the equation PQ) =a — ar’ —bx-c= has (a) two negative roots (b) all the four imaginary roots (©) one positive, one negative and two imaginary roots (@) two positive and two negative roots. 87, 89, 91 92. 93. 95. 96. Theory of Equations 2.37 ‘SECTION IV INTEGER ANSWER TYPE Ifa, b,c >0,a° = be anda +b + c= abc, find the least possible value of a”. Tf, + x) +x) = Nts + yn, + yy = Land pty = Hh find value of byt + let + bol Find the number of real roots of P(x) = x1 — 24" + 30% — ...— 100 + 101 = 0. Find the number of positive integral solutions xt —y! = 3879108 ind the number of solutions of 2 cos? (5) = (0.2)' + 0.2)" Let fix) = ax? + 2bv +c,a, b,c € R. If fix) takes real values for real values of x and imaginary value for imaginary values of x, find the value of a. Find the real value of ~ @ for which the roots Xj Xp ay of x? — Gx? + ax — a = O satisfy (= 3) + (4) — 3)" + 0-3) = 0 ‘Sum of all the real values of x for which a Vi-e43 Let a and b be two integers such that 10a +b = 5 and p(x) =.7 + ax + b. Find the integer n such that (10) pr) = pin). If-x, and x, are roots of the equation x7 +.4+¢=0, © #0, and find value of ~ 4c. Ixy x35 45 are such that att Zar tan? apts tay ap ta xy to a ea 2.38 97. 98. 100, lon. Course in Mathematics for IIT-JEE SECTION V MATRIX MATCH TYPE Leta 6 (q) one positive and one negative root ©) l R be defined by f(x) = @ @-1,B-1 W a@+l, Bel 2 Bx + 64-8 16 Fis onto, then ae [2, 14]. a+6x-8c 2 Statement-2: If the expression y « 2° *3%-4 atx 4 sxe R assumes all real values, then ee (1, 71. 107. Leta. b,p.q © R,p.#q.b #0 and the ratio of the roots of equation ax’ + bx +b =Oisp:q. Statement-1: [e.fe (e=0 Va Vp Va Statement-2: a(p + q) + bpq = 0 SECTION | SINGLE CORRECT CHOICE TYPE - The number of cubic polynomials pix) with inte- gral coefficients such that for three distinct inte- gers a, b and ¢ such that pa) = b, p(b) = c and pie) =a is @o (b) 1 (©) dependent on a,b, ¢ — (@) infinite . Let p(x) be a polynomial with integral coefficients, such that p(0) and p(1) are both odd integers, then the number of integral roots of p(x) = 0, is (a) 0 () 1 (©) 2 (@) none of these . lfm € N, then number of real roots of, texted tee reo f Qn)! is (a) n (2 © 0 (@) none of these |. The number of real values of x satisfying 64204 220 is (a) 0 (bo) 1 ©) 2 (A) infinite }. Let § denote the sum of all the roots of the roots of the equation 27(16") ~ 3036") + 8(81*) = 0 then $ equals. (a) 23 (by 1 © 32 (@) 572 |. The number of real values of a for which 4 a(2')-a+3=0 has at least one solution is. @ 0 (b) 1 ©) 2 (A) infinite The solution set of ——' +4 _ 23 S=4logyx T+log,x is (@) (4.8) (b) (2, 8) © 12,4) (d) none of these . The solution set of 3x! + 46s" = 64 is (a) {9} (b) (27) (©) (81) (@) (16 . Let $ denote the set of all values of a for which 4° +22a+1)2" +4a?-3>0 is satisfied for all real values of x, then S equals 10. Me 13. 14, ‘Theory of Equations 2.39 @ G=,-1) ) (s3/2.<) © (-1v3/2) (@) none of these ‘The number of integral solutions of (Sil mt is (a) 0 (b) 1 (©) 2 (d) 3 The number of subsets of the real solutions of (b) 8 ©) 16 @ 32 ). Let S denote the set of integral values of x for which +l <3r--3 then the number of elements in $ contains is (ao 1 © 3 (4) infinite ‘The equation (2.4)* = (2.6) ~ 1 has (a) no solution (b) exactly one solution (c) at least two solutions (d) infinite number of solutions Let S denote the set of all irrational solutions of GB) = 36 then the number of elements in $ is fa) 0 (b) 1 (©) 2 (@) infinite . Ifa, bye © Rand (b- 1)° x >. > 4245 46> 0, and Az kta tity Bax, ky FAs + phy HAZAy + MMe + ALG CHa 5454 XT then the equation 2x° ~ Ax? + Bx ~ C= O has (a) exactly one real root (b) three positive roots (©) three negative roots (@) none of these Let A, B, C be the angles of triangle and tan A, tan B, tan C be the three roots of x* = ax’ + bx” cx +d =0, then the fourth root of this equation is also a root of @) P+beed=0 (by P-ax-c=0 (©) P= be+d=0 — (d) none of these ‘The equation x7 ~ ax +b = O and x ~cx+d=0 have a common root and x7 ~ ex + d= 0 has equal roots, then b + equals (a) ac (b) acid (©) 2ac (a) Vac If 4 cosectarx? + 2e + (2°-8+5) =o aa roots, then sin? a+ (cos (sin™' B))? equals (34 (b) 7/4 (©) 14 (@) 15/4 Suppose ay, a3, > O and the equation Pox) =x" + ay"! +... +a, x +1 hasan real roots, then least value of P(2) is (a) 3" (by 2" 1 (@) none of these. SECTION I MULTIPLE CORRECT ANSWERS TYPE “The equation x8 *7/200005" JF has (a) a rational root (b) two irrational roots (©) sum of all the roots of the equation 7/2 (@) product of the roots of the equation is less than 1. . Ifa, b,c Rand (a +c)’ (d) both the roots lie in (- 1, 1) If S denotes the solution set of the equation x41) 24 .cos? (rn sin? [%), then x 2 (@) Sis finite (b) P(S), the power set of S has exactly four elements (©) Sis infinite () Sc I, the set of integers. If aw B ye C are such that a+ B+ y= 2, +P +P =6,a°+ B+ YP =8, then (a) apy (by at 4 fi+ HB © a+ Pr pu d) + f+ past Suppose w/2.< @0. If wand A(a< Ph are the roots of (sin Ox? + bx + cos @= 0, then (a 0 0 be such that a+b+e20,a~b+e2 0,42 c,andletx,,.x de the roots of ax? + bx +.¢ = 0, then (@) Lx}, byl > 0 (b) Ixy beg ST (©) Wi < 2a @) byl St SECTION Ill COMPREHENSION TYPE Paragraph for Question Nos. 31 to 35 A root qrof the polynomial equation p(x) = 0 is said to be of multiplicity m € N if (= a)" divdies p(x) but (x - a)"*" does not divide p(x). For instance 2 is a root of multiplicity 3 of @-2 @-NW+)=0 If avis a root of multiplicity n of p(x) = 0, then eis a root of multiplicity n ~ r of px) = 0. 31 32. 33. Values of the parameter a for which the equation pix) = 3x44 4° - 67 - 12x ta =0 has a multiple root (a) 11,-5 @) 1-7 ©) - 11,5 (@) ~ 11,7 15 #0, the relation between a and b for which the ‘equation x° + px‘ + g = 0 has a root of multiplicity twois (a) 3125q = 256p* (c) 256p* = 1025q If the equation ax? + 3h? + 3cx +d = 0 has a root of multiplicity 2, then this root is given by (b) 3125q + 256p* (d) none of these 0 O% abe If #0 and the equation x" ~npx+(n-1)q=0has root of multiplicity 2, then 35. ‘Theory of Equations 2.41 (a) p= qr" () ph + qh =0 © mp+q'=0 (d) p" + ng! = If ais a root of x" +p, x"! +p,.x"" +... +p, =00f multiplicity 2, then @ must be a root of (a) py x! + pyx? +. +p, 20 (b) py 7 + 2p, 2? +. + np, = 0 (©) mp x) + (n= 1) py + + Dy (d) none of these, Paragraph for Question Nos. 36 to 40 Let ix) = ax’ + 2bx +c,a#0, bac 2 0, Both the roots of fix)= and Ifxy= will lie in the interval (x), x3) if o Hn <-l ey a afl) > 0, afl) > 0 ee, we drop the condition a f (x,) > 0 and in case t= %, we drop the condition a ftx,) > 0. 36, 37. 38. 39. If the equation (1 - a?)x? + 2ax— 1 = O has both the roots fying in (0, 1), then (a) a>2 ) -10 by Lt bl+g>0 © p+qrl @) pret Ifa>0,a+lbi+e>0andc2 (b) l 0, ab < 0, ac > 0, then the equation ax’ +2bx? +c =O has (a) four real roots {b) two negative and two imaginary roots (©) two positive and two imaginary roots (d) four imaginary roots aL. 42. 43. 44, 45. 46. 47. 49, si. ‘Course in Mathematics for ITT-JEE SECTION IV INTEGER THE ANSWER TYPE Leta, be R.Ifforid < 1, lax’ + br +el $ 1, find the maximum possible value of [Zax + bl for xel-li}. Leta,d,ce R.Iflar +bx+cl $ 1 and Se eee is maximum, find la +. Let fix) = ax? + bx +c. Given fix) = x has no real roots, find the number of number of real roots of SUR)=X. Let p(x) be a monic polynomial with integral coef- ficients. Let a, b, c, d be four distinct imegers such that fa) = f(b) = fle) = fd) = 5. Find the number of integral values of k for which fik) = 8. If x, y and z are three real numbers such that xty+z=4andx +) + 2° =6, then it is known that x, y, 2 € (2/3, &]. Find k. Ifa, bc are distinct, find the sum of the roots of the equation (era) (x+b) (eta) (x+e) EF O(E+a) (x-a}(x—b) (x=b)(x-0) (x0) (xa) * If three distinct real numbers a, b, ¢ € R satisfy ala? + p) = bib? + p) + cfc? + p) for some p € R, find the value of a +b +c. Find the numbers of values of x € R for which 64! 4.48" = 30" iv 1 Ifa, bce R satisfy L444! = abe atbte find 2007 5 BPO 5 aa 2006 (a+b+ey™ Find the value of the parameter a € R such that the roots @ f, yof x’ + 6x -axr+a=0 satisfy the relation (a+ 3) + (B+ 3) + (y+ 3) =0 SECTION V MATRIX MATCH TYPE ‘The value of k for which the equation -3x+k=Ohas + (a) all three real roots (p) Iki > 2 (b) two equal roots @k=-2,2 (c) exactly one real root (r) Ik < 2 (@) three equal roots (8) no value of k 52, 33, 54, 55. Range of the function 2 @ ree (p) [+ 18, 9) o) S346 @ [o4] eld lex ul o [23] @ BH br +y¥9-¥1 ) Co-S1UB, =) Suppose a, b,c € R, abe #0 and b? — 4ac 2 0. Let a, Bbe the roots of ax? + bx + ¢ = 0, then roots of 2 wee(-B hero oe Ba Yat a Bp (d) (2x + 1 - 0 ae @ Qr+ 1) we © S90 The value of k for which the equation kx? — 3x7 +1 =O has (a) all three real roots (p) lk > 2 (b) two equal roots @ k=-2,2 (©) exactly one real roots (*) [kl < 2 (@) all three equal roots. (s)_no value of & Polynomial p(x) which satisfy (a) pix +3) = (p) no such w+ Tx + 12 polynomial (b) pe - 1) + p+ =3v- 16 @ 24x (©) po? +1) (1) P= 3x ext er-1 (a) pO? +1 = 43 () 6-78 ‘SECTION VI REASONING TYPE . Let p, g € R be such that p? — 4g 20. ‘Statement-I: If the quadratic equation x7 — px + q =O has positive roots, then the equation can be writ- ten as (ax +b)’ = x for some real numbers a and b. the quadratic equation x7 = px + q roots, then > 0 and p 2 2/4. 37. Statement-1: If m, n are positive integers, x” (b" ~ 6") +b" (c" —x) +" (x" ~b") is divisible by ¥ =(b+ oe + be. Statement-2: If a, b are different complex num- bers and r(x) is the remainder when polynomial ivided by (x ~ a) (x B), then (a) P(A), , an(B)- Bala) a-pB a-B 58. Theory of Equations 243 Statement-1: Let a, b, p,q € R and suppose ax” + bx + ¢=0 has no real roots. Let a, & be the roots of ax” + br+¢=0 and peta A a x-@ ar +brte then B Statement-2: If a, b,c € R, and ax’ + bx + c= has no real roots, then b? ~ 4ac < 0. Past Years’ ITT Questions Find all the intes values of x for which Sx-L<(x+l¥ -5 (d) none of these (1979] Ix, y and z are real and different and =x’ +4y? +927 - 6yz—3ex~ 2xy, then wis always (@) non negative (b) zero (©) non positive (d) none of these. [1979] 10. Me 12. Leta > 0, 6 >0 and ¢> 0. The both the roots of the equation ax? + bx += 0 (a) are real and negative (b) have negative real parts (©) none of these {1980} The equation 2 cos? (3) si xe + ct Dex S m2has (a) no reat solution (b) one real solution (©) more than one real solution (1980) Both the roots of the equation (1-5) (2-0) + (4-0) x a) + (a) (x -d) = 0 are always (a) positive (b) negative (©) real (d) none of these. (1980) If.t + px + 1 isa factor of ax’ + bx +, then @atc=-ab Wa -C=-ab (©) @=c = ab (d) none of these {1980} |. Let x and y be two real variables such that x >0 and. ay c= 1. Find the minimum value of x+y. [1981] Leta, b, ¢ be non-zero real numbers such that Jiarcot nia? tort ede = f,0+00s' n(ae’ +br+ eye ‘Then the quadratic equation ax’ + bx +c = O has (a) no root in (0, 2) (b) at least one root in (0, 2) (©) a double root in (0, 2) (d) to imaginary roo. (1981) 16. 17. 18. 19. 20. 21. 22. 23, 24. 25. 26. 27. (Course in Mathematics for IIT-JEE |. The number of real solutions of the equation: bi -3h1+2=0is (4 1 {) 3 @2 Show that the equation e** — e"*** 4 = 0 has no, real solution. (1982) If 2 + iV3 is a root of the equation x7 + px +q=0, where p,q € R, the (p, g)=(-, —). [1982} ‘The largest interval for which xo Petr L> is @) -4O0and.?-3x-4 5 0. 1983] ‘The equation x 2 xl (a) no root (b)_ one root (ce) two equal roots (d) infinitely many roots [1984] For real x, the function @— =P) iy assume all real values provided @a>b>c (b) ache ©arc>b True/False MWa Find the se of allxfor which 5555 > 7° [1987] Solve x? + 4x43 +2x+5=0 (1988) If @ and P are the roots of x" + px + q = 0 and at, 6 are the roots of x7 - rx + 5 = 0, then the equation x — 4gx + 2g? — r= 0 has always (a) two real roots (b) two positive roots (©) two negative roots (@) one positive and one negative root. [1989) Leta, b, ¢ be real numbers, a #0. If ais a root of a +bx+0=0, Bisa root of ax -bx—c=Oand 0 0 (© a) =0 (@) g@) 2 0 [1990] Show that value of “whenever defined lies wander between 1/3 and 3, [1992] Let (x) and [x] denote the fractional and integral part of a real number x respectively. Solve 4Ux) =x 4 bd If p. q. rare positive and are in A. quadratic equation px’ + gx + r = 0 and all real for : eas ans (c) all p and r (d) no p and r (1995) Ifp and q are roots of x? + pr +q=0. Then @ (b) @ pal (b) p= lord (© p=-2 @) p=-2or0 [1995] Let a, b, ¢ be real. If ax” + bx +c = 0 has two real roots crand B where @<~1, > 1, then show that r+ S42 {1995} ala The real numbers x), x>,.x; satisfying the equation a 7 + bx + y= O are in AP. Find the intervals in which Band 7 lie, 11996) ‘The equation Vx+1—Vx—1 = Vix=7 has (a) no solution (b) one sotution 6) two solutions (d) more than two solutions 11997] Find the set of all solutions of the equation <0 2-2 +1 [1997] ‘The sum of all the real roots of the equation be- 21? + b- 21-2 = 0is —. [1997] Let p and q be roots of the equation >= 2x +A =0 and let r and s be the roots of the equation a? - 18x+ B=0. If p4 — (1999) ‘The harmonic mean of the roots of the equation (5+ V2 - + V5 x48 + V5 =0is (a2 (b) 4 © 6 @ 8 {1999} In a triangle POR, 2 R = #72. Mf tan (P/2) and tan (Q/2) are the roots of the equation ac? + bx+=0(a #0), then (a) a+b (b) b+ arte @ b= 11999] If cand A (a < Bi), are the roots of the equation x + bx +¢=0, where ¢ <0 <6, then @) 0 a, then the equation (x—a) (x- 6) ~ (a) both roots in (a, 6] (b) both roots in (- *, a) (©) both roots in (b, + «) (@) one root in (~ , a) and other in (b, + =) (2000) For the equation 3x° + px +3 = 0, p > 0, if one of the roots is square of the other, then p is equal to (a) 13 (o) 1 © 3 (d) 28 12000) If @ Bare the roots of ax” + bx + c= 0, (a #0) and a+ & B+ are the roots of Ax’ + Br + C= 0, (A #0) for some constant 6; then prove that b-4ac _ B-4AC 3 a (2000) Let ez Bbe the roots of x —x+ p= 0and x de the roots of 7° ~4x+q=0.1F @ By Bare in G.P., then the integral values of p and q respectively, are @ 32 (b) = 2,3 © -6.3 () = 6, - 32 [2001] Let ix) = (1 + bx? + 2bx + Land let m(b) be the minimum value of fix). As b varies, the range of (db) is (e) (0 ‘| (a) (0, 1) 2.86 61. 62. 63. 6. Course in Mathematics for LIT-JEE © [S] (@) 1) (2001) . Leta, b,¢ be real numbers with @ #0, and let a Abe the roots of the equation ax” the roots of a°x? + abex + + bx +6 = 0. Express Oin terms of crand fL {2001} Let -1 S$ p S$ 1. Show that the equation 4° ~ 3x — p = 0 has a unique root in the interval (1/2, 1] and identify it. (2001) The set of all real numbers x for which Pat Atx>0,is @) G- 242») (b) (= V2) U (v2. =) © Em DU.» @) (V2, Ifae (04) then the expression Vi? +x 2 44 LS. in always greater than or equal to Vrte (@) 2 tan a@ © {2002} (b) 2 (d) see2 @ {2003} L Lfflx) =? + 2bx + 2c and g(x) =~ 2? —2cx+ Hare such that min ix) > max g(x), then the relation between b and c, is, a) no relation (©) let < Wt (b) 0 why? (2003) . Ifa, b, € R, then find all the values of a for which the quadratic equation x’ + (a~b)x-a-b+1=0 has unequal roots for all b € R. (2003) ). If one root of the equation x? + px + q = 0 is square of the other, then p and q satisfy the relation (a) p' + gp - 1) +g =0 (b) p? ~ gGp + 1) + q?=0 © p’-qGp-)+e=0 @ p+ qBpt+I+g=0 1f the quadratic expression 2+ 2ax-3a+10>0WV xeR, then @)a>s (©) S0 (@) If x > 5, then F(X) satisfies (8) fo) <1 (2007) REASONING TYPE Leta, b. ¢, p,q be real numbers. Suppose @, fare the roots of the equation x° + 2px + q = 0 and @, [Bare the roots of the equation ax? + 2bx +c = 0, where f° ¢ {—1,0, 1} Statement-I: (p? ~ q) (i? ~ ac) 20 ‘Statement-2: b # pa orc # ga {2008} The smallest calue of &, for which both the roots of the equation x°~ Skx + 16(A7~ k + 1) = 0 are real, distianct and have values at least 4, is [2009] a You have either reached 2 page thts unevalale forvening or reached your ieving tit for this book. 2.48 Course in Mathematics for IT-JEE, SECTION VI 104. (©) 107. (d) 103. (b) 106. (a) SECTION | 1 @ 2 @ 4. () 5. © 7.) 8. (a) 10. ©) 11. (b) 13. (b) 14. (b) 16. (©) 17. ©) 19. (b) 20. (a) ‘SECTION I 21, 4a), (0), @) 2. 23. (a), (b), (A) 24, 25. (a), (b), (©) 26. 27. (a), (). (©) 28. 29. (d) 30. SECTION Il 32. (@) 35. (b) 38. (c) 31. @) 34. @) 37. (b) 40. (a) SECTION IV 42.1 45. 2 48.4 4. 44, 41. 50, wecoe SECTION V $1 pars *@@OO + O@O® * ©@O® O©OO® S20 pars * @©@O® O@O® « |©@OO O@OO) 2 105. (a) 3. ©) 6. (@) 9 @) 12. @ 18. (b) fa), (b), (©) (a), (b), (@) (a), (b) (a), (b), (©), (d) 0), (©), 33. (b) 36. (a) 39. (c) 43. 46. 49. ous 53. 34. 35. 32. (a) 57, (b) pars C@@O O@OO @@OO®) OOOO parts @@Oe@| C@OO) O@@®| O®O® pars O@OO| O@@® @®@O® @@O® SECTION VI aoe oe aoe 58. (b) PAST YEARS' IIT QUESTIONS: 3 Sid (q-- 8) + ptr - p) (q - 8) + gtr ~ py: (9-8 + hr p) (Q- 8) + ar py = © 7. () 8. @) ) 10. @) lL. © . (c) 13. 2 14, (b) @) 17.4.7) 18. Gv) . False 20, (a) -ISx1 66. (©) 67. (c) 68. (d) 69. 1210 70. «a 71. Wd) 72. 73. (by pPqgers ‘WOOD » |P@OO| *@@O® 41 @@O@) HINTS AND SOLUTIONS 1. pO) pl) <0, therefore pix) = O has at least one root in (0, 1). 2. x =0 is not a root. Divide both the aumerators and denominators hy x and put «+ 15/x= y to obtain 14, yok > y=%0.10. But.x + [S/x = 20 and x + 15/r = 10 do not have rational roots. 3. x= 0 is not a root. Divide both the numerators and: denominators by x and put x + 3/8 = y (0 obtain 4 yet yas = 5 hay (wo irrational roots and v+ Mr = 3 has imaginary roots. 4. OF tre deo ert de a0 axe -1tv3i, (3471) 4 5. The given equation gives (Paxt Par Se 6. User + dre 2=(r+ Dit 2), 2 -9r+ 20= (6-494 = 5) to obtain (2 + 3x42) Ue — 904 20) =r -3x-4) 0 -3y- 10) Now, put.” ~ 3x =v and rewrite the equation as (FDO Hr 10) 30 Roots of this equation are 5. 4 V3 10, 14 16, As x Theory of Equations 2.49 Write the equation as uch? +4 + f5cr4n? 49 =S- C4 1P and observe that LHS > 5 and RHS <5 for.x#-1. 6Vs+7 5 5 tl = x=00rr +720. ole +7 =. we must have x and al Ie=tl x Fd are of the same sign ie, ~—20 * 2 r=0orx-1>0 Clearly -20= (a-2)>0 3xt4>0 Vix, (a+3rt2>0VE 5 © (at3P0,k-5>0 => k>5. - 4 eos? a~ 2 sin ex(sin a+ cos a= 0 = 1+ 3c0s2e@=sin2a For 3714 < er x left side is positive and right side is negative. 2.50 Course in Mathematics for LIT-JEE 18. a? +f = (sin @- 1) +208’ = 3-2sin ¢-sin’ g 4=(sin 9+ 1° 19. be +28 = 12-4x—x7 P44 4b) 0+ 1620 ) For unique solution (4 +b)’ ~ 64 =0 | => b= 4, ~ 12, b =~ 12 is not possible. | 20, 2x7 + 3x + 4 = 0 has imaginary roots, Thus, the given equations have identical roots. a_b_e a fe 28 oki sary thoy = a+b +.¢= 9k. For least value take k= 1 21, Let D, =p’ -4q, Dy =r? ~ 48 Now, D, + Dy =p? +r? -4(q +5) =ptP-pra(p- 20 => Atleast one of D,, D; 20. cosee 8+ cot 8: ~ bla, cosee Bcot O= cla 1, we get cosec @- cot O=-alb. As cosec? @- ct 1 Thus, if = at —b'=—4ach™ = ash (b—4ac)= 078 ?—4>0ifa<—2andD 0, tb) < 0 and g(c) > 0, g(x) = 0 has exactly two real solutions. . As fis continuous on [- 2009, 2009) and fassumes only irrational values, f must be constant. Thus, (fix) = (0) Vx € [- 2009, 2009) and as such the ‘given equation has only rational roots. Asq.r€ R, @—iBmust be a root of x + gx +r=0. Let the third root be 7: then 21. 28. 3. 32. 33. 34. a+ iB+ a-ipry=0 = ys-2a As yisarvot of x + gxtr=0,-8a@ -2ga+r=0 * fequired equation is Sx° + 2gx — r= 0. Let a+ if, @~ iB, y where B #0 be roots of Searth= As a+ iB + a if'+ 7=0. we get yis real. @ = (a+ iB+ a iP) (2a) + (a+ i) (aif) =~ 3a + P is real. Also, b= 2a(a+ if) (aif) Let common root be a, then 3a* + pat | =Oand 2a’ + qa+l=0 Multiply the second expression by 3/2 and subtract from the first to obtain =2a(a' + P) is real, Thus, 3 + p(2p - 39) + 2p - 347 =0 Write the given equation as eye) = 28,1 > v= -13, 113 and LHS > 0 Vx. (ate 0 *. fix) = O has at least one root in (c,d). cosa sina 0 cos sin 0 t oo. where 4, = = (y= Dr + y+ 3+ Qy- 5) 20 As xis real, +3 -W-DQy-5)20 = y-By-450 a+ B=-b<0, aBzc<0 As a< B, we get <0, 8>0and B0 = -Texe2ore>s ‘Thus, the smallest integer for which the inequality is satisfied is ~ 6, Da 4a =e —4a°h* = (d+ 2ab— A) (a + 2 + Dab) = a= bY =e} (at bY ~ "| >0 ay —dabe >0 yt VD «one is positive and other b+ a p= =! is negative, ma 47. 49. 50. Si. 52. 53. 55. ‘Theory of Equations 2.51 ASLHS 2 Ix 21. Write the given equation as = PP - Nox = 54. It satisfies the given equation, . Put y= Vet Vat2 = y'a 20424 We + 2x ‘The given equation becomes y"/2 ~ | + y = 3. As y>0. we get y= 2. paxt2ed- We tas It satisfies the given equation, See solution of Question number 45. v4 For equation to be meaningful, x 2 1.x 2-3, x $2. Thus,re [1,2] Pur Vr-l4+Jr43 = y to obtain y+» =>y=2asy 2 0. This givesx=1 Put (2x - 1)" = yor y' = (2e- 1). ‘The original equation becomes e221 + B-yl=-Yr-y) Dore +y tay = = yas other equation is not possible * Qe-Le8ore-2r+1=0 ~ U4 V514.-0-V5)4, Replace x by x ~ 2 to obtain PQx)exr 4-4. As PU? + 2) is of even degree, no such polynomial exists. rs Clearly, a,? = a, => a, Let k be the largest non-negative integer a, = 0. A contradiction. None of x, 13. 43. is 0. Also, Ay ty =O.a sg Thus g + pq + q=0-2g 4 pt l=0>p 0. Itholds for each p € R if and only if @~ cb 4b <0 ee = 2be) (a? = 2 = P+ Ibe) <0 Plat +0) (a~b~c) (at b-0) (ac +b) <0 Let a= a+b, B= ab. Now, @ (a + 1) = 1, and 6 (b+ 1) = | gives b= (ab) (a+ b+ ab+ 1) 1 =f (a+ B+) Also,a' +a'=b' +b) a #b (a +a) +(b' +b?) + ab (a+ b) + ab =0 tals ops peo ab => a(l+f)+h=0 = (J-#-toemree =0 => B+ p+ p-P-1=0. Write ax’ + bx te =alx’ + pet ly (xt 2) where 4 is some constant. = 0 =ald+p),b=a(patl)czad Thus, 0 =c+ap (sr e=ad) and =a(\—p*) = abaa’—c Also, (2a - b)* = b+ 407 =pisarootof ax’ +h +ex+d => -ap"+ bp’-cp+d=0=pld Also, cp - d= p" (b~ap) = p'Kep~d) Ifd=0, plap? - bp +) =0. P4Ore8>0.0 +604 881 22+ 2x43>0,27 42043 41 63. JOB s 242e9368 200 = PH6r+ BDO + 6rt7 20 2642043 >00 4x41 20 and 7 - 2x = 20 + 2x43 = x=-1,-3.Butr=-3 does not satisfy 2° + 6x +8 >0 pi) = 127-3 = 12? - 1/4) > Of W 0= ; cos” (p). 64. f(0), fC1), FI) are all integers 65. 67. 68. Sc,atb+c,a—b+ care integers. = c,d +b, a~b are integers = c,a+b.a~b, 2a are imegers D= 16a*b? — abla + b)? > 0 as ab <0 and product of roots < 0. (\- Purysa’+— or a= x P P r o eply-2}+q fy-4 +re0 ( i a a spy —paiyt gir As at B+ 720, B+ ‘The desired equation can be obtained by changing xto~, a You have either reached 2 page thts unevalale forvening or reached your ieving tit for this book. 89. m1. 92. 93. 95. |. For x #0, (0.2)' + (0.2) > 2 Course in Mathematics for IIT-JEE Both x and y must be odd or both must be even. In both the cases LHS is divisible by 8 but the RHS is not divisible by 8. For.x= 0, LHS = 2 = RHS. 2a BY a 20,fey=a (3) + Forx= -2 +i, x9 is real 2a A contradiction. Thus, a = 0. Let Sy sayfa eas’, $= 6,5: =5; $y = 68. Now,0 = (x) = 3)° + (1, - 3)? + @-3)° 2agry + yey + yy) = 36 - 2a aS, +30 = 216 21a. = Sy—95, + 275, - 243 =-27-3a =-a =9. Put Y2=x + rad, 1Ase2 3, V2=x 21 =x Let pl) = (x @) (x £) PALO) p11) = (10 ~ ay (tt ~ A) (11 - @ (10 - fy = [110-10 (a+ By- a+ af) [110 - 10 (a+ B) - B+ a} = (110+ 10a +b ~ a (110+ 10a + b~ By = (115 - ay (11S - B) = pis) Given relation implies Bees 42 a) +4 42k, tay) ty =O Buty #4) =~ Leap gives 4c? + 5e=0. Asc #0,—4e=5. | Cans + xy + 4p ay tay) tay) = eas = Oy tay tay) Oy? tay tay = (2) (64 I= 14 29444) =-2 Equation whose roots are x),.03, 45 18 -2P-4422054=2,1,-1 Thus, ~ (x3 ~ 43) (= 4) (8) =) . (a) fla) > 0,Ab) <0 {b) fla) + as x — ound fla) <0 {o) fle) <0, fid) > 0 (a) flb) <0. fle) > 0 . (a) Use a+ B+ 7=0 {b) For lange values of x. La) inthe expansion of “77. In the present case sw =3e+p Therefore fy) _ Bee | FQ)” Seprtq a+ Bia yf 2 2p? 1 if,_sy' (©) and (d) =--{1-- xa ak a-ifeees ) Thus, a+ B+ y* £0) = Coefficient of x7 in the expansion of — aw FQ) But fo i) Thus, a+ B+ ¥ 99. 101. 102. bioae (@) -= >0, © >0=9.4,~b,careof the same sign @ a tb -2 <0, £ > 0-54,h,carwotthe same sign @ (©) © <0=9.a,—e are of the same sign. a Discriminant of the equation is D= Aa +1)’ -49a~5)=4 (a~ 1) (a-6) Equation will have real roots if a < | or a 2 6 and imaginary roots if | 0 and Roots will be negative if D 2 0; -(a + 1) <0 and 9a-5>0 ° a>6. (a) Use loggrsx t (b) (196) + (49)! = (4.25) (98)"" vs = 2ms(3) =) es 2! 232,28. 4 ‘Thus, there are two values of x, (©) Put fr-T srorx= 1 +P wobtain VP aaa + JP att 3 [r-24lr- Tata ises2 2 there are infinite roots. (@) Put logy x = rto obtain = 1539 (a) al? + 2x4 1) +b04+ I+ e=0 sxtls@poxsa-1p-1 (b) ax? — bx (x1) + er = 1 asl psi © BO? = D+ le 103. 108. 106. ‘Theory of Equations 2.55 @) a(x? -2x4 1) + Wx I +c=0 Sx-1s@B > xel+qleB Let @# I be a cube root of unity. Putting x= 0 o° we get PU) + OQ) =O and = Pl) + @ Ql) = 0 => PU) = Oand QU) = 0 24s Now, Dj + Dy =p? +r? -4(g +s) =p +r —2prle. Ug+s)=prl =(p-rP20 ‘one of D,, Ds, is non-negative, ax? + 2bx ~ 3c = 0 has no real roots 2 4+ Rac<0 = ac<0 Ife>0, thena <0 In this case ax +2bx-3e<0V ER = a2") +26(2)-3c<0 > asbe *. A contradiction Thus, ¢ < 0. For statement-2, see Theory. ax +3x-4 at3r—4e (at 3x40 y= a 4 3x4 = (+ 4y) P+ 3(1 yn (4 4 ayy = 0 Asxe R, X1—y)" + 4art 4y) (4+ ay) 20 9 (9+ 16a) +2 (2a? + 23)y + 16a+920 This is true for each y € R. = 9+ 16@>0 and (2a” + 23)" - (16a+9) <0 = Qa + 23+ 16a+ 9) (2a +23 - 16a-9) 50 = 9+ 16a>0 and 4(a+ 4)" (@- 1) (@-7) 50 = ae [l,7] ye fix) = eet will be onto if ar6x-8e The fu fe jon (2/2) Q2)NFBE—S kes all real values (a/2)+3—40 = aide (1, Tor ae [2, 14} 2.56 107. Course in Mathematics for IIT-JEE - Expression {f fe have meanings if p/q 20 and bla 20. au [2 [2 eo = a\pNa This is not possibie. pa+qas— = a= @ a(p +a) ‘Now, use paris a root of ax” + bx + c= 0. a You have either reached 2 page thts unevalale forvening or reached your ieving tit for this book. Note that if r #1, then S,, aA = WL < Lothen lim x" ‘Therefore, the sum of the infinite G.P. 1 =o. Cl << poy CH 0 2D. then log ay. log ds, log ay. => is an A.P, In this case the converse also holds Suppose a. + isa GP. Let a, = ARM', where A is the first term and R is the common ratio of the G.P. Then log a, = log A+ G- 1) log R ‘This shows that log a), log ds, log ay, -+ is. an A.P. whose first term is log A and the common difference ix log R. Conversely, assume that log a), log a>. log ay. is an A.P. Suppose that log a; =a + (i 1d, where a is the first term and d is the common difference of the A.P. Then 4 We at(at!-! where @ is the base of the logarithm. This shows that ay, a, ay. isa G.P., with the first term a and common ratio a4, 3.4 ARITHMETICO-GEOMETRIC SEQUENCE SUppOSe ys ys sys“ AS aN ALP ad Byy Boy 5 Bye is a G.P. Then the sequence dy By. ty By. > iy By 18 said to be an arithmetico-geometric sequence. An arithmetico-geometric sequence is of the form ab, (a + Abr. (a + 2dbr?, (a+ 3d\br*, « @ ‘Sum to nterm of an Arithemtico-Geometric series. Let S, =ab + (a+ ddbr + (a + 2d br + oe lat (n—2)d |br"*+ [a+ (n= Vd yor"! (1) ‘We multiply each term by rand write the first term below the second, the second below the third and so on. 1S, = abr + (a + dybr + + [a+ (n— 3d |r"? + la + (= 2adlbr™' + la + (n= Id Yor" Subtracting, we get (= S, = ab + dbr + dbr? + + + dor"? + dr" ' —|a + (n ~ Vd Jor” opel aah BOD ta sin tydlbe*™ =r (el) ab, dbr(i=r""') _ [a+ (a= Dd lbr® #1) if -l Where a, # 0 for each iis said (o be in harmonic progression (H.P.) if the sequence 5 = ab Glered) Vays May. May isin ADP Note that a, the nth term of the H.P,,is given by a, = ——_. where a = 1 and gd = 1-1, atin—ba a a Some Facts About H.P. 1. If @ and b are two non-zero numbers, then the harmonic mean of a and b is a number H such that the sequence @, H. is an H.P. We have boiays 2ab tet(iyt H 2 ( ats) a+b May. dy, ©, a, are m non-zero number harmonic mean H of these numbers 1 at(tete +t) Ho nla a, The n numbers H,, Hy. . H, are said to be harmonic means between a and b if a, Hy, Hy coy Hy b are in HLP., that is, if Ma, 1/H\, + UH,, Wb are in A.P. Let d be the common difference of this A.P. Then or He then the given by \ t= 2-1 pay mete a-b (r+ Dab Ma-b) 1d ‘ nla-b) a (at hab A, a (ne Dab From here, we can get the values of Hy, Hy, --, Hy 3. If @ and b are two positive real numbers and A, G. H are arithmetic mean, geometric mean and harmonic between a and b respectively, then A= G > Hand the equality holds if and only if Progressions 3.3 4, Wh dys dy. sons dy are mt positive real numbers, 1 and a My Haat Hae C= @y ay and He aaete. nay ay then A2G2H. 3.6 SUMMATION OF SOME SERIES OF NATURAL NUMBERS. 1. Dk ale 2ese ene tne m= nin +1) Qn+ 1) M 6 _\ oP enone (Sa] = S(4+2+3 4.0407 = [Jemeof «4 2 4 4, To find} kt and Yk we can use the identi te a win + WP ties: Bak and #—(k-1)° = 6) The first one gives yw-« ik -10K + 102 - Sk + 1 iS — 15K + 20K — 15K? + 6-1 ye -p}=s} K-10 ros m i +0 > e-5 zy ken ror i = Wst0 {i +1 a0{Lnn+nansn}as{tacen} 3.4 Course in Mathematics for IIT-JEE 3.7 SUM OF THE PRODUCTS OF TWO TERMS = | then the number of clements in Sis OF A SEQUENCE | ao 1 ‘To obtain the sum Su, «,. we use the identity wy 2 (A) infinite ay Ans. (b) 2D aa,= (a, tay +... +4) ay #a3+..442) | Solution We can write a) 1 2 of ‘ie 3.8 METHOD OF DIFFERENCE FOR SUMMATION | (+=) ar e249. ee 2010 OF SERIES 2 0sre eee tem If possible express th term as difference of two terms as follows: fret o(emec! Lafa-fre = (ro oP aap tas Now, it is easier to obtain 2424042 tod ‘as explained below." erty etle For instance ifthe given series is 1.3.5 435.7 +5.7.94 2x 1005 200 v 1 + (2r—3)t, + 8, = (2r- hy 45 (a) das > ay Ub) tty 1 Say 5 Basler l)—fin ©) a, 44 () a, ,)- 4, = Va Ans, (b) where fle) = r= 3), Solution We have ) (2r~ 1 2r+ 1) 2r+3) =r 1) 1 ¥(! one Mk This gives 8Yn= Y Ure -son sft fy = = [or + 1)? 91 4+ 1° = 1) -4-9 4-1) = 16(n + 1) 400r+ 1 +9415 = DY t= 2+ v-5e4 +3. mm 1 = inline 2d SOLVED EXAMPLES. => fy > tig SECTION! Example 3 Sum of the series 2-2 43-4? +... — 2008" + 20097 is SINGLE CORRECT CHOICE (a) 2019045 «b) 1005004 Example 1 Let § denote the set of all reat values of « (e) 2000506 1d) hone: of thes sos thet | Ans. (a) 1”, GPE YL tat eae Solution We can write Sas S=(1=2)01 4248-43 +4) 4.04 (2007 ~ 2008) (2007 + 2008) + 2009 -[142+3 444... +2008] + 2009" ~ 5 (2008) (2008) + 20097 = 2019085, Example 4 The value of n for which 704 + : (704) + 4 (704) +... upton terms = 1984 — i (1984) + $0984) =. upton terms is @5 — o)3 ©4 Wo ‘Ans. (a) Solution According to the given condition 1) _ 19842) f,_ (704) @) {1-3} = SE {i = 128= 2422 ~ 1984 If misodd, we get 2" If mis even, we get 12 Example § Ifa, b, care three unequal numbers such that a,b, care in AP. and b~ a, c~b, a are in G.P,, then ratio, a:b: cisequal f@ 1 (c) 2:3: Ans. (a) Solution By the hypothesis, b - a = ¢ - b and (c- by = alba) => =a) alba) = b- = n=0. 3 (b) 1: 4 @i: bea) (b) es y? (d) none of these log, xe led = xayit log. y= 142d = yar? ™ and -15 log,z=1+3d = zext* Mis Jd ler ded ~ saya + 2H c+ Sens Progressions 3.5 = (14d) (142d) (1 + 3d) 15 => 6d° + 11d? + 6d + 16=0 => (+2) (6d -d+8)=0 > d=-2 [Note that 6d? - d + 8 = 0 has complex roots} A -! ~4 x y Example 7 If log 2, log (2* — 1) and log (2* + 3) are in AP., then x is equal to (a) 3 (b) log,5 (©) logs (@) 2 Ans, (b) Solution 2,2 ~1,2%+3 are inG.P. = 2" 1P = 22% + 3) = M4x2%-520 = ‘As 2" cannot be negative, we get 28-5=0 + 2=5 or x= log,5. Example 8 The sum 5, to n terms of the series 25-5) 2+ Ds0 teddy Be 274° 816 is equal to fa) 2" -n-1 (bp 1- 2°" () D"+an-1 (d) 2-1. Ans. (©) Solution Let, denote the rth term of the series, Then t, = 1~(1/2'), We have 21/2") 2 If 1? +2743? +... +2003? = (2003) (4007) (334) and (1) (2003) +(2) (2002) +(3) (2001) +... = (2003) (334) (x), n-142", Example 9 +(2003) (1) then x equals (a) 2005 (b) 2004 (c) 2003 (a) 2001 Ans. (a) Solution Let 5 = (1) (2003) + (2) (2002) + (3) (2001) + see # (2003)(1) a You have either reached 2 page thts unevalale forvening or reached your ieving tit for this book. Length of a side of S, = Length of a diagonal of S,,., Ba, fn + Thus, dj. a>. dy, ... is a G.P. with first term 10 and common ratio 1/2. Therefore, a, = 1o(t/ v2)" Area (S,)=a2 <1 = = Also, = odN2y'Pc, b+ c= 1a >aandc +a 2b = 3c 13a/9 > b, therefore a, b, ¢ are the sides of a triangle. Also, as a is the greatest side, let us find angle A of AABC. Be cos A = 2+ =- Be aac be Hence, A ABC is an obtuse-angled triangle. Example 16 If x)... x, are non-zero real numbers, such that (xj + tal DOP +g +o eas Cpt tak ty GF them ap Aa, oo Ky ae iM (@) AP. (b) GP. (©) HP. (G) none of these. Ans. (b) Solution We shall make use of the identity (ai tad t+ ah) bp + bbe 4 - | (ayby + dads + + Oy by)? = (ayby ~ ayb\¥° + (ayy — ag)? oe + (Gq 1 Pe ~ Gy bg Thus, Of tah +e tad _ Od +d + ead) = (yy tay tt a SO Gy ae)? + Ory — ney? + + Wy -2 tn — Fn et SO +..%, are real, this is possible if and only if | tee 0 Progressions 3.7 = bette My ety os a, are in GPL Example 17 if three positive real numbers a, 6, ¢ {> a) are in H.P., then log (a + ¢) + log (a equal to (a) 2 log (e ~ 6) (©) 2 log (€ - a) Ans.(o) Solution a, bc are in H.P., b= Qacyi(a + ¢). We have log (a + €) + log (a ~ 2b +e) | = log (c - a)’ = 2 log (c= a) 2b + chis () 2log a+ (a) log a + log b + log c. = log (a + ©) + log [ero = log (a + c - dac] Example 18 If @,, ay, >. a, are in A.P. with common difference d #0, then the sum of the series sin d [cosec a, cosec ay + cosec ay Cosee ay +o + ‘e0see di, ., OSCE Ay} is (a) sec a,— seca, —(b) cosee a, ~ cose a, (©) cot a,-cota, — (@) tan ay ~ tan a, Ans. (©) Solution We have d= y= My: Thus, sin d cosec a, cosee a, + cosec a, cosee ay + cosee a, cosee ay] = Sin(a; a) , sin(as~a2) , sina, Sina, sina, sinay = (COL a, = Cot dy) + (COL dy = COL ay) +o + (cot a, _, ~ cot a,) = cot a, ~ cot a, Example 19 Sum to n termns of the series 1-3-S43-5-74 5-7-9 4 oe (a) Bn! + 12n? = 2n- (b) n(8n? + LIn® — n = 3) (©) n(Qn* + Bi? + Tn ~ 2) (@) none of these. Ans. (e) Solution The rth term of the series is given by 1, = r= 1) r+ 1) Or +3) = 8P + 127 - 2-3 3.8 Course in Mathematics for ITTJEE. ‘Therefore, 5, the sum to terms of the series is given by | Lynne D | 2 = 2a? (n+ 1)? + Anon + 1) Qn + Wnt + 1-30 | = f2tehT a =n (Qn + dn? + n+ dn? + 6n + 2—-n— 1-3) =n (Qn + 81? + In - 2). Example 20. Sum to n terms of the series «) ——_"_ Mn+ lint 2hne3) ( o mw +6n? -3n | 6+ 2Vine ayn ed) | 15? +70 nine Din 5) ne +6n7 +11 1&ns Dias ined (d) Ans. (@) ‘Solution The rth term of the series is given by 1 a rr +1) (P+ 2K +3) Splitting 1, into partial fraction, we get 1 l 1 1 oS 1 Lyi Wi) | Aft) dt \ar it) | 6 n+l} 3\2 n+2) 6\3 n+3) | | _ i +6n? + Un © BOD (42 | Example 21 Sum of all the products of the first positive integers taken two at a time is @) Hen ns 1) Gn 2) tb) em = 2) n= tye? © tums nnn ms5) (4) none of these. Ans. (a) Solution We have (1+ 24+ ny Le +E rs neh E mone 2net fs ]-" M2ntY Sirs Panel _ nin+ DQn+1) 4 6 = ck nin +1) 130? + 3n 200+ D) 2» = sp m+ 1 = ay oD + D Bn 2). Example 22 For a positive integer n, let lait 1 ama te ep pte (b) (100) > 100 (d) none of these Then (a) u(100) < 100 fe) (200) < 100 Ans. (a) Solution We have any = te (Lot Thus, (100) < 100, Example 23 5, the sum tom terms of the series GP =P) 4 207 ~ 22) 4 Ma? — P+ is w tarp dae? 4 4 «0 (a) nF =) Ans. (a) Solution S,, (E4J-ze Ly 2 wintl- = msde z Qn = 1 oP 1). Example 24 Let.xbe the arithmetic mean and y, z be the two geometric means between any two positive numbers. +2) Then value of is . xy (a) 2 (b) 3 © 2 @) 32 ‘Ans, (a) Solution Let two positive numbers be a and b. Then x= (a+ bY. Also, a, y, z, b are in G.P. If ris the common ratio of this GP. then b = ar = r= (bla)! We have +2 a'tar® _ oltr) xyz xan(ar*) x i (a+ byl2 Example 25 Let H, = Lede, then the sum a to n terms of the series POP +2? P4247 Fete Bea rae 4 4, 1 =m [An + dee + 1 = Sr] = (2m = 1) an (nt V2. + Qn = 1 + 22m" Example 28° The interior angles of a polygon are in AP. If the smallest angle is 120° and the common difference is 5°. then number of sides in the polygon is. (a) 7 () 8 © 16 Ans. (©) Solution We know that the sum of the interior angles of a polygon of nm sides is (n 2)a We are given that a= 120 and d=5. 2 FH220) + (a — 1) (391 = cw ~ 2) (180) 3.10 Course in Mathematics for ITT-JEE, = nl48 + (n ~ De (n ~ 2) (72) = WP -25n4 4 =0 = (n-N (n= 1620 3 2 =9, 16, But when n = 16, the greatest angle is equal to a+ (16 ~ Iid = 120° + 15 x S° = 195° and other interior dangles are 190°, 185° and 180°, But no interior angle of a polygon can be equal to 180°. Therefore = 16 is not possible. Thus, = 9. Example 29 Let A, = upto n term) then lim A, is 3 ' 5 3 bi 3 ws om © > wy ‘Ans. (a) Solution The numenitor of A, = ¥ e-peen ns Liven meinen n+ W) n+ 2) re = Dee y-@s ph (n+ Dred) ‘Example 30 If the ratio of sums to n terms of two A. is (Sn + 3): (3n +4), then the ratio of their 17th terms is (@) 172: 99 (b) 168 : 103, () 175: 9 (@) 171: 103, Ans. (b) Solution Let wo A.P.’s be a.a+d.a+2d.a+ 3d, and A, A +D.A+2D,A+3D, According to the given condition, a+(n- Vid] 2 - Spasin-no) 3t4 ® = Lag 2 @ = Sn+3 aa tal” ined 2 Now, but "=! = 16orn=33. Example 31 If Hy, Hy. .... H, are n harmonic means Hyta Hath between a and b{#a ), then value of remo equal to @ net (net (©) 2n (d) 43 Ans. (¢) Solution AS a. Hy. Hoy ve Hy, b are in HP. Dad od Vaeinar. aH H, Hy > Let d be the common difference of this A.P.. then i i +(n4 Id and 7 mF = (n= Id Hyta_WatVH,_Va+VH, Now, z A A os Va-VH, =a ang Hath Vb+ Vit, Vos Vi Vb-VH, d = Tins thd + (w= tydl =n Example 32. If a, b,c are three positive real numbers. bad then least value of the expression (a +b +c) {b+} +) is (a) 3 (b) 33 (©) 6 Wo Ans. (@) Solution Since A.M. > G.M., we have 3 tha e2 (abo “pas a Multiplying the above inequs wi tfled 3a b ties, we get “gensa (hates g atbee) @ Bc 2I Jes i => farbso (ete ab The least value is attained when @ = Example 33° Let a. ... a, be positive real numbers such that a, dy ...d, = €. where ¢ is a fixed real number. The expression a) + 2ay + Say + ... + na, cannot be less than fa) ate (by mint o)!" (e) el” (d)_ none of these Ans. (b) Solution We have © (a, + 2ay + 30+ ” -+na,) 2 lla) Ray) Bay... nat!” =intel” Example 34 @ (adh +a + 2d) ~ (at MP + is (a) a? + 3nd? (b) a? + nad + nin ~ Iya? (c) a? + Smad + n(n ~ Vd? (d) a + 2nad +0 Qn +t) Ans. (d) Solution We can write the sum upto (2n+ 1) terms as la + (a + Ahi t- d) + [Ca + 2d) + (a + 3d) (~ dd) + Ma + (n= 2)d) + (a + Qn Vd) (—d ) + (a + Indy? B(-d) last d+ at det ‘The sum upto (2n + 1) terms of the series a+ Qn ~ Id) + (a + 2nd? = d) 2 fa bat Qn— 1) dh + (a+ 2nd? = 2nad ~ n(2n = Wd? + a? + 4 nfad) + 4 =a" + 2nad + nn + Vy Example 35. Value of Progressions 3.11 fa) 0.9 (b) 08 (c) 0.6 (d) 0.25 Ans. (b) Solution We have ' + esha" Jog(t/2) tA?) Tog (0.68) - Togio2sy °° OM =l082 Jog (0,64) = 1 tog (0.64) = 08 =log4 2 > O08. 2,6 Example 36 Let ax’ + 2 ¢ forall positive x, where a> (and b > 0, The value of the expression 27ab” cannot be less than fa) 4c" (b) 4e* () 8c @e Ans. (a) Solution As A.M. >G.M., we get 2 oy "2 (on 2x 2x wy! 7) b 4 2 = Dab? > 4c Example 37 If a, b and c ate three positive real numbers, then the minimum value of the expression be cta ath a + is a cS @t (b) 2 3 @6 Ans. (4) Solution We have 1(b a). [ba aGts Ras Ife a 3.12 Course in Mathematics for IIT-JEE Adding the above inequalities, we get bse seetybss) bre, baceb a Thus, the minimum value of the given expression era ath ( Famed coy fF min +2) I tans @ t nin + 5) Ans. (c) Solution We have t, = S,~S,.,¥n22 4, =f tH tn 949007 (n+ Bin-n+ DI = J lon —6n+ 2+ 900-1) + 13) = En? + 120+ 6)= (0417 Also, i, =5,=4= (141)? » Sire Lew Lint in t2)-1 = = Pas, Example 39 If 0 <8, @ < 7/2 and x= sin @y=S cos 6 = = DX 60s" (6+ &) cos" (#6), then = (a) ye +1 = yee (©) aye ~ ay = yz = ce Ans. (c) Solution We have 1 1 * Tosin®@ * costo? Also, cos (8+ 9) cos (8-9) and = cos’@ — sin’ @ = | As. 0 < cos? @< 1.0< sin? o< 1, wort = cGy-yt say or ye ay = ye-az Example 40° If ay.ds,.... 4, are in A.P. with common difference d # 0, then sum of the series sind [sec a, see ay + Sec ay See a +...4800 4, _) SEC 4) is (a) tan a, ~ tan a, (b) cot a, ~ cot a, (c) sec a, ~ see a, Ans. (a) (2) casec a, ~ cosec a Solution AS ay. dye dy. ott da, jd are in ALP, sin d [sec a, sec ay + see ay sec ay + sin(ay~as) Sa) 084, COSA; cosa, cosa = (tan a ~ tan ay) + (tam ay — tan dy) Fo (tan a, = tan ay) = tan a, ~ tan ay Example 41. Value of fim Len (a) ah (b) xia tc) 1 (d) none of these Ans. (b) Solution We have Ort))-Qr-)) 1+Qr¢DQr=d * ar tan! (2r-+ 1) = tan Qr =1) ¥ fan "2r+t)tan"2r~ »} a tan"! Qn + 1) ~ mid £ RZ 204 4 as Example 42. Let /, = ftan" x dx. Then fp + ly. Is o Ugg My lao lg + bs (a A (©) HP. Ans. (©) fare in b) GP. () none of these Solution We have for r 21, Ld, 4 2 2=f tan’ x (1+ tan? x) de 1 ey = [tan x sec? x de = — +i Thus, the given sequence becomes, which is clearly an Example 43 1 Then tim & a is ae le (2 Ans. (a) Solution We have, for n2 1, (b) 3 © 32 (a) 6 = fas De d-ne = tamed. me ft ad-shl-ih = in Bh sin a{t--15) =20-9=2 Example 44. If #, = ledeatd, then value of n syntededs.a% (@) H,+n (b) 2n-H, © (@-+H, — @) Hy + 2n Ans. (b) Solution 5, a-ns(2-2}+ 4(2- ) 2 n =n Hy. Progressions 3,13 Example 45. Sum to n terms of to) 2n- () 2 id) 2 1 A. ord mn-i+t Ans. (0) Solution MULTIPLE CORRECT CHOICE TYPE Example 46 Ifa, b.¢ are in H.P.. then the expression 1 (d) none of these, Ans. (a), (b). (©) Solution Asa, b,c are in HLP., Va, 1/b, Ve are in AP. Let the common difference of this A.P. be d Now, Example 47 terms of a G.P. with common ratio r, the value of r for which the inequality b, > 4, ~ 3b, holds is given by If b,, by, by (by > 0) are three successive fa) r>3 (@ r=35 Ans. (a), (b), (). (d) ib) r 2 => an >n. Example $1 Sum ton 742077 fa) () Set wn +2) @) Ans, (2). (b) terms of the series P4244) 454 1O +. is n(n + 1)? when n is even ) when in is odd ) when n is odd n(n + 29° when nis even, Solution Let n= 2m. then s 1 = Qm) Qm 6 mm +1 1 > = Snel? When n = 2m ~ 1, then S=m(2m + 1~ Pa Pat + my) +P eae soe + mp) +i Gms I+ £ mim m+) D [4m + 1 + 2m + 2) ~ 22my* am vin © nt =mam— =k rine. Example 52. If a. b. HLP,, then fa azbee are in A.P. and a?, b°, ¢? are in 0) a,b, ~ + eae in GP. (©) a, b, © are in HP. @ ~ ha.d.6 are in GP. Ans, (a), (b). (A) Solution From the given condition, w=are 22 and b= Progressions 3.15 Eliminating b, we get ate)! _ 2a% ()-35 @ +2) + due (a +A) ~ Bae? = 0 => [a+ cP + 2ae] (a ~ oF a > a(vras Jae 0 > a 0 or & I amcwegeta= i Wa = 5 ac. then either ab, be are in GP or = Fa, b, © are in GP. Example $3 In the nth row of the triangle 2 3 (@) Last tem = 5 n(n + 1) (b) First term = 5G? =n #2) (©) Sum = > n(n? + 1) (@) Sum, semen | Ans. (a), (b), (e) Solution Last term of nth row S142¢34..405 a(n +1) 2 As terms in the nth row forms an A.P. with common difference 1, first term — = last term ~ (n= 1) (1) =f nm+l-nel (=n +2) 4 ata [et _nenetor Sum = olde ma2y9 ho +n] 2 u 2 1 2 1 nes) 3.16 Course in Mathematics for IT-JEE, +9 upto e. then Example $4 If $= 1 37 fa) (0.25) 8) = 4 (©) (0,008)"5 = 4 Ans. (a), (b) (b) (0.008)"#5 = 8 (d) (0.252) = 8 y3__t tion We have $= —— = = Solution We have S= T= = 5 Thus, (0.25)? Example $5 Let a, = (111 oo names (a) dgy> is not prime. (©) Gggq is not prime Ans. (a), (0), (6). (A) Solution AS ay2.do5, and dy are divisible by 3, none of them is prime. For ay), we have (b) ays, 18 not prime, (A) ayy is not prime. 1 1 ig == (99... 9) = —(10" -1 a = 5 y= 5 ) 1 times qo’)! = 1 107-1 = a0y" y= 1-1] | 9 (eto) a-| [= | = (107)? + (107)! +... #107 + 1 [10+ 10+... +104 TT =2 dg is not prime. Example $6 [f'2.and 31 appear as two terms in an A.P., then (a) common difference of the A.P. is a rational number all the terms of the A.P. must be rational. all the terms of the A.P. must be integers, sum to any finite number of terms of the A.P. must be rational. Ans. (a), (6). (0) Solution Let d be the common difference and a, (b) «) @ a, = 31, then (n - mid = 2a = d = which is» nm rational. Also a, - a,= (r — mid = a, is rational. As each term is rational, sum to any finite number of terms must be rational, Example 57 Lot a and h be two positive real numbers. Suppose A), As are two arithmetic means; G,, G, are two geometric means and Hy. Hy are two harmonic means between « and b; then iG, Athy a BS 2 HH, H+, 2fa by (by =[242 alé+2) © _ Sab (Qa +b)(a+ 2b) @ 2 Ath Ans. (a), (b). (ey Solution A= a+ 5 (b-anAr=b+ > -a) % ALKAy ene y x Sinitay. 6 = o(2) 6: of) ss and Now, os ath At “ah GG, 5 GG, AMA, HH, Wy +H Now, _ _Sab(a+by (a+ 2a +b) Bb) + Sab ab Thus, a2), bea Example 58 Let three termy of a non-constant positive GP. be the sides of a triangle, If ris the common ratio of the G.P. then (a) L 1 then car’ is the greatest term. 6 a@Pcatarae-r-1<0 (b) 2sin Ie rd (d) none of these = H-WB) ttcrc LSen = IncaseO0 are -4 B+) orr> Lis Thus, 2sin 18%2anda=2'+|.b=14+2+.42", then (a) hef (a,b) = 1 (b) a is prime implies is composite (©) a and 6 cannot be simultaneously prime (d) none of these Ans. (a), (b), (©) Solution If p isa prime that divides both a and b, then p is odd and pl(a ~ 6) or pl2. Example 60 Leta and & be two natural numbers greater than 1. If a ~ 1 is prime then f@a=2 (b) & is prime {c) k = 2" for some m € N (d) none of these Ans. (a), (b) Solution a ~1=(a~\yn where omsad®'+a'?+..+a+1 As ak>tm>t. Since a‘ ~ 1 is prime, a - 1 = 1 or a= 2. Ifk is not prime, the k= rs where r, 5 > 1 In this case 2! ~ 1 is not prime. A contradiction. Progressions 3. Example 61 Let F, = 2°" +1, wheren €N. (a) Last digit of F, is 7 Von 2 2. (b) hef (F, Fy) = 1 Yon em. (©) F, ~ 2 is divisible by F,, if n > m. (d) none of these Ans. (a), (b), (c) Solution For n > 1, last digit of 2° is 6. Let n>m,a= 2" and k= 2", then F,-2=a'-1=0F, where asa‘ "+a? +. 4a-1>1 Any prime p dividing both F,, and F,, must be odd, but then p must divide 2. Example 62 Let a(n) = 1 @ (b) @Q2n) <1 Vin (©) @Q2Qn) 205 Vn (6) 05 < ain) <1 Yon Ans. (a), (0), (6) and a(2n) 2L24yn. Qn 2 (d) is not correct as a1 3.18 Course in Mathematics for INT JEE Example 63 Let @(7) be as in Example 62, 28 = a(n) % Where Pix da € Nand hef (py. 4,) (a) 1979 \Pisiy (©) gy is even for each > 1 (©) Py is odd (d) none of these Ans. (a), (b), (©) 1 19) = (1318) + —— Solution ar(1319) = @(1318) + 7 ety 1318 1319 [See Example 62) (rine) (asia) ett 660 661 = 1979 2 where p, ge N. 4 = @rsy9 = 1979 pay. As 1979 is a prime and all the factors of q are smaller than 1979, 1979 must divide ps0. You may check yourself that form > I, p, is odd and q,, is even. Example 64 Let /,= f° x" tan"! x de. If Gy Lyya + Oy by = Cy V2 Athen (@) ay, ay, ay (b) By, by, by (©) cys C35 C56 ae in HP, are in AP. no @ Fra. Ans. (a), (b), (A) Solution Iygg= —= ne = (2+ Dyer Similarly, are in AP. | 5 FD hg tnt Dl rE fla, #1 a ee 2 x 1 saan eI, d, one lege Za 2 nal Example 65 ‘The numbers 1, 5, 25 can be three terms (not necessarily consecutive) of (a) at least one A.P, (b) at least one G.P. (©) infinite number of A.P.'s (A) infinite number of Ans. (a), (b), (¢). (d) Solution Let 1, 5, 25 be the pth, qth, rth terms of an A.P, with common difference d, then (q-pM: 1 and (7 ~ pyd= = Kisay) + 6k where & is any natural number. Let 1, 5, 25 be the pth, gth, rth terms of a G.P., with common ratio R, then RO 25 r-p=4y-2p => r+p- ‘There exist infinitely many triplets of natural numbers satisfying this relation. ‘SECTION II ‘COMPREHENSION TYPE ROP Paragraph for Question Nos. 66 to 70 ‘Sum of the following three series is given » = log 2 a) | “WR m x Matsa a 2 Example 66 Sum of the series 1 1 1 } 1-2 ++ tne NS) 7)" ADAH is (a) AZ) I= 1 (6) RA MA YrRId hd Progressions 3.19 Example 67 Sum of the series is 69, Multiply (1) by ; ‘and subtract from (3). fa) J. log 2 (b) £ log 2 4 6 | 70. ©) i log 2 (@) log 4 Example 69 Sum of the series ote . upto | Paragraph for Question Nos. 71 to 75 * | Given a sequence f,. f, .. if its possible to find a function © ai z FCP) such that @s log V2 oF + logv2 Lake Raji (c) m~ log 2 (d) w+ log 2 then Example 70 Sum of the series ¢ Xs, =Ant+ b-f) ( 1 i) (5 1 i) (3 L et jes |a[oooe ele 2473 6 BF \S 0 | Example 71 Sum of the series is | (@) + tog 2 by + tog 2 1 ae" 3 ic) log 4 (d) #~ log 2 | (2 ib) (ce) 12 dd) V4 Ans, 66. (€), 67. (b), 68. (a), 69. (a), 70. (a) Example 72 If u; = 1, uy. = 2u, + 1, then u,,, equals Solution 66. ‘The given series can be written as (241 (1 -(f-pei-2 Lou ) x | (©) 2-2 (a) 2-2 35°79 HW O13 | 7: Si —_— 67. Add series (2) and (3). Example 73. Sum of the lame is 68. ge —— __.. 1 12 V4 @) 18 (4r=3)(4r = 2)(4r = Example 74 Sum of the series Srir+1)(r +2) is rl @ 40%) th) 6 CQ) (an ) tn 3.20. Course in Mathematics for IIT-JEE Example 75 Sum of the series}. r(r +3) (r+ 6) is ‘= @ jn +3 (n+ 9 (b) a! + In? + 200 © fra +3) ($5) (0 +9) (@) none of these Ans. 71. (c), 72. (b), 73. (c), 74 (b), 75. (d) Solution 71 Sar -1 Now take limit as 1 = 2 Hyg He = Aly yd * (yt ~ Mea) (u, ba tn) = Alga = My + DEH gy + 2 = yg 4 4 © My + 2% ssn + De 2 arty 1 73, Let i7- —— rireD(r42) = n= 1 = (r+ 3, (r+ Dr +2) “ = tr + Die = Ut eI = 204. = 4 at, = . 1 1 = 23, - +-—1_ 2 2 (ae D(n+2) I 1 - 2 2 74, Let f= rt 42) 3+ 3H, - ri = 0 > 4, = Ty ~~ De = a¥y, = rt, = 4! (PC). m (r+k-1) Tip: In general Sop(r + 1) m = ime $V oo (ne 75.4,=7(7+ 3) (r+) r{P + 9r+ 18) rl(rt 1) (#2) +6 (r+ 1) + 10) (rt 1) (r+ 2) + 6r(r+ 1) + 10r Now, use the above tip. Paragraph for Question Nos. 76 to 80 For k,n € N, we define Bk, n) = 1.2.3.4 +234... (K+) tectnint (nek 1, Sy(@)=n and Sy(n) = 1h #24 + nk To obtain value of B(k, m), we rewrite B(k, 2) as follows: tk, my = Kt [(i)4() (42) + "r)] in+k) (nt) = nine kel Example 76 S,(n) + S\(n) equals () BQ, ©) $20, m © dae, n) (@) none of these Example 77 S,(n) + 3S(n) equals * @) BG, n) (b) B(3, n) ~ 2502, n) (©) BG, n) - 2B(1,n) (d) BG, a) + 2B(1, n) Example 78 (*1') syn) + (*3') Sam). (2) sums (E21) Sum equals @) (i+ Df ©) nb-(n- 1 () (n+ DF =1 @) (n+ DS (n- 1 Example 79 SA* equals a fa) Eoin +1) n+ 1) Gn? + 20 = 1) tb) Ent + 1) 2n# 1) Gr? # 30 — 1) © Lins NQn+HG (d) none of these. Example 80S kk + Dk + a equals 1 (a) 55 mint Din 24+ 3) An + 15) 1 (b) min + 1) (n+ 2) 0 + 3) Bn + 12) apn Die De GH + fe) Sante + 1) n+ 2) On + 3) Qn + 13) 1 (a) spr + Dn De Dns 1d) Ans. 76, (a), 77. (c), 78. (b), 79, (d), 80. (a) Solution 76. Syn) + Sy) = de+ Ye= Shas y oi i = BQ, n). TT. Syn) + 3840) = Seu«ry= Dees eer-2 2Ee = B(3,n)~ 281m) 78. (4) + (SS) +. + (Sy + (Et) sven (2)e+()] = Sie+ I-A sine lod pF =se mm Progressions 3.21 | But by Example 78 5 88 = + D8=t- (3) 8,00 - (3) = (i)S\p-n = +1 ae { | | | arene games | wafede ede] 2 3" 6 3 (6n* + 15n° + 107? — 1) = FD On+ 1) Gr 43D. | Alternative Solution | Write | Kis k(k+ 1) (K+ 2) (k+ 3) + ak (kt 1) (e+ 2) + bk(k + 1) + ok | Put &=-1,~2,-3 to obtain lbs7,a=-6 | Thus, Syn) = BOA, n) ~ ‘6BG, n) + 7B(2, n) - BCI, n) = inns Dent Dine dined) = Sant 1) n+ 2) (043) + Zain + tn 2)— Lain + pits + D6 + 2) (n+ 3) in #8) 45 (n + 2) (n+ 3) + 70K + 2) ~ 15] Spite 1G? +9? +n — 1) = Spits # 2n# 1) Gn? + an D, 80. K+ nde rFenbenwen EE” = kk + 1) +2) Lhd +) Kk + 27 = BEA, a) ~ BG, 0) | > a | 1 = gin + 1) (n+ 2) n+ 3) a dy = fan + 1) 1 +2) (+3) nin + 1) (n+ 2) (a + 3) Gn + 1) 3.22, Course in Mathematics for 1T-JEE Paragraph for Question Nos. 81 to 85 Let V, denote the sum of first r terms of an arithmetic progression (A.P.) whose first term is r and the common difference is (2r~ 1). Let T,=V,,4-V,-2: 1 =r, plas @=T, 4, Wa X,=3% for r=, The sum V, + ¥, +. 1 TW, 81. +V, @) Sam +1) Gr =n+ dD 0) FO + DOR +m +2) ©) pm? ned (@ Fen =a +3) T, is always: (a) an odd number (b) an even number (©) a prime number (@) a composite number Which one of the following is a correct statement? (@) Q). Oy Qs... are in AP. with common difference 5. ©) Q. Os 0, difference 6. © OQ, Os Qs difference 11. (d) Q, = Q:= 03 =. Wy, Woy Wy... are in (a) AL (c) HP. Xi, Xy Xp (a) AJ (c) HP. Ans. 81, (b) are in AP. ith common are in A.P. with common (b) GP. (d) none of these are in (b) G.P, {d) none of these. 83. (b) 84. (c) 85. (b). 82. (d) Solution 81. V, n+ -D2r-D) 2 lope = 50h -P +n 2 i “Sde 5d 1 = inte 4 Anes bOned l + omni) 4 Kan + DBnw + = Qn+ 43) n(n + 1) Gn? +n +2) 82. 7,2 V,4)~V,-2 stirs Dr - Luee Pr] + ieen-2 2 =GPsare0- Lorene t-2 = 3r42r-1=Gr- Wir) 2. Tis always « composite number. Thai T= Mert WP =r} + rt De} M2re 1) +2 83. +6r 2+ Oy Qa.» forms an A.P. with common differ ence 6. et ep rete T, Alrtt) (3r-Hlr +l)” a(r +t) W,. Wy, Wy. tre in HAP. ASQ). On. Oy. are in ALP. Xj. Xq, Ny. are in G.P. Paragraph for Question Nos. 86 to 88 Let Aj, G,. Hy denote the arithmetic, geometric and harmonic means, respectively of two distinct positive numbers. For n 22. let A, ; and #1, has arithmetic, ‘geometric and harmonic means as A,.. G,. H,, respectively. 86. Which of the following statements is a correct statement? (a) G, > G, (d) G, < G,< Gy < and G, > G,> G> Which of the following statements is correct? (a) Ay > Ay > (b) Ay < Ap < Ay < (©) Ay > Ay > Ay > and Ay < Ay < Ay <> (d) Ay < Ay < Ag <> and Ay > Ay > Ay 88, Which of the following is a statements is correct? (a) Hy > Hy > Hy > (b) Hy < Hy < Hy < a You have either reached 2 page thts unevalale forvening or reached your ieving tit for this book. a You have either reached 2 page thts unevalale forvening or reached your ieving tit for this book. a You have either reached 2 page thts unevalale forvening or reached your ieving tit for this book. a You have either reached 2 page thts unevalale forvening or reached your ieving tit for this book. a You have either reached 2 page thts unevalale forvening or reached your ieving tit for this book. 3.28 Course in Mathematics for IT-IEE (a) Ysin® @ cos 6= = (b) Stan a ~ Lsin* 6 cos” = ~~ Leos" B sin” O= Example 104 Let a), a3 dy, progression such that @) I=sin? @ cos‘ @ . be a geometric login (ay) = 7 and logiola,) = for two fixed positive integer m and 1, with m 0,r>Oandr2 1. and As AS bysaby=al +n by=al trey bezaltrerer), by — by = are ar?=b,—b, by, bas bys by are notin ALP. tot bb by, by, bys by cannot be in HP. —_—s at all+7) by - a Example 106 Statement-1: There exists no A.P. whose three terms are V3, VS and V7 I. 1, and f, are three distinct terms of an is.a rational number. Ans. (a) Solution Suppose J3, V3 and J7 are the pth, qth and rth terms of an A.P. whose common difference is d, then and 1-=r-p)d 4,=(q-p)d = 22 which is a rational numbers. q-P vi-N3 is a rational numbers, is rational @ V35—V15+V21 is rational, say r. Now, 35 -V15 + V2 = vis - Vi = V35 -r Squaring both sides, we get 15 +212) 6) V35 = 3547 => ¥35 is rational, ‘This is a contradiction Hence, 3,5 and V7 cannot be three terms of an A.P. Example 107 Statement-1: 7 Because 1 1 2s + MQ) Ans. (c) Solution Let Qr-NQr+) foal = Dare) 1 * @r-DGreh + 3x5) Progressions 3.29 enn +1) = 1, = 2 sone Statement-1 is true. But statement-2 is false since y 1 SQr-nerset Example 108 Statement-1: For & 20, let Sny= Vee tal, then Sale = 55 mln 1) Qn4 1) Gr? +n 41) ket kal Statement-2: |" 7 |sun+[" }si A) (‘3 sine (**") sien = cn 1 5) HOE 1) Se Ans, (d) Solution We have (ra tate (eee a (‘1") . dross rt r ‘ kel Stes yay mat =( Foon (ES\s yin) Fel) seme (tt!) sam Hg JO Lc) ‘Statement-2 is uue. That statement-1 is false can be checked by putting n= 4 in the statement -2. In fact > inane! 3.30 Course in Mathematics for IT-IEE Sdn) = 3 nt + 1) Qn + 1) Gn? + 3n = 1) See solution to Example 79. Example 109 Statement-1: If n is odd, 17 + (3) (2%) +3? + (3) (4) + S? +... upto m terms » nin 1) 4n—1) 6 ‘Statement-2: If n is even, 1 + (3) (27) +3? + (3) (4) +S? +... upto n terms. = Ems cane 5) Ans. (a) Solution If n = 2m, then statement-2 becomes 12+ BV Q4 FH +B) G4 S +... Hm)? = £ 2m 2m+1) m+) 3 P48) 2)4P +3) G)4S4...432m- VF = = (2m) (2m + 1) (Bm + 5) ~ 32m)? aie 2) [(2m + 1) (Bm + 5) — 36 m} 0 m (16m? — 18m +5} - $m Qm ~1) (8m~5) For n= 2m ~ 1, we get 1 + (3) (2?) +3? + (3) (4) +5? +... upto n terms - 4(*24) mcan=1)= Enns Dn Example 110 Statement-1: If x > 1, the sum to infinite number of medefet}ot Statement-2: If 0 < y < 1, the sum of the series Le3y + Sy? +I +... is I+y. (1-y/ Ans. (a) Solution Let Sel+3y+ Ween a) ys y+ 3 + Sy to @Q) Subtracting (2) from (1) we get (l= y)S = 1 + 2y + 2p? + dye SINGLE CORRECT CHOICE TYPE 1, Given a sequence of four numbers such that the first three are in G.P. and the last three are in A.P. with common difference 6. If the first and the fourth number are equal, then common ratio of the GP. is (a) -2 (b) 2 (©) 3 (d) -3 2. The sum of the series 7 19 37 re ~ upto is (a) 3 (b) 2 ft (d) LS 3. fis 2 sm +n 40° +2 +...40°) ad Sn’ equals (a) 9/10 (b) 1/10 (c) 2 (d) 13 Low 4. lim) —) °° juals tafe") 1 1 1 — (b) — — d) 0 ot wf of w 5S. Let Sn) = Yor* then tim SLMS) ~ S46 Lt 's(m) a You have either reached 2 page thts unevalale forvening or reached your ieving tit for this book. 3.32 Course in Mathematics for ITT-JEE 20. Lets, = 5 2n? + 9n + 13), then >. = equals (a) din » (b> jm +3) ©) +e @v 21. Leta, = LI The remainder when aja is divided by 271 is @23 2% @27 Ww 22. Three arithmetic means, three geometric means and three harmonic means are inserted between 1 and 5. The cubic equation whose roots are 3rd AM., 2nd G.M. and Ist HM. is @ o- Lars WS) P+ 5-58 0 we bare WEP a s+ Bye - 5 =0 © 4° - 21+ WS) 8 + 5 + 25) VS =0 (d) none of these 23. If r, denotes the rth term of an A.P., and f= 1/4, 1, = I/p, then which of the following is a root of the equation (p+ 2q4 30) <7 + (q+ 2r~3p)x + (r+ 2p-3q)=0 @ t, (b) 1, (©) tpg @ tony 24. Ifthe pth term of an A.P. is Lg and qth term is 1/p and sum of pg terms is 25pq, then p and q are con- nected by (@) paseo (b) p=4q-1 (d) none of these ©) pq=4+p isa polynomial in x, then must be (a) odd (b) even (c) greater than 5 (d) tess than 5 ‘equals (b) 100 iis (d) none of these 28. 29. 30, 31 33. 34 35. 36. . Let S, (a) = 1 +2 =a) + 3(1 = @) (1 2a) + 4(1 a) (1 ~ 2a) (1 = 3a) +... upto m terms, then 1 ce (5) equals (a) 10099 (b) 1199 (©) 200 id) 99 ~e0s2nx Toe them ey dy ae in (a) AP. (b) GP. (c) HP. (d) none of these fs,= i= Emin 1) (n+) Vn21, then 7 ane lim }— is mo ratte (@ 1 ©3732 @2 @sn If unit's digit of 1? + 2? +... +n? is 5, then unit's digit of 11° +2! +... +n'Pis @t (b) 3 @S @7 Four geometric means are inserted between 2"! ~ 1 and 2'' + 1. The product of these geometric means is (a) 24-1 (©) 4-2 41 b) 24-2 41 (d) none of these 1 Lets, es «then value Qay of Sys is (a) 25/54 (b) 25/53 {c) 25/51 (d) 1/2 Suppose 1,= 12 +2? +... 4°. Let hantathe Laine Dansk then value of k is faynet (b) 2n+ (©) 3n-1 @n Leta, b,c >Oand a, b,c be in AP. Ifa? b,c are in GP., then @ jas Sum of the infinite series 1 3 3S 7 + (3X7 IRIS) (by a4 B= ct (d) none of these 3°)” GAD is @12 1B 4 4 44s La $= 245450 1919 19° (©) 6 (d) 14 upto «=, Then S is equal to a You have either reached 2 page thts unevalale forvening or reached your ieving tit for this book. a You have either reached 2 page thts unevalale forvening or reached your ieving tit for this book. a You have either reached 2 page thts unevalale forvening or reached your ieving tit for this book. a You have either reached 2 page thts unevalale forvening or reached your ieving tit for this book. a You have either reached 2 page thts unevalale forvening or reached your ieving tit for this book. a You have either reached 2 page thts unevalale forvening or reached your ieving tit for this book. a You have either reached 2 page thts unevalale forvening or reached your ieving tit for this book. “ole 35. 36. 3. ragraph for Question Nos. 36 to 40 Course in Mathematics for HT-JEE Wa, b, ¢ are in GP. as well as sy... 4 ae in | GP. then pg. rate in | (a) AP. thy GP. fc) HP. a x, are in G.P., then its common ratio is © |& « {2 ap bq Hxyoxs.ay-x) are in AP., with common difference | d, then a _ b-4ad _ Sad? = 2hd +e Se - # q r (ey Go bred _ a? P 4 r () fw brad _ ad poo r none of these @ Arman is to receive Rs P, at the end of ith year for m ‘years. Assume that the tate of interest is Rs r per rupee per year and the interest is compounded annu- ally. IFP,= PV i, then present value of income stream is, @ 2 eden r w ? Wer-d4+n"] fe) PIL- ery" (d) none of these If P= PV é and n>, then present value of the income stream is (a) P (b) 2P (©) Pir (@) P+ Pir If P, = iP Wi, then present value of the income stream is Pier) (a) -de+ery"| a (by POF yg yy OP r rier)" POF) easy - +n () none of these 39, If P= iPY jand n>, then present value of the income stream is (a Par try 2 (©) rP Ww re 40. IP, = iP Y ‘and m ><», then value of r for which present value of income stream is 3P is @O1 (by 02 W038 Wh OS SECTION IV INTEGER ANSWER TYPE 41, Three numbers a,b, care chosen from the numbers 1,2, 3, 100. Find the number of ways of choos- ing. a. b, © so that these are in A.P. 42, Find the largest positive term of the A.P. whose first two terms are 2/5 and 12/22, k eR aximum value of the function ere lo andr im IWS sa + ar tar +, find JS Find the value of 2551 § where $= a cl 45. Ifa= 29 then find [a], where [x] denoted the = greatest integer $a, 46. Let a, = LI, Find the number of primes in the SequUENCE dy, 5 ss. 47. ind the sum of the series 5 us Aaah ay SECTION V MATRIX MATCH TYPE n= Land [x] denotes the greatest integer $x. a) [S+a,|+ftsa]eft san] (2 so)" {50° soo" 0b) La, + ayggl + lay + cag) #2 + ajo + a1 @t ©) la, tay Fay (3 @) lay tay tay 2 (0 a You have either reached 2 page thts unevalale forvening or reached your ieving tit for this book. a You have either reached 2 page thts unevalale forvening or reached your ieving tit for this book. a You have either reached 2 page thts unevalale forvening or reached your ieving tit for this book. a You have either reached 2 page thts unevalale forvening or reached your ieving tit for this book. a You have either reached 2 page thts unevalale forvening or reached your ieving tit for this book. a You have either reached 2 page thts unevalale forvening or reached your ieving tit for this book. a You have either reached 2 page thts unevalale forvening or reached your ieving tit for this book. a You have either reached 2 page thts unevalale forvening or reached your ieving tit for this book. a You have either reached 2 page thts unevalale forvening or reached your ieving tit for this book. a You have either reached 2 page thts unevalale forvening or reached your ieving tit for this book. a You have either reached 2 page thts unevalale forvening or reached your ieving tit for this book. 3.52 Course in Mathematics for HTJEE n. a(Seter] as fo) r (beter e z Divide (2) by (1) to obtain d- From (2) and (3) GB) 2a = (-2}-4 Putting this in (2) we get 2? SoM (See) = r = (-l) +3 4¢ @ Baia elb 3 = 9526") = nes. = le loar3=0 => @= 1/4, 34. Diagonal of (n + 1th square = V2 a, , Now a, = V2 age, = ans => 2a? ~2a+ lar > 12. Psd P,= 4a, = (1 - 2a + 2a°y “(y” V8 _ V2(VE+ V5) BG 3 = $(++si0). Since a, x, y, z and b are in A.P., we have atbaxeczedy Alo xty+c=15 = 2ytyels = yet or y= é a+b=10 a Since a, @% B, 7 and bare in H.P., we have a You have either reached 2 page thts unevalale forvening or reached your ieving tit for this book. a You have either reached 2 page thts unevalale forvening or reached your ieving tit for this book. a You have either reached 2 page thts unevalale forvening or reached your ieving tit for this book. Cuarter 4 LOGARITHM 4.1 EXPONENTIAL FUNCTION For x €R, we define It can be proved for every x € R, the number e* is a finite number. We call e* the exponential function. Properties > ex 9. weer € is defined for each x € R. e* > 0 for all xe R. esl Ifa, be Rea 0, a # 1) of the number.x(x>0) is the number y such that a” = x. Thus y= log ¢x means a’ = x. For a= 10, log jo is called the ‘common logarithm and for a= e, log ,x is known as natural logarithm. Properties 1. log, 1=0,a>0,a#1. 2. loga=l,a>0,a%1. 3. loge’ =x V xe Rand ee =x Vx>Oie, log, x is the inverse function of exponential function e*, log , (ay) = log, x + log, y for all x, y > 0 and a>0,a41 log, (I/x) =~ logy x Vx>0,a>0.a41. 6. 12, log, (x/y) = log, x — log, y, x, y > 0, a > 0, a#l. If ae Rand x > 0, then we define Yetog,2)" = log x? = y log x VW x>0, ye R. (Change of base formula) If a > 0, a # 1 then vai hers log, x = 8%. In fact if a, b,c > 0 and a log. a,c #1 then log, b= =, log a If 0 1 then log x > 0 and if 0 1 and 0 log, y and a* > a’. See Fig. 4.1 and Fig. 4.2. y wat.e>1 yologss ” yer SS yeahocacl ro" yotoniae yooniae Fig. 44 Fig. 42 Boe For il < 1, log (14x) =~ 4 orb < 1, log (1 +x) 22-4 ‘ log (1-3) = {aise = +] and Jafeed ete] a You have either reached 2 page thts unevalale forvening or reached your ieving tit for this book. a You have either reached 2 page thts unevalale forvening or reached your ieving tit for this book. a You have either reached 2 page thts unevalale forvening or reached your ieving tit for this book. 1 Togsyn 1 log, 53. fa) (b) 1 © (@) none of these Ans. (@) Solution ‘The given expression is equal to log, 2 + log, 3+... + log, 53 53) = log, 33! = — logy Example 20. The number of values of x [0, nz], n€ 1 that satisfy loguin « (1 + cos x)= 2 is = log, (2.3 ... (a) 0 (b) n ©) mH (d) none of these Ans. (a) Solution “The equation is meaningful if Isin xt #0, 1 and L+cos x #0 sox 4 km k=O, 1a k=O, 1, * merken %, n-1. Now, login a (I+ 608 x) = 2 ° 1 + cos x = Isin x > (1 + cas x) (cos x) = 0 oe cos x = 0 of cos x = So. cos x= 0 => = 2k + 1) AQ. there is no x which satisfy the given equation. ‘SECTION I MULTIPLE CORRECT CHOICE TYPE Example 21 The sct of all x satisfying 4° -(9)2"*? +8 <0 consists of (a) infinitely many (b) finitely many points from the set of all natural numbers, (©) finitely many points from the set of all inte- gers (@) exactly two integers Ans. (©), (@) Solution Setting 2" *? = 1, the original equation reduces toP ~91+8 20912 1,81 f= 1 then 2°? =e + 2= 0 but this has no solution. If 2" *? = 8 then x7 +2=3 Sxeel ees Example 22. The equation has on Logarithm | 4.5 (a) exactly three real solution (b) at least one real solution (©) exactly one irrational solution (4) complex roots Ans. (a), (b), (c) Solution Taking logarithm of both the sides, we get 9 3 (cos ay “| tors 5) logs x = 3 2 (logs x) = 9(logs x)? + 10 logy x - 3 = 0 > (logs x — 1) (logs x ~ 3) (2 logy x - 1) 0 © logs x = 1, logs x = 3, 2 logs x = 1 ° x=5, 125, V8. Example 23 If log, (3°~? + 7) = 2 + log, (°~' + 1) then x equals fa) 0 (1 2 (d) none of these Ans, (0), (©) Solution The given expression can be wi log, (3°*~? +7) = log, 43%"! + 1) 3-24.72 4%" 41) Bl 4G" 4+3=0 Br t= NG t=3) sists 3 Ox- 1 re xo Hdudoug Example 24 Solution of og. 5 6++8 1OBa +2443 OF — 2x) = Os (a) a natural number (b) a negative integer @-1 (@) none of these Ans. (b), (c) Solution We must have x? + 6x +8 > 0, 20° +2x+3>0 and x? + 648 41,2074 204341 = xeGe-JUCZoxe-3202 and x°-2x>Qi.e.x€ (- 9,0) U (2,09) ‘The given equation can be written as logan s2e+3) GF = 2x) = 1 > B= Oe = WP 4 2043 Ee Pedre3s0are-1-3 x= ~3 does not satisfy (1) sox= ~ 1 Example 25%" > 5 implies f@) xe @,. =) () x € 0, 1/5) U5, ») a You have either reached 2 page thts unevalale forvening or reached your ieving tit for this book. a You have either reached 2 page thts unevalale forvening or reached your ieving tit for this book. a You have either reached 2 page thts unevalale forvening or reached your ieving tit for this book. a You have either reached 2 page thts unevalale forvening or reached your ieving tit for this book. a You have either reached 2 page thts unevalale forvening or reached your ieving tit for this book. a You have either reached 2 page thts unevalale forvening or reached your ieving tit for this book. a You have either reached 2 page thts unevalale forvening or reached your ieving tit for this book. a You have either reached 2 page thts unevalale forvening or reached your ieving tit for this book. a You have either reached 2 page thts unevalale forvening or reached your ieving tit for this book. a You have either reached 2 page thts unevalale forvening or reached your ieving tit for this book. a You have either reached 2 page thts unevalale forvening or reached your ieving tit for this book. a You have either reached 2 page thts unevalale forvening or reached your ieving tit for this book. a You have either reached 2 page thts unevalale forvening or reached your ieving tit for this book. a You have either reached 2 page thts unevalale forvening or reached your ieving tit for this book. a You have either reached 2 page thts unevalale forvening or reached your ieving tit for this book. a You have either reached 2 page thts unevalale forvening or reached your ieving tit for this book. a You have either reached 2 page thts unevalale forvening or reached your ieving tit for this book. a You have either reached 2 page thts unevalale forvening or reached your ieving tit for this book. a You have either reached 2 page thts unevalale forvening or reached your ieving tit for this book. a You have either reached 2 page thts unevalale forvening or reached your ieving tit for this book. a You have either reached 2 page thts unevalale forvening or reached your ieving tit for this book. a You have either reached 2 page thts unevalale forvening or reached your ieving tit for this book. a You have either reached 2 page thts unevalale forvening or reached your ieving tit for this book. 9.8. Course in Mathematies for IT-JEE ‘This, we can arrange V and N in = *P, = 20 ways Thus, the number of ways in which letters can be arranged is 4 x 20 = 80. Example 28 The number of integral points (integral point ‘means both the coordinates should be integer) that lie exactly in the interior of the triangle with vertices. (0, 0), (0, 21), (21,0) is (a) 133 (b) 190 Ans. (b) Solution ‘The integral points tying on the line also lie in the interior of AOAB are (1, 1), (1,2), there are 19 points lying on the line x= 1 (c) 233 (d) 105 8 (az) (0.0) 21.0) Fig. 5.1 Similarly, the number of integral points on the line x= 2 is 18, and on 17 and so on. The number of integral points on x= 19 is just 1. ‘Thus, number of integral points lying in the interior af the triangle is 19+ 184... +1 = 190, Example 29 The range of function F=7™'P, sis (@) (1, 2,3, 4) (b) (0, 1, 2,3, 4,5) (©) (0, 1, 2, 3} @) (1, 2,3, 4, 5, 6 ‘Ans. (c) Solution For ’~*P,_, to be defined, we must have 7-x21and x-320. => 38x56 *=3,4,5,6 Now, (3) = ‘Py = 1.f(4) =°P, =3, £(5)=P, =2 and f(6)=0 range of f = (0, 1, 2, 3}. Example 30 The number of ways of arranging letters of the word RACHIT so that the vowels are in alphabetical order is (a) 120 (b) 240 (e) 360 (d) 480 ‘Ans. (c) Solution We can arrange letters of the word RACHIT in 6! ways, Out of these exactly half, that is, there are ; (6! RACHIT so that vowels are in alphabetical order. 360 ways to arrange the letters of the word Example 31 If "~'C, = (K? - 3) ("C,, ,), then K | belongs to (a) {-V3, V3] (b) @, ~ 2) (co) 2, =} (@) (3, 21 Ans. (4) Solution "~'C= (K? -3) CC, ,\) = (n=) (+ INMn= rt riin=I=7) a a HL n But OSrsn-1 3 1Srt+isn = berth ey teweasi noon n = teasKesa n => 30 = -26K<-V3 or V9 100° = 2° = 1 (mod 7) => 1007 = (1008) = 1 (mod 7) ‘Thus, 100" = 2 (mod 7) 72. «m3 (mod 49) 02-3449 m where me I Putting this in the second equation, we obtain = 349m = 2 (nod 11) = Sm=$ (mod 11) = m= tomo 1) => m=1+llrwherere 1 w= 46 +5395 where se 1 73. 2008 cannot divide 2¢— J as 2x~ 1 is odd 74, Write the equation as Sv=— | (mod 17) and note that 102 + m = m (mod 17) cH 29, 1122 10, 13 = 11, 1142 12 (mod 29) Now check x = - 1 (mod 29) 7S. As hef (a, $)= 1. there exists ke N, 1 Sk S4, such that a = k (mod 5), But for 1S $4, 4% = J (mod 5) Paragraph for Question Nos. 76 to 80 For n€ N, let a(n) denote the sum of all divisors of nA number is said to be perfect if aur) = 2a. Example 76 Ife Nand (dy. dy... 4) is the set of all its divisors, then 1. is equal to «ay ou) fey oui thy non) (d) none of these Example 77 If p isa prime, then (p= 1) (p") equals fa) pt thy ph = er pet (d) none of these Example 78 ‘equals (a) otp) + ay) (e) qa) + pow Ip. q are 1wo distinet primes, then (pg) (b) ofp) og) (a) mone of these Example 79 Uf ris perfcet number. then + equals o (by) 2 (a) none of these Example 80) The number of natural numbers a for which ain) = 10. is fart tera ao (by 1 ty 2 «3 Ans. Th. (c). 77. tb). 78. (bi. 79. (b), 80. (a) Solution 76. Used {nes | n ain) me fs hee! 71. Daw) 7 =(p- DC spee et eptnt +p) 78. 79. lemueg ' ony Le om As aon sins 129. Bat for 1 Sk $9, 0(k) #10 SECTION IV INTEGER ANSWER TYPE 80. Example 8116" °'C,:" "P= 57: 16 find n. Ans. 19. Solution 7216 oot nt ney! Stun 6)! in 7 w 16 = ine Done tm t= > a You have either reached 2 page thts unevalale forvening or reached your ieving tit for this book. a You have either reached 2 page thts unevalale forvening or reached your ieving tit for this book. a You have either reached 2 page thts unevalale forvening or reached your ieving tit for this book. $.20 Course in Mathematics for UT-JEE, (b) The number of w 20-4 4017. () The prime tuctorization of 210 is 2-3-5 7. We can assign 2 to any of x). Ny. OF xy. That is, 2ean be assigned in + ways, Similarly, 3(5) [7] ean be aysigned in J ways. A variable which isnot as- cd any prime will be assigned value 1. Thus, unber of solutions of the given equation is (4) AY) A) = 286, Note that we cannot use 0 at any place, The re- quired number of five digit numbers is equal to the number of ways of selecting 5 digits out of 9 Wd) OXNKTXO 432 Example 98 Consider ull possible permutations of the letters of the word RA CHITIHCAR (a) The number of words (p) 56700 containing the word ACHIT is (b) The number of words beginning with RA and ending with AR is (e) The number of words in which vowels occur the oak places is (a) The mumber of words which the word IIT appears is Aw, oP 4 eS a oc I(r) (8) bI@M@O® © M@MOO 1MADOG) ion 16. (q) 630 (9) 45360, (s) 2520 Solution (x) We wan peemute ACHIT, R, R, 1, H.C, Ain F 22520 ways 2 . (b) We ean permute ©, C,H. HL. 1, T at 7 places in rn <=, = O80 ways te) 4 places out at 6 odd places in °C, ways and 4 arrange ALA. Lin. Therefore, vowels can arranged in Coy i ) ays. Remaining letters od RLR.C, C,H, HT ean be a ways. ‘Therefore, the desired number of ways ” us) 4 = 56700 (d) The number of words in which LT appears = number of ways of arranging HT, R, R.A, ALC. CL HOH = 2 45360 22212 SECTION VI REASONING TYPE | Example 99 Statement-1 40° “\ ( 40 \") + + rho r-thl attains maximum value when r= 50, The expression ‘2n) Statement-2: ( | is maximum when r= r Ans. (a) Solution We have © rQn=nt rét Since for 0S r-Sn— (o}{7) < (2r){an",) <~< (en) 2n Thus, ? ) is maximum when r= Next, (8)-(79)- a You have either reached 2 page thts unevalale forvening or reached your ieving tit for this book. a You have either reached 2 page thts unevalale forvening or reached your ieving tit for this book. a You have either reached 2 page thts unevalale forvening or reached your ieving tit for this book. 30. a. Course in Mathettaties for HTJEE Out of (0 white, 8 black and 6 red balls, the number of ways in which one or more balls can be selected 4s given by (a) O81 th) 69 {c) 679 (d) 692 Ais a set containing # elements, A subset P of A is chosen. The set A is reconstructed by replacing the clements of # A subset Q of A is again chosen. The number of ways of choosing P and so that PO contains exactly wo elements is tay 9x "Cy thy 3" ~ "C; fe) 2x °C, (d) “C3? |. Ten different letters of an alphabet are given, Words with five letters are formed from these given letters. ‘The number of words which have at least one of their letters repeated is (a) 69760, (b) 30240 (c) 99748 (d) none of these The number of ways in which we can select four numbers from I 10 30 so as to exclude every selection of four consecutive numbers is (a) 27378 (by 27405 (e) 27397 (@) none of these IEC, "Cy and "C,, are in ALP. the value of n can be 4b wo @s The letters of the word SURITI are writen in all possible orders and these words are written out as in a dictionary. Then the rank of the word SURITI is (a) 24 thy 24S te) 307) SIS . A candidate is required t answer 6 out of 10 questions which are divided into two groups, each containing S questions. He ix not permitted to attempt more than 4 questions from either group. ‘The number of different ways in which the candi- date can choose six questions is (a) 50 tb) 150) 200 (A) 250 Let A= [1.2.3 .. mh B= (a, b, cl, then the number of functions from A to B that are onto is (a) 3-30" — 1 tb) 3° 2 ©) 32-1 () (a= DE Mf a= "Cy, where me 2, then the value of “Cy is given by fay" 'C, doy "Cy "Ce, ay 3" 8 'C) There are white and black balls marked 1, 2, sem. The number of ways in which we can arrange these balls in a row so that neighbouring balls are of different colours is Qn)! any (a) at (by Quy! (e) 2a? (a) 32. 33. 36. 37. 38, 40. 42. 43. ‘The number of squares which we can form on a chessboard is fa) 64 (b) 160 ‘The number of ‘exactly once is @ 4) O (©) 7) 9) «26 ah 208 5 digit oumbers that contain 7 th) 87) (0) w@) Go) . The units digit of 17" + 1" 7" is f 1 ib) & wr? wo a en If i, cc," then value of nis (a) 3 thy 4 ot (a) wone of these At an election there are five candidates und three members are 0 be elected, and a voter may vote for any number of candidates not greater than the number to be elected. The number of ways which the person can vote is (a) 25 {b) 30 (ep 35, If m is odd and "CG, <"C, < "Cy < maximum possible value of r is ay 2-2" <"C,. then L 1 fa) s(n) aby SD 2 2 an (4) none of these ‘The number of arrangements of the letters of the word BANANA in which two N’s do not appear adjacently is (a) 40 (b) 60 (e) 80 id) 100 The number of positive integral solutions of yp My Xy Ay = 840 is (a) 3125 (b) 6125 (ay 7350 The number of ways we can put $ different balls in 5 different boxes such that at most three boxes: is empty, is equal to (a) S45 (by S*— 10 () °-5 (d) none of these ‘The number of ordered pairs of integers (x.y) satisfying the equation «° + Gx + y" = 4 is fe) 6250 fa) 2 tb) 4 6 WR ‘The number of the divisors of 20! is (a) 4140 (b) 41040 (©) 4204 id) 81650 ‘The letters of the word COCHIN are permuted and all the permutations are arranged in an alphabetical order as in an English disetionary, ‘The number of words that appear before the word COCHIN is (a) 360 th) 192 (©) 96 (a) a8 a You have either reached 2 page thts unevalale forvening or reached your ieving tit for this book. a You have either reached 2 page thts unevalale forvening or reached your ieving tit for this book. a You have either reached 2 page thts unevalale forvening or reached your ieving tit for this book. 5.28 98. # Course v9 Mathematics tor IT SEE A man is allowed to take at most to steps either forward or backward, Statement-1: The 1 be just two steps away 572 her at ways from hi which he will initial position gatement-2: The number of ways of in whieh he will be just Iwo steps forward fron: his initial posi- SECTION! SINGLE CORRECT CHOICE TYPE tion is ECP Phe number of S-digit numbers of distinet digits in which the middle digit iy the ka wo De > a ues is ter 10078 cal) none of these Let = 2° bon 201 then ef umm is (a 41 ub 208 fe) 12S 4d) 1023. ‘The ten’s digit of 23! x 25" ay fa)? tod wy a7 Loi = 100° 43, the suin of the digits in the deci- imal notation of 1 is fay 880, thy SOR WP NYS (A) NOT legral solutions of her of non-neg. Bis thy 1225 I js] denote the gre: the value of wis Wt mot] fnt2] [ae] [nes], 2yta { silt ta 50 thy 78 cer 100 wh 128 vf Hint -use |S )+} f= fay The aumber of eros al the end of (S"™"~ tt is ca) 100 wo 4 io 1 gin fer | a5! 26 ah | 6s! ~ 400) : 4 last digit of (ihe #2007 iy 1 (by 2 wT we 10. 14, ‘The remainder when Che 2 20078" is divided by 7 fay | th? ws Ww The number of ways of choosing three distinet rmumbers from the set {0 1.2... 29] $0 that sum fof the numbers is 30, is fay 78 (hy 80 8S (4) 98 a) toa ; SW yt tent yg =. them remainder when a is divided by 49 is a0 by (2 ian 3 . Let [a] = greatest integer $x. Two rational num- bers x and y which satisfy x + y= 2 and 20" 3 4 + Oe = 99 Hogs 13 = 0, then rv equals fa) 0 wt w2 (ay-t Find the number of ways of choosing two distinct ‘natural numbers < 100, such that they differ by at most ten is (ay 935 iby 940 1) 945, (d) 950 [Hint: Use the following diagram. eH tle bee ol athe = 100 a2 0,05b59] 1 w and Lic TE 14 4 | then § equals wersmen GC; (ay nal “) (er mal (ay na ‘The number of ways of di ibuting 10 identical aabjects among X persons is aw Cy aby VC, ter 10 aay 8” Let = 14454 71+ 0+ 4001, then ten’s digit of Sis ta) ih 2 (6 @)7 - A teal is torn from a book. If sum of the numbers: of the remaining pages is 4.851 the missing pages are (ap St 52 (©) 77, 78 (b) 49, 50 (d) 37, 38 a You have either reached 2 page thts unevalale forvening or reached your ieving tit for this book. a You have either reached 2 page thts unevalale forvening or reached your ieving tit for this book. a You have either reached 2 page thts unevalale forvening or reached your ieving tit for this book. $32 AR io 0 Course a Mathematics foe IT JEL done is sr 6 whe 40 fey 600 ey 3125 [9x9] Lighten wuests have ty he seatev, hull on each side of a long table, Four particular guests desire t0 sit particular side and three others on the other which the he 11991] There are four balls of different colours and four axes of the samc colours as those of the balls. The jhumiber of ways in which the hulls cout be placed fone each in a be. such that 3 ball does not go to a side. Detertnine the number of ways Sitting arrangements ean be box of is own vs {1992} wy ws vhen show that h, =" C,,5, [1992] Let pape FL 22 aY The number af equations of the tar ps ao 1S wr 7 #4 FO hava coal reds is wb ah x [1994) A commute of 12 is tw be formed from nine we way ways can this st five women have We be included int vight men, In how a be done if at k ina committee? In how many oF these committees 420) the wenn are i mapority? 49 tlhe me majority [19941 Let u and & be positive integers such that a 2 bik ats The number of solutions (6)... he 4, 2 A.all integers. satisfying x) + nis 11996} und nine boys ean be ‘nunabered seats, 3 tty In how many ways three girls cated ia Wo vans each havin in tront and 4 at the back? Hew many seating are rangements sire possible if & girls sit together ina back row on adjacent seks! Now, fall seating are rangements are cqually likely, what ts the probabil- ity of 3 girls sitting together in a back row on adja 11996] Number of divisors of the form 4 4 2 G1 2 0) of integers 240 is cent seats? wd (by x ter ia [1998] 1 6 then SE equals « 2 c tw on = ba, ibs wa, wr bn (a mone these 26. 32, 11998) Ao irsligit number is a positive number with ex- nn digits. Nine hundred distinct a-digit num- ers are 1 be formed using only the three digits 2, Sand 7. The smatlest value of for which this is posible is wo ter ® (by 7 wo 11998] . Ina callege of 400 students, every student reads 5 newspapers and every newspaper is read by 60 stu- dents. The number of newspapers is (ay at teat 0 (b) at most 20 (ed exactly 25 (d) none of the above 11998] For2srsa, et ww ft) wn [ 2 tei) (rsa) fnt2) (n+2 w wy" } {20001 le) (re How many different nine digit number can be formed from the number 888 by rearrang- ing its digits so that the odd digits occupy even po- fap Io (by 36 (er 60 (db 180. [2000], For any positive integer am. cssith 1 an), Net ( ") Prove that m Jone) =(20) Hence or otherwise. prove that (ay (mony fama n | jt? |+3! +..+inem +b Cia ae m ne « ( ) (2000) ms? Let T, denene the sumber of triangles which can be formed using the vertices of a regular polygon of sides. IFT, ~ T, = 21. then n equals ws (by 7 (6 a4 [2001] Let f= (1.2.3.4) and F= (1,2), Then the aum- ber of onto function from E to F is wa thy 16 (or 12 ws a You have either reached 2 page thts unevalale forvening or reached your ieving tit for this book. a You have either reached 2 page thts unevalale forvening or reached your ieving tit for this book. a You have either reached 2 page thts unevalale forvening or reached your ieving tit for this book. 5.36 15. 16. 7. 18. 19. 2 22, 23. 28. |. The letters of SURITI can be arr Course in Mathematics for HTIEE For the first place we have nine choice, for each of the next four 10 choices. At this stage add the num bers already written and the choose digit for the unit's place is 5 ways. A; is obtained by putting x= ~ every where in the expression except in. +k. Therefore m! Ae ——_—_—_——" — DG DER FA ED km pt mt . Khe) (kD im~&)! Consider the product 4042.04 Consider the product aey (164) (5) (MM). (AA) (TT). EH Le ‘We can arrange 4 distinct letters in *P, ways, 2 % 4 identical and 2 distinct in ("C, 2 pairs of identical letters in °C) oy ways CMCC) "Cy-n=90 0+ NRHN GET AC) Choose two elements "C. and rest of the (2) elements in 3"? ways. 10° "p, . Cy 27 “cja"C, 4" "Oy Gy WC Y=", P= Gee ear vd in 6172! += 720/2 = 360 ways. The alphabetic order isl, RS. T and U. The number of words beginning Lis *P, 1 = 120, and those beginning R number 54/2! = 120/2 = 60. Then come words beginning SI, numbering 4! = 24, and SR, numbering 44 We then get 'P, = 3 words beginning SUL, after which come the words SURUT and SURITL ‘Thus the rank of SURITL is 120 + 60424412 +642= 2M. ‘The number of ways the candidate can choose questions under the given conditions is enumerated below. 2, 30. 3 32. RB. Mt. 35. 36. 7 38 39, 40. 41 42. 43. Group 1 pf 3 Pe Group 2 2 4 3 4 No.of ways | COPCEY|PEGLE COD ¢ ‘2 From this we see that the required number is COP COI COO COV +COVCE (5) 10) + 10) C10) 4 (10) 05) = 200, amber of functions = ¥ ‘of functions with exactly two elements in ange is 'C))42" — 2) anal with exaetly one element in range is 3 Il 1 a= rnin Vand aber 1 ‘The halls must be of alternate colours Fora square of size AA. we have to choose (+ D consecutive horizontal amd (4 + D consecutive ver- tical Tines from the chess board, and this can be done in (9 17 is used Ay (9 = b3 ways. 1 first place, the number of numbers is 9! and tor any other four plice it is 8 x 9°, Pa Unit's digit of 17 is, ic, Se See Wor is od. “©, ty maxim when r= 1 Gr Lor BAAA can he ate be aliusted i wed ind ways and two N's can “Cy ways. Ske Sx7 Hin EOC nS 5x5 ways. ays and dal number af ways — number of ways in whieh exactly Hour hoxes remain empty. wear yedenhy a= All the words bey ‘occur before COCHIN. Ba ctiethyssior SET TX LR 17 1D # with CC. CH. Chand CN 46. nth iy for 1 nes, a You have either reached 2 page thts unevalale forvening or reached your ieving tit for this book. a You have either reached 2 page thts unevalale forvening or reached your ieving tit for this book. a You have either reached 2 page thts unevalale forvening or reached your ieving tit for this book. 5.40 Course in Mathematics for ITJEL ‘Thus, the number of solutions is 18474 IS +. +34 1 = 99 93. Required number of ways = number of non-negative integral solutions of APH tH EG HS 6 Cs “ 6 = The number of different ways of arran; and 4B", nan will be just two steps forward from his initial position (in at most 10 steps) if his moves ment is as follows 2K or 3: or 48, 28 of SE. SH or 6F, 4B “Thus. required number of ways BN AIC CCM Hatemient-2 ts Lalve Also, the number af ways in which the man is just two steps backwards = 286, *. required number of ways = 2286) 86 572, a You have either reached 2 page thts unevalale forvening or reached your ieving tit for this book. a You have either reached 2 page thts unevalale forvening or reached your ieving tit for this book. a You have either reached 2 page thts unevalale forvening or reached your ieving tit for this book. a You have either reached 2 page thts unevalale forvening or reached your ieving tit for this book. a You have either reached 2 page thts unevalale forvening or reached your ieving tit for this book. a You have either reached 2 page thts unevalale forvening or reached your ieving tit for this book. a You have either reached 2 page thts unevalale forvening or reached your ieving tit for this book. 6.8 Course in Mathematics for [ITJEE 2S = PCy + PC, +... + Ogg = 2” => s=2 Example 23 The sum of the rational terms in the expansion of (24° +43)" is @ 7b) 8s (c) 97 (d) none of these Ans. (A) Solution (r-+ Yh term of (25 +3)" is tn = °C, (2"50~" (2) = Bc, 245 (ay? As 2 and 3 are relatively prime 24 ~"* and 3” will be rational if both 4~ r/S and r/2 are integers, i.e. if 1/5 and P12 are integers. This is possible if r= 0, 10, 20. ‘Thus, the sum of rational terms is C2") (3°) + *Cyq 2") (3*) + Cap (2") 3") As this is more than 71, 85 or 97, the answer is (d). Example 24 If sum of the coefficients of the first, second and third terms of the expansion of iG + 4) is 46, then the coefficient of the term that does not contain xis (a) 84 ‘Ans. (a) Solution We are given "Cy + "Cy + "Cy = 46 =9 2m + m(m — 1) = 90 2+ m9 = 0 m=9asm>0 (b) 92 (©) 9B (d) 106 = om Now, (r+ Uy tern of (x? +4) is soe’ (2) arco For this to be independent of x is Im-3r=0r=6 “+ term independent of x is °C, = 84. Example 25 Value of YY (C,) (C,_)is m (@ *C, ws err 7C, = 7c, © "C42 (d) none of these Ans. (b) Solution We have We have Coxe Pt. 4 Ot a x(l +x)" and a G+ te. x Now, Loe» a = constant term in [eveets : +e5| x ¥ x[ertqe eres] = constant term in (1 +4) (l+ayx coefficient of x" in (1 + x)" 6. nt 1 pan + am Also, = C'*7C, 1) = 0, = **Cy1) *Cy-1 Example 26 Value of SHLKIXIX4HIXIKEKS Feta ntl) (nt D (n+ 3) is @ Ems 142) 43) (na) w Lee) Line © 50rd (@ none of these Ans. (a) Solution k(k-+ 1) (k+2) (k+3) ik+ pea) =a! 4 (14! me ‘Thus, SHS OC + Cy + OC) Coefficient of x in (1 + x) + (4+. ea tt) " 4! [Coefficient of x° in (I +2)" * 4) a You have either reached 2 page thts unevalale forvening or reached your ieving tit for this book. a You have either reached 2 page thts unevalale forvening or reached your ieving tit for this book. a You have either reached 2 page thts unevalale forvening or reached your ieving tit for this book. 6.12 Course in Mathematies for IT-JEE. Example 38 The remainder when 2°" is divided by 17 is (a) 1 b) 2 ©8 (@) none of these Ans. (¢) Solution Weave 2° = (24) = (17 ~ =. 17 — MC, 1 + — Cyt? + 1 = 17 m+ 1, where m is some positive integer. = MF 829) = BLT m+ 1) = 1708 m) +8 This show that the required remainder is 8. Example 39 The interval in which x (> 0) must lie so that the greatest term in the expansion of (1 +x)" has the greatest coefficient is ( n nat) © Ta Carer @) Enel net y™ (d) none of these Solution Greatest coefficient in the expansion of Casi C,, We are given "C, x" is the greatest term. Gat! <*e,2 and eT ae es ac nC. = Ee aay Sea Example 40 The largest term in the expansion of (3+ 20), where x= 1/5. is fa) Sth (b) 6th (c) 8th (d) Oth Ans. (b) Solution Greatest term in the expansion of (x + yy" is kth (n+Dy term where k fens} In the present case | ‘Thus, 6th term is the largest term. 50+) 29] _[SDOIS 3e2x || 3425 |=° ‘SECTION It MULTIPLE CORRECT CHOICE TYPE Example 41 expansion of Values of x for which the sixth term of the ; = (27 pornos) ise. @ 1 (b) 2 3 (d) none of these Ans. (a), (e) Solution Put y= gv" => Jogyy log, Vo" for = 3 (rvsmeort-n) 1 a 5 loa: (4937 - 9 - 9" ene y75 B= (8) {cays => 567 = 2143") ((4 3-9) = = 143" By = 27= 4) 38-9) 3-7 = 4u’ — 9? — 27 = 0 where uw = 3° Note that _u = 3 satisfies this equation’, gen ly-2= =3 = bax-2521 xe2tls3orl. Example 42 IfC,= (") then sum of the series SeCh+ hehe FC}+~ upio (ns 1 terms is 1 (2n4t 1 (2n42) ae » | ® a(n) ” Ea nt) ans 4 © af nt } ca Gari n+l on (naayP From avallable answer, possible values of w are 32° = 9 = 3 for the ‘hsices (a ad (©) and 4° 9 = 3° 1 forthe choice th. a You have either reached 2 page thts unevalale forvening or reached your ieving tit for this book. a You have either reached 2 page thts unevalale forvening or reached your ieving tit for this book. a You have either reached 2 page thts unevalale forvening or reached your ieving tit for this book. 6.16 Course in Mathematics for HT-JEE Bal Yep Gg = - Leng a ro Devt "k= Yen! KC, a imo =0 and Ye, == YCp eG, =0 a i Substituting these in (1), we get Yeo! G @- KF m Example 53 If. R, and Ite 1420 set-¢ t+ +6, "Teme * @ deny? + upto (n + 1) terms C 143x ny then S (a) equals ° (b) equals 1 (©) equals 0 (@) is independent of x Ans. (6). (d) Solution Putting —— = y, we can write lene S=ICQ-Cryt Gy - +E IG = ay [C,- 20, y + 3C, + Gt inc, 4 =d-yft+ ot i -oh,. dt * = (yy = nay (y= =(1-y)" >! hy my} = (=F! [L- + ney] = 0 Example 54 If 22% 4 2F 4 2y10 Say+ xt ar +..+az" aw then value of ay-bay-datas-tay-tatags. 2 2 “2 is {a) less than 2 (b) greater than 0 (©) equals 2 (a) none of these Ans. (a), (b) Solution Let a I be a cube root of unity Put x= 0 o7 in (1), so that lsaqta ore a+. wd Le ay 4 ae? + a0 .. @ B) Adding (2) and (3) and using @+ @* = ~ 1, we get 2 = 2ay - ay - ay + 2ay +. Example $5 Let (1 + v2)" =x, +y, V2 | where x ¥q are integers, then (@) x2 -2y2 = 1" (b) x, + 2 © 42 -2y3=1 @ Yee +n Ans. (a), (b), (d) Solution We have Jaa 20 (1+ V2) 2x, + y,V2 w = (l= V2)" =x, - yev2 Q) From (1) and (2) we get d+ V2)" a = V2y 2? = © If =a} - yz Next, ty 41+ 9n41 V2 = + V2)"*! =(1 + v2) Gy, + v2») = (a, + 2yq) + V2 Oy + yy) ‘Thus, Snot Hint yy Example 56 1 Ith term in the expansion of em) (@) an irrational number (b) a rational number (©) a positive integer (4) a negative integer Ans. (b), (6) Solution We have OF ty 61 Ay ~ 2y, 7 1 42 => 9484 120 qth =4¢ 2) 1 642% 4 G+ 23) 7 1 3 3- py 32 ~7GrWy=5 v2 ‘Thus, 11th term in the expansion of veu(i) Cat which is a positive integer and hence a rational number. 0 a You have either reached 2 page thts unevalale forvening or reached your ieving tit for this book. a You have either reached 2 page thts unevalale forvening or reached your ieving tit for this book. a You have either reached 2 page thts unevalale forvening or reached your ieving tit for this book. a You have either reached 2 page thts unevalale forvening or reached your ieving tit for this book. a You have either reached 2 page thts unevalale forvening or reached your ieving tit for this book. a You have either reached 2 page thts unevalale forvening or reached your ieving tit for this book. a You have either reached 2 page thts unevalale forvening or reached your ieving tit for this book. 6.24 Course in Mathematics for INT-JEE Similarly, a Example 83 Remainder when N is divided by A. (a) N= 99" and A = 10 (7 (b) N = 27 + 2008 and A = 9 @o (c) N= 9° — 8(2008) - 9 and A= 64 @1 @) N= 7° and A = 1000 (s) 19 pars @@OOQ O@OO) ®©@OO| 4 @@@O) Solution f@) 99' =c10- = 10! ~ !, (10) + Cc, (10) ~ vee =! Cqq (10) + Ans. oe =10m+t for some me N. (b) P=9 = 229-1) = 9m ~ 1 for some me N ‘Thus, when 2° + 2008 is divided by 9, the remainder is 0. (©) Use (1 +8)"-8(n~1)-9 1+ 80 +64 ("Cy = "C84 + +"C, 8°] -Bn- 1 = 64 m for some m € N. (d) As 7? =50~1, we get 71 = (1-50) = Cy + C, (- 50) + %C, (SOP +50" m where m is some positive integer. = 1 +2500 (25 x 49 — 1) + 125000 m + 1000 [4 + m] where m is some positive integer. ‘SECTION Vi REASONING TYPE Example 84 Letne Nandc,sandfr(").r=0, 1,2, r Statement-t:1fs= ( Ostcyen ) Statement-2: 16 7= FD ( zi" T=nA aeicjen Ans. (a) Solution Note that = B3c+2)] = nA Using C,= C,_, for Sis n, we get se rE" G Jeans Ostefen = WenTenA = Saran Example 85 Leimne Nani c=(")for02 where C,="C,. {1989} ‘The expression (x + (x° = 1))* + x= G2 = 1)')* is a polynomial of degree @ 5 (b) 6 7 @s8 (1992) WE ose a= Sno 3)’ and a,=1V k2n, a show that 5, arly (1992) Let n be a positive integer txt)" = ag taint + ay, show that Gap taj-~+a5=, (1994) 17, 18. 19. 21. 22, 23, The sum of the rational terms in the expansion of (v2 +3) is 11997) Prove that aig Leos =f) [1997] If in the expansion of (1 +)" (1 ~ 2)", the coeffi- cients of x and x° are 3 and ~ 6 respectively, then m is (a) 6 (©) 12 b) 9 (d) 26 [1999] . In the binomial expansion of (a ~ 6)", # 2 5, the sum of the 5 and 6" terms is zero, Then ; equals, n-4 o> 5 6 On @ ‘The coefficient of Pin (1 +7)'? (1 #4) (1+ 4) (2001) is (a) ®o, +3 (b) nos +1 © °C. @ * nC +2 (2003) If n and k are positive integers, show that A) -2 Ol) #03) - wee (CC) -(0) vit) sar, «() ‘i i) (3m) ia }--Ccl) i) » (i) ( @ (i) (3) 12003] stands for "C,, the value of [2005} a You have either reached 2 page thts unevalale forvening or reached your ieving tit for this book. a You have either reached 2 page thts unevalale forvening or reached your ieving tit for this book. a You have either reached 2 page thts unevalale forvening or reached your ieving tit for this book. 6.40 Course in Mathematics for IIT-JEE 17. $=} *C, sin (kx) cos (n— kx a) ms Writing the terms in the reverse order _ 4 sin (n ~ K)x cos (kx) = Dey sin (n - b)x cos (kx) @ a Adding (1) and (2), we get 2S = y "C, {sin (kx) cos (n ~ k)x ro) sin (a — k)x cos (&x)} as =F Cc) sin (ng) = m => S=2"~! sin(nx) ” sin (nx) 18. $,~S,= J) "C; [cos (kx) cos (n~ Kx ~ rr) sin (kx) sin (n— k)x) = YC, cos (k +n Dx ms = 2" cos (nx) 19, Putx=1 20. Since gett”) . meal) =e ‘We consider the expansion of (e + J ® 1 eee" alg, =o For coefficient of x°, we set 33-6r=9 => rad Thus, desired coefficient is "C, 21. Using "C,+"C,_,="* 'C,, we can write the given ‘equation as BIC a* Cs ae = r=P-1B2orr +P - 182=2n+1 As the last equation is not possible reP-18 = r=i4 Since n 2 r~ 1 we get least possible value of mis 13, Cette "Cy, ~ nny + eMC LG) a a oa be on Col om 4 2, ment 23. UC)="C 14's MGrat "ot ot =2= rel P+naryPen nan —4nr+ 47 = (n= 27) 3m Parent = (1-24437 ‘Thus, coefficient of x" is °C, 26. Forn 23, (# o(sl) casn = (n+ Drcnt*! 27. 6 term is indendent of x. Thus 1(@ = "°C, [(sin ~ ' @) (cos ayy? 5 Thus, U(@I s °C; () 28. Ca" "43, xa"? + °C a? 1 teen! d ye % = oy 1 ie s a oY Multiplying by x and putting a= 1 ~x, we get, the sum of the series as x 29, > ¥ G+ Fahne a You have either reached 2 page thts unevalale forvening or reached your ieving tit for this book. a You have either reached 2 page thts unevalale forvening or reached your ieving tit for this book. a You have either reached 2 page thts unevalale forvening or reached your ieving tit for this book. 6.44 56. 37. 38. 39. Course in Mathematics for IIT JEE =the number of ways of selecting n persons from n ‘men and women and appointing a men's leader and a women’s leader =a @-7¢,_5) ‘Thus, (2) C"C,) — 1? "2G, 2) 2 ‘Sum of coefficients the integral powers of x is = Cy + Cy (27) + C2) + + Cy (2) (a +2)% +0 -2)")=4 a+ 1 2 For the other part, use sum ofall the powers of xis, * 25,(2) = ["Cy sin (Ox) cos (nx) + "C,, sin (nx) cos (Ox)) +1°C; sin.x cos (n—1)x+"C,_; sin (n— 1x cos x] + (°C, sin 2x.c08 (n= 2)x + "Cy sin (mn 2)x cos 2x) ++ upto (+ 1) terms = °C, [sin (Ox) 60s (nx) + sin (rx) cos (Ox)} + °C, {sin x cos (m1) .x + sin (n - 1)x cos.x} + "Cz [sin (2x) 60s (n -2)x+ sin (n—2)x 608 2x] + +°C, [sin (nx) cos (Ox) + sin (Ox) cos (nx)) = ("Cy H"C, #"Cp+ > +"C,] sim (nae) = 2" sin (ox) = SQ) =2"*! sin (nx) We have k ("C,) = 2n P""'C,_ 4) Thus, Sir= PwC!) im = (my? = ne ') [See Theory] S(Qn) = nj"! = n(2"") [See Theory} For Sn) and S(4n) use "C,= 0 r>n. S(n, 1) = coefficient of x” in (4 xtit to) (Cy Cyr t Cy? + C1" G2") Coefficient of x” in (1 — xy"! (1 =x)" enc) Now, (~ 1) (""'C,) = 28 This is possible ifn = 9, r=2 orn =9 or r=6. =10-1 = 35 (10~ 1)" = 10m +1 where me N. 17 = 289 = 290-1 => 17% = (290-1) = 10m = 1 where me N Last digit of 17°7is 9 Similarly, last digit of 7® is 9 and last digit of 19° ist 61. 62. 63. REE 3 22 70. Te 7. B. 14. 5. 16. 71. 78. 79. 80. 81. S=*C, which is maximum when r= 15. S=™C, is least when r= Age "Cy"Gy1 °C, "Cy aa t#"Cy 1 "Co =4G, 24 Now, Ayer. 2n42)! (n- D+! A, nint2it nyt RAFAT SIS, 2,4 nin+2) 4 Putm=n,r=n Use "C,="C, E 42% "Gey" . B= 2 Qrs 64D ead es ayet toy z m= coefficient of x’ in £, where Ex (xt 20 +t 2 theme ee oy = (+27 (+ 2 srl (+2)! =(042)" x4 ‘Sum up the series using A.G.P. (+x) (-x4 P= (14x) 0-2) Use formula for (a + b)" - (a — by" 2 = 4096=2"? =) n= 12 Greatest coefficient = "Cj, Use 2x7 +x~3=(x~1) (2e+3) 8Cs (¢-*°9 (7 logiox)° = 5600 = ¥ logipt)’ = 100 Use multinomial expansion. 5° = 5(26 - 1) = 5 (26m -1) where m is some integer. Use 1 +.x~2x7= (1-2) (1 +2x) ss Buta,= 7? S24) 251-9 r Ea (e445) a+ Thus, coefficient of x? = 'C, = 4950 (a) SC, TM Ly will free from radical sign © risa multiple of 9 and 0 < rs 656 a You have either reached 2 page thts unevalale forvening or reached your ieving tit for this book. a You have either reached 2 page thts unevalale forvening or reached your ieving tit for this book. a You have either reached 2 page thts unevalale forvening or reached your ieving tit for this book. where 4,3 = cofactor of (1,3)th element in A ie. ny An=lay ay 8. If A= diag (Aj, Ag, 2) then A”! exists if 4, #0 Vi and AM = diag O's Reo" oy ed Also, A"= diag (2%), 23...2)) if me N 9. If a square matrix A satisfies the equation ay + a,x + a, + +a, x = 0, then A is invertible if dy # 0 and its inverse is given by dn 2 la, +a, Ate 4a, A™] 4% 7.4 SOME DEFINITIONS AND RESULTS A square matrix is said 10 be symmetric if A’ = A and skew symmetric if A’= ~ A. A square matrix A Hermitian if 4 Results 1, The main diagonal elements of a skew symmetric ‘matrix are zeros, i. a, = 0'V i. 2. Determinant of a skew-symmetric matrix of odd order is zero and determinant of a skew-symmetric matrix of even order is a perfect square 3. Every square matrix can be written uniquely as a sum of symmetric and a skew symmetric matrix, ie. if A is a square matrix, then there exists a symmetric matrix P and a skew symmetric matrix Q such that A=P+O Hermitian if A* = A and skew In fact pel eatvand ont =) 4. Forevery square matrix A, matrices A+ A’, AA’ and AVA aré symmetric and matrices A ~ A’ and A’ ~ A are skew symmetric. ‘A square matrix is said to be orthogonal if AA’= A=. In other words A is orthogonal matrix if and only if At = A’ Aiis said to be unitary if A*A = 1 Results 1. If A is an orthogonal matrix, then |Al # 0. In fact Al = + 1. and A, A% A® are orthogonal also. 2. If A and B are two orthogonal matrices, then AB and BA are both orthogonal matrices. ‘A square matrix A is said to be a nilpotent matrix if there exists a positive integer m such that A” = O. ‘The least positive integer m satisfying the condition A™ = is called the index of the nilpotent matrix. Matrices 7.3 A square matrix A is said to be idempotent matrix if A? =A and is said to be involutory if A? = I. IFA =(a) , 9 i8 a square matrix, the trace of A denoted by (© (A) is sum of all the main diagonal elements, i. wy Ay + gy oe + yy 7.5 RANK OF A MATRIX IEA = (den i8 a matrix, and B is its submatrix of order r, then IB1, the determinant is called as r-rowed minor of A. Definition: Let =(ay)q:9be a matrix. A positive integer ris said to be of rank of A if (i) A possesses at least one rrowed minor which is different from zero; and Gi) every (r + 1) rowed minor of A is zero. From (ii), it automatically follows that all minors of higher order are zeros. Result The rank of a matrix does not change when the following elementary row operations are applied to the matrix. (a) Two rows are interchanged (R, «> Rj) (6) A row is multiplied by a non-zero constant, (R, > RR, with k # 0) (©) A constant multiple of another row is added to a given row (R, > R, + kR)) where i # j Note : The arrow —+ means “replaced by”. Note that the application of these elementary row operations does not change a singular matrix to a non-singular matrix or a non-singular matrix to a singular matrix. Therefore, the order of the largest non-singular square submatrix is not affected by application of any of the elementary row operations. Thus, the rank of a matrix does not change by application of any of the elementary row operations. ‘A matrix obtained from a given matrix by applying any of, the elementary row operations is sad to be equivalent to it. If A and B are two equivalent matrices, we write A ~ B. Note that if A ~ B, then p(A) = p(B). By using the elementary row operations, we shall try to Tatalech Go ghd mais toe Dollosing forse ors oo. * 0 0 O- # where * stands for zero or non-zero element. That is, we shall try to make a, as 1 and all the elements below a, as zero. a You have either reached 2 page thts unevalale forvening or reached your ieving tit for this book. a You have either reached 2 page thts unevalale forvening or reached your ieving tit for this book. a You have either reached 2 page thts unevalale forvening or reached your ieving tit for this book. _[ coscosp-sinasinf# cosasin+sinarcosp ~ [-sinacosB-cosarsinB -sinasin B+cosa cosh _[costa+ fy sin(a+ By =| -sin (a+ A) ek asp Example4 Let A and B be two 2 x2 matrices. Consider the statements (i) AB=O 2 A=O00rB=0 AB=1,3A=B" (A+ BY =A? + 2AB + BP (a) (i) is false, (i) and (iii) are true (b) (i) and (iii) are false, (i) i (c) @) and (ii) are false, (A) (ii) and (ii) are false, (i) is true Ans. (b) Solution Gi) is false. o 1 1d a=[) i]ee=[5 of wf She AB = but neither A = O nor B= 0 is false since matrix multiplication is not commu- tative. Gi) is true as the product AB is an identity matrix, if and only Bis inverse of the matrix A. Example 5 Let A, B be two n x n matrices such that A+B =AB, then (a) AB = BA (b) AB = i, (©) A=1,orB=1, (A) none of these. Ans. (a) Solution A+B=AB =>1,-A-B+AB=1, => U,-A)U,-B)=l, => 1, Ais invertible and its inverse is 1, ~ B. (y-B)y-A)=h, B+ A= BA. Thus, AB=A+B=B+A=BA. Example 6 If A and B are symmetric matrices, then AB-BAisa (a) symmetric matrix (b) skew symmetric matrix (c) diagonal matrix (@) null matrix Ans. (b) Solution We are given A’ = A, B’ = B Now (AB ~ BAY = (ABY ~ (BAY BA-AB BA~ AB = = (AB -BA) ie. (AB — BAY’ = ~ (AB ~ BA) Hence AB — BA is a skew symmetric matrix. Example 7 If D = ding (d,, dz, ....d,) where d; #0, for f= 1,2, ....m then D™ is equal to (a) D () 20 (©) diag Gy", dy"... dy) (6) Adj D Ans. (c) Solution See Theory. Example 8 The inverse of a symmetric matrix (if it exists) is (a) a symmetric matrix (b) a skew symmetric matrix (©) a diagonal matrix (d) none of these Ans, (a) Solution Let A be an invertible symmetric matrix. We have AA‘ =A‘ A =J, > aa AY = (1, = AYA = Atay =], = A'YA=AA = ays [inverse of a matrix is unique} Example 9 The inverse of a skew symmetric matrix (if it exists) is, (a) ‘a symmetric matrix (b) a skew symmetric matrix (©) a diagonal matrix (@) none of these aya’ say TY CA) = ADA ay =a) inverse of a matrix is unique] a You have either reached 2 page thts unevalale forvening or reached your ieving tit for this book. a You have either reached 2 page thts unevalale forvening or reached your ieving tit for this book. a You have either reached 2 page thts unevalale forvening or reached your ieving tit for this book. = Ook =0orxX=0 is the only solution of the system of Example 29 If A= ie 1 Jesten A sequal 10 w(t °) ® (so i} © laa 1 re (4) none of the © (3s of (@) one of these Ans. (a) 1 oVf1 @ I 0 Soliton a {ee Mee 1) (owe ‘) @ aa ald ve te (saa) i Continuing in this way, we get as (wote ) = i H In fact eel tynew ml Example 30 If 1 -wney 1 ney (ue 1 } -tne@ 1 J then (a) a=b=1 (b) @ = cos 28 b = sin 20 (© a= sin 28 b = cos 20 (@) a= 1, b= sin 20 Ans. (b) Solution We have 1 tan) 1 1 ~tan une 1 I+tan? @ [tng 1 oOleo Tle | Matrices 7.11 tan? 6 2tand = cos? ol use I-tan? 6, sin26 cos2 a= cos 28, b = sin 26 SECTION It MULTIPLE CORRECT CHOICE TYPE [ie case] Example 31 Let A and B be two matrices different from / such that AB= BA and A° B*isinveribe for some postive integer n. If A" - 8" = A"*! — B+! = a"*?_B**? then (a) J - Ais singular (b) I~ Bis singular (©) A+B=AB+I (@) W~ A) U~ B) is non-singular Ans. (a), (6), (©) Solution AT*? Bt? = (4 BAB) — AB(A" - B") => A" BY = (A + B) (A" — BY) — AB(A" ~ B’) Pe A+B-AB [ye => (-A)(-B)=0. As A, B #1, we get 4 A" — BY is not invertible] 1 ~ A and 1 ~ B are signular matrices. Example 32 If A is matrix of size nx n such that A? + A+2=0, then (@) A is non-singular (6) A #0 (©) lal #0 @ a Ans. (a), (0), (6), Solution We have A(A +1) = ~ 2 = WA + Dis +20 => UI + n= 2)" #0. ‘Thus, IAI # 0. Also, A {4 + a} “1 = Ate has. Clearly A #0 for otherwise Al = 0. a You have either reached 2 page thts unevalale forvening or reached your ieving tit for this book. a You have either reached 2 page thts unevalale forvening or reached your ieving tit for this book. a You have either reached 2 page thts unevalale forvening or reached your ieving tit for this book. ‘SECTION Ill COMPREHENSION TYPE Paragraph for Question Nos. 46 to 50 Twon xn square matrices A and B are said to be similar if there exists a non-singular matrix P such that PIAP=B Example 46 If A and B are two singular matrices, then (a) det (A) = det (B) (b) det (A) + det (B) = 0 (©) det (AB) #0 (d) none of these Example 47 If A and B are similar matrices such that det (AB) = 0, then (a) det (A) = 0 and det (8) = 0 (b) det (A) = 0 or det (B) = 0 (©) A= Oand B= 0 (d) A=O0rB=0 Example 48 If A and B are similar matrices such that det (A) = 1, then (a) det (B) = 1 (b) det (A) +det (B)=0 (©) det B) (@) none of these Example 49 If A and B are similar and B and C are similar, then (a) AB and BC are similar (b) A and C are similar (©) A+ Cand B are similar (d) none of these Example $0 If A and B are two non-singular matrices, then (a) Ais similar to B —(b) ABis similar to BA (c) AB is similarto AB (d) none of these Ans. 46 (a), 47. (a), 48. (a), 49. (b), 50. (b) Solution As A and B are similar matrices there exists a ‘non-singular matrix P such that A=P" BP => det (A) = det (P! BP) = det (P™) det (B) det (P) 1 ——— det (B) (det P) aay OP) det B ‘Thus, det (A) = 0 > det (B) = 0 and det (A) = 1 det (B) " Matrices 7.15 Next, A= P! BP,B=Q"CQ, = A=P'@' COP =(OPy' COP ‘Thus, 4 is similar to C Also, as AB = B"'(BA)B, we get AB is similar to BA. Paragraph for Question Nos. 51 to 55 Let A be a square matrix of order 2 x n. A constant A is said to be characteristic root of A if there exists an x 1 matrix X such that AX = AX Example 51 If Ais a characteristic root of A, then (@) A-a=0 (b) A = Ais singular (©) A=AL is non-singular (d) none of these Example 52 If a constant Ais such that A - Af is non- singular, then (a) 4=0 (b) 240 (©) Ais not a characteristic root of A (d) none of these Example $3 If 0 is a characteristic root of A, then (a) A is non-singular —_(b) A is singular @A=0 @A=i, Example $4 If dis a characteristic root of A and n € N, then 2" is a characteristic root of (a) A” (o) ant () a* (@) A-A" +A Example 55 Let P be anon-singular matrix, then which of the following matrices have the same characteristic roots. (a) A and AP (b) A and PA (©) A and P' AP (@) none of these Ans. 51. (b), 52. (©), 93. (b), 54. (a), 55. (€) Solution Since X # O is such that (A AX = 0, 4 ~ dl = 0A — A/is singular. If A ~ Al is non-singular the then ‘equation (A ~ A) X= O => X If A=0, we get Al = 0 => A is singular. & X= A(AX) = A(AX) = AX) = ax, A& X= AAX = AD = BAN) = BUX) = BX Continuing in this way, we obtain AXs XV NEN, Also, IP! AP - N= IP (A - ADPA Wola ~ AN IPL = lA - An We have a You have either reached 2 page thts unevalale forvening or reached your ieving tit for this book. a You have either reached 2 page thts unevalale forvening or reached your ieving tit for this book. a You have either reached 2 page thts unevalale forvening or reached your ieving tit for this book. ‘We have (AB)B™ = AB" = B™' A [using (5) = BOB" A) = BAB") = (BA)B" (using (4)] Thus, (AB)B™ = (BA)B" ‘As Bis an invertible matrix, we can cancel B" from both the sides to obtain AB= BA = A'BA=B and B'AB=A etc. Example 72 Match the expression/statements on the Jeft with the expression on the right. (a) The minimum value of Peder xe? (b) Let A and B be 3 x 3 matrices of real numbers, where A is symmetric, B is skew symmetric, and (A+ B)(A- B= (A ~ B) (A + B). If (ABy = (1) AB, then possible value of & are (©) Let a = logslog,2. A integer & satisfying 1< 204° "<2, must be less than @ ("7) =(@-3) (i): then possible integral value of k. pars @@@O® @O@O®| @@O®| @O@@®) Solution (a) Put x+2=1, then £>0. Also Esdras = 27 420-244 x42 7 xD -2 @)o @! (2 (s)3 Ans. noes Matrices 7.19 «+ least value of eatest is2 which is attained when x = 0. (b) (A+B) (A+B)=(A+B)(A-B) = AB= BA. Now, (AB = BA’ = (- B)A =~ BA =-AB 2 CI s-1ko13, (©) 1< 22 30<-k+3"<1 ms keo* kel. => k< (log2y' A =A (: oh (! -») cd) dale a) If detA = 1, then a=d,b=-b, a You have either reached 2 page thts unevalale forvening or reached your ieving tit for this book. a You have either reached 2 page thts unevalale forvening or reached your ieving tit for this book. a You have either reached 2 page thts unevalale forvening or reached your ieving tit for this book. 33. MM. 35. 36. 37. 38. 39. If @ A. yare three real numbers and 1 cos(a-B) cos(a-y) A=| cos(B-a) 1 cos(B-7) cos(y—a) cos(y-A) 1 then (a) Ais symmetic —_(b) A is orthogonal (©) A is singular (@) A is not invertible. Let Cy = "CG, for 05k Sn and a Ae (So ° for k2 1, and Ay +A, +... +A, 0G k 0 -(3 t) then (a) ky =k (b) ky + ky = "Cy, +1 (©) k= *C,-1 @) k= "O64 sind icos. 14= (2, sing (a) A(8) is invertible for all Be R (b) A(O)" = A(- 8) (©) A(@y! = A(z 8) (4) ACB) + AGE +8) is a null matrix Dis. a3 x3 diagonal matrix. Which of the follow- ing statements is not true? (a) =D (b) AD = DA for each matrix of size 3 x 3 (©) D” if exists is a diagonal matrix (d) none of these. If A is an invertible matrix, then which of the fol- Jowings are true: @A#0 (b) Adj. A #0 ©) al#0 (@) AT = Ul AdjA 1-1d fA=/2 -1 0}, then 100 (@) APT (b) At = a? (©) A"=AWn#4 — (@) none of these If D, and D, are two 3 x3 diagonal matrices, then (a) D, Dy is diagonal matrix (b) D, Dz = D; D, (©) D} + D3 isa diagonal matrix (@) none of these san 4. 42. 43. 45. Matrices 7.23 t]yaen b) timt ante]? © | in 00 lim > Aw = © lim A [° 4 (d) none of these Leta, B and C be 2 x 2 matrices with entries from the set of real numbers. Define * as follows: A*B= FAB + BA), then @)AtB=BtA = (bh) A*A=AP ()A*(B+OQHA*BHAC GAtI=A With A, B, C as in Question No. 41, define o as follows: AoB= (AB ~ BA) then (a) AoA = 0 (b) Aol = 0 (©) Ao(B + ©) = AoB + AoC (@) AoB = BoA 0 sing sinarsin -sina 0 cos asin 8}, -sinarsinf -cosacosf 0 Let A= then (a) Ul is independent of a and f (b) Av! depends only on @ (c) Av! depends only on f (d) none of these |. Let A and B are two matrices such that AB = BA, then for every n € N, (a) A"B = BA" (b) (ABy" = A“B” (©) (A + BY = "CA" + "C, AT B+ "CATT B + +"C, BY (a) A™ - BM = (A" — BY) (A" + BY) If A and B are square matrices of the same order ‘such that A? = A, B = B, AB = BA = O, then (a) ABP =O (b) A+ BP =A+B (©) (A-B)=A~B (A) none of these. a You have either reached 2 page thts unevalale forvening or reached your ieving tit for this book. a You have either reached 2 page thts unevalale forvening or reached your ieving tit for this book. a You have either reached 2 page thts unevalale forvening or reached your ieving tit for this book. 15. aa [* ;] and f(A) = det (A = aD), then (a) fd) = # - (a+ dA + ad ~ be (b) fl) =0 (c) A? = 0 implies A’ = 0 VW r22 (d) none of these 16. Let A be as in problem 15 and A* = 0 for some k > 3; then fa A=O (b) A°=0 (©) A is singular (@) A is non-singular 17 Ifa? +b? 4c? =—2, and Ita’x O+b*)x (te? yx Az |(+a*)x 14x (+c%)x|, then A is (ta*)e (+b?)x 1+e2x non-singular if (@x=0 (b)-1 © @2 18. Let A, B and Cbe 2 2 matrices with entries from the set of real numbers. Define * as follows: AYB= i (AB + A’B) (a) A*B=BtA ) AtA=A () A*B+O2A*B+atc @AtI=A+a 19. With A, B, Cas in Question No. 18, define o as follows: AoB= 3 (AB ~ BA‘), then (a) AoA = 0 (b) Aol = 0 (©) AB + ©) = AoB + AoC (a) none of these 20, LetA bea symmetric matrix such that A* = O and B =1+A+A* +A‘, then Bis (a) symmetric (b) singular (©) non-singular (@) skew symmetric SECTION Il) COMPREHENSION TYPE Paragraph for Question Nos. 21 to 25 A square matrix A is said to be an idempotent ma- trix if A? =A, ai. 22. 23. 24, 25. Matrices 7.27 If is 8 non-singular idempotent matrix, then @ A=1, () A=0 (A+A=0 (@) none of these If A and B are idempotent matrices and AB = BA, then which of the following is an idempotent ma- trix A+B (b) AB ()A-B @)B-A ‘Suppose A is an idempotent matrix, and B= J —A, then (a) Bis an idempotent (b) AB = I, (©) BA’ = 1, (@) none of these If A is an idempotent matrix and B = J — A, then which of the following is not true (a) AB=0 (b) BA (Bast (d) AB=1, IA, B and A + B are idempotent matrices, then (@) AB=BA=0 — (b) AB=BA=1, (c) AB=BA#O (d) AB = BA #1, Paragraph for Question Nos. 26 to 30 26. 2 28, 29. 30. Let = (ay)puqe a matrix such that ay € C. Define trace of A, denoted by #r(A) as follows: mAy= Sa im Let A, B and C be three nxn matrices such that C’C=I,and Awe C. tr(AA + 4B) equals (@) Aw) + 40GB) (b) (A+ A +B) (©) Ag tr(A + B) (d) (A+ ) (AB) tr(AB) equals (a) tr(A) (8) (b) tr(BA) (©) te(A) + tr(B) (@) none of these tr(AA’) is. (a) positive (b) negative (©) non-negative (@) non-positive (CA C) equals (a) tr(A) (b) tr(CA) (©) WAC) (d) tC) tr(ABC) equals (a) w(CBA) (b) t{BAC) (c) (ACB) (d) WABCO) a You have either reached 2 page thts unevalale forvening or reached your ieving tit for this book. a You have either reached 2 page thts unevalale forvening or reached your ieving tit for this book. a You have either reached 2 page thts unevalale forvening or reached your ieving tit for this book. 73. @) 75. (a) 76. (b) SECTION I L@ 2 @) 3. (b) 4. @) 5. ©) 6. (a) 7. (a) 8. (b) 9. (b) 10. ©) 11. (b) 12. (©) SECTION It 13. @,(b) 14, (@) 15. (a), (), (©) 16. (b,c) 17. (a), (b), (d) 18. (a), (©), @) 19. (), ©) 20. &) SECTION It! 21. (@) 22. ¢b) 23. a) 24. (a) 25. (a) 26. (a) 27. (b) 28. (©) 29. (a) 30. (a) SECTION IV 31.7 32.1 31 34.3 35.3 36. 100 37. 25 SECTION V 30 Pas + @@0® *1@@O® © @@O® 1 @@O® 39, Ps + @@O® © @@OO® ‘@@OO8) SECTION VI 40. (@) 41. (b) 42. ©) 43. (b) 13. 14, Is. 16. 7. Matrices 7.31 PAST YEARS’ IIT QUESTIONS 1 @ 24 3. (b) 5. @) 6. () 2 Bs (unit)? 8.) 9%) 10. @) i. @ 12.0) 13. @) HINTS AND SOLUTIONS Daze] UP =U =-27 Als Use AA! =F Used =A’ SetA=0 Ast sa+b += 1 and be +cat ab=0 Also, A= 1 = l= +1. But ll = 3abe = (a? + b* +c) (a+b +0) (a? +b +c be ~ca~ ab) (a+b+e) Asa+b+c>0, we get Al =~ 1. Pte +=T 0 -b -c teae[h 0 Cy - Cy, we get 1 x ol FO) 22-1) ]2 x-1 2 3ox-23 =0 Thus, f(500) = 0. Example 7 The number of positive integral solutions of the equation A = 12, where yer zy As|z ztx x yoo oxty is (a) 3 b) 15 ©4 we? Ans. (a) Solution Using C, > C, + C, + Cy, we get yer zy Az2Qletx ctx x xty ox xt)! Using C; > C;-C,, Gy 9 Gy-C), we get yte atx 0 -t xty -y 0 A=2| 0 -y -2 jx 0 -z x -y 0 {using C, > C, + C, + C,) o1 | =2 2yz|1 0 1 11 0) Determinants 8.8 211 201 2 1 «0 {using C, + C, + G+ GI ‘Taking 2 common and applying Cy Cy Cy C39 Cy Cy, we get 10 0 1-10 10 -1 Now, 4nz = 12 = yz =3 ‘The number 3 can be assigned to any of x, x, z. Therefore, the number of positive integral solutions of 4 = 12 is 3. Example [fay ay 03, .n.neaoey dy are in HP. anda = 5, as = 4, then value of the determinant = Dayz A = 4nz 4nz 4 a as A= lag as % oy a equals 1 50 3 7 95 OF OF 5% Ans, (b) Solution Leta,= —— for sis 9. avid We are given Ss : and 4= : . aad asa” = atdd=t,atsd= s 4 eel. 20° Thus, a= = for 1 $k$ 9, and hence = Wx 15x24 21° a You have either reached 2 page thts unevalale forvening or reached your ieving tit for this book. a You have either reached 2 page thts unevalale forvening or reached your ieving tit for this book. a You have either reached 2 page thts unevalale forvening or reached your ieving tit for this book. Solution We have =3 2 3-i -3 wii) =|34i 0-1-1 a “3 -1ri 4 3 Zis purely real. Example 22. If ai+b+c, aa,+b, +e, a,a,+b,+c, bb+c, Bee, byby +e, 6 C62 é a b Gy and Ay=/a, 6 ¢,|,then “ is equal to a oe ° (a) a, by cy (b) ay a a5 (©) dy bye, (d) a by © + ay by cy + ay bs Cy ‘Ans. (a) Solution Taking c; common from R; and applying Ry > Ry Ry and R, - Ry ~ Ry. We obtain a+b, aja, +b, aa, +b, bib 3 babs Cy io cy ‘Taking b, common from R and applying Ry > Ry ~ R; we get As c5 Ja? aay aay breyy bby oe & Gy Oy ay By cx)by by bs| = aybyes Ay Sy C2 Cy A = eae, 5 Example 23 For a, b,c, x,y, 2€ R,if (a-x? (b-x)7 (e-3)* A= |a-yP (b-y¥ (e-yP (@a-2 @-2) (-7* | Determinants 8.9 (+ax)? (1+ bx)? (+ ex)? A:= | tay)? (+byy? (+cy)?| then (lazy? +b2)? U+ez)? fa) A, = Ay (b) 4; +4, =0 (©) dy = 2d, @ A, =-2% | Ans. (a) Solution We can write the determinant A, as product of two determinants as follows: x -2x | a a? Ay={y? -2y allt b 2 -22 illc 2 We interchange C, and C, in the first determinant and absorb the resulting negative sign in C; to get 12x x71 @ a?| Ay=|1 2y y? i]t bb? 1 22 27]ft ¢ ¢? (tax? (+b)? (tex)? =|Ctay? A+by)? (tey?| =A (tag) (+62)? (+e)? Example 24 Let secx cosx sec’x+cotxcosec x F(R) = cos? x cos? x cosec?x 1 cos? x cosec*x vid , then value of J" Fade is @ 0 (b) 48 ) -2-—* (d) none of these Andy 2 se ° Solution Applying Ry > Ry ~ Ry, we get cosx sec? x+eot.xcosec seox S@)=|-sin?x 0 0 1 cos? x cosec*x aay fOo8x sec? +cat xeosee x eosee*x = sin? x [eos x cosec? x = 008? x (sec? x + cot x cosec x)] cos?x=c0s x sin? x~ + (1 cos? x) 2 a You have either reached 2 page thts unevalale forvening or reached your ieving tit for this book. a You have either reached 2 page thts unevalale forvening or reached your ieving tit for this book. a You have either reached 2 page thts unevalale forvening or reached your ieving tit for this book. rou 1 A=(atb+c-x\c b-x a boa Applying C, > C,~C, and C, > Cy ~ C;, we obtain 1 0 0 As-xlc b-c-x ane [[ratb+c=0) b a-b c-b-x Expanding along Ry, we get A= ib -c-2) (€-b- 2) -(@- 6) (@-0)] =al(a- 6) (@-0)- (+ b-0) (b+ 0] sala? = ab = ac + be = x7 + b+ c? ~ be} La? + B +c? — be ~ ca ~ ab ~ 2°) A= 0 implies x=0 or =a" +6? +c?—be-ca—ab. Nowx?=a? +B? +c? ~ be ~ ca ~ ab =P4+B+2-Lfarbsot-a-P-e} =2W@+% ee) t eek Rese se) Example 37 The determinant atb+c=0) a baat 4 b © bate a+b bate 0 is equal to zero if (a) a,b, ¢ are in AP, (b) a, b, ¢ are in GP. (©) a,b, ¢ are in HP, @) ais a root of ax? + Ibx +c = 0. Ans. (b), (d). Solution Applying R; + Ry - @R, -R,, we get ab a+b be bate 0 0 ~(aa? +2ba +c) = @ ~ ac) (aa? + 2ba +o) by expanding along Ry. Now A = 0 if either b? ~ ac=0, or ad + 2ba+c=0, that is, if either a, b and ¢ are in G.P., or @is a root of ax? + 2bx += 0. Determinants 8.13 Example 38. If x2 -Sx430 2x-5 3 AG)=| 3x7 +244 6xt1 9 Tx? -6x+9 M4x-6 2 = ax’ + bx’ + cx + d, then (@) a@=0 (b) b=0 @c=0 @ d=47 Ans. (a), (b), (¢), @) Solution Differentiate both the sides with respect to x Qx-5 2x-5 3| [at-Sx43 23 AG) = | 6x41 Gxt] 9/4/37 4x44 6 9 14x-6 14x-6 21) |7x7-6x+9 14 21 =0+0=0 => AG») is a constant. Thus, @=0,b=Oandc=0. For value of d, put x = 0 ho ghed ke Example 39 if [yf yt*? yte?] * ghogkth ght = (xy) yz) (@=a) (Lda + Ly + Wie) then (@) k=-3 (b) k= -1 (kel d) k=3 Ans. (b) Solution We can write the given expressions as ty 2 dy 1 Bz FT NO DED OTH At) 1x x3} where A, =|1 y? y?| 12 2] = Prt ge =G-WO-DE-NOrtetH) Note that degree of A, is 5 degree of LHS = 3k +34+5=3k +8 Also degree of RHS = 5 3k+8=5 or k=-l, Example 40 If -x a b b =x a |, then, ab -x a factor of A is a You have either reached 2 page thts unevalale forvening or reached your ieving tit for this book. a You have either reached 2 page thts unevalale forvening or reached your ieving tit for this book. a You have either reached 2 page thts unevalale forvening or reached your ieving tit for this book. Ans. (a), (b) Solution The given system of equations will have a non-trivial solution if sin3@ -1 1 A=|cos28 4 3) =0 2077 Applying R + R, + 4R, and Ry» Ry +7R,, we get sin3@ “11 cos2@+4sin3@ 0 7 2+7sin3@ 0 14 Expanding along C;, we get 4 =7[2 (cos 20+ 4 sin 3) ~ (2 +7 sin 30) = 0 a = 2cos 26+ sin38-2=0 = sin 0(2 sin @~ 1) 2 sin @+ 3) =0 As 2 sin 84 3.can never be zero, we get sin 8=0 or sin @= 1/2 = sin (196) => O=nm or O=mn+ (-1)" 216 (n, me LD) ‘SECTION Ill COMPREHENSION TYPE Paragraph for Question Nos, 51 to 55 Let 2x9-3x7 5x47 2 40-7x 3x+2 1 [Te -B ox-1 3) aw ota xt tayxt To evaluate a, we differentiate A(x) i times w.rt..xand put = or divide A(x) by x* put x= 1, differentiate (4 ~ i) time wert rand put r= 0. Example 51 a, equals @O 1 @2 @3 Example 2 a, equals @O 61 © 16 @ 191 Example 53 a, equals @ 41 O)-48 @-41 @ 43 Example 54 a, equals (a) -70 ()-73 @-74 Wo Determinants 8.17 Example $5 a; equals (a) 46 (b) 43) 41 @ 0 ‘Ans. 51 (a), 52 (6), 53 (b), 54 (b) 55 (@) Solution $1. A(0)=0 6x*-6x Sx+7 2 $2. AG@)=| 127-7 3x42 1 2x —16x x-1 3) 2-3 S$ 2 +/4x7-7e 31 Tx -8x7 1 3) a, =A) = 161 2-Wx S+7/x 2 33, AD laa? aeare 1 “| 7-8 Ie 3 Taking limit as x >», we get 2 5 2 a= |4 3 1]=-43 713 54, Put Vx= tin AGx)ix* and write 2-M S+7t 2| 4,0 = [4-77 342 1 7-8 I-13 -3 54% 2 | 2-30 72 AO = |-i4e 34% 1] + )4-77 21 -~8 1-1 3 | 7-8 -13 = @ = M40) = 73 55. Asag= Owe get ay + gx + yx? + a,x? 2x°-3x Sx+7 2 =] 4x7°-7 3x42 1 Tx -8x x-1 3) Differentiating we get a; + 2ayx + 3ax? a You have either reached 2 page thts unevalale forvening or reached your ieving tit for this book. a You have either reached 2 page thts unevalale forvening or reached your ieving tit for this book. a You have either reached 2 page thts unevalale forvening or reached your ieving tit for this book. Ans.0 Solution We have Loren 6nin+2) fin) 321-6 gin) Linney nine yens2) him = (n+) But 3 @r+1) =2 Srena2n 7 mt rt +n=n(n42), S2- and ¥ tare lin ee » z re yer a a r-4, a _ Catalase _nint1)Qn+1) 2 6 = Emin +D@+2 Thus, . n(n +2) 6nin+2) fin) La- a1 62-1) gin) = 0 ” nln +0042) nine D642) ond {'s C, and C, are proportional] Example 71 If 0 < 8 «, and the system of equations x= (sin @) y + (cos 8) z y=rt (cos 6) x z= (sin A xty has a nontrivial solution, find 46/2. Ans. 3 Solution We rewrite the system of equations as follows: =x + (sin @) y + (cos 0) 2=0 (cos @)x-y+z=0 Gin @)x+y~-z=0 As this system of equations has a non-trivial solution 1 sin@ cos8 cos@ 1 1 j= sind 1-1 => -1+sin? 6+ cos’ 8+ 2 sin 6 cos 8+ 1 = = 20=3x/2 => sin2@=-1 => 40ln=3 Determinants 8.21 Example 72. Ifthe system of equations axr+hy+e=0 @ he + by + f=0 @) and ax’ + 2hxy + by? + 2gx + fy torre 0 G3) has a unique solution, and abe +2 fgh - af? - bg? - ch? = 8, find 1. ‘ab Ans. 8 Solution Since we should be able to solve (1) and (2) for xand y, we must have ab — h? #0. We can rewrite (3) as, Max t hy + g)+y (int by+f)+ grt fytert=0 @ ‘Since any solution of (1) and (2) must be a solution of (3), the given system of equations can be rewritten as acthy +g = 0, he + by +f=0 and grtfrctr=0 Since the above system of equations is consistent, we must have - ah ¢ hb f|=0 is f ct ah gl lah o hb fl+]h b 0 efclle fe = [abe +2 fgh — af? — bg? — ch’) +t (ab- W) =0 ; abe + 2 figh - af? — bg? - ch® W ab Example 73. Find the number of values of 1 for which the system of equations (a+ 2x + by + z= 0 be + (c+ 2) +az=0 cx tay + (b+ 2) 220 has non-trivial solutions. Ans. 3 Solution ‘The given system of equations will have a non- trivial solution if att ob ec Os) bb c+h © a > a |=0 bed Clearly A is a cubic polynomial in ¢ and has 3 roots. Example 74 Find the number of real roots of the ‘equation 6-Sx x-2 4-44 x-2 2-x 0 |=0 4-4x 0 3-5 AQ) = a You have either reached 2 page thts unevalale forvening or reached your ieving tit for this book. a You have either reached 2 page thts unevalale forvening or reached your ieving tit for this book. a You have either reached 2 page thts unevalale forvening or reached your ieving tit for this book. A(x) = 0 has only one real root, viz. -6. It lies in [-7, 7] and [-7, 0}. (d) Denote the determinant by A(x). Applying C, + C, + C; + Cy, we obtain L4-x 2 AG) =(7-29)]1 20 dx 1 4-x 2-2] Use Ry > Ry — Ry, Ry > Ry Ry 14-4 2 AG) =(7-24) [0 x-2 2-4 0 0 =x = -WG-HE-a ‘Thus, real roots of A(x) = 0 is are 0, 2 and 7/2 which lie in [-7, 71, [0, 4] and (0, 61. ‘SECTION VI REASONING TYPE Geos Gnas Example 80 Let A,= | dy. dyes p47 Gne2 Anes noe Statement-1: Ifa, > OV K2 1 and ay, a, ay... are in GP. then A,=O0Vn21 Statement-2: If a), a). a3... are in A.P. then A,=O¥n21. Ans. (b) Solution Leta,=ar~'Wk2 1, then 11 4 Dn Ges dave |r or oF Next, if a, = + (k~ I)d, then using C, > C,~ C, and C; > Cy Cy, we get in 3d Bd A, = /a4; 3d 3d] =0 qr 3d 3d Cy and C, are identical] a 0 cosx -sin x} Example 81 Letfix)=|sinx 0 — cosx cosx sinx 0 Determinants 8.25 ‘Statement-1: If sin 2x = 1, then f(x) = 23 Statement-2: f(x) = 0 if sin x= cos x Ans. (d) Solution Multiplication of two determinants leads us 2 fa=j-y 1 y yy where y= sin x cos x Using C) > C,- C,, C+ G+ Cy, we get 10y 10 y F(x) = U1 + yy 1 yl=d+y*]0 1 2y) out ord = (1+ (1-29) When sin 2x = I, y= 1/2 and f(x) =0 When sin x = cos x, 1 - 2y = 1-2 sin*x = cos 2x=0 fe) = 0. Example 82 Suppose x > 0, y > 0, > 0 and log? 3. 15-+log (a")| Ala, 6, 6) = |ylog3 5 25+log(b?) zlogS 7 35+log(c*) Statement-1; A(8, 27, 125) = 0 Statement-2: Ans. (b) Solution Using log (6) Cy Cy -SC, we get xlog2 3 xloga Ata, b,c) = | ylog3 5 ylogb clogs 7 zloge A(8, 27, 125) = A(2', 3°, S*) = O as in this case Cy and C; are proportional. Simitarly, AU2, 13, 1/8) = AZ", 31,5 Example 83 Let a#0,p #0 and ¢ log and applying abe A=|0 pq pqo Statement-1: If the equations ax’ + bx += 0 and px +q=0 have a common root, then A= 0. a You have either reached 2 page thts unevalale forvening or reached your ieving tit for this book. a You have either reached 2 page thts unevalale forvening or reached your ieving tit for this book. a You have either reached 2 page thts unevalale forvening or reached your ieving tit for this book. a You have either reached 2 page thts unevalale forvening or reached your ieving tit for this book. a You have either reached 2 page thts unevalale forvening or reached your ieving tit for this book. a You have either reached 2 page thts unevalale forvening or reached your ieving tit for this book. a You have either reached 2 page thts unevalale forvening or reached your ieving tit for this book. @ a 12 () > (@ - 54 (©) f(2) © V4 @ FO () 18 x oid 82. If0SxSlandf(x)=|-1 x 1), then fix) has [1-1 x (a) least value 4 (b) greatest value (q) I (c) a maximum at mo (@ a minimum at ® -1 83, Let 2ootx -1 0 f@) 1 cotx =I 0 1 2eots| then @ [fod @ -16 Ii P () fala) @ - 163 © fais) 8 (a) f(r) (3) 0 x oy oxty 84. Lt d=| y oxty x sty oxy 1 y y Ay=ll xty oy Lox xtyl, xty x42y x 0 “1 ok 2 1 then (@) A (p) 37 (w+ y) (b) a: @xy ., © 4; () - 2% +y) (d) Ay (s) &@-y)O-DU-x» 85. Ifa, b, ¢ are distinct, match the right side with de- gree of the polynomial on the left side. x@ ob @ |x? bo ieee (p) 0 Determinants 8.33 wa xtta? xtal (b) [xP +b? xb? xt qt xtc atte? xte toe x? ©) jt ott m2 ee et 14 0 @ii-2 5 (s) 3 12x Sx? 86, Match the expression on the left with the appropri- ate value on the right sin(x+a) sin(x+) sin(x+7) (@) If fix) = leos(x+a2) cos(x+A) cos(x+7)} (p) - 1 isin(B+y) sin(y+a) sin(at, A and (0) = 0, then} f(j) equals is (b) If sin 2x = 1, then 0 cosx -sinal® fx) =|sinx 0 cosx| equals @o x osinx 0 x] osx xl sinx (©) If fix) = Jooseex tanx secx xsinx 2 Bs then J f(x) dr equals an (@) Maximum value of the determinant A +sin? x sin? x (s)3 cos*x — sin2x I+cos?x sin 2x cos*x 1+sin2x) SECTION VI REASONING TYPE 87. Statement-1: If sin? x 5 io: x sinx—cosx sil x ein Sia wll a You have either reached 2 page thts unevalale forvening or reached your ieving tit for this book. a You have either reached 2 page thts unevalale forvening or reached your ieving tit for this book. a You have either reached 2 page thts unevalale forvening or reached your ieving tit for this book. 28. 29. Let f(x) = a + bx + 6x7 and @# 1 be a cube root of unity, and ja be As|lbeca cab then factors of A are (@) £0) ©) fo © fw) (@) none of these. ‘The system of equations (+ ax tye ayel x + (14 Ay tay =a my +a, + (1+ Aly = 2 (a) is in consistent if 4=0 or A= -3 (b) has a unique solution if A(2 + 3) #0 (c) has infinite number of solution for A = ~ 3 (d) none of these Let Lay) > last + lay), Lb! > byl + Lbs! and Ley! > lel + leg! and let a, ay ay Bald by by a ae then (@ 420 () A=0 (©) lAt> 0 (d) none of these SECTION Ill COMPREHESION TYPE Paragraph for Question Nos. 31 to 34 ayy zh) IAs lay a ay 4) yy yy - Affe xo and cy = (-1)# (determinant obtained by deleting ith row and jth column), then fy ta Os ey Gn Oa] = a, en ea es xe =7 and 32. 33, 34, Determinants 8.37 @a (b) A = 343 @ A=-49 @ a= 49 Suppose a, b,c € Rua +b+c>0,A=be- B= ca-b and C= ab-c, A BC and BC Aj=49, CAB abe then bc a| equals ea 6 (a) -7 (b) 2401 (ec) - 240) (d) 7 Ifa’ +b +2 — 3abe =~ 5 and A, B, C are as in Question 32, value of aA + B+ cC is @-5 5 ©2 @ -25 If A, B, C are as in Question 32, 3ABC - A’ - B -C=# and bea Sale a b abe then sin? (A) + cos’ (A) equals (@) 1 @)=1 (OA) mone of these Paragraph for Question Nos. 35 to 39 35. Sometimes it is easier for us to evaluate a deter- minant if we can express it as product of two determinant. For instance, if Sy =a +b’ + ct, then SSS S, S; S| can be written as A? SS tid A=|a b a Bt which is quite easy to evaluate. If a, > a; > a, b; > by > by and a,b, #1 for 1 liz x = = 1 on OF (1988) ‘The value of the determinant La abe 1b bP =cal is (1988) 1 ¢ c*-ab) Let a-l on 6 A,=|(a-)? 2n? 4n-2 (a-1)° 3n° 3n? -3n show that ¥ 4. = 6. a constant. [1989] Let the three digit numbers A28, 3B9 and 62C, a You have either reached 2 page thts unevalale forvening or reached your ieving tit for this book. a You have either reached 2 page thts unevalale forvening or reached your ieving tit for this book. a You have either reached 2 page thts unevalale forvening or reached your ieving tit for this book. 87. (a) 90. (a) 10, 13. 16. 19, 21. 23. 27. 29. 31. . (b) 37. 43. 46. 48. 49, 0) @ ©) © ©) © fa), (b), (©) ( (a), (b) da) (d) (b), (€), d) fa), (b), (©) (@), ©), (@) aoge SECTION VI 88. (a) 89. (d) 91. (©) LEVEL 2 SECTION I 2 @ 3. @) 5. (b) 6. (d) 8 () 9. (a) 11. (@) 12. (b) 14. (b) 15. @) 17. ©) 18. (a) 20. (b) SECTION I 22. (a), (b), (©) 24, (a), (b) 30. (a), (©) SECTION Itt 32. (@) 35. (a) 38. (d) SECTION IV 4nd 44.17 47. 12 SECTION V Paris ©9200 @O@O® © 1®@@® OOO® pars @O@O® @O@@O® ®@OO® O@O® 33. 36. 39, 42. 45. 26. (a), (b), (€). (@) 28. (a), (), (©) @) d) a) 5 9 10. 1. 12. 13. 15. 16. 0 19. 20. 2 23. 21. 30. 32. 33, 37. NOW eee Determinants 8.45 50. pars @@OO) @@OQ) © @®@@O C©QO®) pars @@@O @@OQ * @@OO SC@O®) SECTION VI $2. (b) 33. @) PAST YEARS' IIT QUESTIONS k=332 0 x=S5A2,y=A32=4A=6 @ x=ly=0,2=0,w=0 m< —1S/2orm>30 ee o SI. e 0 {-1,2} (b) False 27 x= (4 ~ SRT. y = (13k - 97 z= kwhere ke R. 0 (a), (b). (©) False (b), (e) O= ngor = ma+ (- 1)" 116,n,me I. = (15+ 32/60 @), © 0 2 0 Az sin 2a+ cos 2a; a= maor a= kx+ 4 where m,ke 1 @) 4d* © a(a+d) (a+ 2d) (a +3d)"(a + 4d) @ 35. @) 36. 0 () 38. @) 39. (a) a You have either reached 2 page thts unevalale forvening or reached your ieving tit for this book. a You have either reached 2 page thts unevalale forvening or reached your ieving tit for this book. a You have either reached 2 page thts unevalale forvening or reached your ieving tit for this book. But cos (2773) = cos (— 1/3) = — cos (7/3) = ~ 1/2. and cos (47/3) = cos (+ 1/3) = —cos (#13) =~ 1/2. 44. First take a, 6, common from C,, C3, Cy and then from Ry, Ry, Ry A(x) = 3a°b*e7x? (3 + x) 45. Expanding the given determinant along C,, we get eosr 1 | il 0 = 2 cose Ns we 1 2c0sx|7]1 2c0s x = (2.08 x) 4 cas* x 1) -2.c08.x=8 cos'x~4 cos x 46, Multiplying C; by sin fi the given determinant is wd 1 | ome sinarcosf sin a |e in? As ‘in| sin @ anesp cos asin’ 8 0 -sin 8 sin Boos B Then C, > C; + (cos AIC) gives cosa sinarcosB A= 5|-sina cosercosf simp] 9 sing sin a(cos? B+ sin? ) cos a(cos* B+ sin? B) 0 cosa sinacosf sina = ——|-sin@ cos@cosf cos 0 ~sing 0 Finally, expanding along Rj, the determinant is L cose sine A= ——(sin i sin B =sina cosa = cos? @+ sin’ a@= 1. 47. By the law of sines we can write sin A = ak, sin B= bk and sin C = ck, so that the given determi- ‘nant equals a bak cak 1 bk ck bak 1 cosA| =a? {bk 1 cosA cak cos A I ck cosA 1 1 sinB sine =a'|sinB 1 cosAl. sinC cosA 1 After C+ C, - (sin B) C, and Cy Cy = (sin ©) C,, this becomes 48. 49. 1 0 0 1sin? B sinC cosA—sin BsinC which can be expanded along R, to give @ (cos? B cos’ C — (cos A - sin B sin C)"), a But since A, B and C are the angles of a triangle, we have cos A = cos [4 (B + ©)] =- cos (B+ OC) =~ (cos B cos C~ sin B sin C) = cos A ~ sin B sin C= ~ cos B cos C. 2| sinB Putting this back in (1), we see that the value of the determinant is 0. Doing C;— C~ C, and Cy > Cy ~ C; to the given determinant, we get 1 0 ° l+sinA — sinB-sinA —sinC-sinA |, sin A+sin? A’ sin? B—sin* A sin?C~sin? A which, after taking sin B - sin A common from C, and sin C~ sin A common from C;, becomes (sin B ~ sin A) (sin C - sin A) 1 0 0 I+sind 1 1 sinA+sin?A sinB+sinA sinC+sinA = (sin B~ sin A) (sin C= sin A) (sin C- sin B). Since the determinant is zero, we must have sin B = sin A or sin C=sin A or sin ‘in B, i.e., B=A or C=A or C= B. Inall three cases we will have an isosceles triangle. On expanding the given determinant, we will get six terms of the form (1 + x)" = g(x) (say). The coefficient of x in each of these terms is then m = (0). Therefore the coefficient of x in the given determinant A(x) is 4’(0). Now, ahd xeh-! +x) d+ Aa) =] a, 0 +9729"! dae +a) abt xh! +x (40% | (+a ab+x% 7) 42% +) C40 a.dd+9%*"! den C+ ah +y*-! 49% a You have either reached 2 page thts unevalale forvening or reached your ieving tit for this book. a You have either reached 2 page thts unevalale forvening or reached your ieving tit for this book. a You have either reached 2 page thts unevalale forvening or reached your ieving tit for this book. 82, 85. Ry, we get ax ax- f@= | & bei =F) = ax +b Integrating, we get SQ) = ar +bx+e where ¢ is an arbitrary constant. Since fhas a maxi- mom at.x= 5/2, S'G2)=0=5a+b=0 ay Also {O)=2=c=2 and f()slseatbtest a at+b=-1. 2 Solving (1) and (2) for a, b we get a= 14, b= 5/4 Now, f’ (0) = b =~ 5/4 ‘Use fo) = x" + 3x Use R, + R, +R, followed by C, > C, ~ C;, to get £43) = Acotxcosec*x =f (ay=8 F'@) = = 4 cosectx = 8 cot*x cosectx = f'(als) = - 16 Finally, [E90 ae=- 2 cot? x} 3,=- 16/3 and f(22)=0 Apply C, > €, + C, + €; followed by Ry Ry ~ Ry, Ry > Ry— Ry t0 obtain 4)=-20° +y). In As, apply Ry > Ry ~ Ry, Ry > Ry -R, to obtain A=ay Apply R; > Ry +R, to obtain Ay= 3x7 (x+y) It is easy to show A, = (xy) (y- 1) (1-2) (a) Expand along C,. ) Use Ry > R ~ Ry Ry Ry~ Ry and the expand along Ry. (o) Use C) > Cp =). (@) Expand along Ry. (a) Show f'(x) = 0. 1-2 v2. v2 v2 1 (b) fix) Determinants 8.53 (©) Show fis an odd function, (d) Use C, > C, + C,+ C3 to obtain (fis) = (2+ sin 24) A, where cos? A, =I L+cas? x sin 2x sin 2x cost 1 +sin 2x) 0 0 Using Ry > Ry ~ Ry and Ry Ry ~ Ry. Maximum value of f(x) is 3. 87. Let A be a skew symmetric matrix of onder 1, then Wi = © DTA) = l= "AL If mis od, then (Al = — (Al => 241 = 0 = l= 0. We have 0 sin(@/12) cos(/12) (2) «| -sin(a12) 0 sec(#/12) 47 | -costei2) ~sec(a/12) 0 using statement-2} 88. Statement-2 is true, See theory page 4.3. The roots of x” = p are p'” o” where de 2x @=cos = +isin =, alue of determinant A depends an a, a... . a. If we put a =p"? a, then 1a of eS |l o a 1 @ oF However, if a= p"”, @ = p'” a, ay = pl? a, a = p'” an a = a, = p'? oh, ae, = p'” of, a =p ah, a =p of, then 1 a of A=—'X\ 0 o oF ow &f = p' Qa - w - ob) #0. 89. Statement-2 is true. Using R; >» Ry - VIR, and Ry > Ry- VIR, we get a You have either reached 2 page thts unevalale forvening or reached your ieving tit for this book. a You have either reached 2 page thts unevalale forvening or reached your ieving tit for this book. a You have either reached 2 page thts unevalale forvening or reached your ieving tit for this book. 9.7 TOTAL PROBABILITY THEOREM Let (#4). for i#jand (J H,= S. Suppose that P(H,) > 0 for 1 Sis m= n, Then for any event A P(A) = ¥ PH) P(A 1H) rt 9.8 BAYES' RULE +2, rn, be m events such that 0. Hy = 0 Let $ be a sample space and H, . H, be n mutually exclusive events such that|_) H; = $ and P(H,) > 0 for = J=1,2,-m. We an think of the Hs as the “causes (or the hypotheses) that lead to the outcome of an experiment. ‘The probabilities P(H), j = 1, 2, , w are called prior probabilities. Suppose the experiment results in an ‘outcome of event A, where P(A) >0, What is the probability that the observed event was due to cause H,? In other words, we seek the conditional probability P(H,1A). ‘These probabilities are frequently called posterior probabilities. The information that A has occurred makes us reassess the probability P(H,) assigned to H,. 9.9 THEOREM (BAYES' RULE) Let {#,} be mutually exclusive events such that P(H,) >0 for 2, mand S= UJ H,, Let be an event with P(A) rs > 0. Then for j= 1,2, ++, PH, 1A) = Pu )P(ALH,) > put rainy) t Fig. 9.4 9.10 INDEPENDENT EVENTS: We say that two events A and B are independent if the ‘occurrence or non-occurrence of A(B) does not affect the probability of occurrence or non-occurrence of B(A), that Probability 9.3 is, if P(B | A) = P(B) provided P(A) # 0 PIBA A) PA) Thus, two events A and B are independent if and only if P(A. B) = POA) PCB). If two events are not independent, they are said to be dependent. We note that if P(A) = 0 then for any event B, 0S P(A.1.B) PIA) = 0, so that P(A A B) = PIA) PCB) and A is independent of any other event. = P(B) = PIA B) = P(A) OB) = Difference between Mutual Exclusiveness and Independence We advise the reader to distinguish between inde- pendent and mutually exclusive events. Independence isa property of probability, whereas mutual exclusion is a set-theoretic property. If A and B are mutually lexclusive events with P(A) > 0 and P(B) > 0, then P(A B) = 0# Pt) PB), so that A and B cannot be independent. In fact, P(A |B) = 0 = P(B | A), so that if ‘A occurs, B cannot occur and vice versa. Thus A and B are strongly dependent when A 0B = @. We also note the following fact: two events A and B are independent if and only if (i) A and BY are independent, or ii) A’ and B are independent, or (ii) A’ and B are independent. Three events A, B and C are said to be pairwise independent if P(A 0 B) = PCA) PCB), AB A ©) = PIB) PCO), PC 9 A) = PO)P(A) Three events A, B and C are said to be mutually independent if P(A © B)= P(A) PCB), PB. ©) = PB) PO), PC A A)= PO) PAY and P(A 9 BA C)= POA) PUB) PC) 9.11 RANDOM VARIABLES Let $ be a sample space. A random variable X is a function from the set $ to R. the set of real numbers. If X is a random variable defined on the sample space $ and is a real number, then {X =r) = (weS1x Gwe) =r} is an event. If the random variable X takes n distinct values By ay kp then (X = om}. (X = a), {X = x,} are mutually exclusive and exhaustive events. (Fig. 9.5). a You have either reached 2 page thts unevalale forvening or reached your ieving tit for this book. a You have either reached 2 page thts unevalale forvening or reached your ieving tit for this book. a You have either reached 2 page thts unevalale forvening or reached your ieving tit for this book. (a) 7 Ans. (d) Solution Suppose the coin is tossed n times. Let X be the number of heads obtained. Then X follows a binomial distribution with parameters m and p have POX2208 = 1-PX = 1G y? pr z08 > (2) s02=4 3725 This show that the least value of n is 3. Example 14 For the three events A, B and C, P (exactly one of the events A or B occurs) = P(exactly ‘one of the events B or C occurs) = P(exactly one of the events C or A occurs) = p and Pall the three events ‘occur simultaneously) = p", where 0 < p< 1/2, Then the probability of at least one of the three events A, B and C occurring is m6 OS Wi 2 2 (a fee ) fae w@ 2 cay 32420" Ans (ay? 4 Solution We know that Plexactly one of A or B occurs) = P(A) + PCB) — 2P(A 9 BY ‘Therefore, P(A) + P(B) - 2P(A NB) =p a) Similarly, P(B) + P(C) - 2PB 9 C= p @) and PIC) + PA) - 2(C 9 A) =p @) Adding (1), (2) and (3) we get 2{P(A) + PB) + PO) - PA 9 B) ~ PB AO = PEM AY} = 3p = PIA) + PLB) + P(C) ~ PA 9 B) ~ PBC) PCO A)=3p2 (4) We are also given that P(A A Bm C) = p* 6) Now, Pat least one of A, B and C) = P(A) + P(B) + PCC) - PA 9 B) =PBAO- ACOA) + PANBNO 3 3p + 2p? aha pe BO fusing (4) and (5)} Example 15 Nine identical balls are numbers 1, 2, ..9, ate put in a bag. A draws a ball and gets the number a. ‘The ball is put back the bag. Next B draws a ball gets the number b. The probability that @ and b satisfics the inequality a -2b + 10 > Ois 52 55 @ a (b) u of @ & Probability 9.7 Ans, (c) Solution a-2b+10>0 SIs b<(a+12 Thus, b & [1, (a + 10972) When a= 1,2, be tl, $} When a= 3,4, be {1.61 When a= 5,6, be (1.7) When a= 7,8, be (1.8] When a =9 be [1,9] ‘Thus, the inequality a — 2b + 10 > O is satisfied for 2x 5 + 2x642%742%8 +9 =61 ordered pairs (a, b). Total number of ways of choosing (a, b) is 9x9 =81 required probability is e Example 16 An unbiased die with faces marked 1, 2,3, 4, 5 and 6 is rolled four times. Out of four face values ‘obtained, the probability that the minimum face value is ‘not less than 2 and the maximum face value is not greater than 5 is then, (a) 16/81 Ans. (a) Solution Let _p = probability of getting face value not less than 2 and not more than 5 in a single throw of die 416 = 23. n= number of times die is rolled X = number of times we get a number not less than 2 and not more than 5. Then X ~ B(n, p) Required probability = P(X = 4) = 4C, pt = (23)* = 16/81. (b) 1/81 (©) 80/81 (4) 65/81 Example 17 If the papers of 4 students can be checked bby any one of the 7 teachers, then the probability that all the 4 papers are checked by exactly 2 teachers is (@) 7 (by 12/49 () 32/343 (@) none of these. Ans. (b) Solution ‘The total number of ways in which papers of 4 students can be checked by seven teachers is 7*, The number of ways of choosing two teachers out of 7 is "Cy. The number of ways in which they can check four papers is 2°, But this includes two ways in which all the papers will be checked by a single teacher. Therefore, the number of ways in which 4 papers can be checked by ‘exactly two teachers is 2* = 2 = 14, a You have either reached 2 page thts unevalale forvening or reached your ieving tit for this book. a You have either reached 2 page thts unevalale forvening or reached your ieving tit for this book. a You have either reached 2 page thts unevalale forvening or reached your ieving tit for this book. both X and Yare binomial variate with parameters n = 3 and p = 1/2. Probability that both A and # obtain the same number of heads is PX = 0) PY = 0) + PX = DRY = D+ P(X = 2) PUY = 2) + PX = 3) PUY = 3) pe ftVT pelt pel abe (tT -| (3) ‘| <(3) | 4 (5) 4 (3) iy 205 = (3) o+9+94n= Bak. Example 32. The probability that a student is not a swimmer is 1/5. The probability that out of 5 students exactly 4 are swimmer is ay w (4) (4) @ (3) (ay © 5 (3) Ans. (b) Solution Let p = probability that a student selected at random is a swimmer = 1 ~ 1/5 = 4/5. ‘n= number of students selected = 5 and X= number of swimmers ‘Then X ~ B (n, p) where n = 5 and p = 4/5 Probability that exactly 4 students are swimmer is mene I GI mete OET Example 33 Six distinct numbers are selected from first 150 natural numbers. The probability all the six numbers are divisible both by 3 and S is (a) 1/33 (b) 1/35 (©) a9 (4) none of these Ans. (d) Solution Total number of ways of selecting 6 numbers out of 1, 2, ..., 150 is Cy, ‘A number is divisible by 3 and 5 both if and only if itis divisible by 15. Thee are 10 numbers viz. 15, 30, 150° that are divisible by 15. Thus, probabi required event " of me, Example 34 There are two balls in an um whose colours are not known (each ball can be either white or black), A white ball is put into the urn, A ball is drawn from the urn. The probability that it is white is (@) 4 Ans. (0) 1B ©) 43d) 16 Probability 9.11 Solution Let E, (0S iS 2) denote the event that urn contains i white and (2 ~ i) black balls. Let A denote the event that a white ball is drawn from the urn, Wehave P(E)= 13 for i= 0, 1, 2. and PA | E\)= 13, PAL Ey) = 2/3, PAL Ey) = 1. By the total probability rule, PA) = PE) PA |B) + PLE) PALE) + IE) PA | E3) if! 2 | 2 = s|a+sti[== 3|3°3 3 Example 35 A natural number x is chosen at random from the first one hundred natural numbers. The probability that sae oy <0is r (a) 1/50 (bo) 3/50 (c) 325 (d) 7725 Ans. (2) Solution Sign of E is same as that of sign of (= 20) (x= 30) (x= 40) = Fisay). Note that F <0 if and only if 0 ined +1241) _ 2nst 6 3a” P 69. P(E\IA) = 2 1 an n(n+l)n2n+1 =i _, mn 1)Qn+1)" P(E, )P(ALE.) P(A) 70. PCE;A) = Probability 9.19 2k kan “nlnti) a n+l a? = ies i)(Qn4 1) = Dereia=1. mo Paragraph for Question Nos. 71 to 75 Suppose E), E;, Ey be three mutually exclusive events such that P(E) = p, for i= 1, 2. 3. 1 Example 71 If p; =} 0-p, pie 5 +2) and p, = z (2 + 3p), then p belongs to @ G12 () © 1/3, 213) ©) © 1, 147) (@) (-1/2,-7/23) Example 72 If), pp p3 are the roots of 29x) ~ 27x + ax-1=0 then value of a is (a) 9 ©3 (b) 6 (@ (0, V5) Example 73 W p,= F043, pie 5 ~p)and Py i (1 ~ 2p), then p belongs to (a) © 13,1) (b) (= 109, 1/4) (©) (- 23, 23) @ C10 Example 74 P(none of Ey, E;, E3) equals (a) 0 (b) py + P2 + Py © (=p) =p) A= py (A) none of these Example 75 P(E, 0) Ey’) + P(Ey 0 By’) + PUES Ey’) equals (a) pl = 2) + Px ~ py) + ps — Py) (>) Pipa + P2Ps + PsP1 (©) Pi + Po + Ps (d) none of these Ans, 71. @), 72. (@), 73, (0), 74, (@), 75. ©) Solution 71. p, 20, p20, p32 0 and Pit Prt Psst a You have either reached 2 page thts unevalale forvening or reached your ieving tit for this book. a You have either reached 2 page thts unevalale forvening or reached your ieving tit for this book. a You have either reached 2 page thts unevalale forvening or reached your ieving tit for this book. If-n positive integers are taken at random together, and p, is the probability that the last digit of the product is 2, 4, 6 o¢ 8 find 125p5. Ans, 56. Solution The last digit of the product will be 1, 2,3.4,6, 7, 8 or 9 if and only if each of the n positive integers ends in any of these digits. Now, the probability of an integer ending in 1, 2, 3, 4, 6, 7, 8 or 9 is 8/10 = 4/5. Therefore, the probability that the last digit of the product of integers is 1, 2, 3, 4, 6, 7, 8 or 9 is (4/5)". Next, the last digit of the product will be 1, 3, 7 or 9 if and only if each of the n positive integers ends in 1, 3, 7 or 9. The probability for an integer to end in 1, 3, 7 or 9 is 4/10 = 2/5. Therefore, the probability for the product of 1 positive integers to end in 1, 3, 7 or 9 is (2/5)" . Hence, the probability of the required event is ( Put n=3. Example 95 Two integers rand s are drawn one at a time without replacement from the set 1, 2.9. If Pa Pir Sk 18'S), find 4p, if n= 25. Ans. 1. Solution We have Ps kis < hy = Mrs Wand (s $0) Pissk) Also, Pissk) = PUir sb) and (5S )) = Thus, re = kDa") Putn = 25 and k=7 Example 96 A biased coin with probability p(0 < p < 1) of heads is tossed until x head appears for the first time. If the probability that the number of tosses required is even is 2/5, find 3p. Ans. Solution Let X denote the number of tosses required. ‘Then P(X =r) = (1 —py¥'p for r= 1,2, 3, --- Let Edenote the event that the number of tosses required is even. Then PE) = PUK = 2) Ka 4) UKE HU] = PIX = 2) + PX = 4) + PIX = 6) + = (1 =pp+ (=p) p+ pi p+ Probability 9.23 = lap). p= p) Io G-pr 2p-p?> 2-p As we are given that PUE) = 2/5, we get S(1 — p) = 22 p) 9 1-3p =0 or pos Example 97 There are 10 pairs of shoes in cupboard, from which 4 shoes are picked at random. If p is the probability that there is at least one pair, find 323p. Ans, 99. Solution ‘That total number of ways of choos-ing 4 shoes (in order) out of 10 pairs (or 20 shoes) is 20x 19 x 18 x 17. The number of ways in which no pair is selected is 20x 18 x 16x 14, Thus, the probability of not getting a pair is 20x18 x 16x14 _ 224 2Ox19x18xI7 323 Hence, the probability of getting at least one pair is 1 = 224/323 = 99/323 Example 98 The probability of « bomb hitting a bridge is 1/2 and two direct hits are needed to destroy it, Find the least number of bombs required so that the probability of the bridge being destroyed is greater than 0.9. Ans. 8. Solution Let n be the least number of bombs required and X the number of bombs that hit the bridge. Then X follows a binomial distribution with parameters and p= 1/2. Now PXZY>09 3 1-PX<2>09 = AX S04 PK 21 < 01 ~ of) oly Qed 2 2) \2)"10 nth od et 2 10 By trial and estor, we get 2 8. Thus, the least value of nis 8, Example 99 Suppose X follows a binomial distribution with parameters n = 6 and p. If 9P(X = 4) = P(X = 2), find 4p. = > mec? Ans. 1. Solution 9P(X = 4) = PIX = 2) = 9 °C, pig’ = °C; pq* = = = apt a You have either reached 2 page thts unevalale forvening or reached your ieving tit for this book. a You have either reached 2 page thts unevalale forvening or reached your ieving tit for this book. a You have either reached 2 page thts unevalale forvening or reached your ieving tit for this book. +. probability of the required event _413tay 1 9 (7es)(*e2) (b) We can permute O7L1,N, D,1A.N in ways. *. probability of the required event _ 2421312! 719) Pavan (©) Fixing an /at the first place and L at the last place, we can permute the remaining letters viz. A,DLANN, Oi ways. 212! + probability of the required event is, 2rarsiar 1 “798” BePejed (d) Vowels can be arranged at odd places viz 1". 3, 5*7 and 9" in Says ‘The remaining letters can be arranged at 4 even places in © ways. ‘probability of the required event REASONING TYPE Example 108 Statement-1: Let A and B be two events such that P(A W B) = P(A 0 B) then PAN B= PAO B)=0 Statement-2: Let A and B be two events such that P(A UB) = PA 9B) then P(A) + PUB) = Ans. (c) Solution P(A B) = AB) $= P(A) + PB) = PLA 9 B) = PAB) 9 [P(A) ~ PA 9B) + (P(B)- PA B=0. (1) But P(A), P(B)2 P(A 9 B). ‘Thus, (1) is possible if and only if P(A) ~ P(A 0B) = O and PUB) ~ P(A 9) B) = 0 = PIA) = PLB) = P(A) B) + P(A) + PCB) = 1 need not hold. Next, P(A 9 BY) = PUA) ~ PA 9B} Similarly P(A" 9) B) = PCB) PAB) = 0 Probability 9.27 Example 109 Statement-1: we (+ 5p) 4 (1+ 2p), 4 1 =p) and ya = 3p) are probabil exclusive events, then p can take infinite number of values, Statement-2: If A, B, C and D are four mutwally exclusive events, then PIA), PCB), PCC), PAD) = 0 P(A) + PKB) + PCO) + LD) S| Ans. (a) Solution Statement-2is true, See theory. s of four mutually = p2-WUS,p2-12,pSl,ps 3 =-USps-tWil. Q) From (1) and (2) we get~ 1/5 Sp $— WIL => there are infinite values of p. Example 110 Statement-i: A natural number is chosen at random. The probability that sum of the digits of its square is 93 is 0. Statement-2: A number is divisible by 31 if and only if ‘sum of its digits is divisible by 31. Ans. (c) Solution Let n € N, be such that sum of digits of its square is 98. ‘Then 3in? => 3m => 9b Example 111 Let m ¢ N, and suppose three numbers are chosen at random from the numbers 1, 2,3, .... m. Statement-1: If m = 2n for some n ¢ NY, then the chosen numbers are in A.P. with probability 2@n-1) Statement-2: If m = 2n + 1, then the chosen numbers are in A.P. with probability — re Ans. (b) Solution We can choose three numbers out of m in"C, ‘ways. Let the numbers be x,.¥)..x5. Now, “tye Xge ty are in A.P. if and only if 1X) + ty = ty, that is, if and only if either both x),.x5 are odd of both x), x) are even, a You have either reached 2 page thts unevalale forvening or reached your ieving tit for this book. a You have either reached 2 page thts unevalale forvening or reached your ieving tit for this book. a You have either reached 2 page thts unevalale forvening or reached your ieving tit for this book. 22. 23, 21. 28. 29. 30. If X and Y are independent binomial variate B (5, 172) and B (7, 1/2), then P(X + ¥ = 3) is (a) 55/1024 (b) 55/4098 (©) 55/2048 (@) none of these Suppose n (2 3) persons are sitting in a row. Two ‘of them are selected at random. The probability that they are not together is 2 a @ 1-2 Olen wit (@) none of these A bag contains four tickets marked with numbers 112, 121, 211, 222. One ticket is drawn at random from the bag. Let E; (i = 1, 2, 3) denote the event that ith digit on the ticket is 2. Then which of the following is not true (a) E, and E; are independent (b) £; and E; are independent (c) Ey and E) are independent (d) Ey, Ey Ey are independent If the letters of the word PROBABILITY are writ- ten down at random in a row, the probability that two B's are together is (a) 2 tb) LO) Bt @) OIL If four positive integers are taken at random and multiplied together, then the probability that the last digit is 1, 3, 7 of 9 is (a) 8 (b) 7) 11625. (A) 16/625, ‘Two contestants play a game as follows: each is asked to select a digit from 1 to 9. If the two digits match they both win a prize. The probability that they will win a prize in a single trial is (a) 81) 781) 9) VAL If A and B are two events, then which of the following does not represent the probability that exactly one of A, B occurs is (a) P(A) + PB) ~ PA BY (b) PCAN BY) + P(A’ OB) (©) P(A’) + P (BY) ~ 2P1A’ 9 BY) (d) P(A) + PB) ~ 2PA 9 BY If A and B are two events, then which one of the following is not always true (a) PAB) 2 POA) + PCB) ~ 1 (b) P(A > B) S$ PIA) (c) PA’ 9 BY) 2 PIA’) + PLB) = 1 (d) P(A 9 B) = PIA) PB) A sum of money is rounded off to the nearest rupee, The probability that the round off error is ai. 32. 33. 35. 36. 37. 38. Probability 9.31 at most 10 paise is (@) 10 (b) L110 (©) 325 (d) none of these The probability that an electric bulb will last 150 days or more is 0.7 and that it will last at most 160 days is 0.8. The probability that the bulb will last 150 to 160 days is (a) 05 () 1/3 (©) 056 (@) none of these A bag contains three tickets numbered 1, 2 and 3. A ticket is drawn at random and put back in the bag, and this is done four times. The probability that the sum of the numbers drawn is even is (@) 4081 (b) 481 ©) 1427 (@) 13/81 Two numbers x and y are selected at random from the set {1, 2, 3,.... 3M). The probability that x” ~ y* is divisible by 3 is (a) GN - 1¥3N (b) (N — 1Y/N (©) (SN — 3M(9N — 3) (d) none of these Three persons A, B and C are to speak at a function along with 7 other persons. The probability that A speaks before B and B speaks before C is (a) 470 (b) 16 ©) 7 (d) none of these men and a women are seated at round table in random order. The probalility that they can be divided into nm non-interrecting pairs so that each pair consists of a man and a woman is (a) V2n (b) 22" -1)"c, (©) In/™*C, @ vec? If @ is an integer lying in [- 5, 30], then the probability that the graph of y =x" +2 (a + 4)x —Sa + 64 is strictly above the x ~ axis is (a) 6 (b) 7/36 ©) 49 (@) ¥5. ‘Two numbers X and Y are chosen at random (without replacement) from the set {1, 2, -- SN}. ‘The probability that X*— ¥* is divisible by N 4(4N-1) py) 4aN-D 10 Sy © xsN-D 1IN-5 —_—— (d) of these, © Jorn (4) none of these ‘There is a five-volume dictionary among 50 books arranged on a shelf in random order. The prob- ability that these volumes stand in increasing or- der from left to right (the volumes are not neces- sarily kept side-by-side) is a You have either reached 2 page thts unevalale forvening or reached your ieving tit for this book. a You have either reached 2 page thts unevalale forvening or reached your ieving tit for this book. a You have either reached 2 page thts unevalale forvening or reached your ieving tit for this book. 8. 82, 84, 85. 87, 88, oO. ‘SECTION IV INTEGER ANSWER TYPE Let P(A) = 0.4 and P(A U B) = PA 2 B), find SP(B). A pair of fair dice is rolled together till a sum of 7 or 11 is obtained. Let p denote the probability that 7 comes before 11. Find the value of 4 p. In a test an examine either gusses or copies or knows the answer to a multiple choice question with ‘m choices out of which exactly one is correct. The probability that he makes a guess is 1/3 and the probability that he copies the answer is 1/6. The probability that his answer is comect given that be copied it, is 1/8. If the probability that he knew the answer to the question given that he correctly an- swered itis 120/141, find m. wt +4), i (1=p) and 4 (1.=2p) are the prob- abilities of three mutually exclusive events, find 2p. The digits 1, 2, 3, 4, 5, 6,7, 8 and 9 are written in. random order to form a nine digit number. Let p be the probability that this number is divisible by 36, find 9p. Two squares are chosen at random on a chess- board. If p denotes the probability that they have ‘exactly one side in common, find 144p, In a multiple-choice question, there are five alter- native answers, of which one or more than one are correct. A candidate will get marks on the question if he ticks all the correct answers. If he decides to tick answers at random, find the least number of choices should he be allowed so that the probability of bis getting marks on the question exceeds 1/8? Two persons each make a single throw with a pair of dice, If p denotes the probability that their throws are unequal, find 648 p, In a sequence of independent trials, the prob- ability of success is 1/4. Ifp denotes the probabil- ity that the second success occurs on the fourth ‘trial or later trial, find 32 p. If X follows a binomial distribution with param- eters n= 100 and p = 1/3, find r for which P(X =r) ismaximum, ‘SECTION V MATRIX MATCH TYPE Three players A, B and C alternatively throw a die in that order, the first player to throw a 6 being 92. 94. Probability 9.35 deemed the winner. A’s dic is fair whereas B and C throw dice with probabilities p, and p> respectively of throwing a 6. (a) If py, = US. py) = 4, Op) the probability that A wins the game, () Ip, = US. p= Vt the probability that C wins the game (©) Value of p, so that «r) ; P(A wins) = PUB wins) (2) Value of p; so that the (s) t game is equiprobable to all the three players. Consider a town with N people. A person spreads a numor to a second, who in turn repeats it to a third, and so on. Suppose that at each stage, the recipient of the rumor is chosen at random from the remaining (N ~ 1) people, Suppose rumor is repeated (2 3) times, then probability that it will not be repeated to (a) originator wli-5 yt (1-4) (3) "tp, LN =)" (b) first recipient (c) second recipient (@) any of the earlier recipient . A prisoner escapes from a jail and is equally likely to choose one of the four roads I, ILI or IV to reach away from the hands of law. If he choose I road, he is successful with probability 1/6 and for |. THT and IV this is 1/8, 1/10 and 1/12. Ifthe pris- ‘oner is successful, the probability that he chose road @1 (p) 12/57 © 0 (q) 15157 «© Ml (1) 20/57 @y (s) 10/57 Sixteen players 5,, Sp... Sig play in a tourna- ment. They are divided into eight pairs at random. From each pair a winner is decided on the basis of a You have either reached 2 page thts unevalale forvening or reached your ieving tit for this book. a You have either reached 2 page thts unevalale forvening or reached your ieving tit for this book. a You have either reached 2 page thts unevalale forvening or reached your ieving tit for this book. SECTION I) COMPREHESION TYPE Paragraph for Questions 31 to 35 Let x and y be two numbers chosen at random from the set {1, 2, 3, .... ) with replacement. Let Q,(p) denote the probability that (x?! — y”"') is divisible by p. 31. Qps(3) equals (a) 0.5648 (b) 0.5678 (©) 0.5698 (d) 0.5628 32. Quy(5) equals (a) 0.52 (b) 0.64 (c) 0.68 (@d) 0.75 33, If [x] denotes the greatest integer < x then Q,(p) ‘equals nl], tifa] ot 2}eS [2] (A) none of these 34. lim Q,(p) equals @ 1-242 1-2 pp P @i-4 (@) none of these 2 35. Q,(p) equals (a) 2 1 (c) Ve @ 0 Paragraph for Questions 36 to 40 A permutation of n objects is a one-to-one function from A={1, 2,05 m1) onto itself. Ifa, ay, .. ay are n distinct ‘elements of A such that a function f sends a, tas, a3 t0 ay, oe Qj Fay aNd Gy —¥ ay, tHEN Gy, ay ny are Said 10 form a cycle of length k. ‘A permutation is chosen at random from the set of all permutations of the set A= {1, 2, ... 1}. 36. The probability that chosen permutation is a cycle such that 37, 38. 39, 40. 41. 42. 4s. Probability 9.39 192935..9@-1)> Lis (a) Unt (b) In = 1)! {c) Uat id) 0 Let dys 3, or iy By, Ba one by € A be Such that a's are distinct and b's are distinct. The probability that the chosen permutation sends a, —> b, for F212, on kei (a) Alin! (b) kt (n= 8) ©o @) (n —BYn! ‘The probability that the element i forms a unit cycle ici iis (a) Vat (b) MMe = 1)! (©) Un @o If p, denotes the probability that there is at least ‘one cycle of unit length, then p, equals (ay t- Sew > Sets + ad @1- ze zi (a) none of these lim p, equals (a) Ve (b) 1 - Me @o (4) none of these SECTION IV INTEGER ANSWER TYPE ‘Two persons each make a throw with a pair of cu- bical die and let p denote the probability that their throws are equal. Find the value of 648p. A bag contains m + | coins. It is known that one of these coins shows heads on both sides, whereas the other coins are fair. One coin is selected at ran- dom and tossed. If the probability that the toss re- sults in heads is 7/12, find n. ‘Two squares are chosen at random on a chess board and let py denote the probability that they have exactly one side in common. Find 36p. ‘Two integers r and s are chosen one at a without replacement from the numbers 1, 2, 3, .. 100. Let p be the probability that rs 25 given that s $25. Find the value of 33p. A child arranges 3 distinet black balls, four green balls, and five red balls in a row randomly. If p is the probability that two red balls are not together, find 99p. Two numbers x and y are chosen at random from the numbers from the set {1, 2, 3, ... 300). Let p a You have either reached 2 page thts unevalale forvening or reached your ieving tit for this book. a You have either reached 2 page thts unevalale forvening or reached your ieving tit for this book. a You have either reached 2 page thts unevalale forvening or reached your ieving tit for this book. 36. a 38. 39. aL. 42. B, which option should A choose so that his prob- ability of winning is higher? (No game ends in a draw. [1989} . Let A and B be two events such that P(A) = 0.3 and. P(A B)=0.8, If A and B are independent events, then P(B) (1990) Aisa set containing m elerhents. A subset P of A is chosen at random. The set A is reconstructed by replacing the elements of P. A subset Q of A is again chosen at random. Find the probability that P and Q have no common elements. {1990} If the mean and variance of a binomial variate X are 2 and I respectively, then the probability that X takes a value greater than one is equal to 1991) For any two events A and B in a sample space P(A) + P(B) (AIB) = | p(B) # 0, is al- (a) POAIB) Pia) ) #0, is al ways true (b) P(A 9 B’) = P(A) ~ P(A 9) B) does not bold ©) PAL B)= 1 ~ PAY) PY, if A and B are independent (@) PAU B) = 1 PA’) PB), if A and B are disjoint (1991) In a test, an examine guesses, copies or knows the answer to a multiple choice question with four choices. The probability that he makes a guess is 1/3 and the probability that he copies the answer is 1/6. The probability that his answer is correct given that he copied it is 1/8. Find the probability that he knew the answer to the question, given that he answered it correctly. [1991] . ‘Three faces of a fair die are yellow, two faces red, and one blue. The die is tossed three times. The probability that the colours yellow, red and blue appear in the first, second and the third tosses re- spectively, is___. [1992] India plays two matches each with the West Indies and Australia, In any match, the probabilities of India getting 0, 1 and 2 points are 0.45, 0.05 and 0.50, respectively. Assuming that the outcomes are independent, the probability of India getting at least 7 points is (a) 0.8750 (b) 0.0875 (e) 0.0625 (@) 0.0250 41992) A lot contains 50 defective and 50 non-defective bulbs. Two bulbs are drawn at random, one at a time, with replacement. The events A, B and Care defined as 43. 45. 47. Probability 9.43 A= (the first bulb is defective} B = (the second bulb is non-defective} C= (the two bulbs are both defective or both non-defective) Determine whether (i) A, B and C are pairwise independent and (ii) A, B and C are independent. 11992} An unbiased die with faces marked, 1, 2, 3, 4, 5 and 6 is rolled four times. Out of four face values obtained, the probability that the minimum face value is not less than 2 and the maximum face value is not greater than 5 is then, (a) 16/81 (b) 1/81 (©) 80/81 (@) 65/81 (1993) Let £ and F be two independent events. The prob- ability that both E and F happen is 1/2 and the probability that neither E nor F happens is 1/12. Then, (a) P(E) = 13, POF) = 1/4 (b) P(E) = 1/2, P(F) = 146 (o) PCE) = 1/6, POF) = 2 (d) PCE) = 14, PCF) = 3 [1993] Numbers are selected at random, from the two digit numbers 00, 01, 02, ... 99 with replacement. An event E occurs if and only if the product of the two digits of a selected number is 18. If four nam- bers are selected. find the probability that the event E occurs at least 3 times. 11993] If two events A and B such that P(A") = 0.3, P(B) = 0.4 and P(A A B) = 0.5, then P(BKA U B)) [1994] A box contains 24 identical balls of which 12 are white and 12 are black. The balls are drawn at ran- dom from the box one at a time with replacement. ‘The probability that a white ball is drawn for the 4th time on the 7th draw is 5 27 b) OG OF 5 1 =. > 1994] OF > (1994) ‘An unbiased die is tossed until a number greater than 4 appears. The probability that an even num- ber of tosses is needed is (a) 12 © us (b) 2/5 (@) 23 [1994] a You have either reached 2 page thts unevalale forvening or reached your ieving tit for this book. a You have either reached 2 page thts unevalale forvening or reached your ieving tit for this book. a You have either reached 2 page thts unevalale forvening or reached your ieving tit for this book. 52. @) 53. (©), (d) 54. (a), (b) 55. (a), (b), (©), (d) 56. (a), (b), (©), (A) 57. (a), (b), (©), (B) 58. (a), (b), (©) 59. (a), (b), (©) 60. (@), (©), (@) (b), (©), (d) 62. (a), (b), (©) (>). @) 64, (a), (b) ). (b), (©), (@) SECTION C 66. (b) 67. (c) 68. (a) 69. (a) 70. (a) 71. @) 72.0) 73. ©) 74. (€) 75. (a) 76. () 77. (a) 78. (b) 79. (©) 80. (c) SECTION D 81.2 823 82S 8h 85.2 86.7 87. 488. 73 89.27 90. 33 ow 92. Gee Ceo OOO SO 93. SQ ©e OQ ©) Q Se iO e 95. 96. 100. 2 23. 25. 27. 29. 31. 35. 39, 41. 45. Probability 9.47 Pars *@@OO *@@OO © @@@O 4@@0® pars *O@OO *@@OO® *@@OO *@@0® SECTION F (b) 97. (a) 98. ©) ~— 99. (@) © no SECTION A oo 2 36 4. dd) o 6@ 720 &@ @) 1.6) © 12@ © 14 @) 1S. @) 16. ) ) 18. @ 19%. @ 20 @) SECTION B @), (@) 22. (a), (b), (), (d) (a), (6), 24. (a), () @, @ 26. (a). (b), ©. @ (a), @) 28. (a). (0), (©) (a), (), (©), (@) 30. (b) SECTION C @) 32) 33. @) 34. (@) () 36. (a) 37.) 38. ©) (a) 40. (b) SECTION D B45 420 44.8 7 46.3 47.9 48.10 a You have either reached 2 page thts unevalale forvening or reached your ieving tit for this book. a You have either reached 2 page thts unevalale forvening or reached your ieving tit for this book. a You have either reached 2 page thts unevalale forvening or reached your ieving tit for this book. 58. 39. m=1_, P(ADB) _ =i Now P(B1A)= a (m1)? me Also P(B) = P(A) PBLA) + PAY P(BLA’) “Cale e-) 4) = PAN B)= = PUB) =1~ PB) = PIAQB) _ P(A) (ANB) PAL Bt) = FACE). PAH PANS) ae ae) PB) _om-t © 2m m= (m-1) (m ° P(AUB) = -—— +! -|- . mm m P(B) = P(A) P(B|A) + POA) PBA’) = P(A) P(BIA) + P(A’) [1 ~ PBA’) =a(1 ~ a) +(1 a) [1 ~~ a} =2001 ~ and pea'ipy = LAOB) _ PLB)~ PAB) PB) PUB P(B)= P(A) P(BLA) ~ PCB) a ROG BU ~ 2al-a) 2 vi py = PAOBD PAL B) PAB) = Tay 1- PB) and P(A UB) = P(A) + PCB) ~ PA. B) = PX2n)= PPX w= x “4 PRE m4alX 2m)= P{X=m+n) PRs meni X2m)= P(X 2m) A and B are independent events. PEE,) = °Cy6? = 5154, PCE) = 616° = 1/36, and P(E, 0 E,) = 1/36. 62. Probability 9.51 For divisibility with 9, leave one of (1, (3, 6) or (4, 5). For divisibility with 3, leave one of (1,2), (1,5).(1, 8). 2, 4), 2 Ty 3, 6). B, 9, A, 5), 4, 8), (5,7, QD SB For (d), list all the possible 14 combinations. See Theory, P(AMB) = Piboth show odd numbers) = (3/6) ete, AM BOC means a, band a +b are odd. Impossible. Use definition. ). Let E denote the event that the ball lost is white. we b P= 5 PE) PIALE) = + PALE) = wb- Now use the total probability rule. Similar to 67. wr ). Let E, (0S r< 1) denote the event that r white balls are lost, and Now use the total probability rule and the identity 1 70. n. n. 73. 74, LD w-neereg_p=we "ep. A 1 1 lin (i) = —_—, tn ™ Tore = 140 a’ lis. isible by 10 if and only if last digit of a is Lord, If r=0, there are 2k ways to choose a. 2 Py=2kin, If r= 9,then there are 2(k + 1) ways to choose a and Py = Uk+1 Yin. If 1 S'$8, a.can be chosen in (2k + 1) ways. © Py = (2k + Lin When r= 0, 2, 1s, now DENA When r= 9.Pa= Tos rik ayn and hence k > e. Similarly, for 1.$ 15.8. a You have either reached 2 page thts unevalale forvening or reached your ieving tit for this book. a You have either reached 2 page thts unevalale forvening or reached your ieving tit for this book. a You have either reached 2 page thts unevalale forvening or reached your ieving tit for this book. (ii) a€ Qand ae R (ili) a,€ Qand ae R (iv) a,¢ Qanda,¢ R ‘Thus, total number of choices for picking up Q and Risa" For a, not to belong Q 7 R, we have 3 choices. ‘Therefore, 9 R = oin 3" ways. Thus, probability @.0 R = gis (3/4)", Next, a, € Q UR in 3 ways, Therefore, QU R= P in 3" ways, ‘Thus, probability Q UR = P is (3/4)". . Ifa and b are both even, then a + b is even, there- fore PAA BAC) =0. 100. Probability 9.55 We have Pa)= 4 PUB) = 3, PAB) Now, P(A OBO DIA vB) Placsad)n(avs)] _ Pian) * P(AUB) ~ * P(AuB) v4 Lt * Wasi2- ia ~ 3 Statement-2 is false. P(P and Q contradict each other) = pl 2p) +2p(1 =p) = U2 => 8p? — 6p + 1 =0. => (2p - 1) Gp-1)=0 > p= V2, 4. a You have either reached 2 page thts unevalale forvening or reached your ieving tit for this book. a You have either reached 2 page thts unevalale forvening or reached your ieving tit for this book. a You have either reached 2 page thts unevalale forvening or reached your ieving tit for this book. 10.4 Course in Mathematics for IT-JEE identities, we require the properties of complementary and supplementary angles. For example, if A+ B+ C= x, then 1. sin (B + ©) = sin A, cos B = ~ cos (C + A) 2. cos (A + B) =~ cos C, sin C= sin (A + B) 3. tan (C+ A) =~ tan B, cot A = ~ cot (B+ 0), AtB oe © Ln ALE 4 =sin ©, c& cos “5 in Ss cos S = sin “S Some Important Identities A+ B+ C= x, then 1. tan A + tan B+ tan C = tan A tan B tan C. 2. cot B cot C+ cot C cot A + cot A cot B= 1. 3. tan F tan © stan © tan As tan 4 tan & 2 2 2 2 2 2 Bi cot Biccor S-arcot Bcc Bret £ 4. ct A + cr ® + ct © = cor A cot 5 con €. oot 4 + cot $ $ # oot co £ 5. sin 2A + sin 28 + sin 2C = 4 sin A sin B sin C. 6. cos 2A + cos 28 + cos 2C —1-4 cos A cos B cos C. 7. cos? A + cos’ B + cos? C = 1 = 2 cos A cos B cos C. 8. sin A + sin B+sin C= 4 cos 4 cos # cos £. 2 2 2 A c ‘ Be 9. cos + cos B+cos C= 1+4sin 5 sin > sin. SOLVED EXAMPLES ‘SECTION | SINGLE CORRECT CHOICE TYPE Example 1 If cot a+ tan or= mand cos @=n, then 0s @ (a) m (mn) — n(n (b) m(m?ny' — n(n)" = 1 (e) n(n)" ~ minm?y'® = 1 (€) n(m?ny'® — mimn?y'? = 1 Ans. (a) Solution Clearly a# 0. cot @+ tan a@=m = 219 1+ tan? @=m tan a@ a Ya@=nseca => tan! a=n seca = tan? = tant c= n?m tan a [by ()) = tan a= nm => tan = (nm) and see? a= m(n'm)'® (by (DI => mtv m)'° — (my? = 1 If a cos’ @ + 3a cos @ sin? @ =m Example 2 $ and a sin’ @ + 3a cos’ asin a= n, then (m + n°? + (m—n)*? is equal to (a) 2a (by 2a" (©) 2a”? (A) 2a, Ans. (c). Solution From the given relations, we get m+ =a cos @+ 3a cos asin? a+ 3a cos’ asin @+ a sin’ @ = a(cos a+ sin Similarly m—n= a(cos asin a (me n+ m= ny = a {(cos a+sin a? +(cos asin a] = @ [2 (cos? @ + sin? a) = 2a? 2sin a 1+ cos a@+sin@ Example 3 if =y, then 1-cos@+sin a i+sina ) y © 1-y @1+y. is equal to @) Vy Ans. (b) cos @ + sin @ 1+ sina Solution cos @ +sin @ 1 +cos @+sin & I+sina — l+cosatsina _ {i +sin @? = cos? @ (1+ sine (1+ 0s + sin @) ‘1+ 2sin a +sin? @-1+ sin? @ (1+ sin @) (1 + cos @ + sin a) 2sin ae{I + sin @) (sin @) (I+ cos @ + sin @) sina =n sy, 1+ cos a+ sina a You have either reached 2 page thts unevalale forvening or reached your ieving tit for this book. a You have either reached 2 page thts unevalale forvening or reached your ieving tit for this book. a You have either reached 2 page thts unevalale forvening or reached your ieving tit for this book. 10.8 Course in Mathematics for IIT-JEE om ain Winsecd 3 Solution sin l= —7=- => cos te 2tan 6 ~tan? and — tan (a + 28) _ tane+tan 28 743/425 tan @tan 28 1-7/9 Since 0 < # < a/2 and tan 28 = 3/4 > 0, we get 0<2f< 2/2. Also,0 Ba a/a-o Example 22 If 3/4< ar< z, then 2 cot e+ —1— is equal to sin? @ {a) 1+ cota@ (b) - 1 -cota@ (©) l-cota@ (d) — 1 + cota Ans. (b). Solution eva+—— = Y2eoratcosec? a sin? a qRoota+l+cot? a = Vil + cot a)? =11 + cot a! Since cot @<~ 1 when 34/4 < a< x, we have 11+ cot al=-1-~cot a Example 23 (2V3 +4) sin.x +4 0cos x lies in the interval (@)(-4, 4) (b) (-2V5, 2V5) (©) (-2 +V5, 2 + V5) (a) (-22 +V5), 22 +V5)). Ans. (d). Solution The given expression is equal to 2{(V3 +2) sin x+2cos x]. Put V3 +2=rcos Band 2 =r sin 6, so that P= (J3 +2)? +2? = 11 +43. Then the expression can be written as 2ir cos @ sin x + r sin @ cos x) (say) Since 11 +4¥3 <9+4V5, we have Wiss GraJ5 => filea5<2+V3 1) we ~ 1 Ssin (4x51 = 2r $2 sin (84 x) S2r s2rsin(@+x=y Also, = -2yil+4v3 sys2vil+4v3 > -22 + V5) y?s2ax-(1-b%)2 Also, from a+ 8: a= y/x + band a 2 Example 66 For 0< @< 2/2. if = Cy + bx and tan & x tan ° 2 = Y cos” 9 sin™ 9, then - fa) aye =az+y ©) mzaxty Ans. (b) and (c) a You have either reached 2 page thts unevalale forvening or reached your ieving tit for this book. a You have either reached 2 page thts unevalale forvening or reached your ieving tit for this book. a You have either reached 2 page thts unevalale forvening or reached your ieving tit for this book. 10.20 Course in Mathematics for 1IT-JEE Ans. (a), (c), (d) Solution We are given that sin’ woo ; b. @ cost = sint a+ cost a+ = sint a+ = cost a=1 a = (sin* a + cos? a}? = 2 sit = 2 si? roost a+ & cost a= 0 oO Bet ak+bk= = ke! at (a+b) costa _ (a°k)* (hk)? a e = ak? + bk? = (a + bk? I 1 (a+b ath) cos" @+ sin" @, then -1 =(a+b) Example 82 If P, (a) 2P, 3 (b) 2P, = 3P,=1 (©) 6Pyy ~ 15Py + 10P, = 0 (@) 6Py ~ ISP, + 10P, = 1 Ans. (a), (4) Solution 2P,~3P, +1 = cos’ G+ sin® A) ~ 3(cos* 8+ sin* @) +1 = 2{(cos? 8 + sin? 8) ~ 3 sin? cos? @ (cos? @ + sin? @)) = 31(cos? 8+ sin? = 2 sin® cos? B]+ 1 = 21-3 sin? 8 cos? 6) = 3(1 - 2 sin? 6 cos? 8) + 1 = ‘Again for n 24, we have Py - Pao cos" 8+ sin" @— (cas? 8+ sin”? 8) cos”? @ (cos? 8-1) + sin”? 0 (sin? @= 1) = - sin’ @cos"? @ — cos? @ sin? 8 ~ sin® @ cos” @ (cos** @ + sin** @) ~ sin? 0.608? OP, 6Pjo ~ 15P, + 10P, ~ 1 = 6Pyo~ Py) ~ (Py = Py) + (Py = Pd +Py-P; = sin? 8 cos? 8 (6P, - 9P, + P; + Py) = ~3 sin’ @ cos’ 6(2P, - 3P,) A cos? A(1 + 2) Le Pps 1, Py = 2) = 3 sin? @.cos? @(— 1) - 3 sin? tole [e. 2Py = 3Py + 1 = 0 (as proved)] Example 83. The equation sin’ x + cos* x =a has areal solution for (a) all values of a (b) a = 12 © a=m0 @a=t Ans. (b), (c). (d) Solution We have sin’ x + cos* x $sin? x + cos’ x, as Isin xls Land kos xis} = asl ay Next, sin! x + cost x => (sin® x + cos! =2sin? xcos?x=a => foi 2e= 1-0 = l-asin Te sin? 2x 1) = sa Q From (1) and (2) we get 1/2 Sa L. Note that a= 1/2 for x= al4anda= | forx= x/2. Example 84 If sin @(1 + sin ® +cos @(1 +cos A= xand sin @(1 ~ sin 8) + cos @(1 ~ cos 6)= y then @) xe sin 20 (b) y? + 2y (c) xy = sin 20 @) x-y=2 Ans. (a), (b). (©), sin 26 Solution sin 8+ cos B= x—1=y+1 sin 20 = (sin @+ cos @ - 1 =x? - 2x . =y¥42y ays (sin 8+ cos 8 ~ 1 = sin2@ 2 Example 85 For a positive integer n, let fy) = tan (B12) (I + sec 8) (1 + See 28). (1+ sec 2") then (a) f(7/16) = 1 (b) f,(482) = 1 (©) f,lf64) = 1 (Gd) fs(2/128) = 1 a You have either reached 2 page thts unevalale forvening or reached your ieving tit for this book. a You have either reached 2 page thts unevalale forvening or reached your ieving tit for this book. a You have either reached 2 page thts unevalale forvening or reached your ieving tit for this book. a You have either reached 2 page thts unevalale forvening or reached your ieving tit for this book. a You have either reached 2 page thts unevalale forvening or reached your ieving tit for this book. a You have either reached 2 page thts unevalale forvening or reached your ieving tit for this book. a You have either reached 2 page thts unevalale forvening or reached your ieving tit for this book. 10.28 Course in Mathematics for IT-JEE cose tank ab ¢ @a+e (p) W7x* ) +e +e qi? (c) be (6) Wak L ak (dd) Praarers (s) alk Solution a + b’ @+P ee ze tins = alk cot x + T+ cos. = cosx(l+cosx)tsin?x 1d sin.x(1 +084) Example 124 If sin A = sin B and cos A = cos B, then (a) sin (12) (AB) (p) 0 (b) sin 2A + sin 28 (q) cos 24 (©) cos (A + B) ol (d) tan (A + BY/2) (s) 2 sin (A + B) sin A~sin B= cos B-cosA =0 APB ig AB oa si Solution A-B => 2cos A-B cos 2 => Bither sin A= A+B | in “== sin 2 A+B S Ast) = sin =sin =0 sin 2A + sin 2B = 2 sin A cos A +2 sin B cos B sinx ak’ sin A cos B+2.cos A sin B sin (A +B) cos (A + B) = cos A cos B~sin A sin B 0s" A ~ sin? A = cos 2A. sinda ‘xample 125 iis Bxample 125 2a * iat le lan (@) positive wae (3) , lax 1k (b) negative (@) @e (‘= re (a ue) (c) positive or negative () ae {—*,—* 48° 48 (@) not defined () a= 2x4 ros pq * @@OOQ) + @@OO * @@OQ) 4@@O®) 1s ax a (SEIS 48° 48 = (#- 327/16, #- 22/16) ie. See I quadrant = sin 3a>0 (= ie) ( ioe) 2ae | —,—|=|# = 24° 24 a ie. 2ee I quadrant = cos 2a<0 _ sind cosda Ans, Solution ae ( is negative. Similarly we ( = sin 3arcan be positive or negative. 2ae (»-E ss) cos 2eris negative. 24 sin3 @ be positi sative, cad a Sat! be postive or negative ir and cos 2ais - ve so we is positive and if @ = 23014 then cos 2a = 0 s0 932 is not defined, a You have either reached 2 page thts unevalale forvening or reached your ieving tit for this book. a You have either reached 2 page thts unevalale forvening or reached your ieving tit for this book. a You have either reached 2 page thts unevalale forvening or reached your ieving tit for this book. a You have either reached 2 page thts unevalale forvening or reached your ieving tit for this book. a You have either reached 2 page thts unevalale forvening or reached your ieving tit for this book. a You have either reached 2 page thts unevalale forvening or reached your ieving tit for this book. a You have either reached 2 page thts unevalale forvening or reached your ieving tit for this book. a You have either reached 2 page thts unevalale forvening or reached your ieving tit for this book. a You have either reached 2 page thts unevalale forvening or reached your ieving tit for this book. a You have either reached 2 page thts unevalale forvening or reached your ieving tit for this book. a You have either reached 2 page thts unevalale forvening or reached your ieving tit for this book. 10.40 Course in Mathematics for IT-JEE. 17. 18, 19. 20. 2. 23, 25. 26. ‘The number of all possible triplets (ay, ay, a) such that ay + a3 cos (2x) +9, sin? x= 0 for all xis (a) zero (b) one (©) three (@) infinite (€) none of these [1987] The sides of a triangle inscribed in a circle subtend angels a, f, y at the centre. The minimum value of the arithmetic mean of (1987) cos (a+ 12), cos (B+ 712), cos (7+ #72) is equal 10. (1987) Prove that tan @+ 2 tan 2a+4 tan 4ar+ 8 cot 8a= cota [1988) ‘The value of the expression 3 cosec 20° - see 20° is equal to (a) 2 (b) 2 sin 20°%in 40° 4 (d) 4 sin 20°%sin 40° [1988] i on 3% gi, SH 1k The value of sin 7 sin TF sin TF sin 7 on ie. Be sin [7 sin TE sin —F isequal to... [1991] If f(x) = cos [2°]x + cos [~ x”]x where [x] stands for the greatest integer function then (a) f (#2) =-1 (b) fo) (©) f- a) =0 (d) fray {1991} ‘Show that the value of tan x/tan 3x, where ever defined, never lies between 1/3 and 3. [1992] For 0 < 0< 2/2, if x = Loose, y = Ysin™y, 2 = a = Scos* gsin® ¢, then = (a) xyz = az +y © ayzexty tz (b) xyz = xy +z (@) xyz = yet [1993] If k = sin (2/18) sin (52/18) sin (72/18), then the numerical value of & is {1993} IfA>0,B>Oand A +B = 2/3, then the maximum value of tan A tan B is {1993} For all values of A, B, C and P, Q, R show that lcos(A~P) cos(A-Q) cos(A- R) cos(B—P) cos({B-Q) cos(B~R) =0 leos(C-P) cos(C-Q) cos(C-R) [1994] ‘The number of points of intersection of two curves ys 2sinxand y= 5x7 +2x+3is 29. 30. 3. 32, 33. 35. 36. 37. fa) 0 1 © 2 (d) 2 11994) Let 0 Statement 1 is false. xsin 8. sin? 8+ y cos 8. cos’ @= sin cos @ => xsin 0= sin cos @=> x=cos O=>y=sin = x+y" = 1, so statement 1 is true. Also state~ ‘ment 2 is true but does not imply statement-I. a You have either reached 2 page thts unevalale forvening or reached your ieving tit for this book. a You have either reached 2 page thts unevalale forvening or reached your ieving tit for this book. a You have either reached 2 page thts unevalale forvening or reached your ieving tit for this book. 114 Course in Mathematics for ITT-JEE Since - v2 x= 92, y=-mQorx=— 2,y= m2 So there is only one pair — 272 & 2 satisfying the given equation. Example 14 The number of solutions of the equation tanx ~sec!x + 1 = 0 in (0, 10) is @3 6 © 10 @o ‘Ans. (a) Solution sec*x —sec'x = 0 sec*x (1 - sec¥x) =0 1 = sect = 0 as sec*x > 1 => costx = 1 = cose = +1 mnet x= (4 2x, 32} € (0, 10) Huuug © [- Ans. (c) (d) none of these {sin x Solution. \sin x12 0 => Tisinal So the given in equation becomes [sin x} 22 Telsinxl > 3 1 = 3 Isin.xl ciel =slsin xis i+lsinxl haere eel 2°2. Example 16 Number of solutions of the equation cos(aVx)-cos (wVx=4) = 1 are @ 1 (b) 2 (3 @)o Ans. (a) Solution cos(m/x~4) is defined if x>4 cos(avx) cos(nJx—4) = 1 cos (Vx) = 1, cos(aJx-4) = 1 avx = 2mm nied = 2nz(mnel x= 4m, x— 4 = 4n? mn =lamealn=0r=4 So x=4 is the only solution of the equation Vuuds Note If'we take cos(#Vx) =—I, cos(yx-4 We will get (2m +1)? - (Qn + 1)? =4 > (m+ n+ Im —n)=1 which does not give any integral solution of m & n. Example 17 Let @ f e[- = mj be such that cos (a=) = | and cos(ar+ A) = Ve, The number of pairs of a, satisfying the above system of equations is @o 1 O2 @4 Ans. (4) Solution — nS a, BS x=-2nSa-BS2n So cos(a@- A) =1 = a 2m, 0.2 = B=a-2x, a, a+2e As cos(a+ ) = Ie, cosdar= Ve As—24S2aS2nor-aS asx ‘And cos2a= I/e, there are four values ar and hence at least four pairs (a, A). Example 18 The number of solutions of the pair of equations 2sin7@—cos20=0 and 2cos*@— 3sin@= 0 in the interval (0, 2] is (0 @)1 ©2 @4 Ans. (c) Solution 2sin°8-cos28=0 = 2sin?@ - (1 - 2sin?® = 0 = 4sin°@ = 1 = sind = +12 and 2cos?@ - 3sind = 0 = (1 - sin*@ ~ 3sin = 0 a You have either reached 2 page thts unevalale forvening or reached your ieving tit for this book. a You have either reached 2 page thts unevalale forvening or reached your ieving tit for this book. a You have either reached 2 page thts unevalale forvening or reached your ieving tit for this book. 11.8 Course in Mathematics for IIT-JEE Solution We can write rsin (x+ @) = 2y?~8y +21 where r= Vi44+25 = 13 and tan a= 5/12 LS $13 and RUS = 2(y- 2) + 13213 Roots of the equation exist if L.H.S. = R.H.S = 13 => y= Zand sin (x + tan" (5/12) = 1 2 we! 5 ocotre = Sore 3st oat (2) = 3864 « 5 = 322 x 5 = 1610. SECTION V MATRIX MATCH TYPE Example 36 (@) cosx=- 12 (p) x= 7a (b) sin.x = V3/2 (q) x= 19216 (c) anx = V3 () x= 828 (d) cotx=-1 (s) x= a4 Ans. s pat *@@OO > @@OO a OJOTGTO) *@@OO) Solution cos. x= ~ 1/2 = cos (22/3) => x= na (293), nel > = 8713 for n= 1 sin x= 3/2 = sin (4/3) = x= n+ 1" AB, ne EL = x = TRf3 torn = 2 tan x =1/V3 = tan (716) => x= nz + m6, ne L x = 19216 for n = 3. = cot (32/4) => x =nt+3adnel = x= Ml a forn=2 Example 37 (a) (2 sin x - cos x) (p) sin x = 12 (1 + cos x) = sin? x (b) 1 + sin2x = (q) tanx=-1 cos x + sin x (©) 4x4 + 2° + sin? Sx = 0 (d) sin x + sin 2x = 0 (x=0 (8) cos x Ans. Solution (2 sin x cos.x) (1 + 608 2) = sin? x => 2 sin x (1 + cos 2) ~ cos x ~ cos? x sin? x = 0 (1 + cos x) 2 sins - 1) = 0 which holds if sin x = 1/2 1 +sin 2x =cos.x + sinx > = which is satisfied if cos x + sin x= Oi, tanx=- 1 4x! 4 x° + sin? Sx = 0 ifx=0 sin x + sin 2x = sin x (1 + 2cosx) = 0 Example 38 (a) 4 sin? x + sin? 2x = 3 (0) 4 cos? 2x + 8 cos* x= 7 (p)x = tan! 1 @x = sin! 1) O30 +siny el scos2e (x= tas (Yi) (@)4 sin’x ~ 2 sin? x - 2 (s).x = sin”! (1/2) sinx+1=0 An. Pat 8 *@@OQ b O@GO O@O®| 4 @Q0® Solution 4 sin? x + 4 sin® x cos? x= 3 (sin® x + cos? x) sin’ x +4 sin’ x cos” x-3 cos’ x=0 = => tan? x (1+ tan? x) +4 tan? x—3(1+tan?x)=0 => tant +2tan?x-3=0 => (tan? x~1) (tan? x43) =0 which is satisfied by x = tan! 1, Next, 4 cos? 2x +8 cos? x= 7 = 4 (cos? x - sin’ xP +8 cos? x= 7 | 40 tan? + 8 (1+ tan?) = 7 (1 + tan? x? a You have either reached 2 page thts unevalale forvening or reached your ieving tit for this book. a You have either reached 2 page thts unevalale forvening or reached your ieving tit for this book. a You have either reached 2 page thts unevalale forvening or reached your ieving tit for this book. 11.12 Course in Mathematics for IIT-JEE 1 it Wehave tan(45* san 30°) = — 2a V3#1 , 488 1 Wei 2 1 *B =22 3 So From (i) tan@= tan(45° + 30°) Sa « = 15° or 715° or O= ~ or = = O=1S* of 15° of B= or TF Showing that statement-1 and 2 both are true and statement-2 leads to statement-1 SINGLE CORRECT CHOICE TYPE 1. cos 2x +a sin x = 2a ~7 possesses a solution for (2) all value of a (b) a>6 acd @) 2 c then B-C_b-e tan BIE. 2 bte gives the value of (B ~ CV/2, while Bee | or cosA= co A 2 A 2 so that B and C can be evaluated. The third side a is given by 90° — sin A sin a=b or @ = b+ — Ibe cos A 3. If two sides b and ¢ and the angle B opposite one of them are given, then angle Cis given by sin C = (cfb) sin B, A = 180° ~ (B + C) and the third side a is given by a= (sin Alsin B). If b A+B=90", we have 2 4p? oe sin (A ~ B) in? A + sin? (90° - A) = Sin At tin” (=A) sin (4 - sin (A+ B)sin (AB) "4 in? 2 Man Ate A ln iwitia We Bh (A+B) in Example 11 If ina triangle ABC 2ewA , cosB , 2eosC_ a , b a ©. be ca then (@ A= 90° (b) B= 90° (b) C= 90° (©) none of these Ans. (a) Solution ‘The given relation can be written as UWP are ra? 42a? +b =) _ a+ abe => BB + cP + a? = 2a? + BY => P+e-a=d = cosA=0 = A= 90". Example 12 In a triangle ABC, AD is the altitude form A (Fig. 12.9). Given b> ¢, £ C= 23° and AD = FS then 2 B. is equal to abe =e 4 5 = ™ D a 4 c Fig. 12.9 (a) 113° (b) 123° (@):147° @) 157° Ans. (a) Solution We have AD = b sin C abe 5 = bsin€ sin? A sin (B+ ©) sin (BC) 3 sin@®-Q=1 [+ sin(@+ C= sin Aj a => B= 90° + C = 90° + 23° = 113°. Example 13 In a triangle ABC, a:b: =4:5:6. The ratio of the radius of the circumecircle to that of the incirele is (@) 154) 1S) 167 @) 1683 Ans. () 2,2. r 44° 4 4a7 sabe * dss =a) (s-DG=2) 4k x Sk x 6k {Bx ~41)(Bu-s4](Bx-o1) 2 2 2 _5x6x8 16 “Tx5x3 7° Example 14 If for a triangle ABC, abe bea cab then sin’ A + sin’ B + sin’ C= (@) sin A + sin B+ sin (b) 3 sin A sin B sin C (©) sin 34 + sin 3B + sin 3C (d) sin’ A sin? B sin? C Ans. (b) 0 lbe lea lab Solution A= =(a+b+e) aes Boe eas a You have either reached 2 page thts unevalale forvening or reached your ieving tit for this book. a You have either reached 2 page thts unevalale forvening or reached your ieving tit for this book. a You have either reached 2 page thts unevalale forvening or reached your ieving tit for this book. 12.10 Course in Mathematics for ITT-JEE b> (b+e-a)(a+b- ol” a> ((c+a-b) (a+b)? Similarly and ‘Multiplying the above inequalities we get abe > (b +c ~a) (c+a~b)(a+b~c) | Note Ita, b, care any distinct positive numbers at most one ofb +¢-a,c+a~b,a +b ~c can be non-positive and | in that case also the given expression is negative) Example 27 The perimeter of a triangle is 6 times the arithmetic mean of the sines of its angles. If the side ais 1, then A is equal to a) 30° () 60° (©) 90° (@) 120° Ans. (a) Solution a+b+e=6 Sees) > a+b+c= 2AsinA + sinB + sinC) a 1+ SBE SOC = asia + sind + sinc) faring OE ow TH ty 88 Sind ~ SinB ~ SinC => sin + sinB + sinC = 2 sind (sinA + sinB + sinC) > sin = 3 = A = 30° Example 28 If in a triangle ABC, cosA + 2 cosB + os C= 2 then a, b, ¢ are in (a) AP () GP (© HP (@) none of these Ans. (a) Solution cosA + cosC = 2(1 ~ cos) = 2cos 44 cos 4=E = 2 x2 sin? 2 2 2 B A+C 2x2 sin = cos 2 (s-be-0) , [e=a-5) 1 ssa) sis 3 = 3(s-b)=s=—2s=3b = atbeen baba Se = ab.carcinAP. Example 29 In a triangle ABC, bisector of angle C meets the side AB at D and circumcentre at E. The maximum value of CD. DE is equal to (@) as (o) 7/4 © cM @) (a+ 4 Ans. (©) Solution CD. DE = AD.DB < AD + DB=c Since AM. 2 GM. D on > AD-DB A 8 gE 2 oe > AD.DB Fig. 12.13 => AD.DB s 7/4 and the required value is c7/4 Example 30 In a triangle ABC, a > b 2 c. If , atte ene maximum vane ofais sin? A+ sin’ B+sin’ C (@) 12 (b) 2 © 8 @) 64 Ans. (b) (2R)*(sin* A + sin®B + sin°C) sin’ A+ sin’B +sin’C = R= 1, the radius of the circumcirele is 1. Greatest length of a side of a triangle inscribed in a circle can be equal to the diameter of the circle and hence the maximum value of the greatest side a is equal to 2. Example 31 If p denotes the perimeter of the triangle ABC, then b cos" teeth is equal to (a) p (b) 2p (©) pr @ Pp Ans. (c) Solution we have Solution boost +ecostS s-0) 5, 16-0) = bx ab ac a You have either reached 2 page thts unevalale forvening or reached your ieving tit for this book. a You have either reached 2 page thts unevalale forvening or reached your ieving tit for this book. a You have either reached 2 page thts unevalale forvening or reached your ieving tit for this book. 12,14 Course in Mathematics for ITT-JEE. Solution Let ABC be the isosceles triangle with AB = AC = b and 2B = ZC = a (Fig. 12.19). Let AD be the perpendicular bisector of the side BC. Since AABC is isosceles, AD is also the bisector of angle A, so that O and ‘Tboth lie on AD. We have OB = Rand ID Also, since Gis the circumcentre, we get OA = OB = R. Therefore, from isosceles triangle OAB op AB sin (90° = a) © sin 2a beosa 1 2sinacosa 2 ‘so that (a) is correct. = R= Again 4 = BD-AD = b cos ab sin a= 8" sin 2ar 0 that (b) is not correct, 5? sin 2a . Also, r= <5 ieee 5 St b+2beowa) A+ cosa) 80 that (¢) is correct. Further Of = 10D + Dil = 10D + ri because @< 4/4, A > a/2 and O lies on AD produced. Now, from right-angled triangle ODB, we get OD? = OB ~ BD? = R° - (b cos a)? 16 4 sin? 8 cos? a 244 sin? cos? = P= 4 sin* 2008? @ pom fp 4:sin® Eos’ @~sin® ay 4 sin’ a = Pecos 2a @sin ay beos2a 2sina = oD= bsin 2a Of = |e 2(1+ cos @) bcos 2a -| b sin 2@ ‘4sin (@/2) cos (@/2) ae 4 cos? (@/2) | b sin 2a sin leii sem tace(e12)| 4.c0s (a@/2) sin (a2) cos (a2) _ | _bc0s Gal2) 2 sin a cos (@/2) ‘Thus, (d) is also correct. Example 41 If H is the orthocentre of triangle ABC, then AH is equal to (a) 2B cos A (©) a cota (b) 2R sin A (a) 2ab6 cos A. 4 Ans. (a) and (c). Solution Referring to AAHB (Fig. 12.20), we have AH . © sin (90°- A) sin (A + B) cos A = or sin (180° - C) sin C Fig. 12.19 ‘so that (a) is correct while (b) is not correct. Also, Abe og 4 = abe AH = 2R cos A Tk EAR cos A and AH = 2R cos A = —*—-cos A =a cot A sin A so that (¢) is correct, while (4) is not correct. Example 42 if the tangents of the angles A and B of a triangle ABC satisfy the equation abx” ~ cx + ab =0, then (a) tan A = a/b (b) tan B= ba © cos C=0 (d) sin? A + sin? B + sin? C = 2, Ans. (a), (b), (©) and (a). Solution From the given equation, we get tan A + tan B= Vab and tan A tan B= 1. Since a You have either reached 2 page thts unevalale forvening or reached your ieving tit for this book. a You have either reached 2 page thts unevalale forvening or reached your ieving tit for this book. a You have either reached 2 page thts unevalale forvening or reached your ieving tit for this book. 12.18 Course in Mathematics for 1IT-JEE Example 52 A and B are two points 30 m apart in a line on the horizontal plane through the foot of a tower lying (00 opposite sides of the tower. The distances of the top of the tower from A and B are 20 m and 15 m respectively, if the angle of elevation of the top of the tower at A is @and the height of the tower is f, then (a) cos 6 = 29/36 (b) cos = 43/48 (©) h= SVSIV3 (a) h = 5455/12 Ans. (b) and (@) Solution Let OP be the tower of height /, then AB = 30, AP = 20 and BP = 15 (Fig. 12.25) P 220%30 1075 _ 43 and = 20sin 0:20 x “455. SVS. a 2 Example 53 A pole 50 m high stands on a building 250 m high. To an observer at a height of 300 m, the building and the pole subtend equal angle @ If the horizontal distance ‘of the observer from the pole is x, then (@) x= 25V6 m (b) x= 253 m (©) tan @= J273 — (d) tan B= 26 Ans. (a) and (6) Solution Let PQ be the pole on the building QR and 0 be the observer. Then PQ = 50, QR = 250 (Fig. 12.27) Fig. 12.24 => PR = 300 so the observer is at the same height as the top P of the pole. So OP = x. Then from right angled triangles OPQ and OPR, tan 6 = © and tan 29 = 300 x 5 so thar 2409-300 Intan?@ ox 2x50 300 = ed “ayo” : s-(8)}= y 2 s0v3 2 (2) fo Sp 8 sof and tan 9° SR 3" Example $4 A monument ABCD stands at a point A on a level ground. At a point P on the ground the portions AB, AC, AD subtend angles a, A, ¥ respectively. If AB = a, AC =b, AD =, AP =x and a+ B+ y= 180° then @) 2 = (a4 b + cVabe (b) x? = abcila +b + 0) (©) tan a+ tan B+ tan y= (a +b + 0c)? /Vabe (@) tan @tan Bran y= (a +b + 0)" /Vabe Ans. (b), (c) and (d) Solution We have a = AB = AP tan a= x tan @ (Fig. 12.28) b=AC=xtan B and ¢=AD=xtany so that a+b +¢= x (tan @+ tan B+ tan 7) =xtan @tan Bian y (v @+ B+ y= 180°] and abe = x tan tan B tan ¥ abe atbte so that tan a+ tan J+ tan y= tan attan Btan y=(a+b+c)hx = (a+b+ 0) Vabe. a You have either reached 2 page thts unevalale forvening or reached your ieving tit for this book. a You have either reached 2 page thts unevalale forvening or reached your ieving tit for this book. a You have either reached 2 page thts unevalale forvening or reached your ieving tit for this book. 12.22 Course in Mathematics for IT-JEE = ha 106 ft = X= 4he24 = 400 ft => OA’ = 400 + 300° = 500 ft Paragraph for Question Nos. 64 and 65 A diving stand (vertical is situated at a point on the edge of acircular swimming pool of radius r. At another point P on the circular edge the angle of elevation of the top of the diving stand in 30°. After moving 10 metres along the circular edge, the angle of elevation is again 30°, and further going along the edge 5 metres, the angle of clevation is 45°, Example 64 Height of the diving stand is @r (b) 2r © QB) (@ 3 Example 65 The perimeter of the swimming pool is (a) 15 m (b) 20 m © 30m (@) 35m Ans. 64, (a), 65. (€) Solution Let OR be the diving stand of height h and C the centre of the swimming pool of radius 7: Let P and Q be the two points on the edge of the pool where the angle of elevation of R the top of the stand is 30° and R’ be the middle point of PQ (Fig. 12.34). Fig. 12.31 Then arc PR’ = arc R'Q = 5 m, as are PQ = 10 m. Let S be the point where the angle of elevation of R is 45°. Then are QS = 5 m. We have OP = OQ =h cot 30° = hV3 and OS = OR cot 45° = OR = h. Let 2COP COQ = |QOS = a From isosceles triangle COP cr. V5h r sin (180°- 20) * Fn@ ~ 2sinOcos® ~ sind = gst a 2r —4_. . 2sin 20 cos 28 ~ sin 20 => cs2=t = ar Zoos O-1 2 Sh peo 4r Or 2 = 3(4) -4 -2=0 ror =» 2x = Gh+29(h-r)=0 = her From (1) cos @= V3/2 = @=n/6 20r= PR =5 sre lSin So the perimeter of the pool is 27 x 8 =30m. SECTION IV INTEGER ANSWER TYPE Example 66 Ina triangle ABC; A= n/3, b = 50, c= 30, AD is a median through A, then AD’ is equal to Ans, 1225. Solution B aD ah c Fig. 1232 a You have either reached 2 page thts unevalale forvening or reached your ieving tit for this book. a You have either reached 2 page thts unevalale forvening or reached your ieving tit for this book. a You have either reached 2 page thts unevalale forvening or reached your ieving tit for this book. 12.26 Course in Mathematics for INT-JEE, wo VS (s) 6 (©) ED (d) AE => BD = 49 => BD = 7 = A BCD is isosceles = ED=12CD=2 and BE= (7? =? = V45, AE =6 Example 79 In a triangle ABC, sin A cos B = 1/4, 3 tan A =tan B, (a) B=A (p) 60° (b) A+ BR (@) 90° () A+B (1) 30° (@) A+ C2 (s) 75° Ans. PG FS * OOO + @@OO @@OO) 4@@OOO) Solution 3tan A =tan B = 3 sin A cos B= cos A sin B = 00s A sin B= 3/4 =9 sin (A +B) = 1/4 + V4 = 1 => C= and tan A = tan (90° — B) = cot B = tan?A= 13 > tan A= x A= 30" 3, B= 60" Example 80 The angles of a triangle are in the ratio 2:3:7 (a) smallest side = 2 (b) smallest side = 2 © s= 34342 (p) largest side = J +1 (@) largest side = 6 +/2 (f) largest side = 1 (s) largest side =2 + V3 @ A= (V3-1)/4 Solution Angles of the triangle are 30°, 45°, 105°. a b ie sin 30° sin 105° sin 45° > Ga Wi” Vann able EI" Ga =k (say) ‘The smallest side a= 2 kand largest side c= (v3+i)e ates 1/2 ifa=2,k= V2,c= V6 +V2 Ifa= ¥2,k=1e= V3+1 Is=3+ V2+V5,k=2,c=24 NI Ba IfA= 4 3 «then from 4 = i bc sin A we have . ; x2kx (+1) k sin 30° wee _ a = Waey 4 2 and hence ¢= 1 a You have either reached 2 page thts unevalale forvening or reached your ieving tit for this book. a You have either reached 2 page thts unevalale forvening or reached your ieving tit for this book. a You have either reached 2 page thts unevalale forvening or reached your ieving tit for this book. 12.30 Course in Mathematics for IT-JEE Ans, (a) Solution Statement-2 can be proved (by using A + B+ C = A) to be true, (conditional identities) From which we get 3-2 (sin? A + sin® B + sin? C)=— 1 ~ 4.608 A cos B cos C =} 3-2(2)=—1-4 cos A cos B cos C > cos A cos B cos C= 0 = one of the angles A, B, Cis equal t0 90°. Example 90 Statement-1: In any triangle ABC cos A + bcos B+ccos CS. Statement-2: In any triangle ABC sin (A/2) sin (B/2) sin (C2) < 1/8 Ans. (a) Solution Since L.H.S of the inequality in statement-2 is a symmetric function of sines of the angles of the triangle, its maximum values is attained. When A = B = C = 60° and hence the statement-2 is true. Statement-t is true if 2K (Sin A cos A+ sin B cos B+ sin Coos C) SR (sin A + sin B + sin C) or if sinA sin B sin C$ 4 cos (A/2) cos (B/2) cos (C2) (From conditional identities) or _if'sin (A/2) sin (B/2) sin (C/2) < 1/8 which is true by statement-2 So statement-1 is correct. Example 91 Statement-1: In any triangle ABC, if'a: b re=4:5:6then® 7 Statement-2: In any triangle ABC a as ras Ans. (©) Solution Statement-2 is false as R= ©, p= 4 4a" s R _ sabe sabe wo X she, __sabe_ > 4a? Asis ans — bX 4x5x6 x6x8 x53 (EfE9" and the statement-1 is true. Example 92 Statement-1: Manan observes that the angle of elevation of the top P of the tower OP at a point Aon the ground is a he then walks a distance AB towards the foot O of the tower and finds the angle of elevation as B. be again walks a distance BC in the same direction and observes the angle of elevation now is 7 He notices that a+ B+ y= 180°; a B yare in A.P. and the distance of C from the foot of the tower is half the distance of B from the foot O of the tower. If the height of tower is A, the distance of A from the foot O is 5h/3V3. Statement-2: If the angles of elevation of the top of a tower at three points on the ground are in A.P. then the distance of the points from the foot of the tower are also in AP. Ans. (c) Solution Statement-1 a= 60° ~ 8 B= 60° y= 60° +0 OC = (1/2) OB Fig. 1245 => 2h cot (60° + @ = h cot 60° = N31 - V3 tan ® = V3 + tan O = tan = 3/7 and OA = h cot (60° ~ 8) a(t+V3tané) sy, “i= tan 5 Showing that statement-t is True. But statement-2 is not ‘True. Because if the angles of elevation are 30°, 45° and 60° and the height of the tower is A, distances are h/ V3, , hN3 which are not in A.P. EXERCISE Le ‘SECTION | SINGLE CORRECT CHOICE TYPE L. The expression (arb+oy(b+c~a)(c+a~bi(atb~o) abi? is equal to (a) cos? A (b) sin? A (©) cos A cos B cos CG) none of these a You have either reached 2 page thts unevalale forvening or reached your ieving tit for this book. a You have either reached 2 page thts unevalale forvening or reached your ieving tit for this book. a You have either reached 2 page thts unevalale forvening or reached your ieving tit for this book. 1234 Course in Mathematics for ITT-JEE SECTION | SINGLE CORRECT CHOICE TYPE |. In a triangle ABC, if a is the arithmetic mean and b, (6 # c) are two geometric means between, any sin’ B+sin’ C sin A sin B sin two positive real numbers then is equal to (@ 0 1 ©2 wa In a triangle ABC, if cot A = (xe +. + x)!?, cot B= (x + x! + D'? and cot C = a+ a'y"®, then the triangle is (a) isosceles (b) obtuse angled (©) right angled (d) equilateral mtn, Ina tangle ABC, "TE is equal to abe ath abe abe bm) Stb = wo wi wo oF oF In a triangle AC, if S84 = sl ome and a = 1/V6 then the area of the triangle in square units is equal to (a) 1/24 (b) 1/8 (©) /8v3 (a) 1/24V3 In a triangle ABC, if 5 cos C + 6 cos B = 4 and 6 cos A + 4 cos C = 5, then tan (A/2) tan (B/2) is equal t0 @23 O32 OW Ws In a triangle ABC, sores is equal to (a) 1o08 (py eos T= cos B T¥e0s B oe (Al2) @ nal A sin’ (B/2) sin’ B If r, R are respectively the radii of the inscribed and circumscribed circles of a regular polygon of, nn sides such that © = 5-1, then 2 is equal to r (@) 5 (b) 6 @) 10 (d) none of these In a triangle ABC, if cos A +2 cos B + cos C= 1, then a, b, ¢ are in (a) AP. (©) HP. (b) GP. (d) none of these 10. 12. 14, 15. 16. abe In a triangle ABC, if |b © @ cab sin A sin B + sin B sin C+ sin C sin A is equal to (@) 0 (b) 9/4 om (@) none of these In a triangle ABC, if A = 18°, b - a= 2, ab = 4, then the triangle is (a) acute angled (b) right angled (©) obtuse angled (d) isosceles If for a triangle ABC, a, b, A are given, then which of the following gives us two such triangles ), then (a) absin A anda b sin A and a>b The distance of the incentre of the triangle ABC from A is (a) 4 sin (42) (©) 4R cos (AP) (b) 4R sin (B/2) sin (C/2) (d) 4R cos (BI2) cos (C2) . If dy, ds, dy are the diameters of the three escribed circles of a triangle ABC, then d,d, + dd; + dyd, is equal to abe abe © bea @aetPaee (a) ab + be + ca ©) @+ bey In a triangle ABC, if . then tan? (A/2) is equal to no 2 OB (a) 143/432 (b) 13/33 (© 1139 (d) 1287 A point P is inside an equilateral triangle. If the distances of P from the sides are 8, 10, 12 units, the length of a side of the triangle is (@) 10 (b) 103 (©) 20V3 (a) 30V3 ‘SECTION II MULTIPLE CORRECT CHOICE TYPE If ABC is not a right angled triangle, then which of the following is (are) not possible. (a) sin 2A + sin 2B + sin 2C=4.cos A cos B sin C (b) cos 2A + cos 2B + cas 2C = ~ 3/2 (c) sin A+ sin B + sin C =2V2 cos (A/2) c0s (B/2) (d) cos A + cos B+ cos C= 1 a You have either reached 2 page thts unevalale forvening or reached your ieving tit for this book. a You have either reached 2 page thts unevalale forvening or reached your ieving tit for this book. a You have either reached 2 page thts unevalale forvening or reached your ieving tit for this book. 12.38 Course in Mathematics for IT-JEE 13. 14. 15. 16. 17. 19, 20. AB is a diameter of a circle and C is any point on the circumference of the circle. Then (a) The area of the AABC is maximum when it is isosceles. (b) The area of the AABC is minimum when itis isosceles. (6) The perimeter of ABC is minimum when it is isosceles, (a) none of these. (1983] In atiangle ABC, if cot A, cot B, cot Care in AP. then a*, b°. c* are in ... progression. (1985) In a triangle ABC, the median to the side BC is of length and it divides the angle A into Vio angles of 30° and 45°. Find the length of the side BC. (1985} There exists a triangle ABC satisfying the condi- tions: (a) bsinA = a,A < RZ (b) b sin A> a, A> m/z (@) bsinA a () bsinAni2,b=a [1986] If in a triangle ABC, cos A cos B + sin A sin B sin C= 1. Show that a slit: v2 [1986] Ifa, 6 and ¢ are distinct positive numbers then the expression (b+¢-a)(c+a~b)(a+b—c)-abcis (a) positive (b) negative (©) non-positive (d) non-negative (€) none of these {1986} In a triangle, the lengths of the two larger sides are 10 and 9 respectively. If the angles are in A.P. then the length of the third side can be (a) 5- v6 (b) N3 © 5 () 5+ V6 (©) none of these [1987] If the angles of a triangle are 30° and 45° and in- cluded side is (V3. + 1) em, then the area of the triangle is ... 11988] - ABC isa isosceles triangle inscribed in a circle of radius r. If AB = AC and h is the altitude from A to BC, then the triangle ABC has perimeter P = 2 (ir A? + Vir) and area A = —____ also tim 4 =___. (1989) oP 2. 23. 25. 26. 27. 28. 29. 3h. 32. ABC is a triangle such that sin (24 + B)=sin (C-A) =~ sin (B +20) = 1/2 If A, B andl C are in arithmetic progression, deter- ‘mine the values of A, B and C. {1990} The sides of a triangle are three consecutive natu- ral numbers: and its largest angle is twice the small- est one. Determine the sides the triangle. [1991] . In a triangle of base a, the ratio of the other two sides is r(< 1) show that the altitude of the triangle is less than or equal to ar/(1— 7), {1991} If in a triangle ABC 2cosA , cosB , eos _ a, b a b © be ea then the value of the angle A is ... degrees. [1993] Ina triangle ABC, AD is the altitude from A. abe Given b ><, | = 23° and AD = Pre then |B =... 11994] A circle is inscribed in an equilateral triangle of side a. ‘The area of any square inscribed in this circle is... [1994] Let Aj, Ap, ... A, be the vertices of an n-sided regu lar polygon such that 1 1 1 —— = +=. find the value of n, AA A Ay AA [1994] Consider the following statements concerning a tri- angle ABC. {i) the sides a, b, ¢ and area A are rational Gi) a, tan (B2), tan (C/2) are rational (ii) a, sin A, sin B, sin C are rational. Prove that (i) = (ii) = (iii) = (i) (1994) ). If the lengths of the sides of triangle are 3, 5, 7, then the largest angle of the triangle is (a) aR (b) S216 © 298 @ 3/4 [1994] In a triangle ABC, |B = n/3 and |C = n/4. Let D divide BC internally in the ratio 1 : 3 then sin |BAD inca “8 (a) U6 (b) 13 © We (@) V2 {1995} Ina triangle ABC, a:b: ¢= 4:5: 6. The ratio of the radius of the circumcirele to that of the incircle is [1996] a You have either reached 2 page thts unevalale forvening or reached your ieving tit for this book. a You have either reached 2 page thts unevalale forvening or reached your ieving tit for this book. a You have either reached 2 page thts unevalale forvening or reached your ieving tit for this book. 12.42 Course in Mathematics for ITT-JEL 2 8 ny 22 2 48 Az BA DB ABXDB 8. (iv) B. 9. 75° 14, AP. 15.2 18. (b) 19. (a), (d) 20. (V3 +1)/2 sq. em 21. hf Qhr— Ah? 1/128 22. A= 45°, B= 60°, C= 75° 23. 4,5, 6 25. 90° 26. 113° 27. @'i6 units 28. 7 30. (©) 31. @) 32. 1677 33. pe (=, 0) UG + 2V2, ») 34. (b) 36. (a) 38. (b) 39. (a) 42. (@) 44, (b) 45. (©) 47. ©) 48. (d) 49. (b) 50. (©) 51. @ 52. (a), (b) (©) @) 53. (0), (©) HINTS AND SOLUTIONS ? ][a? -(6-<)? 4b be +(b? +e? -a*)] be — (0 + 4b? __ Zhe +e0s A} 2be(1~c0s A) _ - 4b (b+ 1-cos?A Cl 8 az D an ie Fig. 12.48 b ald ———_— = —""_ Fig, 12.48 sin(A+C—a@) — sin(A-a@) asin o= b¥E sin 4 @ sinB+sinC A Bane sin Fig. 12.48 _ csin4s® _csin4s® sinl125° — cos225° =2csin 22 =2 AD . a (1 + 608 C) +c (1 +008 A) = 3b SateracosC+ecosA=3b Satctb=3b=a+c=2b =a, b,carein AP, a You have either reached 2 page thts unevalale forvening or reached your ieving tit for this book. a You have either reached 2 page thts unevalale forvening or reached your ieving tit for this book. a You have either reached 2 page thts unevalale forvening or reached your ieving tit for this book. 12.46 Course in Mathematics for IT-JEE gg, BD Sim(AI3) From AAD! AD sing “rmAAPA) sin(A/3) AD sin(C +(A/3)) sin(B +(A/3)) AD sin(C +(A/3)) AE sin(A/3) sin _ Sin (B + (A/3)) sin(A/3) AD. sin (C + (A/3)) sin Ny i B D E c Fig. 12.54 c 39. B= 60, sin C= > sin B= and A = 75° => cos (A - C) =cos 30° sin (A ~ C) = sin (B/2), sin (A + C) = sin 2B and sin 2C ~ A) = sin 15° = sin (B/4) 40. AD=h, tan a= 45/28 AC=h= 17 = hoot a= 2h <3 45 (h—17) = 28h =h=45=AB AC = 45-1728 BC = ¥(45) + (2 BD = 452 42. A is acute if b? + c? > a? and obtuse if b? +c? 74 45. Statement-2 is False as tan A + tan B + tan C = tan A tan B tan C b=1,cosC= (true) <0 tan B tan C<1 => Statement-t is True. a You have either reached 2 page thts unevalale forvening or reached your ieving tit for this book. a You have either reached 2 page thts unevalale forvening or reached your ieving tit for this book. a You have either reached 2 page thts unevalale forvening or reached your ieving tit for this book. 13.4. Course in Mathematics for ITT-JEE Ans. (c) Solution cot (A +B) = Sot Acot B . x(l+aWe (+a) (+ xt? => A+ B= C= a2 Triangle ABC is right angled. ax? (txt ry? scot Example 8 The value of cos ($00 i) is equal to (@) 4b) 3/4) YG) 1/4 Ans. (a) Solution Let cos" (1/8) = @ where 0 < @< x Then [r0<$x° ~ 4x holds if tan"! (= 1) + tan”! x + tan (x + 1) = tan! 3x is (a) 0 (b) 1 ©2 @3 Ans. (a) Solution The given equation can be written as tan! (x = 1) + tan"! (x + 1) = tan 3x - tan"! x lextl oun! 3x-x -O-DGFD 143x? 2e y eae ded 2-x 1+32x' = 4¢-x=0 = x47 -1)= => tan! 1/2 none of which satisfies | (a+b) sin = (@ x=2- 9-2 Similar (b) x= 2+ \O-2e imilarly a+b (©) x€ @- (9- 2x, 2+ 9-2) so that asin” x + b cost = Fabsetanb) (@) x>2+ (9-2 en yt 3820 Tsar ene Ans.(e) Example sin eg | tan then.x = Solution Since 32/2 <5 < 2m, we have sin 5 <0, s0 (a) tan 30 (b) 3 tan 8 sin! (sin 5) = 24-5. Therefore, the given inequality can (©) (1/3) tan 6 (d) 3 cot be written as Ans. (©) Qr-S>xP- 4x or Y-4x- (24-5) <0 asin 20 > Solution 5 Gand _ 2806 sindg x 4- J16-422-5) yo St VI6= 408-5) 26 2 2 => k- Q~ 9-2) x- 2+ 9-2 <0 => x@ 2- 9-2H,2+ \9-2t) Example 10 If l0, y >Oand x > y, then tan” (x/y) + tan” [(x + y\/(x = y)] is equal to = (A) [28028 2. 5+4cos 2 so that x = (1/3) tan @ a You have either reached 2 page thts unevalale forvening or reached your ieving tit for this book. a You have either reached 2 page thts unevalale forvening or reached your ieving tit for this book. a You have either reached 2 page thts unevalale forvening or reached your ieving tit for this book. 13.8. Course in Mathematics for ITT-JEE. n-ne 4° 2 Qn mi2n_ilix 258. Ng 4°3°3 12 Example 29 If a, Bare the roots of the equation (tan (2i5))?+ (V3 — 1) tan! (a/5) - V3 = 0, lat > If then (a) a+ B=-Sm12 (b) la~ Bl = 35 #/12 (©) a B= - 25012 ()3a+4fp=0 (a), (b), (©), (@) Solution (tan (2/5) + V3) (tan! (x/5) ~ 1) =0 Ba so that a+b x 3 x Ans. => tan (5) and tan! WiS)= 1 F =F moe Sa Se Ttgwns bey = at B=~5/l2,| a l= 35x/12 @f=-257/12and3 a+4 B=0. Example 30 @= tan" (2 tan? @ - tan“'((1/3) tan 8) if tan Bis equal to (@) -2 Ans. (a), (b) Solution @= an ~' (2 tan? —tan~! (1/3) tan 8) (b) 1 ©2723 @2 2 tan® @ ~ (1/3) tan @ 1+ (2/3) tan? @ 2 tan 8 (1/3) tan | MRSC) 1] 20 ° [ay | which is true if tan 6= 0 2tan @- (1/3) 1+ (2/3) tam’ @ => (tan @~ 1)? (tan 6+ 2) = 0 which holds if tan @= | or tan @= - 2. => tan @= = tan’ @-3 tan 0+2=0 Example 31 sin’! 6x+ sin! 6V3 x=— 4/2 if xisequal to (a) -1/12 (b) 1/6 (©) V2 (d) ~ 6 Ans. (a) Solution The given equation can be written as sin! 6x =~ 2/2 ~ sin’ 6V3x = sin (sin 6x) = sin (-/2- sin = 6x =~ cos (sin! 6V3 x) = 6x=-yi-108x" a) Squaring both sides, we get 36x? = 1-108 = 44x? = 1 => xe4Y2 Substituting x =—1/12 in the given equation, we get 1), gine! 3 4) ein(2) aR -RHS LHS = sin ( 63 2 ‘Therefore, x = -1/12 is a root of the given equation. ‘Again, substituting x = 1/12 in the given equation, we get LHS = /2, so that x = 1/12 is an extraneous root of the given equation. Hence x = ~1/12 is the only root. Example 32 If A = tan"! (1/7) and B = tan” (1/3), then (a) cos 24 = 24725 (b) cos 2B = 4/5 (©) cos 2A = sin 4B (d) tan 2B = 3/4 Ans. a), (0), (€), (4) Solution We have A = tan (1/7), or tan A = 1/7. I-tan? A _ 1-149 _ 48 I+tan? A 1+1/49 50 cos 2A = Also, from 8 = tan (1/3), or tan B= 1/3, we get sin 4B = 2 sin 28 cos 28 = 2-2. Example 33 cos"! x is equal to ii? (@ 2 sin! I> (©) 2 cos" Ans. (a), (c) Solution Let cos” => Cs ne x a You have either reached 2 page thts unevalale forvening or reached your ieving tit for this book. a You have either reached 2 page thts unevalale forvening or reached your ieving tit for this book. a You have either reached 2 page thts unevalale forvening or reached your ieving tit for this book. 13.12 Course in Mathematics for IIT-JEE. Example 52. If @= cot! 7+ cot 8 + cot” 18, then cot® @is equal to Ans. 6561 ; Tx8x18—(748+18) _ 975 Solution cot = 2~8X!8—C +8418) = ion cot O= Se iaa+i26-1 325 => cot @=3* = 6561 Example 53 If tan“! (=#3) tan! (:-2} atest 4 then the value and 250x* + 320? + 137 is equal to * Ans. 1777 Solution We have s42-(1-2) aNd 4 ge ua(e-4) = e=2 $0250x4 + 320x7 + 137 = 250 x4 + 320x2+ 137 =1777 = Example $40 tan! x= 2 tan! 1-8, 2892? + 238. + 9876 is equal to ‘Ans. 9827 Solution 2 tan“ (1/5) = tan! $/12 3 sa, x= tan (tan (5/12) - 2/4) = © ae be 12 =» 17x +7 = 0 = 28937 + 238x + 49 = 0 ‘So the required value = 9876 ~ 49 = 9827 Example 55 Maximum value of 2997 sin x + 3996 cos x is equal to ‘Ans. 4995 Solution 2997 sin x + 3996 cos x 999 (3 sin x +4 cos x) 999r sin (x + @) where r sin @= 4, reos @=3 and r=5 S$ 99x 5 = 4995 SECTION E MATRIX MATCH TYPE Example 56 (@) sin (an x) ° (p) VI-x (b) cos (tan! x) @ = “ov ° ee 1 (©) 0s (sin! 2) © fe) €0s (sin! 2) 0s (d) 60s (2cos x) (8) 2x7 -1 Ans. PQ it § *@@OO LOOT TO) © @@OOQ 4 @@O®@ | Solution sin (tan™! x)= sin @where x= tan @ _ tang x * Tetan'@ iy 1 and (tan™! x) = cos @= aa = ee NO vant” Uae cos (sin"' x) = cos @where x= sin a= Vi=x7 0s (2cos"! x) = cos (29) = 2.cos?p— 1 = 2x71 cos @) Example 57 (a) tan? 3 + tan! 4 {p) 2/2 (b) tan“ (1/3) + tan“! (1/4) (q) tan (7/11) (©) sin (1/3) + cos" (1/3) (®) tan“! (7/11) (@) tan” (3) + cot! (4) (s) tan" (13) An, Pat $ " B@OO) > @@@O © @@OOQ 4 @O0@ Solution tan"! 3 + tan"! 4 = 9+ tan’ x4 [v 3>0,4>0,1-3x4<0] = n-tan (7/11) y3+y4 = a/3y4/4) = tan“ (7/11), and sin” (1/3) + cos"! (1/3) = m2 tan (3) + cot"! (4) = tan“! 3 + tan" (1/4) = tan”! 13 tan”! (1/3) + tan (1/4) = tan a You have either reached 2 page thts unevalale forvening or reached your ieving tit for this book. a You have either reached 2 page thts unevalale forvening or reached your ieving tit for this book. a You have either reached 2 page thts unevalale forvening or reached your ieving tit for this book. 13.16 Course in Mathematics for IT-JEB. is equal to 29/3, Statement-2: (x cos (cot x) + sin (cor! »)}? x= Ysv2 Ans. (d) Solution Expression in statement-1 is tan (tan! 9 — tan” 5) = 2/23 showing that the statement is False. In statement-2, let cot! x=y => x=coty,so Showing that statement-2 is True. Example 69 Statement-1: The value of the determinant wntx cote wiz sin“"(4/5)sin“(3/5) sin" ‘cos"'(3/5)_ cos™'(4/5) 1 is equal to zero for all values of x. Statement-2: 2cos! x= cos! (2x ~I)if-I SxS 1. Ans. (©) Solution Statement-2 is true if 0Sx <1 For-1 $x <0, 2 cos! x= 4~cos (2x*~ 1) So statement-2 is false. If A denotes the given determinant ‘Then A tan” x+cot" x cote ai = | sin"(4/5) +sin"/5) sin '(G/5) si cos“'(3/5)+00s"'(4/5) cos (4/5) 1 (Applying C, > C, + C.) a2 cot'x — a/2 | s( -$43 z| sin“'3/5) sin“ 4_ fh? i-#] cos4/5) 1 3125125 a2 cot!x a2 sin (3/5) sin! cos"'0 cos (4/5) 1 = 0 (veos! 0 = 1) = | sin 1 (C, and C; are identical) So statement-1 is true, Example 70 Statement-1: 3 sin” @) + sin @) « 2af3 and tan” (2V2-1) > a3 Satement-2: If 3 sin’! x= #6, then Gr —8r'-1=0 Ans. (b) Solution 3 sin” (3) +sin? (3) 3: 5 =sin' (sx4-sar3") + sin! (3) ws (8) sam sit 34 art 3 27 $s and tan"! 2V3— 1) > tan" V3 =F [s 22-1218, 43 =17) So statement-1 is true pe % In statement-2, 3 sin”! x= a => sin (3 sin”! x) " = ‘Showing that statement-2 is also true but does not lead to statement 1. SECTION | SINGLE CORRECT CHOICE TYPE 1. The equation sin“ x = 2 sin“ a has a solution for (a) all real values of a (b) a <1 a You have either reached 2 page thts unevalale forvening or reached your ieving tit for this book. a You have either reached 2 page thts unevalale forvening or reached your ieving tit for this book. a You have either reached 2 page thts unevalale forvening or reached your ieving tit for this book. 13.20 Course in Mathematies for IT-JEE 21. 22: 23, 26. 2. SECTION IV INTEGER ANSWER TYPE If a= sin" (4/5), B= cos"! (5/13) and x=sin (a+ A) then the value of * is equal to 146085 and 77= 22/7, then the value 08 x of (2x + 14y)° ~ 7° is equal to x sin (sin! (3/5) + sin! (8/17) y= cos [oos"! (4/5) + cos (12/13)] then the value of 2431 is equal to » If 2 tan"! x = cos! (V5/3) then the value of 60x ~ 540? + 360x + 9261 is equal to Af x = sin (2 tan”! (1/3)) + cos (tan™! (2V2)) then I 1 — + + is equal to Gay rs (=x? SECTION V MATRIX MATCH TYPE (a) sin”! (2/3) + cos (2/3) (p) sinc’ sin (77/6) (b) sin (sin (7.2/6) (q) sin! sin (11216) (c) cos” (cos (#/6)) (#) tan! 2 + cor 2 (d) tan"? 3+ tan"! (13) (3) /2 (a) sin"! x + sin”! 2e= 9/3 @) 43, | 2x-1 173 toytanet 2a) tag BEM! 1f (oytan? stant Fw) SG 1B = tan! = wn 36 28. (a) tan” 1 @ 5/3 () -348 (©) tan cos x= sin (tan“! 2) 3/3 I=cosx I¥eosx <1 cos xtsin x cosxzsinx © 2ran’( ane ) L+cos.x cos 2x 2(cos.x+ sin x) ‘SECTION VI REASONING TYPE (@) sin tan (p) x (b) tar @ t-x @ x2 (8) wax 29. Statement-1: feeles)], cot +cot log(ex’) =z-tn'3 Statement-2 = tan" 1 + tan“"(log x?) 30. Statement-L: Let f(x) = cot! (255). gi) = Past Years’ IIT QuestTIONs Find the value of cos (2 cos"! x + sin"! x) at x= V5 where 0< cos"! x wand ~ #/2 $ sin" xs #/2 [1981] - a(1)_x] , The numerical value of tan j2tan-"|&)-4) is equal to [1984] ‘The value of tan (@) 617 (©) 167 (b) 76 (d) none of these [1983] 4 hpi a [22] (a) - 20/3 ©) 423 (e) none of these. (b) 22/3 @) 523 [1986] a You have either reached 2 page thts unevalale forvening or reached your ieving tit for this book. a You have either reached 2 page thts unevalale forvening or reached your ieving tit for this book. a You have either reached 2 page thts unevalale forvening or reached your ieving tit for this book. 13.24 Course in Mathematics for IT-JEE gat 2x9 49 =2an'9 Den + me fe 2a! 9 =n! 222. sit 2 ~ sin B= 7 = 81 coxec* 1681 80 cosec? @+ 81 cosec® A= 1681 + 81 = 1762 y70-1/99 29 1 1+(1/70)(1/99} ~ 6931 239 9 cot 8+ 890 = 239 x 9 + 890 = 2151 +890 = 3041 24. tan O= ro 26. cos (3 cos”! x) = cos 38= 4 cos’ 8-3.cos @ = 4x’ - 3x [x= cos 9 = as(x+4) +8=4858 3—* (x= tan ie sin (2 cos"! x) = 2xyl—27 27, xzsin @ = cos 20= 1/9 = 1-2sin? @= 1/9 Ssin = +23 x= 1 satisfies (i) and x = ~ 1 satisfies tan x + sin! xt cor! x= a = 24k -costxe x = costs 2°2 =0 28. 30. . In tel, (tan! -(£ ) In statement (tan™" x)’ 6°3 axel Statement-2 is false as tan (sin"' x + cos x) = tan (172) In statement-I,'p = 2 tan srsoes 2 qe Land we getx?—2x+1=0 => x= 1, so itis True. ae 63 = tan! x= w/b or tan"! x= x13 x= V3; V8 => a+ B= (VV3} +3 = 4/J3 = statement-1 is True L.H.S of Statement-2 = sec® (sec™! 4) + cosec? = 16 +25 =41 and the statement-2 is True. sin“ x tan (tan! x + tan"! (1 =x)) = a/2 ssin a= 1 => 1-x(1-)=1 = r= 1 is anon zero solution and thus the state- ment is false. Next, (sin™' x)’ + (cos sin“! x + cos” x) — 3sin x cos! x (sin! x + cos" x) = (2) — Bt2) sin”! x(f2 ~ sin”! x) = (8) = 32014) sin! x + B¥/2) (sin! 2° = (#18) + B?2) [sin x ~ (H/4)P - B°132) = (#132) + Gad [sin x ~ (HA) Greatest and least value of sin” GxI4) and 0 respectively. at is a You have either reached 2 page thts unevalale forvening or reached your ieving tit for this book. a You have either reached 2 page thts unevalale forvening or reached your ieving tit for this book. a You have either reached 2 page thts unevalale forvening or reached your ieving tit for this book. 144 Course in Mathematics for ITTJEE Example 10, OPORis.a square and M, N are the middle points of the sides PQ and QR respectively then the ratio of the areas of the square and the triangle OMN is fay 4:1 (by 221 () 8:3 (d) 4:3 Ans. (©) Solution Taking the coordinates of vertices O, P, Q, R as (0, 0), (a, 0), (a, a), (0, @) respectively we get the ‘coordinates of M as (a, a/2) and of N as (a/2, a) (Fig. 14.1) Mal2,a) RO.) Q(a.2) Ma, a/2) 00,0) fa.) Area of the AOMN Area of the square = a” the required ratio is 8 : 3. Example 11 If p, xj; 6) arein AP., with common difference a and b respectively then locus of the centre of mean position of the points A, Gp yo = 1,2 sao is (a) ax ~ by = aq - bp (b) bx ~ ay = ap - bq (c) bx ~ ay = bp - ag (d) ax - by = bq - ap and 4, », Ei Evi J ca) Solution Let the coordinates of the centre of mean position of the points Ans. (©) [Note Centre of Mean Position is (= aN te ” n) yw BGP 4 2+ +m) ” = sap Mardy = nee) » ta yags tty 2 = => bx - ay = bp - aq. Example 12 If a, 8, 7 are the real roots of the equation .? — 3px” + 3qx — 1 = 0, then the centroid of the triangle wn vee (a) (2) mr) acne i (a) (p.@) (©) (pA, q/3) ©) P+ap-49) (d) Gp, 39) Ans. (a) Solution The centroid of the given triangle is the point tydyt asfty a Boy = (22 bs Pre | a) 3° Sapy =) [v @+ B+ y= 3p, af+ By+ yes 34, apy= 1) Example 13 The number of points (p. q) such that p. q (1, 2,3, 4} and the equation px’ + gx + 1 = 0 has real roots is @)7 (b) 8 ©9 (@) none of these Ans. (a) Solution px? + qx + 1 = 0 has real roots if G-4p20 © G24 Since p,q € (1, 23,4) ‘The required points are (I, 2), (1, 3), CL, 4), (2, 3), (2, 4), B, 4), 4) So the required number is 7. Example 14 If G is the centroid and / the incentre of the triangle with vertices A(~36, 7), B(20, 7) and C(0,-8), then Gi is equal 10. @ V73/3 (by V397/3 (©) V205/3 (@) None of these Ans. (6) Solution We have a= BC = y(20) +(7 +8)? = 25, = V6)" +08 +7)? = 39 and b=CA a You have either reached 2 page thts unevalale forvening or reached your ieving tit for this book. a You have either reached 2 page thts unevalale forvening or reached your ieving tit for this book. a You have either reached 2 page thts unevalale forvening or reached your ieving tit for this book. 14,8 Course in Mathematics for IIT-JEE In Ex, 28. p, (Bate Brut) 3 3 So the coordinates of the centre of mean position of P,, Py vos Pyagis (BF) where eo M2 +3 tet Btyg +2 Hy Mit 29 $39, to 3¥p-9 + 2s He Paragraph for Question Nos. 29 to 32 Given two points A(- 2, 0) and B(0, 4), M is a point with coordinates (x, x),x>0. P divides the joint A and in the ratio 2: 1. C and D are the mid-points of BM and AM respectively. Example 29 Arca of the SAMB is minimum, if the coordinates of M are @ a.) ) 0.0) © (2,2) @) GB, 3) Example 30 Ratio of the areas of the triangles APM and BPM is (a) 25 (b) 1:2 © @ 1:3 Example 31 Perimeter of the quadrilateral ABCD is (a) 2V5 (b) 34305 © 24+N5 (@) none of these Example 32 Area of the quadrilateral ABCD in units is (@) 1 (b) 2 © 3 @ 4 Ans. 29. (b) 30. (a) 31. (b) 32. (ce) Solution Area of the AAMB = vie 1 | 3 (Art2r-8)| which is minimum for x = 0 and thus the coordinates of M are (0, 0). in example 30, as P divides AB in the ratio 2:1 ‘The base of the triangles APM and BPM are in the ratio 2:1 and the length of the perpendicular from the vertex Mf on the base is same. So, the ratio of the areas of the triangles APM and BPM is also 2: 1. Fig. 142 Inexample 31, ABCD is a trapezium with AD = 1, BC =2 oc= i ap 3 2 +4? = V5 So, the required perimeter is 1424 V5 42N5 =3 4 5 In example 32, Area of the trapezium ABCD = Area of the AAMB ~ Area of the ACMD. => (8-253 2 ) Paragraph for Question Nos. 33 to 35 A(p. 0), B(4, 0), C(S, 6) and DUI, 4) are the vertices of a ‘quadrilateral ABCD. Example 33 [f ZADC is obtuse, the maximum integral value of p is ft (b) 2 (©) 3 iW) 0 Example 34 The value of p for which ZADC is obtuse and ZBAD isa right angle is wt (b) 2 (c) -3 (d) -4 a You have either reached 2 page thts unevalale forvening or reached your ieving tit for this book. a You have either reached 2 page thts unevalale forvening or reached your ieving tit for this book. a You have either reached 2 page thts unevalale forvening or reached your ieving tit for this book. 14.12 Course in Mathematics for IT-JEE (PQ) = 29, (QRY = 205, (PRY = 320 16 (PS) = ((2/5) (POP = =e. (PS) = (215) (PONT = SE (057 = 12/5) (POF = 8. 25 38 = PGI) cy 9 iy 218%) FR) #2 Fig. 145 So that (PR)? + (QR) +51 = 320 +205 + 51 = 576 100 (PSP = 4 x 116 = 464 and (PQ)? + (QR) + 25 = 29 + 410 +25 = 464 Next, 2(PR)° ~ 64 = 2 320-64 = 576 2) 50 (QS) — (PQ) = 2x 261- 2% 29= 464 (5) and 5 (QT)? + (QR)* + 2 = 369 + 205 + 76 w Combining these results we get the required answer, SECTION Vi REASONING TYPE Example 46 Statement-1: The points A(3, 4), B(2, 7), C(4, 4) and D(3, 5) are such that one of them lies inside the triangle formed by the other points Statement-2: Centroid of a triangle lies inside the triangle. Ans. (@) 34244 44744 Solution = Sand =5 So D is the centroid of the triangle ABC, using statement 2 which is correct we find statement A is also correct. Example 47 Statement-1: The points A(-2, 2), B(2,~2) and C(1, 1) are the vertices of an obtuse angled isosceles triangle. Statement-2: Every obtuse angled triangle is isosceles. Ans, (©) Solution Statement-2 is not correct as the triangle with angles 120°, 40°, 20° is obtuse angled but not isosceles, In statement-1 AB = 42 ,AC= VIO, BC = VIO AC? + BC? - AB* _ 10x10-32 <0 AACKBC) 2x Vi0 VIO + Cis obtuse So the triangle is obtuse angled isosceles. Example 48 Statement-1: The quadrilateral whose vertices (in order) are A(1, 0), B(O, 3), C- 2, 0) and (0, 2) can not be conver. Statement-2: A quadrilateral ABCD (in order) is convex if and only if when any diagonal is taken then the remaining vertices must be on the opposite sides of it. Ans. (a) Solution Statement-2 is correct because if one angle say D of the quadrilateral ABCD which is not covex is more than 180° then B and D tie on the same side of the diagonal AC of the quadrilateral. Statement-1 is true, using statement-2, the quadrilateral ABCD is not convex, 8 (0,3) cos C= (0.2) ce20) (1.0) Fig. 14.6 Example 49 Statement-1: Three points A(x), y;) Bos, y) and Cry, yy) are collinear if x, +.r +.45 = 9) +32 +3. Statement-2: The points A(x, - x), BO, -5)C(-5, 3) are collinear if x= 1 Ans. (a) Solution Statement-1 is False as the condition for nyt 1)=0, using in statement-2 we Hoyt collinearity is |x, y jxocr have | 7-5 1]= 1 1) => 4x-4=0=9.x= 1 and statement-2 is True. ExampleS0 Statement-1: A(2, 2), B(6,—1) and C(7, 3) are the vertices of a triangle ABC. D, E, F are the mid- points of the sides BC, CA and AB respectively. Area of the ADEF is equal to 19/8 square units. Statement-2: Lines joining the mid-points of the sides of a triangle divide the triangle into four triangles of equal areas. a You have either reached 2 page thts unevalale forvening or reached your ieving tit for this book. a You have either reached 2 page thts unevalale forvening or reached your ieving tit for this book. a You have either reached 2 page thts unevalale forvening or reached your ieving tit for this book. 14.16 Course in Mathematies for IIT-JEE. SECTION | SINGLE CORRECT CHOICE TYPE L. If the points (a, 1), (1, 6) and (@ = 1, b= 1) are collinear, a, fare respectively the arithmetic and geometric means of a and } then 4a - # is equal to fa) -1 (b) 0 © 3 (@) 2 2. If P (1, 0), @ © 1, 0) and R (2, 0) are three given points. The point § satisfies the relation SQ? + SR* = 2SP*, The locus of 5 meets PQ at the point (a) 0, 0) (b) (23, 0) © 32,0) (@) @,~ 2/3) 3. Ajy Any «+ Ay af points whose coordinates (x, y) satisfy y = x and lie in the positive quadrant such that 0A, =n OA, ,, O being the origin. If OA, = | and the coordinates of A, are (2520V2 , 25202 ) then n= fa) 5 (b) 6 7 @) 8 4, The coordinates of the point A, are (na, a"). If denotes the arithmetic mean of x coordinates of the points Ay, A, ... A, and denotes the geometric ‘mean of the ordinates of these points then if a is a ‘variable, locus ofthe point P(a, 5) is ( v=ay (@) none of these 5. Coordinates of the point A,, are (1, 2n) and of B, are (n°, - 2n). Area of the quadrilateral with verti- ces A, B, B, A, is (@) @+1y tb) (n= 1 (©) 2@° + 1) (d) 20? = 1) SECTION II MULTIPLE CORRECT CHOICE TYPE 6, ABC is an equilateral triangle. If the coordinates of two of its vertices are (1, 3) and (~2, 7), the coordi- nates of the third vertex can be fa) [-feais- 28) 28) (b) (4 +2N3,5- 1 3V3 ©) {}-28--24) @ (}+28. 3-38) 7. The coordinates of a point at a distance V3. from the point (3, 4) and having the same abcissa and ordinate are (a) (2, 2) b) (- 2,- 2) © (5) @ 5-5) 8. If two vertices of a triangle of area a? are (0, 0) and (a, 0) then the coordinates of the third vertex can be (@) (a2, 2a) (b) Gal2, - 2a) (©) (al2, aly (@) (a2, - 2a) 9. P(-a,0), O(a, 0) and R (1,1) are three points such (QRY'| = 12 then 7 (b) (QRY = 5 (©) (PRY (a) (OR) = 10. A(O, b), BQO, 0) and C(a, 0) are the vertices of a triangle ABC. D. E, F are the mid-points of the sides BC, CA and AB respectively. If a? + 5? = 20, then (a) (ADF = 9 (b) (BE) = 4 (©) (ADP + (CFY = 25 (d) (AD) + (CFP = (BEY SECTION Ill LINKED COMPREHENSION TYPE Paragraph for Questions Nos 11 to 13 Vertices of a parallelogram are A(O, 0), B(- 2/3, 7/3), Cla, b) and D(S/3, 11. Length of the larger diagonal of ABCD is (a) V2 (b) 58/3 © Va @ i703 12. If P and Q are two paints on the diagonal DB such that DP = PQ = QB then area of the ACP in square units is (a) 3 (b) 2 fo) V4 @ 1 13, Sum of the areas of the triangles ABP, APD, BOC and COD is (@ 2 (b) 372 © @ 12 a You have either reached 2 page thts unevalale forvening or reached your ieving tit for this book. a You have either reached 2 page thts unevalale forvening or reached your ieving tit for this book. a You have either reached 2 page thts unevalale forvening or reached your ieving tit for this book. 14.20 Course in Mathematics for IT-EE 15. 16. 18. 19. 24, 28. 26. ax+an tar s0x41+P 20 ‘ordinates of P and @ are the roots of (7? ~ 1) y* + Wyth-4=0 (feeer eb +tp sre) 5 3a? _ (Ger -2%3q Gq -2x3p ~ 3 . 3 = (3p?—2g, 34° - 2p) | atc= dba tl+c+9=2b +4) cost +sint 3 Asin tcos 1). sint - cost or _ cost + sin) ~ 3 eos f= sin sor sin t= 3 cost fi mm I Area of the triangle | mm’ mtm’ | (my? 2m’ | me 2m = 4 (m’—m) m'-m 0) m’>~m* 2m'—m) 0 = 3 1m’ — my" = 32.=9(m'-m)'=2 64 (OP) + (00) = (POY = 4x; + ¥192 = 0 PO, 1), QU/2, 1), RAP, 7/3) (PRY? = 17/9, (ORY = 65/36 Area of A OAB = 3/2, Area of A PQR = 1/3 |. $116, 10/6), OS = V5 13, OQ = V5 12 AB= V5, PQ = 1/2, PS= ViT/6. @+lah +1 a(a-lP+b-1)P = a=b[as0 4p, distance of the point is least if pp? +4? is least 21. 28. 29. 30. 31. 32, 33. 35. = p=50,q=15 Yioan?=F e+ a - mins Want (nor) 6 2 a4+b=2x28= 56,b-25a=4=0=2,b=54 x= 2ny=2?b=raers3 and Q(x, y) = (6, 18) = OP? + OG? = 2 + 6 + 187 +54 (p- WP + 4 aap? ps 4 a3 P (A, Ay.) = 2 for all n, Required sum (10+ 11 +... +20) = $30 (OA,) =n? + Qn + 1) = Sn? in tant 1 (AA) = (= 1)? + Qn)? = Sn? In +1 (A, A, = (a 1)? + Qn + 1-3)? = Sn? — 100+ 5 Ay. Ay? = 5 ata+2 2054 b+b+6 =0>b=-3 coordinates of P (0, — 3/2), Q (5/2, 0) RW, 3/2). Centroid of AABC and APQR is same (a+3,b+3)=2,0) Statement-2 is True. Using it in statement-1 each side of the square PORS is equal to [/3)? + (2/3)? and hence its area is 5/9. Centroid of ABC is (a ~ 1/3, b = 1/3). if (X, ¥) are the new coordinate of A the X = a -(a—1/3) = 1/3. ¥ = 6 —(b~ 1/3) = statement-| is true. ‘Statement-2 is also true but is not a correct expla- nation for statement-I. a+ Batre locus is (x af? + (y = BP = [x= (a+ NP + Ly Bor = xty—(at P=r= 2x4 2y-21=0 Statement-1 is true. For statement-2, Let the fixed points be (— 1, 0), (1, 0) whose mid point is (0, 0). Locus is Vu+l?+y +Va-P+y = ¢ (constant) ‘which does not pass through (0, 0) for all values of cc. So statement-2 is False. a You have either reached 2 page thts unevalale forvening or reached your ieving tit for this book. a You have either reached 2 page thts unevalale forvening or reached your ieving tit for this book. a You have either reached 2 page thts unevalale forvening or reached your ieving tit for this book. 15.4 Course in Mathematics for IMT-JEE. be the equations of a line and a conic, respectively. Writing the equation of the line as bet m, = | and making § = 0 homogeneous with its help, we get $ = ax” + Zhxy + by? + gk + fy) (et) (etmy sam +e =0 =n =n which being a homogeneous equation of second degree, represents a pair of straight lines through the origin and passing through the points common to S = 0 and L = 0. 7. Some important results regarding pair of lines (i) Equation of the pair of lines through the origin perpendicilar to the pair of lines ax! + 2hay + by? = 0 is bx? — Yay + ay? = oO (ii) The product of the perpendiculars drawn from the point (x), y,) on the lines ax? + 2hxy + by? = 0 is 2 2 | hy, + by; Via—by +4 (iii) The product of the perpendiculars drawn from the origin to the lines ax? + 2hcy + by? + ge + 2 te = Dis © Via— br an (iv) Equations of the bisectors of the angles be- tween the lines represented by ax’ + 2hxy + by? + 2ex + fy + ¢ = 0 are given by (x4) -O-nF _ =) =) a h where (x. ¥,) is the point of intersection of the lines represented by the given equation. (v) If ax? + Qhey + by? + 2gx + By toe 0 represents two parallel straight lines, then the distance between them is 2 ‘a{atb) Every nth degree homogeneous equation agx" +a, x"~' y tay lyt... ta, x" '+a,y"=0 in x andy represents n straight lines through the origin. Mf my, ms, Mm, are the slopes of these 7 lines and a, # 0, then my tomy toe ty = 1 a, and om, my... m, = Gly SOLVED EXAMPLES ‘SECTION I SINGLE CORRECT CHOICE TYPE Example 1 If a, b,c forma G.P. with common ratio r, the sum of the ordinates of the points of intersection of the line ax + by + ¢ = 0 and the curve x + 2y? = 0 is @)-rn (b) ~ (c) rid @ 72 ‘Ans. (¢) Solution “The equation of the given line is ax+ by +.c=0 sartaytar=0 > rtrny+P=0 @ (i) Tntersects the curves x + 2)? = 0 at the points whose ordinates are given by -w+y+P=0 or W-y-P=0 ‘Therefore required sum of the ordinates = 1/2. Example 2 Orthocentre of the triangle with vertices (0, 0), (3, 4) and (4, 0) is (a) (3, 5/4) (b) (3, 12) (©) G, 3/4) (a) (3, 9) Ans, (©) Solution Let 00, 0), A(, 4) and B(4, 0) then equation of OB is y = 0. Equations of the altitudes from A and 0 are respectively x= 3 and y = (1/4) x so the orthocentre is (3, 3/4), the point of intersection of these altitudes. Example 3 The number of integral points (integral point means both the coordinates should be integer) that lie exactly in the interior of the triangle with vertices (0, 0}, (0, 21) and (21, 0) is (a) 133 (b) 190 (c) 233 (@) 105 Ans, (b) Solution Af (x, y) is an interior point of the triangle OAB then x4+y<21. Possible values of xare 1, 2, ... 19 and the corresponding values of y are 19 for x = I, 18 for x= 2and soon | forx= 19 as x+y $20, the total number 5 19x20 of points are 19 + 18+... #1 = —3— =I O21), A alo 21.0) Fig. 18.1 a You have either reached 2 page thts unevalale forvening or reached your ieving tit for this book. a You have either reached 2 page thts unevalale forvening or reached your ieving tit for this book. a You have either reached 2 page thts unevalale forvening or reached your ieving tit for this book. 158 Coune in Mathematics for ITT-JEE Example 18 If a line joining two points A (2, 0) and B (3, 1) is rotated about A in anticlockwise direction through an angle 15°, then equation of the line in the new position is (a) Bx y= W3 te) x + V3y = 3 ‘Ans. (b) (b) V3x-y = 23 (d) x - V3y = 2v3 Solution Slope of AB = If AC is the new position of the line AB then ZCAX = 45° + 15° = 60", and thus its equation is y= tan 60° (x — 2) = ye dS -2) = Vix-y=2VF 8G,1) AQ,0) Fig. 15.7 Example 19 An equation of a line through the point (1, 2) whose distance from the point (3, |) has the greatest value is (a) y= 2x @xtw=s Wd) Ans, (a) Solution Let the equation of the line through (1, 2) be y-2=m(x-1) a If p denotes the length of the perpendicular from (3. 1) on this line, then b) ysael 3x-1 2 = 2 = Ami sdmsi 4, dm=3 m +t m +1 then p” is greatest if and only if s is greatest = P = 5 (say), 2 Now! d5. = Lon? +194) —2mt4m—3) am (nm +17 _ = 2Qm+1)(m=2) (+P ds f alice moe oli2,/2. Blea 0 if 12 < m<2 <0 if m>2. So s is greatest for m = 2 and thus the equation of the required line is y = 2x. Note will be least when (3, 1) lies on the line through (1, 2) and the equation of the line in that case is x + 2y = 5. Example 20. Let 0 < @< a/2 be a fixed angle. If P= (cos @ sin @) and Q = (cos (a— 8), sin (@— 8)) then is obtained from P by (a) clockwise rotation around the origin through an angle @ (b) anticlockwise rotation around the origin through an angle @ {c) reflection in the line through origin with slope tan @ (d) reflection in the line through the origin with slope tan (af2) Ans. (a) Solution OP makes an angle @ with the positive direction of x-axis and OQ makes an angle (@~ @) with the positive direction of x-axis.(Fig.15.8) y (08 8, sin 8) (costar 6), sin (a~ 0) Fig. 15.8 So that ZPOQ = a and thus @ is obtained from P by clockwise rotation through an angle @ around the origin. Note In the anticlockwise rotation, OQ makes (27—- a+ 8) with the + ve direction of x-axis. a You have either reached 2 page thts unevalale forvening or reached your ieving tit for this book. a You have either reached 2 page thts unevalale forvening or reached your ieving tit for this book. a You have either reached 2 page thts unevalale forvening or reached your ieving tit for this book. (a) (~ 8/3) sin? @ (©) ~ 8V3 cosec 2a Ans. (b) (b) (© 8/3) cosec 2a (a) = 4 cose 2a Solution The given equation can be written as (P4y") (cos? O sin? @ + sin’ @ = 7 tan? a 2xy tan @ sind + y* sin? or (cos? @ sin’ a+ sin? @~ tan? a x° +2 (lan @sin @ xy + 6 n= 0 Since the slope of these fines are given as tan @ and tan 8 + tan = 2x (3/4) xsin? @ (8/3) cose 2 Example 35. If two of the lines represented by Jy a syteo bisect the angle between the other two, then the value of ¢ is Bary ter @o Ans. (d) Solution Since the product of the slopes of the four lines represented by the given equation is | and a pair of lines represent the bisectors of the angles between the other two, the product of the slopes of each pair is ~1. So let the equation of one pair be ax’ + hry ~ ay? = 0. o&-1 ©) @) -6 ‘The equation of its bisectors is By hypothesis wtexyrary —y'tyt = (ax? + Dhxy — ay?) (hx ~ ary - hy’) = ahix* + y*) + 2(h? —a*) (Py — xy) - bah’? Compairing the respective co-efficients we get ah= 1 and c 6ah =~ 6 SECTION II MULTIPLE CORRECT CHOICE TYPE Example 36 The point of intersection of the lines x42 24221 and ab” fies on the fine Swraight Lines 18.13 (a x-y=0 (b) (+ y) (@ + b) = 2ab (©) x + my) (a +b) (A) (Ls ~ my) @ +b) Ans. (a), (6), (€) and (@) Solution Solving the given equations, we find that the point of intersection of the given lines is ab ab (S ~) which lies on all the lines given by (a), (b), (e) and (d). Example 37 Equations OK + ~ ay + (a~b) 0 and (ewe (co - ara’ —b= will represent the same line if @) b=e by) esa @a=b Watbtes Ans. (a), (b), (€) and (d) Solution The two lines will be identical if there exists ‘some real number k, such that Bc =kb- 0, c ~a* = ke -a) and a — b= k(a~ by. = b-c=0 © BHC beak, c-a=0 0 CHa +cask and a-b=0 of a+b + ab=k ‘That is, b= core =a ora =b. Next B+ c+ be = 2 +a? + ca = ba = la ~ b) Hence, a = b ora +b+e=0. Example 38 The straight lines x + y=0,3x+y—4=0 and x + 3y - 4 = 0 from a triangle which is (a) isosceles (b) right-angled (©) obtuse-angted (d) equilateral, Ans. (a), (©) Solution Let the given lines be represented by AB, BC and CA, respectively, Then A(-2, 2), B(2, -2) and (A, 1) are the vertices of a triangle ABC, Also, note that AB = 4V2, AC = VI0 and BC = VIO. This shows that AABC is isosceles; it is clearly not right-angled or equilateral. Since cos C= BOL #AC? = AB? _ 10410-32 2 BC) (AC) 210 Vi0 = A ABC is obtuse-angled, Example 39 Equation of a straight line passing through the point (4, 5) and equally inclined to the lines 3x=4y +7 and Sy = 12x +6 is (a) 9 = Ty = 1 (b) 94+ Ty = TI (©) Ix + 9y = 73 (A) Tx = 9yGHNTia! O a You have either reached 2 page thts unevalale forvening or reached your ieving tit for this book. a You have either reached 2 page thts unevalale forvening or reached your ieving tit for this book. a You have either reached 2 page thts unevalale forvening or reached your ieving tit for this book. 15.18 Course in Mathematics for IT-JEE Equation of any line parallel to (ii) is Bx+4y= Since its distance from (iii) is 7 k Tree Example 53 The lines joining the origin to the point of intersection of 3x°+ Ary—4x+1=Oand2x+ y-1=0 are at right angles for (@Az-4 (c) A=7 Ans. (a), (b) and (c) Solution Equation of the lines joining the origin to the points of intersection of the given lines is 4x7 + Ary ~Ax(2e + y) + Lx + y= 0 =7 = kaa(7x5)=235 (b) 424 (d)_no value of (Making the equation of the pair of lines homogeneous with the help of the equation of the line) = #-ay-y =0 which are perpendicular for all values of A. Example $4 If x° + 2hxy + y= 0 represents the equa- tions of the straight lines through the origin which make an angle awith the straight line y+x=0, then (a) secl@=h © 2sina= * Ans. (a), (b) and (A) Solution Let equation of the lines given by a? + hy + y= 0 be y = myx and y = myx. Since these make an angle @with y + whose slope is -1, (b) cos a= (6) cot a= Example SS If y=mu bisects the angle between the lines 2 (tan? @+ cos 6) + 2xy tan Oy? sin? @= 0 when = af3 the value of m is 2-7 @) (© 27 (d) 23 Ans. (a) and (b) Solution Equation of the bisectors of the angles between the given lines is yt je ie OE, O+ cos? O+sin?@ tan 2 os Boyt ay Tun? and 2_y2 = ~ =X 2 ©) when O= a3 143° Wa which is satisfied by y = mx if dont Lm 4 N3 = 3m? + 4m— V3 = 0. = 27 v3 SECTION Ii LINKED COMPREHENSION TYPE Paragraph for Question Nos. 56 to 58 Aly, ¥pde Bley Yo) Cy, ys) are the vertices of a triangle ABC. Lx + my +n = 0 is an equation of the line ZL. Example 56 If L intersects the sides BC, CA and AB of the triangle ABC at P, Q, R respectively then BP CQ AR | Sox x is equal ti Pe oA“ ep eaualto (a) = 1 (b) - 12 ©) 12 @t Example 57 _ If the centroid of the triangle ABC is at the origin and algebraic sum of the lengths of the perpendiculars from the vertices of the triangle ABC on the line £ is equal to 1 then sum of the squares of the intercepts made by Lon the coordinate axes is equal to fa) 0 (b) 4 9 (d) 16 Example 58 If P divides BC in the ratio 2 : 1 and Q divides CA in the ratio | : 3 then R divides AB in the ratio (P.Q.R are the points as in Ex. 46) a You have either reached 2 page thts unevalale forvening or reached your ieving tit for this book. a You have either reached 2 page thts unevalale forvening or reached your ieving tit for this book. a You have either reached 2 page thts unevalale forvening or reached your ieving tit for this book. ae -a 1 = V3, tines in (d) are perpendicular to ‘those in (a) Example 88 (a) If the slope of one of the lines represented by ax” ~ Gry + y= 0 is square of the other then If 9x? + 2a ~ 2) xy + ay" +64 + 4y —3 represent two parallel lines, then Ifa denotes the slope of a line given by 108° + 31xy + 9? = 0, then If a®x? + 1Sxy = 16y" represent a pair of two perpendicular lines then (Das (b) @e 4 q 8 fc) (a «@ asd Ans, Pp ros 4 1O@ 1O@OO| @@OO®| 4®OOO) Solution (a) m+ m? = 6, me and a= 8 or ~ 27. (b) The lines are parallel if 9x4=(a- 2 => a-2=+6 = a=8ora=-4 But the equation represents a pair of lines if a= m=2or-3 9 a-23 a-24 2 | 20orifa=28 3 2 =3 So we geta =8 (c) We have (27x + y) (4e + Slopes are ~ 27 or - 4 ie. a a=-4 (d) Lines are perpendicular if @ - 16 = 0 => a=t4 Example 89 ‘The lines given by (a) 6x° + Sty ~ 6? =r + Sy-1=0 0 ~2 or (p) are parallel Straight Line 15.27 (b) 30x? + 36xy + 6y? ~ 35x (q) are perpendicular -iy+5=0 (ec) 30° + 4lxy + 6y" ~ 35x (r) makeanintercept ~Iy+520 5/6 on x-axis (d) 6x? + 12xy + 6y 7x (s) make an inter- =Ty+1=0 cept 1/6 on Ans, pars @OO O@O® Solution (a) Co-efficient of a? + co-efficient of => line are perpendicular Meet wvaxis at 6x? 1/2 or x = = 3 => intercept on 11s waxis = 5+ = > 2°36 Meet y-axis at - 6)" + Sy -1 = 0 = Qy-D)Gy-)=0 = y= Wory= 13 => intercept on y-axis = 12-13 = V6. (b) The lines are neither parallel nor perpendicular meet waxis at 301° - 3544 5=05 x= lore 1/6 so the intercept on x-axis 16 = Si: Similarly intercept on y-axis = | - 5/6 = 1/6- As in (b). The lines are parallel as 6 x 6 = (6)* (h? = ab) meet waxis at 6° -7x+ 120 v= lorx= 6 © (d) = so intercept on x-axis = 1 - 1/6 = 5/6 intercept on y-axis is also 5/6. SECTION VI REASONING TYPE Example 90 Statement-1: If the circumcentre of a triangle lies at the origin and centroid is the middle point of the fine joining the points (2, 3) and (4, 7), then its ‘orthocentre lies on the fine 5x 0. Statement-2: The circumcentre, centroid and the orthocentre of a triangle lie on the same line. a You have either reached 2 page thts unevalale forvening or reached your ieving tit for this book. a You have either reached 2 page thts unevalale forvening or reached your ieving tit for this book. a You have either reached 2 page thts unevalale forvening or reached your ieving tit for this book. 52. 53, 54, 55. Statement-2; If a and 6 are constants (a, b #0) and cis a variable, then from the lines ax + by +e = 0 and bx — ay — the line farthest from the origin is the one for which Ic! is least. ‘Statement-1: If the area of the triangle formed by the lines y = x, x + y = 4 and the lines through P(h, k) parallel to y-axis is , then P lies on Pty deeded ‘Statement-2: Area of the triangle formed by the lines y =.x, x + y= 4 and the x-axis is equal to half the area of the triangle formed by the line. x + y = 4 and the coordinate axes. ‘Statement-1; Reflexion of the point (6, 1) in the line x + y = 0 is (- 1, -6). ‘Statement-2: Reflexion of a point P(a, f) in the line ar+by+e= is ovr, pit (22% BAF.) lies on the line. ‘Statement-I: The product of the perpendiculars from the point (1, 3) on the lines 15x? + 14 =0is 32/4. Statement-2: The product of the perpendiculars drawn from the point (x,, y,) on the lines ax? + is [24 42h +f | Yla-b) 4h? Statement-1: Angle between the lines represented by 3x7 + 1Oxy + 3)? ~ 2e + 2y 1 = 0 and 3x7 + Oxy + 3y? ~ 11x + 7y - 20 = 0 is same. Statement-2: Equations of the bisectors of the angles between the lines 3x° + 10xy + 3y* = 0 is P-ys0 — 8)? 2hxy + by? ‘SECTION | ‘SINGLE CORRECT CHOICE TYPE Triangle ABC has vertices (0, 0), (11, 60) and (91, 0). If the line y = kx cuts the triangle into two triangles of equal area, then k is equal to (a) 30/51 (b) 47 © 14 (@) 3091 . The distance of the point (1, 2) from the line x + y +5 =0 measured along the line parallel to 3x — y = 7 is equal to @ 4Vi0 (c) Va0 (b) 40 (@) 102 . A ray of light travels along the line 2x~3y + 5=0 and strikes a plane mirror lying along the line 10. Straight Line 15.33 x-+y=2. The equation of the straight line contain- ing the reflected ray is (a) 2x -3y+3=0 (b) 3x-2y +350 (©) 2x-Ty+1=0 (@) 2x4 7y-1=0 Given two points A = (— 2, 0) and B = (0, 4), the coordinates of a point M lying on the line x = y so that the perimeters of the AAMB is least; is @ dD (b) (0, 0) © 22 @) G, 3) If the point (@ @) lies inside the triangle formed by the lines 2r + 3y - 1 = 0,4 + 2y - 3 = 0 and Sx - 6y ~ 1 = 0, then (@) - 12 (Qx~y) (12x? -20xy + 3y") = (Qx-y) 6x~y) Qx~3y)=0 The lines are 2x ~ y= 0, 6x ~ y = O and 2x ~3y= Perpendicular distances from (1, 1) are 1 5 L 5 BAIT "BF a + 37p" = (52) + (SY? = 2729 41. Vertices of the parallelogram are AQ1.82,5).CG, 5.08.1) Equation of AC, 4x ~y = 7; Equation of BD = 4x + y = 13 on Doo: =13+7=20 => Qa +36)? = (63)? = 3969 42. It V2.c=5,(b)m ‘mx + cthen (a) m= similarly for (c) and (4) (6), (A) The lines do not form a triangle if they are concurrent or any two of them are parallel 45. (a) 31441 +7=14 Similarly for other pars 46, Match with the point of intersection of the lines 47. Area of the triangle formed by the lines in 0 0 4] uy @ 5)! i Similarly for other parts 48, (a) +) Ax- 39) (b) (2x = 3) Gx + 4y) (©) =») Qe (a) Gx + 4y) (ey) 49. Vertices are (0, 0), (&/2, 0), (0, &/3). It is a right angled triangle, right angle at the origin. So orthocentre is at the origin for all values of & 2 ‘= 3, 2)ifk= Cireum-centre is ( z “(kk Centos (£, 4). G,2)ifk= , lke Area ofthe wiangle = 5 «5x5 = 192 if P= (487 50. Statement-2 is true (See Text) Equation in statement is x+ V3y+5+ cot 0/3 x—y- 7) =0 where cot & is non-zero finite real number. As @enaid Using statement-2, Statement-1 is True. 51. Statement-2 is false as the distance of any such line from the origin is, a You have either reached 2 page thts unevalale forvening or reached your ieving tit for this book. a You have either reached 2 page thts unevalale forvening or reached your ieving tit for this book. a You have either reached 2 page thts unevalale forvening or reached your ieving tit for this book. S+ AL =0 represents a family of circles passing through these points 5. If L = 0 is a tangent to the circle $= 0 at P, then S+ AL = 0 represents a family of circles touching S = 0 at P, and having L = 0 as the common tangent at P. 6. Coaxal Circles A system of circle is said to be coaxal if every pair of circles of the system have the same radical axis, The simplest form of the equation of a coaxal system of circles x? + y? + 2gx + ¢ = 0, where g is a variable and c is constant, the common radical axis of the system being y-axis and the line of centres being x-axis. The Limiting points of the coaxal system of circles are the members of the system which are of zero radius, Thus the limiting points of the coaxal system of circles ty + der tec = Oare (+ ¥e,0) fc 20. ‘The equation S + 28’ = 0 (24-1) represents a family of coaxal circles, two of whose members are given to be $= O and 5’ =0. Conjugate systems (or orthogonal systems) af circles Two system of circles such that every circle of one system cuts every circle of the other system orthogonally are said to be conjugate system of circles. For instance, 0 and x + y? + 2fy-c =0, where g and f are variables and c is constant, represent, two systems of coaxal circles which are conjugate. 16.6 COMMON TANGENTS TO TWO CIRCLES Sty der te If (x~ 9)" + FP = a7 and (x- g,) + (y- fF = a} are two circles with centres Cy (g,. fi) and Cy (a, fy) and radii a, and a; respectively, then we have the following results regarding their common tangents. 1, When C, C; > a, + a; ie. distance between the centres is greater than the sum of their radii (Fig, 16.2), the two circles do not intersect with each other and four common tangents can be drawn to two circles, Two of them are direct common tangents and the other two are transverse common tangents. The points T,, T; of intersection of direct common tangents and transverse common tangents respectively, always lie on the line join- ing the centres of the two circles and divide it externally and internally respectively in the ratio of their radii, Circles 16.3 centres is equal to the sum of the radii (Fig. 16.3), the two circles touch each other exter- nally, two direct tangents are real and distinct and the tranverse tangents coincide. oo n ea Fig. 16.3 . When C, C; x=a@-aandy=f As AG, y) lies on the circle x7 +’ = a” we have (a-are B @ — 200+ f? locus of M(a, 8) is x2 + y* = 2ax, = > AG») Fig. 16.13 MC a, By Circles 16.7 Example 13 If common chord of the circle C with centre at (2, 1) and radius rand the circlex° + y*~ 2x~6y +6= (is adiameter of the second circle, then value of ris, @ 3 (b) 2 (©) 32 @t Ans. (a) Solution Equation of the circle C is @-27+0-V er = P4y-4x-2+5-P =0 juation of the common chord is (r+ y -4x-2y+ 5-7) -(8 +? -2e-6y +6) =0 = -4y+P4l=0 If it is a diameter of the second circle, it passes through the centre (1, 3) of the circle $o2-4x3+F +1203 29-9723 Example 14 Tangents drawn from the point P(1, 8) to the circle x° + y* - 6x —4y - 11 =0 touch the circle at the points A and B. The equation of the circumeircle of the triangle in PAB is (a) P+ y? + 4x -6y + 19=0 (b) x2 + y? - 4x — 10y + 19 (©) P+ y= 2+ by-9=0 @) P+ y-6x- ay + 19=0 Ans. (b) Solution Equation of AB the Chord of contact of p is Lx + 8y—3(x+1)-20+8)-11=0 = -2r+6y-30=0 3 r-3y4 1520 Equation of any cirele through. AB is + y? - Gx - dy - 11 + Ale ~ By + 15) = 0 It will pass through P(1, 8) if 1+64~-6-32-11+ 21-244 15) =0 = 16-84=0 = A=2 ‘Thus, equation of the required circle is x+y? = Gx - 4y - 11 + 2r- 3y + 15) =0 sox ey? 4r- lOy + 19 = Fig. 16.14 Example 15 Let ABCD be a quadrilateral with area 18, with side AB parallel to CD and AB = 2CD. Let AD be perpendicular to AB and CD. If a circle is drawn inside the quadrilateral ABCD touching all the sides, then its radius is (a) 3 (b) 2 © 32 q@t Ans. (b) a You have either reached 2 page thts unevalale forvening or reached your ieving tit for this book. a You have either reached 2 page thts unevalale forvening or reached your ieving tit for this book. a You have either reached 2 page thts unevalale forvening or reached your ieving tit for this book. 16.14 Course in Mathematics for T-JEE = ZOBP = 6 Equation of the tangent OT at O to the given circte is d+ dy =0 => slope of the tangent = ~ 2/3 So that, if ZXOT = a tan w= 2/3 From geometry, ZPOT = ZOBP = 8 => ZPOX = @-a@ tan @—tan ar 0- a= and tan (O= = an Otan Hence the equation of OP i Example 38 On the line joining the points A (0, 4) and B (3, 0), a square ABCD is constructed on the side of the line away from the origin. Equation of the circle having centre at C and touching the axis of x is (a) 2 + y? - 14x ~ 6y +49 = 0 (bo) P+? = Me - by +920 (©) + y° ~ 6x - ty +49 =0 =xtan (0- @)=>x-2y=0 (d) P+ -6r- My +9=0 Ans. (a) Solution Let ZABO = @ then £CBL= 90" -@ CL being perpendicular 10 x-axis (Fig. 16.27). The coordinates of Care (OL, LO) OL= 08 + BL=3+5 sin 0 = 345 x (4/5) =7 CL = 5 cos = $x (V5) =3 So the coordinate of C are (7, 3) and the equation of the circle having C as centre and touching x-axis is @&- 78+ (y 3y = (CLP = 9 = ey - Me ~ by +49 = 0. Example 39 A circle with centre at the origin and radius equal to a meets the axis of.xat A and B. P (a) and Q (A) are two points on this circle so that a — B= 2x, where y is a constant. The locus of the point of intersection of AP and BQ is (a) xt ay an y= (b) x? + y? — 2ay tan y= a? (©) xP + y? + Qay wan y= a? (d) x? = y? + ay tan y= Ans, (b) Solution Coordinates of A are (- a, 0) and of P are (acos aa sin a) Equation of AP is asin a(cosa@ +1) «ro or tan (a 12) (w+ a) wo ye 0 (a cos fa sin B) ‘Bla, 0) Fig, 1628 Similarly equation of BQ is easing »* Teop-y or y= - cot (8/2) (x - a) ai We now eliminate @ 8 from (i) and (i) From (i) and (ii) tan (a /2) —., tan (82) = atx Now a f= 2 a You have either reached 2 page thts unevalale forvening or reached your ieving tit for this book. a You have either reached 2 page thts unevalale forvening or reached your ieving tit for this book. a You have either reached 2 page thts unevalale forvening or reached your ieving tit for this book. B(O.b) Dial, 2) IN % A(a.0) ars Example 64 A and B are two points on the r-axis and y-axis respectively. Two circles are drawn passing through the origin and having centre at A and B. (a) Equation of the common chord is ax ~ by = 0 (b) mid-point of the common chord is ab ) veh =. ae (©) AB bisects the common chord, (a) AB is perpendicular to the common chord. Ans. (a), (b), (©), (d) Solution Let OA =a and OB = b (Fig. 16.35), so that the ‘equation of the line AB is va + y/b = 1. Equations of the circles passing through the origin and having centres at Aa, 0) and B(O, b) are, respectively, a) and @) Fig. 16.35 Circles 16.21 so that the equation of the common chord OP is ax-by=0 QB) which is perpendicular to AB This has one end at the origin O and the other end P is given by solving (2) and (3). which lies on AB Example 65 Equation of the straight line which meets the circle x? + y* = a? at points which are at a distance d from a point A(@, #) on the circle is (a) 2ax + 2By = 2a? - d?(b) 2ax — 2fy = 2a? +d? (c) 2a@x + 2By = 2a + Pd) 2ax + 2fy + 2a? = Ans. (a), (d) Solution Let M and N be the points on the circle x+y? =a? such that AM = AN = d (Fig, 16.36). The tine joining A(@ A) and O(0, 0), the centre of the circle, meets the chord MN at B, the mid-point of MIN. Now, the slope of OA is fa, so that the slope of MN is—@/B. An equation of MN is ax + fy =k. Then, since (a, 8) lies on the given circle, we have A(a,B) Fig. 16.36 a You have either reached 2 page thts unevalale forvening or reached your ieving tit for this book. a You have either reached 2 page thts unevalale forvening or reached your ieving tit for this book. a You have either reached 2 page thts unevalale forvening or reached your ieving tit for this book. 16.26 Course in Mathematics for IfT-JEE Thus (AE) = (a + a)" + (23a)? = 160" = AE = 4a = 2AB. 8 4(a,9) ~ \ LZ > ff. arr,- iad Fla, 2080) Fig. 16.42 ‘Area of the trapezium ACDB = i (AB + CD) x OL 3 4 - Me. 23, as 6 is an integral root of 4x? - 3x — 126 =0 Paragraph for Question Nos. 79 to 81 224-2 Day -B Example 79 The equation represents a family of circles passing through two fixed points whose coordinates are (a) (- 2, 0) (b) (2, 0) {c) 4, 0) @) (4,0) Example 80 Equation of a circle C of this family, tangents to which at these fixed points intersects on the line x + 2y-+ 5 = Qis ais a variable, (a) 2+ y? = 2x - 8y-8=0 (b) 2 + y= 2+ 6y- 8 =0 (c) x2 + y? = Qe + By - (d) 2+ y- 2x -6y-8 Example 81 If the chord joining the fixed points subtends an angle @at the centre of the circle C, then = (a) 216 (b) is (©) a3 @) 22 Ans. 79. (a), (©) 80. (d) 81. @) Solution Equation of the given circle can be written as (i? + y* - 2x - 8) - 2aly) = 0 which represents a family of circles passing through the intersection of the circle x° + y* - 2x - 8 = 0 and the line y=0, ‘The circle and the line intersect at the points. P(~ 2,0) and Q(4, 0). Let the tangents at P and Q to a member of this family intersect at (h, k), then PQ is the chord of contact of (h, k) and its equation is he + ky (+h) -a(y+b)~ or ath ~ 1) + y(k a) —(h + ak +8) Comparing this with equation y = 0 of PQ, we get hal, heak+8=0, Since (h, &) lies on the given line x+2y+5=0 1+2%k+5=05 k=-3 and 1-3a+8=0— a=3 Hence the equation of the required member C of the family is x? + y" - 2x-6y—8 =0. Centre of the circle C is RC, 3) . Slope of PR = Vand of QR = 1 => @= m2 Paragraph for Question Nos. 82 to 84 A circle C of radius 1 is inscribed in an equilateral triangle PQR. The points of contact of C with the sides PQ, OR, RP are D, E, F respectively. The line PQ is given ‘by the equation JJ x + y— 6 = 0 and the point D is (33/2, 3/2). Further it is given that the origin and the centre of C are on the same side of PQ. Example 82 The equation of circle Cis (a) @-2V3) +Q- IP a1 (b) @ - 23 ++ 12 =I © @- Var +o4+ =I @ &- VIP +O-1%=1 Example 83 Points £ and F are given by (@) (32, 32), (43, 0) (b) (32, 12), (V3, 0) (©) (3, 3/72), (3/2, 1/2) (@) (3/2, V312), (W312, 1/2) Example 84 Equations of the sides QR, RP are @) y= QN3r+ hy = Uae =I ) y = (3s, y = 0 a You have either reached 2 page thts unevalale forvening or reached your ieving tit for this book. a You have either reached 2 page thts unevalale forvening or reached your ieving tit for this book. a You have either reached 2 page thts unevalale forvening or reached your ieving tit for this book. 16.30 Course in Mathematics for UT-JEE. be a pair of perpendicular tangents to the circle a +y? = a? Eliminating m we get fy = may + (my + 4)? = 2a? (1 + me) se +? (©) x cos @= y cot @=a x= asec &y=atan 6 Eliminating @ we get x? — y? = a? (d) Equation of the chord of the circle x? + y? = 2ax in terms of its middle point (@, A) is ox + Py - ale + a) = + P - daa. Since it passes through the origin. @ + P - aa=0, Locus of (@ Piss’ + y= ax Example 94 For the circle x? + y" + 4x + 6y - 19 =0 (a) Length of the tangent from (6, 4) (p) nae to the circle. " (b) Length of the chord of contact (q) VIT3 from (6, 4) to the circle (©) Distance of (6, 4) from the () Vi - 32 centre of the circle (d) Shortest distance of (6, 4) from (s) 9 the circle Ans, pars @O@O® ®@OO O@OO 4 @@O@OO) (a) Required length of the tangent Pra (3416+ 2F TAD «9, (b) Equation of the chord of contact 77" is Gr + 4y +2 (0+ 6) +My +4) -19=0 = ke y+S=0 perpendicular distance of TT’ from the centre C (= 2, = 3) of the circle is oe 5 = [TISz2t+5| | 32 oa+4o | VB and the radius of the circle is CT = J4+9419 = 2 Now (ST) = (CTF - (Cs? 2 = G2) 222 32x81 3 3 Length of TT = 2ST = 2% 36 f% = 72¥226 113 3 (c) required distance is cp = 6+2" +443" = V3 (@) required distance is PR where R is the point of intersection of CP with the circle, Now PR = CP- CR= Vila — J32 Example 95 If S227 +y?+x—-y-2=0, then (a) C2, D lies (p) on the circle S. (b) 2, - 1) lies (q) outside the circle $. (©) ©, 1) lies (©) on the tangent at (1, 0) to S (a) 2, 3) lies (3) inside the circle S. Cn * (O00 +‘ @@OO® © @@O® a OLOLGTO) Solution Let S(x, y)= x+y + e-y-2 then S24 =4+1-2-1-2=0=9(~2, I) liesonS, Sa.-y244142+1-2>03@2,-1 lies outside S. So, p= 0+140-1-2<0= ©, 1) lies inside, Say =4+9+2-3-250 Tangent at (1, 0) is 3x ~ y— 3 = 0 which passes through (2, 3) but not through (2, ~ 1) Example 96 (a) The circle x7 + > + 2x +c =0 and x? + y? + 2y +c = 0 touch each other @ife=t a You have either reached 2 page thts unevalale forvening or reached your ieving tit for this book. a You have either reached 2 page thts unevalale forvening or reached your ieving tit for this book. a You have either reached 2 page thts unevalale forvening or reached your ieving tit for this book. 21. 28. 29. 32. 33. A circle C passes through (2a, 0) and has the line 2x = a = 0 as the radical axis with the X+y sa’, then (a) centre of the circle € is (a, 0) (b) centre of the circle C is (~ a, 0) (c) C passes through (0, 0) (@) C passes through (a, a) Cutty +We-3=0,Cy rey Cy 2 + y? = 2y = 3 = 0 are three circles (a) C; passes through the centre of C; (b) C, passes through the centre of C (©) centres of the three circles form an equilat- eral triangle. (d) sum of the radii of C, and C, is twice the radius of C3. If Pisa point on the circle x° + y° = 9, Q is a point ‘on the line 7x + y +3 = 0, and the line. x— y+ 1= © is perpendicular bisector of PQ, then the coordinates of P are (a) 3, 0) -3=0, (b) 3.0) (ce) (— 7225, 21/25) (d) (72/25, 21/25) If the circle Cy: x7 + y? = 16 intersects another circle C, of radius $ in such a manner that the common chord is of maximum length and has a slope 3/4, the coordinates of the centre of C, are (a) (915, — 1215) (b) (= 9/5, 12/5) (©) (915, 12/5) (d) (9/5, ~ 12/5) If a chords of the circle x° + y° = 8 makes equal intercepts of length a on the coordinate axis then a can be a2 () 3 wad Ws ‘Two equal chords of the circle x7 +)? - 2x + 4y = 6, passing through the origin are perpendicular to each other; their equations are (a) x-2y=0 (b) e+ y=0 (x4 3y=0 (d) ae-y=0 Equation of a circle of radius 3, touching the ‘coordinate axes, no portion of which lies below x axis is (a) C+ y+ Ox + by + 9=0 (b) P+ - Or + by + 9=0 ©) P+ y + bx - by 4950 (d) 2 +7 - 6 -6y +9 =0 34. 35, Circles 16.37 Equation of a tangent to the circle x + y? = a7 is (a) yz axtayiea™ (by y= art aytea® (©) y=ar-ajiva® _ eae ‘The locus ofa point, which is such that the tangents from its two concentric circles of radii a and b are inversely proportional to their radi, is a circle C of area 16 7. (a) centre of C is at the centre of the given circles. (b) centre of C is at a distance 4 from the centre of the given circles (©) a? +B = 16 @ a+ hed ‘SECTION Itt LINKED COMPREHENSION TYPE Paragraph for Question Nos. 36 to 38 37, 38. L, is a line imersecting x and y axis at P(a, 0) and (0, b). Ly isa line perpendicular to L, intersecting x and y axis at R and S respectively. Locus of the point of intersection of the line PS and OR is a circle with (a) centre at (a/2, b/2) (b) ax + by = 0 as a tangent {c) a and as the intercepts on x-axis and y-axis respectively (d) radius equal to (1/2) a’ +b* ‘Common chord of the circles on QS and PR as diameters passes through the point (a. 6) if (a) a= 2b (b) 2a sb ash (d)as-h If the area of the triangle ORS is 4 times the area of the triangle OPQ then equation of PS is, (a) «+ 2y = 2% (©) x- 24250 (b) e+ y= 2a () We-y- 2 =O Paragraph for Question Nos. 39 to 41 AG, 7) and BUG, 5) are two points. C2 4y-4r-6y -3=0isacircle. a You have either reached 2 page thts unevalale forvening or reached your ieving tit for this book. a You have either reached 2 page thts unevalale forvening or reached your ieving tit for this book. a You have either reached 2 page thts unevalale forvening or reached your ieving tit for this book. 16.46 Course in Mathematics for HT-JEE @)3 (b) 2 ©) 32 wt (2004) 60, Find an equation of the circle which touches the line 2x + 3y + 1 = 0 at point (1, ~ 1) and is or- thogonal to the circle which has (0, ~ 1) and (~ 2, 3) as the end points of a diameter. 12004} 61. A circle C touches the x-axis and the circle x7 + (y~ 1)? = 1 externally, then locus of the centre of the circle C is given by fa) (x ys? = ay} U (0, sy $0} () [0 ysl (©) (Oy, 1 Y 1, ys ¥ SO} (@) (ony <0} [2005] 62. Three circles with radii 3.cm, 4 cm and 5 em touch each other externally. If A is the point of intersec- tion of tangents to these circles at their points of contact, find the distance of A from the points of contact. [2003] 63. Let ABCD be a quadrilateral with area K. with side AB parallel to the side CD and AB = 2CD, Let AD be perpendicular to AB and CD. If a circle is drawn inside the quadrilateral ABCD touching all the sides, then its radius is (a) 3 (b) 2 (©) 32 @t (2007) REASONING TYPE 64, Tangents are drawn from the point (17, 7) to the circle x? + y" = 169. The tangents are mutually perpen- Because ‘Statement-2: The locus of the point from which mutually perpendicular tangents can be drawn to the given circle is x° + y* = 338. {2007} Paragraph for Question Nos. 65 to 67 A circle C of radius | is inscribed in an equilateral triangle POR. The points of contact of C with the sides PQ, OR, RP are D, E, F respectively. The line PQis given | by the equation 3x + y ~ 6 = 0 and the point D is (3.3/2, 3/2). Further it is given that the origin and the centre C are on the same side PQ. 65. The equation of circle C is (a) @ = W3P +O -1P (b) Ge WBF + + 12y 67. 68. 70. © @-Vy+y+nel @ @- 3+ 0-17 Points £ and F are given by (a) (3/2, 32), (3, 0) (b) (3/2, 172), (3, 0) (©) (3/2, 312), «3/2, 12) (@) 372, V3/2), 3/2. 12) (2008) ‘Equations of the sides QR, RP are @) y= QN3)x + Ly = YV3)a = 1 (b) y = v3) y = 0 (©) y= 3/2) x4 Ly =(-V3/2)e-1 (2008) @ y= Vax y=0 (2008) REASONING TYPE Consider Ly: 2x + 3y +p-3=0 2x+3y+p+3=0 where p is a real number, and C: x7 + y* + 6x 10y +3020 Statement-I: If L, is a chord of the cirele C, then line L, is not always a diameter of circle C. and Statement-2: If L, is a diameter of circle C. the line L, is not a chord of circle C. (2008) MULTIPLE CORRECT CHOICE TYPE A straight line through the vertex P of a triangle PQR intersects the side QR at the point S and the circumeircle of the triangle POR at the point T. If S is not te centre of i circumeirele, then © 35*o TER oy 2+ ean PS” SP” JOSxSR het 9 pt ST OR 11.4 Oe as 2008; © ae £2008) ‘Tangents drawn from the point P(1, 8) to the circle a? + y? - 6x — dy — 11 = 0 touch the circle at the points A and B. The equation of the circumcircle of the triangle PAB is (a) x7 + y? + de ~ by + I (b) 2 + y= 4x - Oy + 19=0 () P+ y= 2x + by -29=0 (@) x + y? = 6x - 4y + 19=0 {2009} a You have either reached 2 page thts unevalale forvening or reached your ieving tit for this book. a You have either reached 2 page thts unevalale forvening or reached your ieving tit for this book. a You have either reached 2 page thts unevalale forvening or reached your ieving tit for this book. 16.50 Course in Mathematics for IIT-JEE 16. 7 18, 19. 20. 2. 22, Ist circle lies in the first quadrant as it touches both the axes and centre is also in this quadrant. The sides of the square be x= 0,x=1,y (h, k) be any point on the locus, then P+ (LWP ++ (1k = 9 Si +Pah-k-72=0 Locus x? +? ~x—y~7/2 = Ois acircle with centre (1/2, 1/2), the centre of the square and radius equal to 4, greater than the diagonal of the square and hence contains the square. ‘The radical centre is the point of intersection of 2x + 2y = 4 and 4x + 6y = 10 ie. (1, 1) which lies on ys mcif m= 1. ‘The point lies on the line joining the origin to the centre (1, 2) i.e. y= 2x which meets the given circle at the points. (1 + 4/,J5,248//5) , the one which is farthest from the origin is (I+4/V5,2+8/ v5). Equation of AB is xa + y/b = 1. If Pth, &) is the reflexion of O in AB then (h/2, k/2) lies on AB and AB and OP are at right angles. and ~ 2 x£ @ 20° k= a+b Length of the longest ray is equal to the length of the tangent from the point (4, 3) which is N6+9+ 6445441 = 12 radius of the circle = Ji6+81-1 = 12 Equation of the tangent is ax + fy = 17, which meets the coordinate axes at A (7/az, 0) and B(O, 77/8) and the area of AOAB = where ? = 7 or 8. 2ap 2+y-P)arty-aevey- ce 22 =a +c =a be are in AP. ak a are in H.P. Required length is yx°+y? +15Sx-S7y +21 where? + y+ ISx-ITy+=0 = ti-2 =# Saesbacets 24. 26. 27. 28, 29. Area of the quadrilateral = (#45? and 2x5—e x fb tee = 8 =(1I5-c)S+0)9 7 - 10e-11=0 ed =e . P (cos @ sin @, Q (cos B, sin f), Ricos x sin 2 AP = Yosa+l? tsin?'a = 2420s = 2-c0s (a2) AQ = 2 cos (72), AR = 2 cos (#2) Since AP, AQ, AR are in G.P. cos (af2), cos (f/2), cos (#2) are in G.P. = cos (a2) cos (2) = cos? (2) Since a B xare inp TT = p => c08 (@/2) cos (#2) ~ sin (af2) sin (72) = cos B = cos? (#2) ~ cos f= sin (a#2) sin (2) = 1 ~ cos" (82) = sin (a#2) sin (7/2) => sin? (92) = sin (a2) sin (9/2) => sin (a2), sin (872), sin (7/2) are in G.P. Radical axis of the given circles is x + Sy -6=0 which passes through (1, 1). Let the given circles intersect the circle a? + y? + 2gx + 2fy + ¢ = 0 orthogonally then 2g(— 3) + 2-6) =¢ +1 2g(- 2) + fl-se- 1h => 2g(- 1) + 2-5) 2 => — g-Sf-6=0 ‘=> the radical axis passes through the centre (~ g, =) of the third circle. Verify it does not pass through the mid-point of the line joining the centres. Equation of the circle be x? + y7 + 2gx + fy +e = 0, it passes through (2a, 0) => 4a” + 4ag +c =0 Radical axis is 2gx + 2fy + ¢ + a? = 0 which is 2-a=0 gf eta 10° =a => a? +ag=c = ¢=0, g =a and the equation of the circle is x + y* - 2ax=0 ‘Centre is (2, 0), passes through (0, 0) and (a, a) Centres C\(— 1, 0) Cx(1, 0), CO, 1) Vertices of an isosceles triangle. Cy passes through (1, 0) and C, passes through (© 1, 0) radii of each = 2. P(h, k), Oley, ¥)) then h? + P = 9, Tx, +y, +3 =0 wrk yh = f=0,-agec+a hek a-loaty, a You have either reached 2 page thts unevalale forvening or reached your ieving tit for this book. a You have either reached 2 page thts unevalale forvening or reached your ieving tit for this book. a You have either reached 2 page thts unevalale forvening or reached your ieving tit for this book. a You have either reached 2 page thts unevalale forvening or reached your ieving tit for this book. a You have either reached 2 page thts unevalale forvening or reached your ieving tit for this book. a You have either reached 2 page thts unevalale forvening or reached your ieving tit for this book. a You have either reached 2 page thts unevalale forvening or reached your ieving tit for this book. a You have either reached 2 page thts unevalale forvening or reached your ieving tit for this book. a You have either reached 2 page thts unevalale forvening or reached your ieving tit for this book. a You have either reached 2 page thts unevalale forvening or reached your ieving tit for this book. a You have either reached 2 page thts unevalale forvening or reached your ieving tit for this book. => Sm + = 16+ => m= +£2/V3 and equation of the common tangent is 7 = V3yt2x= 47 2 (4v7y ,(47) - (Ay (eg = 31? = 196 = 81/7 +3 = 5295 Example 53 If d is the length of the tangent from the 2 Point (100, 81) tothe ellipse ~~ = I,then dis equal to Ans. 3228 100x100 | 81x81 “er Example $4 Tangents to the parabola at the extremities ‘of a common chord AB of the circle x” + y? = 5 and the parabola y? = 4x intersect at the point T. A square ABCD is constructed on this chord lying inside the parabola then [TC + (7D)? is equal to ‘Ans. 6400 Solution ‘The points of intersection the circle and the parabola are A (1, 2), B (1, =2) Equation of common chord is x = 1, which is the latus rectum of the parabola. Tangents at the extremities of AB intersect on the directrix x =~ 1. Coordinates of T are (~ 1,0) Since length of AB = 4, sides of the square ABCD are of length 4, coordinates of C are (— 5, 2) and of D are (5, 2). (TC) = (TDY = (5 + 1)? +4 = 40. => (TC) + (TD}} = 80? = 6400 Solution d* = ~1 = 3228 Fig. 17.7 Conic Section (Parabola, Ellipse, Hyperbola) 17.19 Example §5 ‘The point of intersection of the _perpendi- ar an "Gs5F circle square of whose radius is Ans. 2906 Solution Any tangent to the ellipse is mx+ Y(41? m? +(35)? and perpendicular to it is z ye tne [OD mn Vw Eliminating m we get the locus of the point of intersection of these tangents. Cy may? + (my +x) = (1 + m2) (C41)? + 35)") cular tangents to the ellipse = | lies on a +35) 29. + y? = G1)? + (35)? which is a circle square of whose radius = (41)? + (35)? = 1681 + 1225 = 2906 Example 56 If x + y = k is a normal to the parabola y? = 12x, p is the length of the perpendicular from the focus of the parabola on this normal; then 34° + 2p is ‘equal to Ans. 2223 Solution For the parabola. with slope 1, is ye-x-2@)(-D-@ C1 = xtys9 so kad Focus of the parabola is (3, 0) 3-9 vz 34° + 2p? = 2223 Example $7 _If CF is perpendicular from the centre C of 2 es = on the tangent at any point P, and 12x, equation of a normal {m=-l,a=3] P | => 2p? =36 the ellipse Gis the point where the normal at P meets the minor axis, then (C.F. PG)? is equal to Ans. 2401 Solution Equation of the tangent at P (7 cos @, 5 sin 8) ‘on the ellipse is 3 cos 0+ : sin @=1 Pxs then (CF)? = 3 —3——5 5 (PY Scot 0+ TiO 25x49) 2Scos* 6+ 49sin?@ Equation of the normal at P is Tx _ SY 2 92_52 cos8 sind a You have either reached 2 page thts unevalale forvening or reached your ieving tit for this book. a You have either reached 2 page thts unevalale forvening or reached your ieving tit for this book. a You have either reached 2 page thts unevalale forvening or reached your ieving tit for this book. Example 74 _y? = 4x is the equation of a parabola, ‘Statement-1: Through (A, A+ 1), 3 normals can be drawn to the parabola, if A< 2 Statement-2: The point (A, A + 1) lies outside the parabola for all 2-1. Ans. (@) Solution Equation of a normal to the parabola is y+ar=2t+P, if it passes through (4, 2+ 1) then Atlt+dedet or P+ Q-AN- 4 1) = 0=fid) say. If f(a) = 0 has three real roots then f’(f) = 0 has two real roots => 31° + (2 ~ 2) = 0 has the real roots which is not true as <2 ‘£() = 0 has only one real root and thus there is only ‘one normal through (4, 2 + 1) to the parabola and the ‘statement-1 is false. Statement-2 is correct as $= y?~4x = (A+ 1)?-44>0if A# | showing that (2, 4+ 1) lies outside the parabola y’ 4x if A#—1, Example 75. Statement-I: If the tangent at the point (2/2, 0/2) on the ellipse x7/a + y°/b? = 1 meets the auxillary circle in two points, the chord joining them subtends a right angle at the centre then the eccentricity e of the ellipse is given by 3e7-2=0 Statement-2. If ¢ and e’ are the eccentricities of the ellipse x/a? + y7/b? = 1 and the hyperbola xa ~ y7/b? = I respectively then e? +e =2 ‘Ans. (b) Solution Equation of the tangent at (a/J2,b/ 2) 10 the ellipse is * +t = V2. The joint equation of the lines joining the points of intersection of this tangent and the auxillary circle x* + y? =a’ is Peyts which are at right angles, if co-efficient of x* + coefficient of y’ = Ind41-S5 505 a=3b7=30%(1-e) In Statement-2, e' =e? +e’? = 2 so statement-2 is also true but does not lead to statement-1. Conic Section (Parabols, Ellipse, Hyperbola) 17.25 SINGLE CORRECT CHOICE TYPE . The axis of a parabola is along the line y = x and the distance of its vertex from origin is V2 and that from its focus is 2.V2 . If vertex and focus both lie in the first quadrant, then the equation of the parabola is (@) @+y -y-2) (b) (ey = + y- 2) (©) @-yP a4 ety-2) ) (-yP = 8 @+y-2) . The normal y = mx ~ 2am — am’ to the parabola y? = dax subtends a right angle at the vertex if, (@) m=1 (b) i241 () m=z 2 (@) y2-1 An equation of a tangent common to the parabo- las y? = 4x and x = dy is (a) x-y+1=0 (by xty-t ©xtytl=0 (d) y=0 The coordinates of the end point of the latus 0 rectum of the parabola (y ~ 1)° = 2(¢ + 2), which does not lie on the line 2x + y +3 = 0 are @ ©2.) (b) © 32, 1) © © 32,2) (d) © 3/2, 0 . The point of contact of the tangent to the pa- rabola y? = 9x which passes through the point (4, 10) and makes an angle @ with the axis of the parabola such that tan @ > 2 is (a) (49, 2) (b) (36, 18) O46 @) (14, 32) Equation of the normal at a point on the parabola y? = 36x, whose ordinate is three times its ab- scissa is (a) 2x+3y+44=0 (b) 2x-3y+44=0 (c) 2x + 3y- 4450 (d) 2x-3y=0 In an ellipse, if the lines joining a focus to the ‘extremities of the minor axis make an equilateral triangle with the minor axis, the eccentricity of the ellipse is @ 34 & 3R |IR dw . If the chords of contact of tangents from two points (x), y;) and (xy, y2) to the ellipse ta + yb? = 1 are at right angles then ty X/y Y2 is equal to (a) ath? (by ba? (©) - abs (a) - ba a You have either reached 2 page thts unevalale forvening or reached your ieving tit for this book. a You have either reached 2 page thts unevalale forvening or reached your ieving tit for this book. a You have either reached 2 page thts unevalale forvening or reached your ieving tit for this book. a You have either reached 2 page thts unevalale forvening or reached your ieving tit for this book. a You have either reached 2 page thts unevalale forvening or reached your ieving tit for this book. a You have either reached 2 page thts unevalale forvening or reached your ieving tit for this book. a You have either reached 2 page thts unevalale forvening or reached your ieving tit for this book. 17.40 Course in Mathematics for ITJEB (o+v7} 2_16+25 _ 41 16 16 Positive end of the LR = (V41, 25/4) Equation of the tangent V1 x~ 4y-16=0 (©) °~y*=4,e? = 2, positive end of LR (2V2,, 2) Equation of the tangent /2 x (d) y?=8 (x—4), focus (6, 0), positive end of LR (6, 4) Equation of the tangent x~ y-2=0 26. Equation of the normal is y = mx = 2m =m", si it passes through (3,0), 3m—2m—m?=0 => m=0, 1 centroid of APQR = (. 0) FO,0) 0,2) ‘l,2) Fig. 17.22 APQR is isosceles. If (x, y) is the circumcentre, then +ys(x- bP 4(y- 27 =- 1 +04 1? = 1=5/2, y=0. oo1 radius = 5/2, Area= 4/1 -2 1 “jbo2 27. (a) °+8x-9=0=9x=-9orx= 1; Forx=-9, 1 is not res (b) P+ x-220 5x51 y=tW2 Hoes 8 x= 222, @ y+ 2y+laeys-Lx= t22 28. Draw the figures in each case. © @ The line joining the centre of the circle and the fo- cus of the conic in each case is a common normal, (a) any tangent to the parabola y’ = 4ax is ty = x + at, which is not a tangent to the circle for any value of 1. They have a common chord and a common point. (b) x = a is a common tangent, x - y =a isa common chord. (a, 0) is @ common point. (©) x =a, are common tangents, each chord of the ellipse is a chord of the circle, (+ a, 0) are common points. (@) interchange x and y in (b) to get (d). 29. (2a, 0)is the focus in (a) and centre in (b), (p) and (@) pass through this point. Equation of any tan- gent to (c) is y = mx ~ 2a/m which is satisfied by (p) for m= I and (7) for m = 1/3. (4) Equation of a normal is y = mux ~ 2am —am° and for m = 1, it represents (s). 30. (a) x(r— a) +y (y-2a) =, (b) y= + 2s, (Cama syst2aandr=0y=0,d)xeaay=tla, =0>y= +220. 31. Statement-2 is true, perpendicular tangents to y? = dar are y = mx + alm, y =m! x + alm’; mnt == 1, they intersect at a point where (m —~ m')x = Me) 5 =~ a, Which is the directx of rman the parabola. Slope of the tangents at the extremi-

(be + ad)? ~ 2ade +d- BP =O (1) 2.18 Course in Mathematics for IT-JEE This quadratic equation will have equal roots if D=4a'd? ~ Abc + ad) (d - b*) = 0 PE + We + adh? - Bed - Fd = 0 ed] = 0 => Bbc + a’d - = Example $9 For real x, the expre will assume all real values provided @a>b>c (b) ab @acccb Ans. (©), (8) (x= ailx-b) . x-e => — P-(atb+y)xtabscy=0 Since x is real, the discriminant (a+b +y) dla + cy) 20 yh + Ua +b - 2ehy + (a + bY - dab 20 y+ Uatb-Wwy+(a-by 20 wo Since y takes all real values, (1) is possible if and only if Ma +b - 2c) ~ 4a- by <0 (@+b-2+a-b)(atb-2w-a+b<0 Ma - 0) (b= 0) <0 (a-c)(b- 0) <0 Solution Let y= => = = = This is possible if ¢ lies between a and b, that is, ifac>b. Example 60 If sin « cos are the roots of the equation ax’ + bx +¢=0,(c #0) then a) 2 — b+ 2ac =0 (b) (@t cP shee (©) @ +B -2ac=0 ) @-cP =H +e, Ans. (a), (b) Solution We have sin + cos a= ~ ©, sin cos = © a a Now, sin? @ + cos? a = (sin @+ cos a) ~ 2sin acos @ 2ac = a? - B+ ac = 0 Bee ay Q) a+ dace Pe > ator = Example 61 If? + mx +1=0 and (b- ox? + (c-a)x+ (a-b) =0 have both the roots common, then (a) m=-2 {b) m=-1 (©) a, 6c are in AP. (d) a, 6, ¢ are in HP. Ans, (a), (c) Solution Note x= = 1 satisfies (1). Thus 1+m+1=0 = ‘Therefore, (1) becomes ¥-2ve1=0 => ‘Thus, two roots of (1) are 1, 1 + product of roots of (2) = 1 a-~b bre a,b,c arein AP. Example 62 If 4? > 4ac for the equations ax! + bx’ +¢= 0 then all the roots of the equation will be real if (@) b>0,a<0,c>0 (b) b<0,a>0,c>0 (c) b>0,a>0,c>0 (d) b>0,a<0,c<0 Ans. (©), (A) Solution Put. = y. The given equation becomes f(ysay +byec=0 oO ‘The given equation will have four real roots if (1) has two non-negative roots. This can happen i satisfies (2) m=-2 a =l = a-b=b-c = =” 30, af 0) 20; 6 —4ac 20 | | Thus @ and 6 must have opposite sign whereas a, ~ | = -ab20,ac20 {yb = 4ac 2 0 is given} bande ‘must have the same sign. = a>0,b<0.c>0 of a<0,b>0,c<0, | Example 63. If a, b € R and the quadratic equation | 2 +(@—byx—a—b+1 = 0 has unequal real roots for all | be R,then (a a>0 (bya>1 @a0 => B+ (4-2ayb+ a? +4a-4>0 @ quality will hold for all b € R if and only if discriminant of the quadratic expression on LHS of <0. That is, if (4 = 2a? = 4(a? + da = 4) < 0 = 16 - 16a + 4a* ~ 4a? - 16a + 16 < 0 = 32-3a<0 = a>l Example 64 If a, # are the roots of the equation ax’ + bx + c= 0 and a+ h, B+ h are the roots of the equations Ax’ + 2Bx + C = 0, then @ a= 2-8 ) pos £ Act+ac & (h= ane Ans. (a) (6) Solution We have as f= and (arms Bem= 22 = and (a By = (a+ (B+ yp = 3 ag} 2 Thus, 1-29 _ 4 40) @ a = Beac _ a B-ac & Example 6S. Let a, Bbe the roots of ° - 4x +A = 0 and x Sbe the roots of x ~ 36x-+ B= 0. If @ A, x Sforms an increasing G.P., then (a) B=81A (©) B= 243 ‘Ans. (a), (b), (6) Solution Let common ratio be r, then Al+N=at p=4,ar (l+n=% o P=9=>r=3. Thos, a= 1 wb) A=3 () A+B = 251 ‘Theory of Equations 2.19 Paragraph for Question Nos. 66 to 70 A polynomial p(x) of degree two or less which takes values yo. ¥1- ¥2 at three distinct values xo. xy, y respectively, is given by Gem) POP = Tae) Example 66 A polynomial of degree 2 which takes values yp Yost POimts Xp, p+ 1,4 (F #0) is given by Gaa)@y-5-9 y, (y=) +O + Gow) (nm), —%y—0 7 oo GrANOy=H- 9, adn Oy = 409-4 - 9 Ga mn 5-9], (2-4) =, +9) (@) none of these Example 67 Let p be a polynomial of degree 2 and @ #0, 1, then (2) pla) = x6 ~ 1) pO) + xp(a)] + x(x ~ 1) pla) (b) pla) = xx ~ a) p(0) + apa) (x(x ~ a) + 7) ©) (©) pw) = pe 4 A=) P@- pO , zisco) py I=a @ (@) none of these. Example 68 In Example 67 if a + 0, then p(2) is given by (@) (1 = 2) pO) + x(1 ~ 9) PO) +X pA) (b) x7 pl) + xp) + pL) (©) 7 pl) + xp’) + xp) + pL) (a) none of these is called as Lagrange’s interpolation polynomial, 2.20 Course in Mathematics for IIT-JEE Example 69 A polynomial of degree 2 which takes | values ~ 1, O and 1 at ~ 1, O and 1 respectively is @ Pox+l (bo) Pax +3 () e4x-1 (d)_none of these. Example 70 Condition under which p(x) will be a polynomiat of degree 2 is % yo [a »? 1] @ fy y to oy) |x, ay dl hy x oy 1 |= ©) |x? =o (@) i* 2 | a 1 bay Ans. 66. (d), 67. (€), 68. (a), 69. (d), 70. (a) Solution 66. (x= -DO-%) (x) x Ea %-DG— 5) a” nly =) (=H) *) * Out) (= H)(t= y=) + yam (x= 4) Qu — 4 0+) (r= ==) (HH 0 Soe an ee (= x)= =D 67. plxy= READ nj p SD (0) aa-) a x(x-a) +a sda eh), aca) ply + a(x, a) pO) ie —Ir-a) where @ (x, a) = ‘a-) a = 22M + 4 y-a] 21 - 20-8) a la-l l-a 68. Take limit as a + 0 in (c) of Example 67. 69. p(x) is a polynomial of degree Pw 70. Af point (xo, yi), (%)..¥))s (ep ¥2) ate not collinear, then ‘parabola can pass through these points. Paragraph for Question Nos. 71 to 75 Let xp. x; and x; be three distinct real numbers. Define three polynomials (x), 4(4), f(x) and (x) as follows: ya) = 2m) G4) —) yay © S220) (= 5)G, =) (x= )r-) and 1) = na) 2) = esa a) Kx) = (&~ x) x) (=) Example 71 p(x) = h(x) + (2) + 4(@) equals @t (b) x @ x @ lente Example 72 px) = (4; +23) ola) + ry +0) OO + Go +) hoo) then p(x) equals (@) x (b) x ; (y+; +.4)) (©) % +4 +2 -2x (@) none of these Example 73. The Polynomial a) a Gm) =a) MO) equals @l (b) x ©xr+x (@) none of these Example 74 § /,(x) x equals & @ 0 A (b) x ox (d) none of these Example 75 1f 1. = 8444 4 then Ta) * F-m “3-5 3-m A(for i= 0, 1, 2) equals 1 ry) (b) —E © TaD 5 & © FS @ Me) Ans. 71. (a), 72. (c), 73. (a), 74. (©), 75. (a) Solution 71. phx) I= 0 for x = xy, x1. ¥. Thus, pix) = 1 ¥ x. 72. pla) = (ip +4) +=) =0 for X= Ny Xp Xp PQ) = ty tx Ve BIW) = (4) my) +m) 8) + &— x) x) This gives 1) =a) Now, use Example 71. 74. Use Example 71. = [g(u) ete. 75. A, can be obtained by putting x = x, in a} after 5 deleting the factor (x —x,) of M(x). Paragraph for Question Nos. 76 to 80 A polynomial PA) = dg X" + a, 2+ | NF Oy ig % Os said to be for 0S i$ (n/2] where [2] denote the greatest integer $x Example 76 {If p(x) is a reciprocal polynomial of odd degree, then one of the roots of p(x) = 0 is fa) -1 @) 1 © 0 @ +R Example 77 If p(x) is a reciprocal equation and a #0 is a root of p(x) = 0, then which one of the following is also a root of p(x) = 07 @ a (6) Va (a-1 (d) none of these Example 78 Sum of the rational roots of 133x-78 * 133=78x 0 is fa) 29 (b) 972 {c) 13/6 (@) 6/13 Example 79 Remainder when p(x°) is pajartedeens lis (a) x4 e+ Example 80 Let m,21¢ Nand pays lx. p(x) will divide p(x") if (a) hef (m,n) = 1 (b) hef (m + 1, m) (©) hef (mn + V @) hef(m+ ine =i Ans. 76. (a), 77. (b), 78. (6), 79. (2), 80. (b) (b) x= 1 as ‘Theory of Equations 2.21 Solution As nis odd (— 1)‘ and (— 1)“ are of opposite sign. Now use the fact that a, = a, 77. As p(x) isa reciprocal polynomial pis) =x" p () x “ pla) = 0 € pila) =0 78, We can write (1) as ‘Tax° - 13x + 133x - 78 = 0 @ Clearly 1 and ~ 1 are root of (2) Dividing (2) by x° - 1, we get quotient as ‘78x ~ 133x' + 78x? = 133x + 78 = 0 @ Dividing (3) by 27 and putting x + I/x= y we get 78y" - 133y - 78 = 0 13/6, — 6/13 Now, x4 Ut = 136 = x= 23, 372 and x + Ux =~ 6/13 gives imaginary roots. +. sum of rational roots is 1+ (1) +23 +92 = 13/6. 79. (x= 1) pa)=x = 1 > Now Pee) = OP - D+ Gh - 1+ G@- 1+ @-D+5 2° — 1) gh) +5 where g(x) = (x'* + x!° +x + 1) +4 ee D4 4D 41 the required remainder is 5. “1 80, poy = I-x ‘We wish to find m, n so that pry _ (ln)a-2) px) is a polynomial. Since both 1 ~ x" and 1 -x"*! divide 1 —2"*" and since factors of | —°"* '™ are distinct, it is necessary and sufficient that x"*" — 1 and x" — 1 have no factor in ‘common except x~1, that is, hef (m + 1, n) = 1. (-x*)0-2"") Paragraph for Question Nos. 81 to 85 A polynomial PAR) = ay x" + a, x where dy, a, ... 4, € 1, dg #0 is said to be irreducible over Tif p(x) cannot be written a: product of two non-constant polynomials of degrees both p(x) + 1 and g(x) + | are divisible by (~ a) (x 5). As deg q(x) + deg r(x) = 4, we have gx) tl =r) tl =~ a) e-d) ga) r(x) = U(x - a) &e - 6) £1 | Since | we get = PO) = pla) # 2G - a) Ge - b) = (x~ a) (xb) = 0. A contradiction. 82. Use Eisenstein’s Irreducibility criterion with p = 5. vol xl 83. p(x) = = = pore ye GEN=1 Sale rh tt Cha tt Cpa For 1 Sisp "C, © (pil As pis prime, p \ (p - i)! and p i! for 1 $ iS p-1. Also, pp! Thus, p1"C, for | $i $ p-1. Asp \ Land p® ¥, °C,.,. p(x + 1) and hence pix) is irreducible over I by the Eisenstein’s irreducible over I. 84. Clearly ais zero of fix) = 7%” - 3. Also by the stein’s criterion fix) is irreducible over L. If g(x) is the desired polynomial of the least degree then by the division algorithm there exist q(x) and r(x) such that FO) = glx) g(a) +r) where Hx) = 0 or deg r(x) < deg g(x) Let Hx) #0, then since ra) = fla - ela) qa = 0 and deg Ax) < deg g(x), we reach a contradiction, Thus, nx) = 0 and flx) = g(x) gtx) and fix) is imeducible over I and deg g(x) 2 1, g(x) must be a constant, Therefore, f(x) is the desired polynomial. 85. Let p(x) = g(x) r4x) where g(x), r(x) are non-constant polynomials with integral coefficients. We have a= d ray= £ | for a=a,b, Also degree of q(x), r(x) and hence of q(x) + x) <4. Ga) + a) = 0 for a= a, b,c. d, gy +r) = Pix) = = (gay? A contradiction as coefficient of x* on the L.H.S. is positive and on the R.H.S. itis negative. ‘SECTION IV INTEGER ANSWER TYPE Example 86 Ifx,y,ce Raxty+cedande + y+ 2 = 6, find the maximum possible value of z. Ans, 2 Solution -2v+y=6 a+ yP-@?+y) (4-2 - (6-2) = 22-82 +10 ‘The quadratic equation whose roots are x and y is P(t yt ays 0 or P~G-m+2-4245=0 Since x and y are real 4-42-4245) 20 32-8 +4 S$ 0 Ge- 2-250 eo WBs282 That z can take value 2 can be checked by taking x= y= 1. Example 87 Find the number of rational roots of pOr) = 2 + 307 + 2 + DHS =O, ‘Ans, 2 Solution p(x) = Ox +3) (7 + +. + D). = (r+ 3) (rt 1) 444 e D x+y 2ay Also, 42% +e P+ 1>1 VER Example 88 Find the sum of all the integeral roots of (log, 2)? + logs, (Six) = 1 ay Ans. 6 Solution Clearly x>O and x # 1/5 logs 5—logs x logs, (Six) = 8s2— 8s * logs, (S/x) og, S105, 5 Putting logs x = 1, then equation (1) becomes Jot oy +e > t- I 4+2) =0 @ £=0,1,-2 So integral roots of (1) are 1 and 5. Example 89 Find the least integral value of m for which, every solution of the inequality 1 $ x $ 2isa solution of the inequality x? - mx +1 <0. Ans. 3 @ feP-2=0 Theory of Equations 2.23 Solution Clearly m > 0. Now x7 —mx+1<0 7 = 3. ge | and Soya 5/2 veuu vs the least integral value of m is 3. Example 90 Find the sum all the real numbers satisfying the equation Y+te- isl a Ans.1 Solution Forx 2 1, (1) becomes P+x-1al r+ DH e-)=0 mrel For x < 1, (1) becomes Pelo-x=l ars. Example 91 Find the greatest integral value of a for which the equation Q-y)&+lea a has non-negative roots. Ans. 2 Solution (1) can be written as waxta-2=0 Its roots will be real if 1 — 4(a—2)>0 = a < 9/4 Also, a — 2 product of roots of (1) 2 0. Thus, 2s a < 9/ 4 Example 92 Find the sum of all the real roots of the equation @-x) + @-5=-32 a) 2.24 Course in Mathematics for IT-JEE Ans, 8 Solution Put 4 —x = t, so that (1) becomes (Do -@+ IS =~ 32 => 10r + 207-30 =0 = @+30-) =0 When x is real, ? +3 >0 Sts tered, 6. Example 93_ Find the number of polynomials p(x) with integral coefficients satisfying the conditions pil) = 2, pG)= Ans.0 Solution Let p(x) be one such polynomial. Let py(x), be such that pa) = 1 = = 3) aa) = p(l) — 1 =~ 2p.) = 1 =- 2p) But this is not possible as both 2 and p,(1) are integers. Example 94 Find the degree of the remainder when x07 | js divided by (7 + 1) G7 +x + 1) Ans, 3. Solution Write as 27 gia) (8 + IGP txt tar tbe texted and substitute x = @, a, i, ~ ito show a = Example 98 Let p(x) be a polynomial with integral coefficients, Let a, b, ¢ be three distinct integers such that pla) = p(b) = plc) =- 1. Find the number of integral roots of p(x). Ans. 0 Solution a, b,c are roots of phx) +1 = 0. Let pix) + 1 = (x a) (xb) 0) gla) ay where q(x) is some polynomial with integral coefficients. Let m be an integral root of p(x). Then (1) gives 1 = (ma) (m—b) (m — 6) qi) Asa, b,c are distinct at least one of m—a,m~b,m—cis different from 1 and ~ 1. This is a contradiction, SECTION V MATRIX MATCH TYPE Example 96 Match the equations on left with the properties on right. Column 1 @ a0,atb+e<0 and equation is ad +br+c=0 (©) b,c, € Land the equ- ation Pebx+co=d ‘has rational roots @ a,b,c,de Rare in G.P._and equation is (a + b+ yx? + ab + be + cae +P eed ad Column It (p) real roots (@) distinct real roots (P) integral roots (8) discriminant 2 0 pars Ans. Solution (a) Let P(x) = (x~ a) (x~ 0) + AUr~ b) (x~d) Note that P is continuous on R and P(a) > 0, P(b) <0, Plc) <0, Pd) >0 <. P(x)=O has a root in (a, 6) and a root in (¢, d). (b) As a> 0, the parabola y = ax’ + br + c open upwords. Also, as y(1) <0, y = ax" + bx +c meets the x-axis wice. + ax? + bx +e =0 has distinct real roots. (©) x7 +bx+c =O has integral roots. See Theory page 2. (@) If is the commion ratio of the G.P., we can write the given equation as (eee) +P er=0 2 WH =O x=-n- 7 Example 97 Match the equations on the left with the properties on the right. Column 1 @ abceR are such that a+b+c=d. ‘Equation is 3ax? + 2bx +c = 0 (b) a,b,c Rare such that 2a + 3b + 6c = 0. Equation is a+ br+c=0 ©) a,b,ce Rare such that @tor 0 (®) at least one root in (0, 1) (8) @ root in (= 22, = 1) ‘of a root in (1, =) 1424S co, a Ans. (b) Consider the polynomial Pays tars Sas Ler and use Rolle’s theorem on (0, 1}. (©) Let P(x) = ax? + br + ¢ Note that P(-1) PU) <0 = PA) vanishes at least once in (-1, 1) => Pla) = 0 has real roots. (@) Let Pix) = ar’ + bx +e and Qtx) = 2 + 2x4 £ atta ast + Ase <0, we get a*a Q-1) or O11) < 0 Since Q(x) —> ee as x p00, we get Q(x) = 0 or Pix) = 0 has a root in (1, ©) or (1, *%. Theory of Equations 2.28 Example 98 If x € R, Match each expression with its range. (a) 2a 2e43 (p) (= &, 3] U 14, ) 2x-8 ® (qR-(1} OR (8) (— », - 29} U (1, &) Ans: 2-8) ya - 243 = 0-1) P-%w-Dx-By+3=0 As re R, dy - 1)? +41) By +320 = U-) Oy +220 a ys-Wory2t But y= 1 =93 Thus, y € (- ~, - 2/9] v (1, ») — 8. Not possible. 12 () y= 2x-7 = x? 2+ Ty- 1220 As xe R, 4y?~ 4 (Ty~12)20 | sy -Ty+22050-H0-420 S yee 3}U l=) # 3x42 AEs o- De +t dre Fax-6 9 ° © y= (6y +2) =0 As xe R, (+ 3) + 40-1) 6y +220 = 2y? - Oy + 1 = (Sy 1) 20 2.26 Course in Mathematies for IT-JEE However, y = 1 => x = 2 for which expression becomes 8. Therefore, y # 1. Thus, range of the ziven expression R ~ (1). oy increases as x increases. Thus, range is R. Example 99 Let a £ ybe three numbers such that toa 9 ataty 7 and a+ B+ y=2. (a) apy (p) 6 (b) By+ a+ af 8 © e+ h+7 (-2 @ a+ fey @)-1 Ans, parts + @®@00 + ®@O® © @@O® 4|@@OO) Solution rear By ya of, BY, 2(at+f+y) =-2 = apy Bytyevoh 1 : Also, aby 7” y+ y+ apa +P +P = (a+ Br yp - Upy+ yer OP =6 Lastlya’ + f° + 7 — 3apy =(at+ f+ plas f+ P - py ~ we- a) Example 100 The set of value(s) of ke R for which (@) kk + Det 2-10 @) (1-2) has no real roots (ii) xP - 204k - Ix + 15 - 2-7 > 0 for each x (b) G- 17) val (iii) Sum of the roots of ¥+Q-k-Ry-P=0 is zero (iv) The roots of ¥ + Qk-Ix+h +250 (©) (- 4} (d@) @, 4) are in the ratio 1: 2 pars Solution (a) (k-+ 1)? ~4k (2k~1) <0, k#0 = 7h - 6k-1>0 = Ck+Nk-1)>0 = k<-Work>1 (b) 4k = 1) - (15? - 2k- 7) <0 =» 6k 48 <0 = (k-2) (kK-4) <0 = 22 (p)3 (b) x - 9-220 @2 © @+NG+DG+NO+4 = (HL (@) (log, x)? + logy, Bix) = 1 (0 Ans, pars *@@OO > |©@@O® © @@OO) 4 @@O® Solution (a) Any rational root of 2° — px? + an integer. But for a € I, a°(p - a! possible if p > 2. As in (i) any root of x!° — x? ~ an integer. Clearly x =~ 1 satisfies the given equation. For, x # = 1, xe I ex- = 2 is not possible. must be (b) 0 must be (©) The given equation can be written as (2 + S44) (8 + Sx 46) = 24 Put x? + Sx = 1 to obtain P+ 10=0 = 1=0,r=- 10. For t= 0,7 +Sr=s05x50,15-5 For t= = 10, x° + Sx = ~ 10 does not have rational roots, (d) Put logs x = ¢ to obtain = +2) 0-1) = 09 1=0,1,-2 This gives x= 1, 3, /9. Example 102 The number of real roots of (a) Vev9 ve = 27 2 vi3-¢ ) ixel at (@infi- | to) Vrt5-Ve nite ( Vee2We-1-yr-We-l =2 «0 pars *©@OO®@ > |®@@OO® « ®@O® | 4|©@G® ‘Solution (a) x20 = Vxt9 23 (b) 3 - Pat lPartx-6=0 =xe-3,2 Ans. (x4Salexe Wr a Ve s2ored @ Purx-1 =F, to obtain + U-t-N=2 which is satisfied for each 12 1. ‘Theory of Equations 2.27 Example 103 Let a 8 be roots of the equation ax’ + bx +c = 0, then equation whose roots are (a) - Va, - VB (p) x? + Qac - Bx + a? = )-@-B a’? + Qac - P+ =0 © @&, & (er - br +a=0 d) ee UP (Sax -be+c=0 Ans. 8 par + @@OO) + O@OO@ a OLTGIO) 4@@O0 Solution (a) Replace x by - 1/x to obtain a- bx +ex* = 0 (b) Replace x by — x to obtain ax? ~ br + ¢ = 0 (c) Replace x by VX to obtain ar+ br +e=0 = a + 2acr + 2 = be = at + Qac- B+ =0 (@) Change x to 1x in the above equation. Example 104° Let @ # be roots of ax” + bx +c = 0. Match the equation on the left with its roots on the right, (a) (x - bY + B(x — b) tac =0 (p) 2a, 28 (b) ar? + 2bx + 4c = 0 @)- ata,- Ba (c) 4a°x? — b? + dac = 0 Maat b, ap+b @) ab? ~ aby +0 =0 (s) a+ bia, B+ bRa Solution (a) Write equation as 2 « (2) +n(Z2) sexo @ @ 2.28 Course in Mathematics for IT-JEE 2 «(3) +6 (3) +e-0 > wM=a8 b* — 4ac © re sat (d) a(- ax)? + bE ax) + = -a=apa x= 124100 Se ize+100"* © (a) Greatest value of y (b) Least value of y (©) Greatest value of y is attained at (d) Least value of y is attained at Example 105 Let y= (p) ~ 10 (q) 10 14 (s) 4 Ans, s pare ©@O®@| @©@@OO) @@OO) 4 @@OO) Solution (x2 + 12x + 100)y = x7 — 12x + 100 (y= 1x? + 12 + Ix + 100 - 1) =0 As xis real 36(y + 1)” - 1007-1)? 2 0 = Y-l)y-4s0 oc e = > Wasys4 | For y= 4, we getx=- 10 | For y=—4, we get.x = 10 Example 106 Let a, be roots of the equation ax’ + 2bx + = 0. The equation whose roots are @) £2aq £28 (p) @ + ex? ~ | ac - 26)? = 0 | (ot) t Vat UP @ @r+ ey? - Afac ~ 2b)? = 0 (tat RP (ax + 8ac- 26+ 16c? = 0 (@) £ Ve, + UP (8) xt + Yac — 2b? + @=0 An Poa ros *@@OO) > ©@O® * MB@O® 4 @@OOO Solution (a) 2@, 2f are roots of ax” + dbx + 4c = 0 and ~ 2a, ~ 2f are roots ar ~ dix + de = 0. Required equation (at + dey = 1 0 ax! + 8(ac ~ 2b? + 16c? = 0 (b) Way, 1/8 ate roots of a + 2bx + cx" = and = ey, ~ 1/6 are roots of a- 2b tex? =0 +. Required equation (or + a - 4b? = 0 x! + ae ~ 2b? + a or or 0 © @, B ate roots of ax+ Ue += 0 (ax + x = at + ac - Wyre = 0. Also,- a, ~ # are roots of ae — Yac - 2b yx + P= 0 Required equation is (es + A} — (ae ~ 267 2 = 0. (d) Replace x by I/x in the above equation. ‘SECTION VI REASONING TYPE Example 107 Statement-1: Ifa, 6,c € R and 2a+3b+ 6c = 0, then the equation ax” + bx + c = 0 has at least one root root in (0, 1]. Statement-2: If a continuous function f defined on R assumes both positive and negative values, then it vanishes at least once. Ans. (b) Solution Consider f: R— R defined by ‘fay= 5 a+ ; bts ex ‘Then fis continuous and differentiable on R and sy= bas 5 bHE= 5 Qa 344+ 60) =0,f(0)=0. By the Rolle’s theorem there exists at least one @e (0, 1) such that f(a = 0 But SW =ar + bere ‘Thus, ax? + bx + ¢ = 0 has at least one root in (0, 1). Statement-2 is true but its not the correct explanation of statement-1. Example 108 Statement-1: If a # 0 and the equation ax’ + bx + ¢ = 0 has two roots @ and # such that a<—2 and > 2, then a +lbl + ¢ and a have the opposite signs. Statement-2: If a > 0 and 7 8(7< 8) are the roots of ax? + bx+ c= 0, then graph of the parabola y= ax? +bx + ¢, for 7 0, graph of the parabola y =.7 + Bae ‘ for @ dp As gp and hef (p,q) = 1 we get q= 1 = = [s hef (p,q) = 1) Thus, 2 = pis an integer. q Statement-2 is false as x? + x + 1 = 0 does not have integral roots. 2.30 Course in Mathematics for IT-JEE Example 112 Statement-1: a +b + > 0, be +ca+ab >O,abeo>0 a>0,b>0,c>0. ‘Statement-2: If p,q, r > 0, then the equation x’ — px? + qx—r= 0 cannot have negative roots. Ans. (a) Solution Statement-2 is true as x <0 implies eps gr—r<0 [Iv pg > 0} Let p=a+b+e,q=be + ca + ab and r= abc, then p, q, r>0. Equation whose roots are a, b,c is (xa) (xb) (xe) =0 or = petqr-r=0 As this equation cannot have negative roots a, b, ¢ > 0. Example 113 Statement-I: If a, b,c R - (0), then at least one ax? + bx +¢=0, bx +ex+a=Oandex tart b= 0 has imaginary roots. Statement-2: Ifa, b,c € R, a# 0, then imaginary roots of the equation ax’ + bx + c = 0 occur in conjugate pair. Ans. (b) Solution Uf each of the three equation has real roots, then 1b? ~ dac 2 0, c? ~ 4ab 2 0 and a® — 4ab 20 = abc? 2 64ab'c*. A contradiction. For truth of statement-2, see Theory. Example 114 Statementel: Let ac = be and let x). x) be the roots of x7 + ax + be = 0 and x, x be the roots of x7 + bx + ca = 0, then x), x5 are the roots af x7 + cx + ab = 0. Statement-2: If a, b € R, then labl < Ja* +b° Ans, (¢) Solution Asac#be,c#0anda#b, We have ay tary + be =0 ay + bx, + ca = 0 Subtracting, we get (ab) +e(b-a)=0 = xy =casaeb, Thus, c? + ae + be Also, x, =b and xy =a. Equation whose roots are x, and x, is 2 — (a+ bx + ab =0 or? + cx+ ab =O, ‘Statement-2 is false can be checked by taking a = b = 0. Example 115 Statement-1: The equation x satb+c=0 Sf) has two pairs of repeated roots. Statement-2: Polynomial equation P(x) = 0 has a repeated root aif Pa) = 0 and P’(a) =0. Ans. (A) Solution If czis a repeated root of f(x) = 0, then f(a) =0 and f’(@) = 0. But a= 0 does not satisfy fix) = 0. For truth of statement-2, see Theory. Example 116 Statement-1: If all the four roots of way) ta —be +150 are positive, then a = 6 and b= 4. Statement-2: If polynomial equation P(x) = 0 has four positive roots, then the polynomial equation P’(x) = 0 has 3 positive roots. Ans. (b) Solution Let xy, x3. x3 x4 be four positive roots of x4) +ax’— he + 1 =0, then xy tay tay tay = and xy x 4) = 1 = HG) ty tay tad = Oy ay)! ‘That is, A.M. aye GM,, but this is possible if and only if msl. ‘Thus, the given equation becomes (-1's0 = a=6,b=4 Statement-2 follows immediately from the Rolle’s theorem. Example 117 Statement-t: If a, b,c € @ and 2" satisfies @ + bx + cx’ = 0, then a = 0, b=0,c=0. ‘Statement-2: A polynomial equation with rational coeffi- cients cannot have irrational roots. Ans. (¢) Solution By multiplying a, b, e by an appropriate natural number, we may assume that a, b, c are integers, We may further assume that a, b,c have no factor in common. Now, at i 4 cM a0 = @a-26+2" 0) = a Thus, 4a) = ~ (b* + 2c ~ 6a,be) => b= ~ 22a) + 8 - 3aybe) ~ 2b? + 2c? + Babe) => 2la = 8a = 2a, for some a, € 1. = 20 Therefore 4b) = ~ Qaj) + & - 6a,b,0) ~2a,' + 2b} ~3a,b,c) = 2c => Me This is a contradiction. = Ub = b= 2; for some b, € L. =e Statement-2 is false as 2!" js irrational but is a root of P-2m0. Example 118 Let a, b,c, q be real numbers. Suppose 4, b are the roots of equation x° + 2pr+q=Oand @ 1/8. are roots of the equation ax’ + 2bx + .¢ = 0, where J € (-1,0,1). Statement-1: (p* ~ q) (b° - ac) 2 0 and Statement-2: b # pa or c# ga Ans. (a) Solution Suppose ae R. Asa, b,c, p,q € R, the other root of x7 + pr +g =Oand ax’ + 2hx +c = 0 must be @. Thus @=f=/fp = # But AP e (-1,0,1) aeR = ABER => D,=4(p?-q)20and Next, let b 4b ac)20 and ¢ = ga, then ax? + 2bx += a (0? + 2px +g) = both roots of the two equations are common. = f=p = # =1. Acontradiction. Statement-2 is true but is not the correct explanation for the statement-1, SINGLE CORRECT CHOICE TYPE 1. Ifa, b,c © R and (a +b +6) € <0, then the quadratic equation p(x) = ax’ + bx + ¢ =O has (a) two negative roots (b) two real roots (©) two imaginary roots (@) none of these. 2. The number of rational roots of the equation PaWSeH1S ANSI Soldxel5 Pol6xel5 12 is Theory of Equations 2.31 (a) 0 (b) 1 (©) 2 (3 3. ‘The number of irrational roots of the equation 3 “2 (>) 1 (c) 2 (d) 3 4. Wha Bx Sare the roots of (txt)? 4 Bai? +44 4)427=0 then lat + |fh+ If + [lis equal fa) 6 (b) 8 (e) 12 (d) 25 5. x= Lisa rootof ? (2 =x4 8-60 (Px 4 DP + Set ‘of multiplicity (a) 2 (b) 3 (e) 4 @ 6 6. The number of irrational roots of the equation (2 = 3x + 1) G2 + 3x42) (7 - 9x +20) =- 30 (a) 0 (b) 2 () 4 (d) 6 7. The number of roots of the equation 3r 46x47 +52 410x414 =4~ 20-27 is (4 (b) 3 (©) 2 @1 8. The number of real values of x which satisfy the @ 0 (b) 1 ©) 2 (@) infinite 9. The number of real values of « which satisfy the equation | +12 ant le=H @ 1 (2 () 5 (d) infinite 10. Sum of all the real values of x which satisfy the equation (b) 2 (©) 75 () 1S 11. The set of real values of a for which the equation 2a? +37 2x 1 ° 2.32 Course in Mathematics for ITT-JEE hhas a unique solution is @ C=, 1) ) G1») @CL) (d) none of these 12.The set of real values of a for which sum of the roots of the equation tat wea a” reane is less than a°/4 is @) ©2302) — ) B,») OCLOVA) @ QQ) 13. The real values of a for which . ve a3et4 for each xe R, is @) © 1,2) (b) (0, 7) © G2) @) 27 14. Ifa, b, ¢ © R are distinct, then the condition(s) on 4, b, c for which the equation t,t 1 Xna a-b x-c x-bmc+a bas real roots is (a) a+b+c20,a20 (b) a-b-c#0,a#0 (©) Wa=bee (d) none of these 1S.1f fix) = at + bx +c, f- 1) < 1, fl) > - 1 and f-<-4 then f@a=0 (acd (©) @>0 — (d) sign of a cannot be determined 16. The set of all values of k for which the equation X + Yk = Ive (k 5) = Ohas at least one non-negative root is @ i.) ©) 11) © m5) @ C=, 5) 17.1 3214 < re x, then the set of values of arfor which Gin ap + @ 60s aor L (sin + cos is square of a linear polynomial is Sa iia] lz] wo few fs} Sa] © {3} de 18.1f 0 S$ ¢< 3, then the set of all values of ¢ for which sum of the squares of the roots of the equation + (sin g— Dir 4 cos? 9=0 is greatest is @ {x2Z.ancr} ) (3a) 5x) 3a] © fox, 7 } @ {*} 19. The set of values of b for which log yap (bx + 28) =~ log, (12 - 4x ~ 27) has exactly one solution is (@) C9 - 14) u (1483, =) (b) [14/9, 9) (©) © = AL (4) [148, @ (4) 20.1f 2x7 + 3x +4 = 0 and ax? + bx +c = 0, where a,b, ¢ © N have a common root, then the least value of a +b + cis (a) 10 () 9 8 @ 6 21. Let p, q. be real numbers such that pr = ler quadratic equations x” + px + q = at trx+s=0. Then (@) none of these has real roots (b) both have real roots (©) at least one has real roots (d) at most one has real roots 22.1f cot 8 cosec @ are the roots of the equation ax’ + bx +c = 0 and 4=b? ~4ac, then a equals (a) ba (b) BA (©) bed (@) «3 23. Let a <0, a #- 2. The equation x° + aixi +1=0 has {a) no real roots (b) at least two real roots (©) exactly four real roots or no real roots (d) none of these 24, Let fix) = 1° + 3x7 + 6x + 2009 and ats). and 2 3 mi FQ)" =f)" ‘The number of real solutions of g(x) = (0 1 ©2 (@) infinite 25.Let f be a continuous function defined on {- 2009, 2009] such that f(x) is irrational for each 2€ {- 2009, 2009] and (0) = 2+ V34+V5. The ‘equation (2009)x* + 2/(0) x +,/{2009) = 0 has (a) only rational roots (b) only irrational roots (c) one rational and one irrational root (@) imaginary roots 26.Let Ba Be R, B40 and a+ if be a root of at + qx + r= 0 where q, r€ R.A cubic equation with real coefficients one of whose roots is @ is (a) O-grer=0 (b) P+ gr tr =O (©) x + 4gx - 8 =0 (@) none of these 27.Suppose a, b are complex numbers and 2 + ax + b = 0 has a pair of complex conjugate ‘roots, then (a) @ is real and b is imaginary (b) a is imaginary and 6 is real (©) both a and b are real (@) none of these 28, Suppose that the quadratic equations 3x + px +1 = 0 and 20° + gx + 1 = 0 havea common root then the value of Spq ~ 2p" ~ 3q° equals @o (byt © -1 @ 2 29.The number of solution of 14 25 = (8) 0!) (a) 0 (b) 2 ©) 4 (d) infinite 30.The equation (2) =-3P742r- nhs (@) no solution (b) exactly one solution (©) exactly two solutions (@) none of these 31.Let a, 6, ¢ € R and a # 0 be such that (a_+ cf < b, then the quadratic equation ax + bx +c = 0 has (a) imaginary roots (b) real roots (©) two real roots lying between (~ 1, 1) (d) none of these 32. The integral values of a for which the quadratic equation (x - a) (x - 10) + 1 = 0 has integeral roots are (@) -1,3 (b) 2,3 (c) 12,8 (d) - 8,-12 33. The number of real solution of 48715 4985 = (10) (6°) is (a) zero (b) one (©) two (6) infinite 34. The number of real solution of 28 4 52°) = Tis ‘Theory of Equations 2.33 (a) zero (b) 1 (©) finitely many (@) infinitely many 35.Let a, b,c R be such that a+b +e <0, a-b+c0. If @and Bare roots of the equation ac’ + be + c = 0, then value of [a] + [Bl is (a) 2 1 © -1 0 36. If roots of the equation x? ~ 2mx + m in the interval (~ 2, 4), then O lie @) -leme<3 (b) lems (©) lla (©) lat + a<2 (2) none of these 2.34 ‘Course in Mathematics for IIT-JEE 43.1 x is real, and y = 2 P2#*©. sakes al real ” w+axt3 values then (a) O 0 is satisfied is given by Sx @ 4 (b) -5 oa (a) -3 If a + 61 > c and a # 0, then the quadratic equation OPES *yx+b=0 {a} has two real roots (b) both positive roots (c) cannot have real roots (d) none of these SECTION II MULTIPLE CORRECT CHOICE TYPE If abe < 0, then the equation ax + 2b +¢—a)x + be = 0, has (a) real roots (b) one positive and one negative root (©) both positive roots (a) both negative roots ‘The equation | + Vi-Jt a? once Ribas {(@) only positive solutions (b) exactly one solution (©) at least two solutions (4) infinite number of solutions. The equation save Vre2 ¢Vi8 +20 =3 has (a) no solution (b) at least one solution (©) only positive solutions (@) infinite number of solutions, Let a, b, ¢ be the sides of an obtuse angled triangle with ZC > 212. The equation ar +P ea cr =0 has (a) two positive roots (b) one positive and one negative root (©) two real roots (2) two imaginary roots, 50. SL. 32. 53. 55. 56. 57. ‘The equation Vani +Vx43 +2V(0-DG43) = 4-28 thas (a) exactly one integral solution (b) all its solutions in (1, 2} (©) sum of all the solutions is 1 (@) no solution ‘The equation .x° + 1 = 2(2x - 1) has (a) one rational solution (b) two irrational solutions (©) sum of roots equal to zero (@) product of the roots as ~ 1. Let P(x) be a polynomial such that P(x+2) =2° + 34-2. Which one of the followings are true statement? (a) There are two polynomials satisfying the above relation. (b) P(x) must be of degree 2 (©) P(x) = 0 has two irrational roots (@) Product of the roots of P(x) = Let Pux) be a polynomial such that PO? + 2) =2'7 — 3x8 +27 ~ 3, then 0 thas at least 34 roots. has exactly 17 roots (©) Sum of the roots of P(x) = 0 is ~ 193. (d) none of these. |. Let PU) = dg + yx +. + ai,t", dy #0 be such that P(x?) = P(x)’, then Let p, q be non-zero, real numbers and x,, x3, 0 be the roots of x° + px +q=0 such that (@) q is a root of x* + pr+q=0 () q<0 (©) p<0 WM aty 1 and ae R be such that sin a# 0, and let p(x) =x" sin &—x sin nar+ sin (n- 1)@rthen (a) cos @ + i sin & are zeros of p(x) (b) p(x) is divisible by x7 — 2x cos @+1 (©) px) is divisible by x° + 2x cos a+ 1 (2) sum of all the roots of p(x) = 0 is 0. Letne N,n> Land ae R be such that sin a0 and let p(x) = (x sin a+ cos @)" — x sin (na) — cos (na), then 58. 60, 61. 62. 63. |. Let fix) = ax? (a) £ dare zeros of pix) (b) p(x) is divisible by 7 +1 (©) £ 1 are zeros of p(x) (d) pla) is divisible by x? - | Leta, b, > 0 and for all p. g € R with p +4 =1 pa? + qb* > pqc’, then @) atb>e ) ate>b © b+c>a W) atb=c . Let a, b be two distinct roots of at 423-150, and pax) =a tx4 4 7 (a) ab is a root of p(x) = 0 (b) @ + bis a root of p(x) = 0 (©) both a + 6 and ab are roots of pix) = 0 (2), none of ab, a + b is a root of pix) = 0 Ifa? + pr + 1 isa factor of ax’ + br +c, then (a) ax + ¢ is also a factor of ax’ + br +.¢ (b) ap+e=20 © @-c = ab (@) Qa ~ bP = B+ 4c? Let a, b, c, d and p be integers such that p #0. If x+ pisa factor of ax’ + bx’ +.cx +d, then (a) pla (b) pPlipe = d) (©) d=0 = pis a root of ax” ~ br +c=0 (@) none of these. ‘The equation log .,,.,,.(log, (2-29) =0 has (a) no solution (b) exactly one solution (©) at least one negative solution (4) at Teast two solutions, Let—1

You might also like